Sei sulla pagina 1di 364

Sucesiones y límites de sucesiones Tema 1

Tema 1

Sucesiones de números reales. Límites de sucesiones

1.1 Conceptos previos


Intervalos y entornos

Definición Dados dos números reales a y b, con a < b , se define intervalo cerrado de
extremos a y b, y se representa por [a , b ] , al conjunto [a, b ] = {x ∈ R / a ≤ x ≤ b} . Su
representación en la recta real es:

a b
[ ]

Definición Se define intervalo abierto de extremos a y b, y se representa por (a, b ) , o bien

]a, b[ , al conjunto (a, b ) = {x ∈ R / a < x < b} .

Su representación sobre la recta real es:

a b
( )

En las definiciones anteriores, los extremos pueden ser infinitos. Al intervalo [−∞,+∞ ] se
le llama recta real ampliada y se denota por R .

Nota Evidentemente, los intervalos cerrados contienen a sus extremos mientras que los
abiertos no; pero hay otra diferencia sustancial entre intervalos abiertos y cerrados. En el
intervalo [a , b ] , a es el menor y b el mayor de los números que pertenecen a dicho intervalo,
llamados mínimo y máximo respectivamente; sin embargo, en el intervalo (a, b ) no hay ni
mínimo ni máximo, pues a no pertenece al intervalo y no puede encontrarse ningún número a’
de (a, b ) que sea mayor que a y tal que entre a y a’ no haya ningún número de (a, b ) . Muy al
contrario, en la recta real sucede que entre dos números cualesquiera, por próximos que se
elijan, siempre hay infinitos números reales.

Definición Se llama entorno de un punto a todo intervalo abierto que lo contenga.

Así, por ejemplo, el intervalo (8,12) es un entorno del punto 9, como también lo es del punto 10
y de todos los puntos del propio intervalo.

Nota Si el punto está en el centro del intervalo entonces, a ese intervalo, se le llama entorno
centrado. A la distancia entre el centro y cualquiera de los extremos del intervalo se le llama
radio del entorno. Así el intervalo (8,12) es un entorno centrado de centro c = 10 y cuyo radio

1-1
Sucesiones y límites de sucesiones Tema 1

r = 2.

Definición Se llama entorno de centro a y radio r (r > 0 ) , y se denota por E (a, r ) o E r (a ) ,


al intervalo abierto (a − r , a + r ) .

Nota Observa que E r (a ) representa también el conjunto de puntos que distan de a en menos
de r unidades. En efecto, el conjunto de puntos que distan de a en menos de r unidades es
{x ∈R / x − a < r }. La desigualdad anterior es equivalente a la doble desigualdad −r < x − a < r
y sumando a a los tres miembros se tiene a − r < x < a + r , cuya solución son todos los puntos
del intervalo (a − r , a + r ) .

Formas de determinar un entorno

Según lo anterior, un entorno centrado queda determinado si se conoce el centro y el


radio o se conocen sus extremos o viene dado de la forma {x ∈R / x − a < r } .
Ejemplos

Ejemplo 1.1 El entorno de centro 3 y radio 5 queda determinado de cualquiera de estas tres
formas:
a) E 5 ( 3 ) b) (− 2,8 ) c) {x ∈ R / x − 3 < 5}.

Ejemplo 1.2 Si queremos determinar el centro y el radio del entorno centrado definido por el
intervalo (− 3,5 ) , se calcula el radio como la semidiferencia de la distancia entre los extremos
5 − ( −3)
= 4 y, como consecuencia, el centro es 5 − 4 = 1 o −3 + 4 = 1 . Por lo que se puede
2
escribir (−3,5 ) = E 4 (1) .

Ejemplo 1.3 Para determinar los extremos del entorno {x ∈ R / x + 5 < 0,5}, como el centro es
−5 y el radio es 0’5, los extremos inferior y superior son respectivamente a = −5 − 0'5 = −5'5 y
b = −5 + 0'5 = −4'5 , por lo que el intervalo que resulta es (a, b ) = (-5’5,-4’5).

1.2 Sucesiones de números reales


De un modo intuitivo se puede definir una sucesión como un conjunto ordenado e
infinito de números reales.

Definición Una sucesión es una aplicación a de N * en R , tal que a cada número natural n
le asigna como imagen el número real a(n ) = a n .

Esto es, n indica la posición y a n es el término de la sucesión que corresponde a esa


posición. De forma análoga, denotamos por a1 al primer término, a 2 al segundo y, así,
sucesivamente. Al término que está en el lugar n , esto es, al a n , se le llama término general
de la sucesión o término enésimo.

Nota Evidentemente, cada término a n de una sucesión tiene un término siguiente que
denotamos por a n +1 , pero no hay último término.

1-2
Sucesiones y límites de sucesiones Tema 1

Ejemplos

Ejemplo 1.4 Los múltiplos de tres determinan la sucesión de términos: 3, 6, 9, 12,....

Ejemplo 1.5 Los números impares forman la sucesión de términos: 1, 3, 5, 7, 9, ......

1.2.1 Formas de determinar una sucesión


Mediante el término general La sucesión queda determinada al conocer la expresión del
término enésimo en función de n. Esta es la mejor forma de expresar las sucesiones.

Ejemplo 1.6 Si el término general de la sucesión es an = 3n , dando a n los valores 1, 2, 3, ...


resulta la sucesión 3, 6, 9, 12,..... Y, recíprocamente, es fácil ver que el término general de la
sucesión 1, 3, 5, 7, 9, ......es a n = 2n − 1 .

Mediante una ley de recurrencia Consiste en dar una ley de construcción en la que cada
término de la sucesión quede determinado a partir de los anteriores.

Ejemplos

Ejemplo 1.7 Dada la sucesión a n definida por la ley de recurrencia a1 = a 2 = 1 y


an = an −1 + an − 2 , los distintos términos de la sucesión se obtienen dando valores a n o, también,
observando que, en este caso, cada término es la suma de los dos anteriores. En efecto, para
n = 3 se tiene que a 3 = a 3 −1 + a 3 −2 = a 2 + a1 = 1 + 1 = 2 . De manera análoga se obtendrían los
restantes, resultando 1, 1, 2, 3, 5, 8, .... (sucesión de Fibonacci).

Ejemplo 1.8 La sucesión 3, 6, 9, 12,.... se puede expresar como a1 = 3 y a n = a n −1 + 3 . Esto


es, cada término se obtiene sumando 3 al anterior.

Nota A partir de la ley de recurrencia es posible encontrar el término general, aunque no


siempre es sencillo.

Nota Existen sucesiones que no pueden ser expresadas de ninguna de estas dos formas,
como por ejemplo las sucesivas aproximaciones por defecto de 2 : 1, 1’4, 1’41,... En lo
sucesivo consideraremos sucesiones que puedan ser expresadas mediante su término
general.

Notaciones Una sucesión de término general a n se puede escribir como {a n : n ∈ N } , (a n )1∞ ,


{a n }, (a n ) o simplemente a n ; por comodidad, adoptaremos esta última notación.

1.2.2 Representación gráfica


Las sucesiones se pueden representar en la recta real, o bien en un sistema de ejes
cartesianos.
2 1 2 2
Así, por ejemplo, la sucesión a n : 2, 1, , , ,..... .... se puede representar como
3 2 5 n

1-3
Sucesiones y límites de sucesiones Tema 1

o bien

1.2.3 Sucesiones monótonas y acotadas

Definiciones Una sucesión se dice que es monótona creciente si a n+1 ≥ a n ∀n ∈ N . Se dice


estrictamente creciente si a n +1 > a n ∀n ∈ N .

Se dice que una sucesión es monótona decreciente si a n+1 ≤ a n ∀n ∈ N . Se le llama


estrictamente decreciente si a n +1 < a n ∀n ∈ N .

Nota Se dice que una sucesión es monótona si es monótona creciente o monótona


decreciente.

Ejemplo 1.9 La sucesión 5, 7, 9, ...., 2n + 3, .... es estrictamente creciente. La sucesión


1 1 1 1 1 1
1, , , ..... , .... es estrictamente decreciente. La sucesión − 1, , − , .....(− 1)n ⋅ , .... no es
2 3 n 2 3 n
monótona.

Definiciones Se dice que una sucesión está acotada superiormente si existe un número
real k , llamado cota superior, tal que a n ≤ k , ∀ n ∈ N .

Se llama supremo de la sucesión a la menor de todas las cotas superiores y si éste


es un término de la sucesión se llama máximo.

Se dice que una sucesión está acotada inferiormente si existe un número real k,
llamado cota inferior, tal que a n ≥ k , ∀ n ∈ N .

Se llama ínfimo de una sucesión a la mayor de todas las cotas inferiores y si,
además, es un término de la sucesión se llama mínimo .

Se dice que una sucesión está acotada cuando lo está superior e inferiormente.

Nota Toda sucesión acotada tiene siempre supremo e ínfimo.

1-4
Sucesiones y límites de sucesiones Tema 1

Ejemplos

1 1 1
Ejemplo 1.10 La sucesión 1, , , ..... , .... está acotada superiormente, una cota superior es
2 3 n
1 que es además el máximo. También está acotada inferiormente, una cota inferior es 0 que es
el ínfimo pero no es mínimo, porque no pertenece a la sucesión.

Ejemplo 1.11 La sucesión 5, 7, 9, ...., 2n + 3, .... está acotada inferiormente por 5 pero no tiene
cota superior.
2n
Ejemplo 1.12 Justifiquemos que la sucesión a n = es monótona creciente. Los primeros
2n + 1
2 4 6
términos son , , ,... y se observa que cada término es mayor que el anterior, pero eso no
3 5 7
asegura que la tendencia no pueda invertirse, así que deberemos comprobar que a n +1 ≥ a n
para todos los valores de n.
Hay dos modos de comprobar esa desigualdad, probando que a n +1 − a n ≥ 0 (criterio
an +1
de la resta) o, bien, cuando la sucesión es de términos positivos, comprobando que ≥1
an
(criterio del cociente).
Como en este caso se tiene una sucesión de números positivos, podemos utilizar éste
a 2( n + 1) 2n 4n 2 + 6n + 2
último, n +1 = : = > 1 , luego es creciente.
an 2( n + 1) + 1 2n + 1 4n 2 + 6n
1
Ejemplo 1.13 Comprobemos que 3 es una cota superior y una cota inferior de la sucesión
2
anterior.
Para probar que an ≤ 3 ∀n ∈ N bastará ver que 3 − an > 0 . En efecto,
2n 4n + 3
3− = > 0 , para todo valor de n.
2n + 1 2n + 1
1
Comprobar que es una cota inferior es trivial, pues se trata de una sucesión
2
2 1
monótona creciente y su primer término es que ya es mayor que .
3 2

1.3 Concepto de límite de una sucesión


1
Considera la sucesión an = .
2n

Evidentemente, la sucesión a n es decreciente y está acotada. Su supremo es1/2 y su


ínfimo es el 0.

Como muestra la figura, a medida que hallamos los términos de la sucesión, la


distancia entre dos términos consecutivos va disminuyendo. Además, cada nuevo término se
encuentra a menos distancia del cero que el anterior. Esto es, los términos se acercan al cero
al aumentar n, hasta el punto de que parece lógico afirmar que en cualquier entorno del cero,
por pequeño que sea, hay infinitos términos de la sucesión; lo que permite dar la siguiente
definición:

1-5
Sucesiones y límites de sucesiones Tema 1

Definición Decimos que una sucesión a n tiene por límite un número real a si, y sólo si,
para todo entorno de a, por pequeño que sea, contiene infinitos términos de la sucesión, y
fuera de él quedan un número finito de términos. En tal caso, escribimos: lim a n = a , que
n →∞

se lee “límite cuando n tiende a infinito de a n es a“.

Nota Si lim a n = a , también se dice que a n tiende a a .


n →∞

De una manera rigurosa, los matemáticos han formulado la anterior definición como
sigue:

Definición Se dice que una sucesión a n tiene por límite a , a ∈ R , y se denota por
lim a n = a si, y sólo si, para todo ε > 0 (por pequeño que sea) existe un n0 ∈ N (que
n →∞

depende de ε ) tal que para todo n ≥ n 0 se tiene que a n − a < ε ; o, lo que es lo mismo,

an ∈ (a − ε, a + ε ) .

Nota Observemos que esto significa que para cualquier entorno de a , por pequeño que sea,
hay siempre un término a n0 de la sucesión, a partir del cual los infinitos términos posteriores
están todos dentro de ese entorno. Y que fuera del Eε (a ) sólo hay un número finito de
términos de la sucesión. En concreto, hay n0 − 1 términos. Veamos, con algunos ejemplos,
cómo se comprueba el límite de una sucesión.

1
Ejemplo 1.14 La sucesión tiene por límite 0.
n
La definición exige demostrar que para todo ε > 0 existe un número natural n0 tal que para
1 1 1 1
todo n ≥ n0 : − 0 < ε ; pero = < ε ⇔ < n . Luego, sí es posible encontrar un n0
n n n ε
a partir del cual todos los a n de orden posterior ( n ≥ n0 ) estén en el Eε (0 ) . El más
1
pequeño posible de estos, el n0 , será: n0 = E   + 1 . (La notación E [ x ] significa "parte
ε 
entera de x", que por definición es el mayor entero menor o igual que x.)

Nota En este ejemplo, si ε = ½ n0 = 3; si ε = 10-2 n0 = 101.

Observa 1) Que n0 depende de ε ; 2) que fuera del E ε (0 ) hay sólo un número finito de
términos y 3) que dentro del E ε ( 0) hay infinitos términos.

Ejemplos

2n + 3
Ejemplo 1.15 Dada la sucesión a n = a) comprueba que es monótona decreciente y
n +1
que 2 es una cota inferior. b) calcula los términos que pertenecen a un entorno de centro 2 y
radio 0’2.

1-6
Sucesiones y límites de sucesiones Tema 1

a n+1 2(n + 1) + 3 2n + 3 2n 2 + 7n + 5
La sucesión es monótona decreciente pues = : = <1
an (n + 1) + 1 n + 1 2n 2 + 7n + 6
2n + 3 1
2 es una cota inferior ya que −2= >0
n +1 n +1
5 7 9 11 13
El entorno es el intervalo abierto (1'8,2'2) y los primeros términos son , , , , ... luego a
2 3 4 5 6
partir de a 5 todos los términos están dentro del entorno dado.

Ejemplo 1.16 Comprueba con la definición que 2 es el límite de la sucesión anterior.

Para comprobar que 2 es el límite no valen casos particulares con un radio


determinado como el del apartado anterior, hay que comprobar si se satisface la definición de
2n + 3
límite para cualquier ε dado. Veamos cuándo se cumple la condición −2 <ε ,
n +1
2n + 3 − −2n − 2 1 1 1 1− ε
< ε⇔ < ε y como es positivo se tiene que <ε⇔n>
n +1 n +1 n +1 n +1 ε
y cualquiera que sea el valor de ε siempre habrá un número natural que cumpla esa condición,
el término que corresponde a esa posición y todos los que le siguen cumplen esa condición
2n + 3
luego lim =2
n→∞ n + 1

2n − 3
Ejemplo 1.17 Prueba que lim =2
n→∞ n+5
2n − 3
Será cierto si dado un ε > 0 ∃ n 0 ∈ N / ∀n ≥ n 0 −2 <ε , es decir,
n+5
− 13 13 13 − 5ε  13 − 5ε 
= <ε ⇔ < n . Luego, sí existe n0 = E   +1 .
n+5 n+5 ε  ε 

1
Ejemplo 1.18 Prueba que lim no es 2.
n→∞ n
1 1 − 2n
Supongamos que sea 2 entonces dado un ε > 0 ∃ n 0 ∈ N / ∀n ≥ n 0 −2 <ε ⇔ <ε
n n
2n - 1 1
⇔ < ε ⇔ 2n-1 < ε n ⇔ (2-ε)n < 1 ⇔ n < , luego, sólo hay un número finito de
n 2 -ε
1
valores de n que verifiquen que n < , por lo tanto, 2 no es el límite.
2-ε

1.4 Sucesiones convergentes

Definición Llamamos sucesión convergente a toda sucesión que tenga límite finito.

Proposición (Unicidad del límite) El límite, si existe, es único.

Demostración.- Lo demostraremos por reducción al absurdo. Supongamos que exista una


sucesión a n tal que

1-7
Sucesiones y límites de sucesiones Tema 1

(1) lim a n = a y (2) lim = b , con a ≠ b , a < b


n →∞ n →∞
De (1), por definición de límite, se tiene que para todo ε >0  E(a) que contiene infinitos
términos de a n y fuera sólo un número finito.
De (2), análogamente, para el mismo ε anterior, existirá un E(b) tal que contendrá
infinitos términos de a n y fuera sólo un número finito.
b−a
Lo anterior será cierto, en particular, para ε= , pero sabemos que
2
E b- a (a) ∩ E b- a (b) = ∅ , luego si fuera del entorno de a hay un número finito de términos es
2 2
porque en el entorno de b sólo hay un número finito, y viceversa, en contra de (1) y (2).

Proposición Toda sucesión convergente está acotada

Demostración.- Sea a n una sucesión convergente ⇔ lim an = a dado  ε >0  ∃ n0 de N tal que
n→∞

para todo n ≥ n0 ⇒ an − a < ε⇔− ε < an − a < ε , esto es, para todo n ≥ n0 se tiene que:
a − ε < an < a + ε .
(
Luego, M = max a1, a2 ,K, an0 −1, a + ε ) es cota superior de a n y también m =
( )
min a1, a2 ,K, an0 −1, a − ε es cota inferior de a n .

Sugerencia: Como ejemplo de la anterior propiedad, considera la sucesión


(− 1)n , demuestra
n
que converge a cero, toma ε = 0′1 y halla M y m. Haz también el dibujo.

Nota El recíproco no es cierto. Por ejemplo, la sucesión -1,1,-1,1,-1,1, ... está acotada, pero
no es convergente.

Nota Sin embargo, añadiendo la condición de monotonía, el recíproco se hace cierto, como
veremos en la próxima propiedad.

Proposición Toda sucesión monótona y acotada es convergente.

Demostración.- Sea a n una sucesión monótona creciente y acotada. Como está acotada, entre
todas las cotas superiores elijo la menor, es decir, el supremo, que llamaré a.
Por definición de supremo: para todo ε > 0, ∃n0∈N,a − ε < an0 . Por otro lado, como a n
es creciente: para todo n ≥ n0 ,a − ε < an0 Y, por supuesto, an < a + ε .
Resumiendo, tenemos que: ∀ε > 0, ∃n0∈N /∀n ≥ n0: an − a < ε .

Nota Pero, hay sucesiones convergentes que no son monótonas. Por ejemplo, 0, 1/2, 0, 1/3, 0,

1/4, ..., 0, 1/n, 0,...; y también la


(− 1)n , no son monótonas y , sin embargo, ambas convergen
n
a cero.

1.5 Infinitésimos (sucesiones nulas)

Definición Se dice que una sucesión es un infinitésimo si, y sólo si, su límite es cero.

1-8
Sucesiones y límites de sucesiones Tema 1

1 n +1
Ejemplo 1.19 Las sucesiones a n = , bn = 2 , c n = b n si b < 1 , son infinitésimos.
n n

Nota Observa que si lim a n = a , entonces lim a n − a =


n →∞ n →∞
( ) 0 ; es decir, la sucesión
b n = a n − a es un infinitésimo.

1.6 Subsucesiones

Si se toman infinitos términos de una sucesión dada, conservando su relación de orden, se


dice que se ha formado una subsucesión de la sucesión inicial.

1 1
Ejemplo 1.20 es una subsucesión de .
2n n

Proposición Toda subsucesión de una sucesión convergente es convergente y converge


al mismo número que la sucesión inicial.

Nota Esta propiedad nos va a servir para ver que no tiene límite una sucesión dada.

Ejemplo 1.21 La sucesión 1 , 1 2 , 1 , 1 4,K no tiene límite porque podemos extraer dos
subsucesiones: la 1, 1, 1, ... y la 1 2 , 1 4,K , que tienen límites 1 y 0, respectivamente.

1.7 Sucesiones divergentes


No todas las sucesiones, como hemos visto, tienen límite.

Definición A las sucesiones que no tienen límite las llamaremos divergentes.

Entre estas sucesiones hay unas que parecen perderse en el infinito, como
n 1 1
2 ,n ,n − n 2 ,(− 1) ⋅ n 2 ,K , y otras como (− 1) ;1, ,1, ,K; a las primeras las llamamos
n 2 n
2 4
divergentes en el infinito, mientras que a las segundas se les denomina oscilantes.

1.8 Cálculo de límites de sucesiones


La siguiente proposición permite obtener los límites de sucesiones complicadas
mediante operaciones entre límites de sucesiones más simples.

1-9
Sucesiones y límites de sucesiones Tema 1

Proposición (Propiedades algebraicas)

Sean a n , bn sucesiones tales que lim a n = a y lim bn = b , a , b ∈ R , entonces:


n →∞ n →∞

i) lim (a n ± bn ) = a ± b
n →∞

ii) lim (a n ⋅ bn ) = a ⋅ b
n →∞

an a
iii) lim = , siempre que b sea distinto de cero.
n →∞ b b
n

iv) lim (a n )
bn
= ab
n →∞

an
Nota En el caso iii) no importa que no esté definido para algún valor de n .
bn
Nota Al estudiar esta proposición conviene que la aprendas también con palabras, es decir,
"límite de la suma es la suma de los límites",... .
1
Como se verá, el resultado más utilizado en el cálculo de límites es lim = 0 , que
n →∞ n
demostramos en el ejemplo 1.14 .
Para calcular el límite de una sucesión a n se descompone en sumas, productos y

cocientes de otras sucesiones, cuyos límites sean calculables y de forma que se pueda aplicar
la proposición 1.5 .

1.8.1 Límites de sucesiones polinómicas

 + ∞ , si k 0 > 0
(
Proposición lim k 0 n r + k 1n r −1 + K + k r =  )
n →∞ − ∞ , si k 0 < 0

 k 
( ) k k
Demostración. lim k 0 n r + k1n r −1 + K + k r = lim n r  k 0 + 1 + 22 + K + rr  =
n→∞ n→∞  n n n 

 k k   1 1 
= lim n r lim  k 0 + 1 + K + rr  = lim n r  lim k 0 + k1 lim + K + k r lim r  = ∞ ⋅ k 0 = ±∞
n→∞ n→∞  n n  n→∞  n→∞ n→∞ n n→∞ n 

Ejemplo 1.22 (
lim n 7 + 3n − 1 = +∞,
n→∞
) n→∞
lim (− 3n + 4 ) = −∞

1.8.2 Formas indeterminadas


n −1
Para calcular, por ejemplo, lim no podemos razonar del siguiente modo: “como
n →∞ n
tanto el numerador como el denominador son polinomios con coeficiente principal positivo, el

1-10
Sucesiones y límites de sucesiones Tema 1


límite es ”. Esto es, no se puede aplicar la proposición 1.5 sin más; sino que hay que

comprobar si se verifica la hipótesis que, evidentemente, no se cumple porque no son
sucesiones convergentes.
Para calcularlo, se debe transformar la expresión dada en otra en la que aparezcan
sucesiones convergentes, si es posible. Así, podría dividirse el numerador entre el
denominador, obteniendo de cociente 1 y de resto –1, quedando:
n −1  1
lim = lim 1 −  = 1 − 0 = 1 .
n →∞ n n →∞  n

También se podría haber hecho:


n −1 n 1  1
lim = lim  −  = lim 1 −  = 1 − 0 = 1 .
n →∞ n n → ∞ n n n → ∞  n
O, sencillamente, como se vio en la demostración del límite de una sucesión
polinómica, sacando como factor común la n en el numerador, se tiene:
 1
n 1 − 
n −1  n  1
lim = lim = lim 1 −  = 1 − 0 = 1 ,
n →∞ n n →∞ n n → ∞  n

lo que vendría a determinar un método consistente en dividir numerador y denominador entre n


(en este caso, o entre la mayor potencia de n para otros casos) tal como sigue:
n 1  1
−  1− 
lim
n −1
= lim n n lim  n  = lim 1 − 1  = 1 − 0 = 1 .
n →∞ n n → ∞ n n → ∞  1  n →∞  n
 
n  


De manera análoga a este ejemplo en el que aparece , se podrían poner otros en lo

0
que se obtiene , 0 ⋅ ∞ , ∞ − ∞ , 1∞ , 0 0 , ∞ 0 sin llegar a determinar cuál es el resultado
0
correcto del límite.

A estas siete expresiones se les conoce por el nombre de formas indeterminadas del
cálculo de límites y se llaman indeterminadas porque, cuando aparecen, no se puede predecir
cuál va a ser el límite.

Junto a las anteriores formas hay otras que no son indeterminadas. De una forma
resumida se dan a continuación en forma de tabla una amplia relación de límites de
operaciones con sucesiones:

1-11
Sucesiones y límites de sucesiones Tema 1

Suma:

lim a n = a a a +∞ −∞ +∞
n→∞

lim b n = b +∞ −∞ +∞ −∞ −∞
n→∞

lim (a n + b n ) = a+b +∞ −∞ +∞ −∞ indet.


n→∞

Producto:

lim a n = a a>0 a>0 a<0 a<0 0 +∞ −∞ +∞


n→∞

lim b n = b +∞ −∞ +∞ −∞ ±∞ +∞ −∞ −∞
n→∞

lim (a n ⋅ b n ) = a⋅b +∞ −∞ −∞ +∞ indet. +∞ +∞ −∞


n→∞

Cociente:

lim a n = a a≠0 0 a ±∞ ±∞ an
n→∞ (∗) lim = +∞
n→∞ bn
lim b n = b≠0 0 0 ±∞ ±∞ 0
n→∞
an
an a indet. (∗) indet. 0 indet. indet. (∗ ∗) (∗ ∗) lim = +∞ en estos
lim = n→∞ bn
n→∞ b n b
casos el límite puede ser ±∞ o no
existir en función del signo de los términos del infinitésimo.

Potencia: ( a n ≥ 0 y a ≥ 0 , pues si no, la potencia no siempre es un nº real).

lim a n = a 0 a <1 a <1 a >1 a >1 1 +∞ +∞ +∞ 0 0 0 0


n→∞

lim b n = b 0 +∞ −∞ +∞ −∞ ±∞ 0 −∞ +∞ +∞ −∞ b>0 b<0


n→∞

lim a n
bn
= a b indet. 0 +∞ +∞ 0 indet. indet. 0 +∞ 0 +∞ 0 +∞
n→∞

1.8.3 Límites de sucesiones racionales

 ∞,si r > t
k n + k1n +K+ k r 
r r −1
Proposición lim 0 t =  0,si r < t
n → ∞ s n + s n t − 1 +K+ s
0 1 r 
k 0 s0 ,si r = t

Demostración. La demostración es inmediata utilizando el artificio de dividir numerador y

1-12
Sucesiones y límites de sucesiones Tema 1

denominador entre la mayor potencia de n.


Ejemplos

2n 3
Ejemplo 1.23 lim = ∞ , ya que el grado del numerador es mayor que el grado del
n →∞ 3n − 1
denominador.

7n 3 − n 2 + 3 7
Ejemplo 1.24 lim 3
= , ya que el grado del numerador es igual al grado del
n →∞ 2n + n − 3 2
denominador.

1.8.4 Límites de sucesiones irracionales



Indeterminación

n +1
Ejemplo 1.25 Para calcular lim , se puede proceder de dos modos. El primero
n →∞ 3 + n
consiste en dividir numerador y denominador entre la mayor potencia de n, tal y como sigue:
n 1 1
+ 1+
n +1 n = 1 =∞.
lim = lim n n = lim
n →∞ 3 + n n →∞ 3 n n →∞ 3 1 0
+ 2
+
n n n n

El segundo consiste en hacer un cambio de variable. Se hace z 2 = n , entonces:

n +1 z2 + 1
lim = lim = ∞ , porque el grado del numerador es mayor que el grado del
n →∞ 3 + n z →∞ 3+z

denominador.

Nota En general suele ser más cómodo el primer método.


Ejemplos

n2 + 3 + 4
Ejemplo 1.26 Para calcular lim se divide numerador y denominador entre la
n→∞ n −1

mayor potencia de n teniendo en cuenta el índice del radical. En este caso será n 2 y resulta

n2 3 4 3 4
2 2
+ 2
+ 1+ 2
+
n +3 +4 n n n n n 1
lim = lim = lim = =∞.
n →∞ n −1 n →∞ n 1 n →∞ 1 1 0
2
− 2
− 2
n n n n

3
2n 2 + 1
Ejemplo 1.27 Para calcular lim , se puede poner todo bajo una misma raíz de
n →∞ n −1
índice el mínimo común múltiplo de los índices que aparecen. Procediendo de este modo
resulta:

1-13
Sucesiones y límites de sucesiones Tema 1

lim
3
2n 2 + 1
= lim 6
(2n 2
+1 ) 2
= 6 lim
(2n 2
+1 )
2
= 6 lim
4n 4 + 4n 2 + 1
=
6
∞ =∞.
3 3 3 2
n →∞ n −1 n →∞ (n − 1) n →∞ (n − 1) n →∞ n − 3n + 3n − 1

Indeterminación ∞ − ∞

Ejemplo 1.28 Para calcular lim  n − 7 − n 3 + 1  , se multiplica y divide por el conjugado


n →∞ 
quedando:
 n − 7 − n 3 + 1  n − 7 + n 3 + 1 
  
 3     n − 7 − n3 − 1
lim  n − 7 − n + 1 = lim = lim =
n →∞  n →∞ n →∞
n − 7 + n3 + 1 n − 7 + n3 + 1
1 8
− 1+ −
− n3 + n − 8 n 2
n3 −1
= lim = lim = = −∞ .
n →∞
n − 7 + n3 + 1 n →∞ 1 7 1 1 0
5
− 6
+ 3
+
n n n n6

1.8.5 Límites en los que aparece el número e


n
 1
La sucesión de término general a n = 1 +  es particularmente interesante, fue
 n

estudiada por el matemático suizo Leonhard Euler y a su límite le llamó número e (¿inicial de
su apellido?), este número es de una tremenda importancia en matemáticas y aparece en el
estudio de multitud de problemas en todas las ciencias, ahora nos vamos a limitar al estudio de
la sucesión anterior y su utilización en la resolución de las indeterminaciones del tipo 1∞ .
Con la calculadora puedes obtener algunos términos de la sucesión:
1 2 3 4
 1  1  1  1
a1 = 1 +  = 2 , a 2 = 1 +  = 2' 25 , a 3 = 1 +  = 2' 3704 .. , a 4 = 1 +  = 2' 441... ,
 1  2  3  4

y si calculas términos más avanzados


10 40 1000
 1  1   1 
a10 = 1 +  = 2' 5937 .. , a 40 = 1 +  = 2' 685 .. , a1000 = 1 +  = 2' 7169 .. ,
 10   40   1000 
se puede observar que:
- Es una sucesión monótona creciente.
- Es una sucesión acotada. Una cota inferior es 2 y una cota superior 3.

Evidentemente, esta observación con cálculo de términos concretos no demuestra


nada, pero puede probarse, mediante un proceso laborioso, que ambas afirmaciones son
ciertas.
Por tratarse de una sucesión monótona y acotada la proposición 1.3 asegura que es
convergente. El límite de dicha sucesión es un número irracional, es decir formado por infinitas

1-14
Sucesiones y límites de sucesiones Tema 1

cifras decimales no periódicas y del que solo se pueden conocer aproximaciones decimales.
Por ejemplo, una aproximación sería el término 200.000 de la sucesión
500000
 1 
a1 = 1 +  = 2' 7170597 .. . Se sabe que las primeras cifras del número son
 500000 

e = 2' 71828182 ... .

Resolución de las indeterminaciones del tipo 1∞ .


3 n −1
 1 
Observemos si se calculan términos de la sucesión b n = 1 +  resulta:
 3n − 1
2 n 14
 1  1  1 
que b1 = 1 +  es el término a2 de a n = 1 +  , que b5 = 1 +  es el término a14 de
 2  n  14 
n
 1
a n = 1 +  .
 n

Por tanto, es razonable pensar que cuando n → ∞ el límite de la sucesión es el


número e.
n 2 + 2n
  n +1
 
1
En el caso de la sucesión c n = 1 + 2  en el denominador y en el
 n + 2n 
 
 n +1 
exponente obtenemos valores no enteros y la observación anterior parece no ser válida, pero
9' 9
 1 
si calculamos por ejemplo c 9 el resultado es c 9 = 1 +  que realmente no corresponde a
 9' 9 
n
 1
ningún término de la sucesión 1 +  pero su valor c 9 = 2' 5925904 .. está comprendido entre
 n
9 10 n
 1  1   1
los términos a 9 = 1 +  = 2' 5811748 .. y a10 = 1 +  = 2' 5937425 .. de a n = 1 +  y
 9   10   n 
por lo tanto también parece poder afirmarse que lim c n = e .
n →∞

Aceptemos pues que toda sucesión cuyo término general pueda escribirse de la forma
an
 1 
1 +  con lim a n = ∞ tienen como límite el número e.
 a n  n →∞

Abordaremos la resolución de las indeterminaciones del tipo 1∞ de dos modos ambos


basados en la observación anterior.

Método I

Se manipula algebraicamente la expresión dada buscando escribir la sucesión en la

1-15
Sucesiones y límites de sucesiones Tema 1

an
 1 
forma 1 +  , con lim a n = ∞ .
 a n  n →∞

Ejemplos

5n
 2n − 1 
Ejemplo 1.29 Para calcular lim   seguiremos los siguiente pasos:
n →∞  2 n+ 3 

1º se suma y resta 1 a la base con lo que


5n
 
5n 5n 5n  
 2n − 1   2n − 1   −4  1
lim   = lim 1 + − 1 = lim 1 +  = lim 1 + 
n →∞ 2 n+ 3  n →∞  2 n+ 3  n →∞  2 n+ 3  n →∞  2 n+ 3 
 
 −4 
2n + 3
2º se multiplica el exponente por y por su inversa, quedando:
−4
2 n+3 −4
⋅ 5n
  − 4 2 n +3 −4
 1  lim 2n + 3 5 n
= lim 1 +  =e n→ ∞
= e −10
n →∞  2 n+ 3 
 
 −4 
2 n+ 3
  −4
  −4 −20n
1
ya que lim 1 +  = e y lim 5n = lim = −10 .
n →∞  2 n+ 3  n →∞ 2n + 3 n →∞ 2n + 3
 
 −4 

n 2 −2
 3n 2 + n − 1 
Ejemplo 1.30 Para calcular lim   se sigue el mismo proceso que en el
n →∞  3 n + 3 
2

ejemplo anterior :
n 2 −2 n 2 −2 n 2 −2 n 2 −2
 3n 2 + n − 1   3n 2 + n − 1   3n 2 + n − 1   n−4 
lim   = lim 1 + − 1 = lim 1 + − 1 = lim 1 +  =
n →∞  3 n + 3 
2
 n →∞  3 n 2+ 3  n →∞  3 n 2+ 3  n →∞  3 n2+ 3 
3 n 2 +3 n − 4
( n2 −2 )
  n − 4 3 n 2 +3
  n−4 2 n 3 − 4 n 2 −2 n +8
1 lim (n −2 ) lim
= lim 1 +  2
3 n 2 +3
=e n → ∞ 3 n +3
=e n→∞
= e +∞ = +∞
n →∞  2
3n + 3 
 
 n−4 

Método II

Aplicaremos el proceso visto en los dos ejemplos anteriores al caso general lim (a n )bn
n →∞

con
lim a n = 1 y lim bn = ∞ . Se tendrá:
n →∞ n →∞

1-16
Sucesiones y límites de sucesiones Tema 1

1
(an −1)bn
  an −1
 
 1  lim bn (an −1)
lim (a n )bn = lim (1 + a n − 1)bn = lim 1 + = e n →∞ ,
n →∞ n →∞ n →∞ 1 
 
a n − 1 

lim bn (an −1)


de donde resulta la fórmula lim (a n )bn = 1∞ = e n → ∞
n →∞
( )

que podemos aplicar para resolver las indeterminaciones del tipo 1∞ .

n+2
 3n 2 − 5 
Ejemplo 1.31 Para calcular lim   aplicamos la fórmula anterior con lo que:
n →∞  
2
 3 n + 3 

n +2  3 n 2 −5   −8   −8 n −16 
 3n 2 − 5  lim  3 n 2 +3 −1⋅(n +2 ) lim  ⋅(n + 2 ) lim  
lim  
2
  n →∞  3 n +3  n →∞  3 n 2 +3 
=e n →∞
=e =e = e0 = 1
n →∞  3 n + 3 
2

1.8.6 Límites de otros tipos de sucesiones


Ejemplos
2
3−
3 n +1 − 2 3n = 3 = 3
Ejemplo 1.32 Calcula lim n
= lim
n →∞ 3 +4 n →∞ 4 1
1+ n
3

Hemos dividido entre 3n y hemos tenido en cuenta que k/ 3n tiende a cero cuando n
tiende a infinito.

1+ 2 + 3 + K + n n (n + 1) 2 n2 + n 1
Ejemplo 1.33 Calcula lim 2
= lim 2
= lim 2
= . Donde hemos
n →∞ n n →∞ n n →∞ 2n 2
aplicado la fórmula de la suma de n términos de una progresión aritmética.

1-17
Sucesiones y límites de sucesiones Tema 1

Ejercicios resueltos
Sucesiones
2n - 1
R.1 Calcula qué términos de la sucesión an = pertenecen al entorno con centro en 2/3
3n + 2
y radio 0'1. ¿Y al E 0′001 (2 3) ?
2
Solución.- i) Hemos de encontrar los a n tales que an − < 0′1 , esto es,
3
2n − 1 2 3(2n − 1) − 2(3n + 2) −7
− < 0′1 ⇔ < 0′1 ⇔ < 0′1 ⇔
3n + 2 3 3(3n + 2) 9n + 6
7 1 64
⇔ < ⇔ 70 < 9n + 6 ⇔ n > = 7′1 ; por consiguiente, desde el término a8
9n + 6 10 9
(inclusive) en adelante todos pertenecen a E0'1(2/3).
ii) Análogo.

2n + 5  1
R.2 ¿Cuántos términos de la sucesión pertenecen al intervalo  0,  ?.
3n + 2  5
Solución.-
 2n + 5   −5 
2n + 5 1  0 < 3n + 2   0 < 2n + 5   2 <n
< ⇔
2n + 5 1 
⇔ ⇔ − 23 
0<
n+2 5  <  10n + 25 < 3n + 2 n < 
 3n + 2 5   7 
Como la primera desigualdad se verifica para todo n de N, pero la segunda no se verifica para
 1
ningún n, podemos asegurar que ningún término de la sucesión pertenece al  0,  .
 5

R.3 ¿Cuántos términos de la sucesión a n = 3n 2 + 4 son mayores que 1000? ¿Y mayores que
k, para todo k positivo?
996
Solución.- i) Trivial: 3n 2 + 4 > 1000 ⇔ n 2 > = 332 ⇔ n > 332 = 18,22 . Por lo tanto, hay
3
infinitos términos de la sucesión mayores que 1000. En concreto, el a19 y todos los siguientes.
 k−4
k -4 n > ,si k ≥ 4
ii) 3n 2 + 4 > k ⇔ n 2 > ⇔  3
3  n > 0,si 0 < k < 4

Por lo tanto, si 0 < k < 4 , todos los términos son mayores que k, y si k ≥ 4 desde el término
k -4
que ocupa la posición E[ ]+1.
3

Cálculo de límites

(
R.4 Calcula lim (-1 )n (n+ 1) .
n →∞
)
 n + 1 , si n es par
Solución.- Puesto que (− 1)n (n + 1) =  y cada una de estas dos
− n − 1 , si n es impar
subsucesiones diverge a más infinito y menos infinito, respectivamente, la sucesión inicial no
tiene límite.
 5 − 4n 1 − 8n   10 − 8n − 1 + 8n  9 3
R.5 lim  −  = (− ∞ + ∞ ) = lim   = lim = .
n →∞ 3 6  n →∞  6  n → ∞ 6 2

1-18
Sucesiones y límites de sucesiones Tema 1

R.6 lim (n 2 )
- 3 n+ 1 - n = (∞ − ∞ ) = lim
n 2 − 3n + 1 − n 2
= lim
− 3n + 1 ∞
= =
n →∞ n →∞
n 2 − 3n + 1 + n n →∞
n 2 − 3n + 1 + n  ∞ 
1
−3+
n −3+0 −3
= lim = = .
n →∞ 3 1 1− 0 + 0 + 1 2
1− + +1
n n2
(1 + n ) n
∞
1 + 2 + 3 + ... + n 2 n + n2 1
R.7 lim =   = lim = lim = .
n →∞ (n+ 1 )3 - (n- 1 )3  
∞ n → ∞ 2
6n + 2 n → ∞ 2
12n + 4 12
2 n+1
0
4 3+2n n2  4
R.8 lim  n 3  =  =1 .
n →∞  5 n - 2 
 5
1 1
+ 2
n+ 1  ∞  n n 0
R.9 lim =   = lim = =0 .
n →∞ n+ 2  ∞  n →∞ 2 1+ 0
1+
n
n 2 + 4n + 1 − n 2 − 8n − 1
R.10 lim  n 2 + 4n + 1 − n 2 + 8n + 1 = (∞ − ∞ ) = lim =
n →∞  n →∞
n 2 + 4n + 1 + n 2 + 8n + 1
− 4n ∞ −4 −4
= lim =   = lim = = −2 .
n →∞
n + 4n + 1 + n + 8n + 1  
2 2 ∞ n → ∞ 4 1
1+ + 2 + 1+ + 2
8 1 2
n n n n
n2
 1  1
n+
R.11 lim   = 0∞ = 0 .
n →∞  2 n 2 - 1 

- n 2 - n+1
 2 n2 + 2  1 1
R.12 lim  2  = 2 −∞ = = =0 .
n →∞  n + n+ 1 
 2 ∞ ∞
n −1
 2+n + 1 3
R.13 lim  n  ( )
= 1∞ = e A , donde
n →∞  n2 
n − 1  n 2 + n + 1  n − 1  n + 1 n2 −1 1
A = lim ⋅

− 1 = lim ⋅ 
 n →∞ 3  n 2   = (∞ ⋅ 0 ) = lim = .
n →∞ 3
 n2  n → ∞ 3n 2 3
n −1
1
 2 + n + 1 3
Luego, lim  n
3
 = e 3 = e .
n →∞  n2 
- n+ 5
 2 n2 + 2 
R.14 lim  2 
n →∞  2 n + n+ 1 
( )
= 1−∞ = e A , donde

 2n 2 + 2   − n +1  n 2 − 6n + 5 1
A = lim (− n + 5 )  2 − 1 = lim (− n + 5 )  2  = (∞ ⋅ 0 ) = lim = .
n →∞  2n + n + 1  n →∞  2n + n + 1 n →∞ 2n 2 + n + 1 2
 
- n+ 5 1
 2 n2 + 2 
Luego, lim  2  = e2 = e .
n →∞  2 n + n+ 1 

R.15 En un círculo de 2 m de radio se inscribe un cuadrado y en este cuadrado se inscribe otro


círculo, en éste otro cuadrado y así indefinidamente. Halla: a) el límite de la suma de las áreas
de los infinitos círculos; b) el límite de la suma de las áreas de los infinitos cuadrados.
Solución.-

1-19
Sucesiones y límites de sucesiones Tema 1

En general, si se tiene un círculo de radio r, mediante el Teorema de Pitágoras es


sencillo ver que su cuadrado inscrito tiene de lado 2 r .
Por otro lado, si se tiene un cuadrado de lado L, es inmediato que su círculo inscrito
L
tiene de radio .
2
Pues bien, llamando r n al radio del enésimo círculo y Ln al lado del enésimo
cuadrado inscrito y aplicando lo comentado más arriba, se tiene:
como r1 = 2 , entonces L1 = 2 2 ;
para L1 = 2 2 , entonces r 2 = 2 ;
para r 2 = 2 , entonces L2 = 2 2 = 2;
2
para L2 = 2 , entonces r 3 = = 1;
2
para r 3 = 1 , entonces L3 = 2 ;
2 1
para L3 = 2 , entonces r 4 = = ;
2 2
y, así, sucesivamente.
1
a) Por lo tanto, la sucesión de los radios de los círculos viene dada por 2 , 2 , 1 , K . Ahora
2
bien, teniendo en cuenta el área de un círculo de radio r, la sucesión de las áreas vendrá dada
π
por 4 π , 2π , π , , K , y la suma de ellas será la suma de n de ellos cuando n tiende a infinito:
2
2 2 −n ⋅ 2 −1 − 4 0−4
n →∞
( )
S = lim 4 + 2 + 1 + 2 −1 + K + 2 2 −n ⋅ π = π ⋅ lim
n →∞ −1
= π ⋅ −1 = 8 π m2 ,
2 −1 2 −1
donde se ha utilizado que los sumandos del paréntesis forman una progresión geométrica de
1
razón .
2

b) Análogamente, la sucesión de los lados de los cuadrados inscritos es 2 2 , 2 , 2 , K


Ahora bien, teniendo en cuenta el área del cuadrado de lado L, la sucesión de las áreas
vendrá dada por 8 , 4 , 2 , K , y la suma de ellas será la suma de n de ellos cuando n tiende a
infinito:
2 4 − n ⋅ 2 −1 − 8 0−8
(
n →∞
)
S = lim 8 + 4 + 2 + K + 2 4 −n ⋅ π = π ⋅ lim
n →∞ −1
= π ⋅ −1 = 16 π m 2 ,
2 −1 2 −1
donde se ha utilizado que los sumandos del paréntesis forman una progresión geométrica de
1
razón .
2

Ejercicios propuestos
P.1 Dada la sucesión 20, 20/2 , ... , 20/n, ... comprueba que si elegimos un entorno de centro
-2
cero y radio 10 , hay 2000 términos de la sucesión que no pertenecen al entorno, pero sí
pertenecen los infinitos términos restantes.
n-5
P.2 Halla el primer término de la sucesión que pertenece al ]0'999,1'001[.
n +1
3n - 5
P.3 Averigua si 7/15 es un término de la sucesión cuyo término general es: an = .
2n + 7
3n + 7
¿Cuántos términos de la sucesión pertenecen al intervalo ]1,2[ ? ¿Cuántos quedan
2n + 1
fuera?

1-20
Sucesiones y límites de sucesiones Tema 1

2n − 3
P.4 ¿Cuántos términos de la sucesión pertenecen al entorno de centro 2 de radio
n+5
-5
10 ? ¿Cuántos quedan fuera?

P.5 Calcula los siguientes límites:

 n 2 + 1   1 + 2 n2 1 - 2 n2  3 n- 1
a) lim  n − b) lim  -  c) lim
n → ∞
 n − 1  n →∞  1 - n 1 + n  n →∞ 1
+1
2n
 2
d) lim  n - n
2 +1 


e) lim  (− 1)
n n 

 n 1 
n → ∞ n + 1
f) lim  (-1) − 
n →∞  2 n- 1 2 n+ 1  n→∞
 n+ 1
 1 2 n  4n - 2  1 
g) lim  2 + 2 + K + 2  h) lim i) lim  3 n 2 + 
n → ∞ n n n  n →∞ 3n + 5 n →∞  2 n+ 3 

j) lim
n→∞
( n + 5 − n) k) lim
3 n- 4 l) lim
n →∞
2 n2 + 3 n - n 2 + 7 n
n →∞ 2
3 n - n+ 1

m) lim (n- n2 + 10 n ) n) lim n2 - 1 - n2 + 1 ñ) lim ( 4 n 2 + 3 n+ 1 - 2 n- 1)


n →∞ n →∞ n →∞

  n−2  q) lim [ n ( n+ 2 - n+ 1 )] 25 n 2 + 1 - 9 n 2 + 1
p) lim n − 2  . n →∞ r) lim
n → ∞  n + 2 
   n →∞ 4 n2 + 1 - 1

P.6 Calcula los siguientes límites:


- n2 n 2n
1   8 n 3 - 1  2 n-1  n+ 3 
a) lim  
b) lim   c) lim  3

n →∞  n  3 n →∞  2 n - 1 
n →∞  2 n + n+ 1 

n 2 −3 n +7 n 2 −3
n+2   3 n 2 − 2n + 6   n+ 4 
d) lim   e) lim  2
 f) lim   ,
n →∞  n+ 1  n →∞  3 n − 2n − 1  n →∞  2 n+ 1 

n +2 n 2 + 2 n −7 n +3
 3n + 8   3n + 8   n+6 
g) lim   h) lim   5
n →∞  3 n − 1  i) lim 1 + 2 
n →∞  n + 6  n →∞  n − 5n + 6 
n +7 n +2
 2 − 2n + 7  n 2  n + 1 n 2 − 3n + 1 
j) lim  2 n 2  k) lim  + 
n →∞  
2 n+ 1 2
n →∞  2n + 8n − 1   2n + 5 n 

Soluciones de los ejercicios propuestos


P.2 desde el 6000. P.3 Desde el a6 (inclusive) están dentro. Quedan fuera los cinco primeros
términos. P.4 Pertenecen todos, menos los 1.299.996 primeros. P.5 a) –1; b) -4; c) ∞ ; d) ½ ;
1 1
e) no tiene; f) no tiene; g) ½ ; h) ∞ ; i) ∞ ; j) 0; k) 3 ; l) ∞ ; m) –5; n) 0; ñ) − ; p) −∞ ; q) ; r)
4 2
1
1
1. P.6 a) ∞ ; b) 2 ; c) 0; d) ∞ ; e) 1; f) 0 ; g) e 3 ; h) ∞ ; i) e 5 ; j) 1; k) .
e5

1-21
Límites de funciones Tema 2

Tema 2

Límites de funciones reales de variable real


En el tema anterior se ha estudiado el concepto de límite de una
sucesión ahora nos adentramos en el concepto de límite de una función en
sus diferentes formas, centrándonos de manera especial en el cálculo de
límites.

2.1 Límite de una función en un punto


Antes de definir de manera rigurosa el concepto de límite veamos una idea intuitiva del
mismo.

Afirmamos que el límite de una función y = f (x ) es el número real L cuando x tiende a


un número a, y se escribe lim f ( x ) = L , si al elegir valores de x cada vez más próximos al valor
x →a
a, los correspondientes a f (x ) se acercan a L, además podemos conseguir valores de f (x ) tan
próximos a L como queramos y serán todos los valores correspondientes a un determinado
nivel de cercanía de los x al valor de a.

Ejemplos

x 3 + 5x
Ejemplo 2.1 Consideremos la función f (x ) = y veamos qué ocurre al considerar
x2 + x
valores de x cada vez más próximos a cero.
Si nos acercamos primero por la derecha y después por la izquierda se obtiene:

x 0´5 0´1 0´01 0´001 0´0001


f (x ) 3´5 4´55... 4´95... 4´995... 4´99950...

x - 0´5 - 0´1 - 0´01 - 0´001 - 0´0001


f (x ) 10´5 5´56... 5´0506... 5´995... 5´0050...

Se observa que cuando x se acerca a cero los valores de la función se acercan a 5;


obsérvese también que ni siquiera existe f (0 ) .

 x − 2 si x < 1
Ejemplo 2.2 Consideremos ahora la función f (x ) =  y observemos qué ocurre
 x 2 si x ≥ 1
cuando x se acerca a 1 por la derecha y después por la izquierda.

x 2 1´5 1´1 1´01 1´001 x 0 0´5 0´9 0´99 0´999


f (x ) 4 2´25 1´21 1´0201 1´002... f (x ) - 2 - 1´5 - 1´1 - 1´01 - 1´001

Como puede verse no puede afirmarse que los valores de f (x ) se acerquen todos a un valor
determinado y en consecuencia no puede hablarse de límite de esa función cuando x tiende a

2-1
Límites de funciones Tema 2

1, sin embargo f (1) = 1. Nótese, pues, que la afirmación lim f ( x ) = L es independiente de la


x →a
existencia o no existencia de f (a) y, en caso de existencia, L no tiene porqué coincidir con f (a).

2.2 Definición de límite de una función en un punto


Sea la función f : A → R , donde A ⊆ R , y un a ∈R tal que existe un entorno reducido
de a incluido en A .
Definición Diremos que el límite de la función f en el punto a es un número real L, y lo
representaremos por lim f ( x ) = L , cuando se cumpla:
x →a

∀ε> 0, ∃δ > 0 tal que si 0 < x − a < δ , entonces f (x ) − L <ε .


Lo que significa que para cualquier entorno V de L existe un entorno reducido U de
a cuyas imágenes mediante f están todas en V.

Notas:
- Observemos que la condición 0 < x − a < δ expresa que x dista de a menos de δ unidades y
que no exigimos nada para x = a . Insistimos de nuevo en que el concepto de límite de f en a
es independiente de la existencia y, en su caso, del valor de f (a).

- Para cada ε hay un δ apropiado, pero no se exige que sirva el mismo δ para cada elección
de ε , es decir δ = δ (ε ) .

- Encontrado un δ , este no es único, puesto que también sirve cualquier número positivo
menor.

Ejemplo 2.3 Demostremos que lim (3 x + 1) = 7 .


x →2
Sea ε > 0 un número real cualquiera, según la definición 2.1 hemos de encontrar un
δ > 0 tal que si x − 2 < δ entonces f (x ) − 7 < ε ⇔ (3 x + 1) − 7 < ε .
Trabajando con la segunda desigualdad se puede llegar a la relación de ε con δ . En
efecto,
(3 x + 1) − 7 < ε ⇔ 3 x − 6 < ε ⇔ 3 x − 2 < ε ⇔ x − 2 < ε ,
3
ε
así que eligiendo δ = se cumple la definición.
3

Si existe el límite de una función en un punto, éste es único.

2-2
Límites de funciones Tema 2

Ejemplo 2.4 Demostremos que el límite del ejemplo 2.1 es único.

Supongamos que no es único, esto es, que lim (3 x + 1) = 7 y, además, lim (3 x + 1) = L ,


x →2 x →2
con L un número real distinto de 7.
ε
Si lim (3 x + 1) = 7 , dado un ε > 0 (y por tanto > 0 ) existe un δ1 > 0 tal que si
x →2 2
ε
x − 2 < δ1 entonces (3 x + 1) − 7 < . (1)
2
ε
Si lim (3 x + 1) = L , para ese >0 existe un δ 2 > 0 tal que si x − 2 < δ 2 entonces
x →2 2
(3 x + 1) − L < ε. (2)
2
Llamando δ al menor de δ1 y δ 2 es evidente que para los x tales que x − 2 < δ se
cumplen a la vez las desigualdades (1) y (2).

Como L ≠ 7 entonces L − 7 > 0 y aplicando (1) y (2) para ε = L − 7 , se tiene


ε ε
L − 7 = L − (3 x + 1) + (3 x + 1) − 7 ≤ L − (3 x + 1) + (3 x + 1) − 7 <
+ = L −7 ,
2 2
así se llega a la contradicción y, por tanto, la suposición inicial es falsa y el límite es único.

2.3 Límites laterales

Definiciones Se dice que el límite por la izquierda de la función f en el punto a es L, y se


representa por lim f (x) = L , si
x→a −
∀ ε > 0 , ∃ δ > 0 tal que si 0 < a − x < δ , entonces f (x ) − L <ε .
Se dice que el límite por la derecha de la función f en el punto a es L, y se representa por
lim f ( x ) = L , si
x→a +
∀ ε > 0 , ∃ δ > 0 tal que si 0 < x − a < δ , entonces f (x ) − L <ε .

 x 2 + 2, si x ≤ 0
Ejemplo 2.5 Estudiemos los límites laterales de la función f (x ) =  para x = 0 .
 x , si x > 0

Como puede observarse, tanto en la gráfica, como en las


funciones definidas a derecha e izquierda del cero, se tiene
lim f ( x ) = lim x = 0 y lim− f ( x ) = lim− x 2 + 2 = 2 .
x →0 + x →0 + x →0 x →0

En consecuencia, no existe lim f ( x ) .


x→ 0

Proposición Existe el lim f ( x ) = L si, y sólo si, existen los límites laterales y coinciden.
x→a

2-3
Límites de funciones Tema 2

Ejemplos

Ejemplo 2.6 Estudiemos el límite de la función f (x ) = [x ] cuando x tiende a 2 . ¿Qué pasa


cuando x tiende a 2,2 ?

Se ve en la figura que lim [x ] = 1 y lim [x ] = 2 ,


x →2− x →2+
por lo que no existe lim [x ] .
x →2
Sin embargo, sí que existe lim [x ] = 2 como
x → 2' 2
fácilmente puede verse en la gráfica.

Ejemplo 2.7 Estudiemos el límite de la función f (x ) = ln x


cuando x tiende a 1 .

Observando la gráfica, es fácil comprobar que lim ln x = 0 y


x →1−
lim ln x = 0 , por lo que lim ln x = 0
x →1+ x →1

2.4 Límites infinitos


Hasta aquí se ha estudiado el lim f ( x ) = L cuando tanto a como L son números
x→a
reales (finitos). A continuación, ampliamos el estudio para incluir límites en los que L es infinito.

Definición Diremos que el límite de f en a es + ∞ , y lo representaremos por


lim f (x) = + ∞ , cuando:
x→a
∀M > 0, ∃δ> 0 tal que si 0 < x − a < δ , entonces f ( x ) > M .
Diremos que el límite de f en a es − ∞ , y lo representaremos por lim f (x) = − ∞ , cuando:
x→a
∀M > 0, ∃δ> 0 tal que si 0 < x − a < δ , entonces f ( x ) < −M .

Ejemplos

1
Ejemplo 2.8 Probemos que lim = +∞ .
x→0 x2
Debemos demostrar que ∀M > 0, ∃δ> 0 tal que si 0< x <δ , entonces
1 1 1 1
f (x ) > M ⇔ 2
> M . Como M > 0 despejando x 2 < ⇔ x < por tanto tomando δ =
x M M M
se cumple la condición de la definición.

2-4
Límites de funciones Tema 2

Ejemplo 2.9 Estudiemos el límite de la función


π
f (x ) = tgx en x = .
2
Como es fácil de observar en la gráfica
lim + tgx = −∞ y lim − tgx = +∞ por lo tanto no
π π
x→ x→
2 2
existe lim tgx aunque sí límites laterales.
π
x→
2

Interpretación geométrica de los límites infinitos. Siempre que lim f ( x ) = ±∞ o bien


x →a +
lim f ( x ) = ±∞ se dice que la función y = f (x ) tiene una asíntota vertical de ecuación x = a .
x →a −
π
En el ejemplo anterior la función f (x ) = tgx tiene una asíntota vertical de ecuación x = .
2

2.5 Límites en el infinito


Se trata de límites en los que a es infinito.

Definición Diremos que el límite de f cuando x tiende a + ∞ es L , y lo representaremos


por lim f (x ) = L , cuando:
x→+∞

()
∀ ε > 0 , ∃ M > 0 tal que si x > M , entonces f x − L < ε .
Diremos que el límite de f cuando x tiende a − ∞ es L, y lo representaremos por
lim f ( x ) = − ∞ , cuando:
x→a

()
∀ ε > 0 , ∃ M > 0 tal que si x < − M , entonces f x − L < ε .

Ejemplos

1
Ejemplo 2.10 Probemos que lim = 0.
x→+∞ x
1
Debemos probar que ∀ ε > 0 , ∃ M > 0 tal que si x > M , entonces − 0 <ε es decir
x
1 1 1
<ε ⇔ <ε dado que M es positivo y por tanto x también, despejando se llega a x > luego
x x ε
1
eligiendo M = se cumple la condición de límite.
ε

Ejemplo 2.11 Estudiemos el límite de la función f (x ) = e x


cuando x tiende a −∞ .
Como se aprecia en la gráfica y se puede comprobar
fácilmente con una calculadora para valores de x negativos y
grandes en valor absoluto los valores de y se pueden hacer tan
pequeños como queramos, por ello lim e x = 0 .
x →−∞

2-5
Límites de funciones Tema 2

Interpretación geométrica de los límites en el infinito. Siempre que lim f (x ) = b o bien


x →+∞
lim f (x ) = b se dice que la recta de ecuación y = b es una asíntota horizontal de la función.
x →−∞
En el ejemplo anterior la recta de ecuación y = 0 es una asíntota horizontal para x → −∞ .

2.6 Caracterización del límite mediante sucesiones


Se podría haber dado como definición de límite de una función en un punto la
siguiente:

lim f ( x ) = L si para cualquier sucesión (an ) cuyos términos están en el dominio


x→a
de f y tal que cumple que lim an = a y an ≠ a, ∀ n ∈N , se tiene que lim f (an ) = L , (donde L
n →∞ n →∞
puede ser finito o infinito).

Esta definición parte del concepto de límite de una sucesión y en ocasiones facilita la
demostración de los teoremas relativos al límite de una función. En todo caso nos da una
nueva posibilidad de abordar determinados problemas de límites de funciones.

 1
Ejemplo 2.12 Veamos que no existe lim sen  .
x→0 x
Vamos a elegir dos sucesiones de términos generales
2 1
respectivos an = y bn = . Se tiene que
πn πn
lim an = lim bn = 0 , sin embargo se observa que
n →∞ n →∞

 1   nπ 
lim sen  = lim sen  (no existe límite) y
n →∞  an  n → ∞  2 
 1   1
lim sen  = lim sen (nπ ) = 0 luego no existe lim sen  .
n →∞  bn  n →∞ x→0 x

2.7 Propiedades algebraicas de los límites


Sean f y g dos funciones tales que existen los límites lim f ( x ) = L 1 y
x→a
lim g ( x) = L 2 . Entonces se cumple:
x→a

1. lim (f ( x ) ± g (x )) = L 1± L 2 .
x →a
2. lim (f ( x )⋅g (x )) = L 1⋅ L 2 .
x →a
f (x ) L 1
3. Si L 2 ≠ 0 , entonces lim = .
x → a g (x ) L2
(x )
4. Si L 1 > 0 , entonces lim (f (x ))g = L 1L 2 .
x →a

2-6
Límites de funciones Tema 2

Nota Cuando se opera con los límites, a igual que en el caso de las sucesiones (incluso
cuando los límites involucrados sean infinitos), pueden aparecer las siguientes
indeterminaciones:
0 ∞ ∞ 0
∞ −∞ , ∞⋅0, , , 1 , 0 , ∞0 .
0 ∞

2.8 Límite de la función compuesta


Por la finura de su condiciones no expondremos la proposición que rige el límite de la
función compuesta. Nos limitaremos a afirmar sin demostrarlo que con las funciones que
vamos a trabajar siempre se cumple que:

si lim f (x) = b y lim g(x) = L , entonces lim (g o f )( x ) = lim g ( x )= L .


x→a x→b x →a x →b

Ejemplo 2.13 lim e senx = e 0 = 1 , lim ln(tgx ) = −∞ .


x →π x →0 +

2.9 Cálculo de límites


- Para las funciones elementales, esto es, para las afines, polinómicas, raíces de índice
natural, exponenciales, logarítmicas y trigonométricas, en su dominio, siempre se cumple que:
lim f (x ) = f (a ) .
x →a

Ejemplos

( )
Ejemplo 2.14 lim x 3 − 2 x + 2 = 13 − 2 ⋅ 1 + 2 = 1 .
x →1
3 3
Ejemplo 2.15 lim x 2 − 12 = (− 2)2 − 12 = 3 − 8 = −2
x → −2
4 4 4
Ejemplo 2.16 lim 5 x 3 + 2 = 5 ⋅ (− 1) + 2 = − 3 , que no existe.
3
x → −1

Ejemplo 2.17 lim e x = e 0 = 1 .


x →0
Ejemplo 2.18 limln(x ) = ln(1) = 0 .
x →1

π 
Ejemplo 2.19 lim tg (x ) = tg   , que no existe (aunque sí existen límites laterales).
x→
π 2
2

- Para las funciones racionales (cociente de polinomios), al sustituir la x por el valor de


a, es posible que resulte un número real, un infinito o que aparezca la indeterminación
0
.
0

Ejemplos

x4 − 1 04 − 1 −1
Ejemplo 2.20 lim = = .
x →0 x 2 − 2x + 3 2
0 − 2⋅0 + 3 3

2-7
Límites de funciones Tema 2

x2 − 4 22 − 4 0
Ejemplo 2.21 lim = = =0 .
x →2 x3 + x − 2 3
2 +2−2 8
x 2
12  1
Ejemplo 2.22 lim 3 = 3 =   , que no existe porque
x→1 x − 1 1 − 1  0 

x2 x2
lim = −∞ y lim = +∞ .
x→1− x3 − 1 x→1+ x3 − 1

x 2 − 5x + 6  0
22 − 5 ⋅ 2 + 6
Ejemplo 2.23 lim 3
= =   . Como resulta una indeterminación, hay
3
x→2 x −8 2 −8  0
que procurar deshacerla. Para ello factorizamos el numerador y el denominador, simplificando
los factores iguales y volviendo a sustituir la x por el valor de a :

lim
x 2 − 5x + 6  0 
=   = lim
(x − 2) ⋅ (x − 3) = lim x − 3 = 2 − 3 = − 1 .
x→2 3
x −8 2
( 2
)
 0  x → 2 (x − 2) ⋅ x + 2 x + 4 x → 2 x + 2 x + 4 2 + 2 ⋅ 2 + 4 12
2

- Para funciones definidas a trozos véase si f está definida de igual o distinta forma a
izquierda y derecha de a. En el primer caso, el límite se calcula como hemos visto
anteriormente. En el segundo caso, hay que calcular los límites laterales.

 x 2 − x si x < 2
Ejemplo 2.24 Calculemos los límites en 2 y en 3 de f (x ) =  . Como
 2 x si x ≥ 2

x →2− x →2
( )
lim f (x ) = lim− x 2 − x = 2 y lim+ f (x ) = lim+ 2 x = 4 , no existe límite en 2. Por otro lado, como 3
x→2 x→2
está a la derecha de 2, se tiene lim f (x ) = lim 2 x = 6 .
x →3 x →3

- En ocasiones, ante cualquier indeterminación, simplemente hay que operar.

Ejemplos

 3 2  3 ⋅ (x + 1) − 2 3x + 1  4 
Ejemplo 2.25 lim  − 2  = (∞ − ∞ ) = lim 2
= lim 2 =   , que no existe
x →1 x − 1 x − 1 x →1 x −1 x →1 x − 1 0
3x + 1  4  3x + 1  4 
porque lim− 2 =  −  = −∞ y lim+ 2 =  +  = +∞ .
x→1 x −1 0  x→1 x −1 0 

Ejemplo 2.26 lim


(1 + x )2 − (1 − x )2 0
=   = lim
4x
=4 .
x →0 x  0  x →0 x

- El producto de una función que tiende a cero por otra acotada es siempre cero.

 1
Ejemplo 2.27 lim x ⋅sen   = 0 .
x→0 x

- En los límites de funciones irracionales algebraicas, la indeterminación se deshace


algunas veces realizando un cambio de variable o multiplicando y dividiendo por el conjugado.

Ejemplos

2-8
Límites de funciones Tema 2

Ejemplo 2.28

lim
x − 2 0
=   = lim
( x− 2 )( x+ 2 ) = lim x−2 1 1
x →2 x−2  0  x →2 (x − 2)( x+ 2 ) x →2 (x − 2)( x+ 2 ) = lim x →2 x+ 2
=
2 2

Este método es eficaz siempre que aparezcan raíces cuadradas.

Ejemplo 2.29
 x2 − 9 − x  ⋅  x2 − 9 + x 
   
lim  x − 9 − x  = ∞ − ∞ = lim     = lim
 2  −9
=0
x → +∞  x → +∞  x2 − 9 + x  x → +∞  2 
   x − 9 + x 
   
Ejemplo 2.30

lim
1+ x − 1
(
= 1 + x = y 6 = lim ) y3 −1
=
0
  = lim
( )
(y − 1) y 2 + y + 1 = lim y 2 + y + 1 = 3 .
x →0 3 1+ x − 1 y →1 y 2 − 1  0  y →1 (y − 1)(y + 1) y →1 y +1 2
6
Hemos realizado el cambio de variable 1+ x = y . Como la variable x tendía a 0, ahora
6
la variable y tiende a 1, porque 1+ 0 = y , de donde y = 1 .

En general, este cambio de variable se realiza siempre que los radicales que
aparezcan tengan el mismo radicando. El cambio consiste en cambiar éste por una nueva
variable elevada al mínimo común múltiplo de los índices de dichos radicales.

Ejemplo 2.31 Calculemos lim  x − x 3 − 1  .


3
x → +∞ 

Si llamamos y = 3 x 3 − 1 y consideramos que x 3 − y 3 = (x − y ) x 2 + xy + y 2 , de ahí se tiene ( )


que x − y =
3
x −y 3
y x − 3 x3 − 1 =
3
(
x − x −1 3
) =
1
,
2 2 2
x + xy + y
x + x x − 1 +  3 x 3 − 1 
3 3
x + x x − 1 +  3 x 3 − 1 
3 3
   
1
en consecuencia lim  x − x 3 − 1  = lim
3
=0
x → +∞  x → +∞ 2
x + x 3 x 3 − 1 +  3 x 3 − 1 
 

- En las indeterminaciones del tipo 1∞ se actúa del mismo modo que en el caso de las
sucesiones.

Ejemplo 2.32
x −1 2 x −3
⋅ ⋅
2 2   x −3 x − 3 x −1
 1 
2 x −3
 2x − 4  x −3  2x − 4  x −3 lim ⋅
lim+   = lim+ 1 + − 1 = lim+  1 +  =e x →3 + x − 3 x −1
=e
x →3  x − 1  x →3  x −1  x →3  x −1 
 
 x −3 

2.10 Cálculo de límites mediante infinitésimos equivalentes

Definición El par f , a se dice infinitésimo si, y sólo si, lim f (x ) = 0


x→a

2-9
Límites de funciones Tema 2

Ejemplos

π
Ejemplo 2.33 Como lim cos(3 x ) = 0 , el par cos(3 x ), es un infinitésimo.
x→
π 2
2
Ejemplo 2.34 También son infinitésimos x,0 , x − 3,3 , senx,0 , …
Unos infinitésimos tienden a cero más rápidamente que otros como puedes comprobar
en la siguiente tabla:

x 2 1 0’5 0’1 0’01 0’001


x2 4 1 0’25 0’01 0’0001 0’000001
senx 0’909... 0’84... 0’479... 0’0998... 0’0099998.. 9'99999 ⋅ 10 −4
1− cos x 1’41.. 0’459.. 0’122... 0’00499... 4'9999 ⋅ 10 −5 5 ⋅ 10 −7

Para comparar la fuerza con la que dos infinitésimos se acercan a cero se observa qué
ocurre con el límite del cociente, de este análisis vamos a descubrir un método de cálculo de
límites complicados que funciona en algunos casos concretos sustituir un infinitésimo por otro
equivalente.

Definiciones Dados dos infinitésimos f ,a y g, a , se dice que


f (x )
- Son del mismo orden si, y sólo si, lim = L , con L un número finito distinto de
x → a g (x )

cero.
f (x ) f (x )
- Son de distinto orden si, y sólo si, lim =∞ o lim = 0 . En el primer caso
x →a g (x ) x →a g (x )
diremos que g, a es de orden superior a f ,a (g tiende a cero más rápidamente que
f). En el segundo caso diremos que f ,a es de orden superior que g, a (f tiende a
cero más rápidamente que g) .
f (x ) g (x )
- Son equivalentes si, y sólo si, lim = 1 o lim =1 .
x → a g (x ) x → a f (x )

Ejemplos

Ejemplo 2.35 Sean las funciones f (x ) = x 2 + x − 6 y g (x ) = x 2 − 4 , que evidentemente son


f (x ) x2 + x − 6  0  2x + 1 5
infinitésimos en 2. Como lim = lim =   = lim = , los infinitésimos dados
x → 2 g (x ) x →2 x 2 − 4  
0 H x → 2 2x 4
son del mismo orden.

x2  0 
Ejemplo 2.36 Como lim =   = lim x = 0 , el infinitésimo x 2,0 es de mayor orden que el
x →0 x  0  x →0
infinitésimo x ,0 .

senx  0  H cos x 1
Ejemplo 2.37 Como lim 2
=   = lim = = +∞ , el infinitésimo x 2,0 es de mayor
x →0 x  0  x → 0 2x 0
orden que el infinitésimo senx, 0 .

2-10
Límites de funciones Tema 2

senx  0  cos x
Ejemplo 2.38 Como lim =   = lim = 1 , los infinitésimos senx, 0 y x ,0 son
x →0 x 0 H x → 0 1
equivalentes.

Proposición Sean f ,a y g, a dos infinitésimos equivalentes. Entonces, para cualquier


función h definida en un entorno reducido de a, se cumple:

- lim (f (x ) ⋅ h (x )) = lim (g (x ) ⋅ h (x ))
x →a x →a
f (x ) g (x )
- lim = lim
x →a h (x ) x → a h (x )

h (x ) h (x )
- lim = lim
x →a f (x ) x → a g (x )

siempre que alguno de los límites involucrados en cada una de las igualdades exista.

Nota La anterior proposición viene a decir que en el cálculo de límites se pueden sustituir
infinitésimos por otros equivalentes siempre que multipliquen o dividan a toda la expresión,
nunca cuando sumen o resten.

Algunos infinitésimos equivalentes, cuando x → 0 , son:

( )
x , senx , tg x , arcsenx , arctgx , ln x + 1 , e − 1
x

k x
x2 (
kx , 1+ x ) −1 x lna , a − 1
, 1− cos x
2

Ejemplos

3 x ⋅ senx  0  i .e. 3x ⋅ x i .e. 3x ⋅ x


Ejemplo 2.39 lim =   = lim = lim =3 .
x →0 2 − 2 cos x  0  x → 0 2 ⋅ (1 − cos x ) x → 0 x2
2⋅
2
senx  1 
− senx senx ⋅  − 1
tgx − senx  0   cos x =
Ejemplo 2.40 lim =   = lim cos x 3 = lim
x →0 x3  
0 x → 0 x x → 0 x 3

 1 − cos x  x2
senx ⋅   x⋅
= lim  cos x  = lim senx ⋅ (1 − cos x ) = lim 2 = 1 .
3 3 3
x →0 x x → 0 x ⋅ cos x i .e. x → 0 x 2

2-11
Límites de funciones Tema 2

Ejercicios resueltos

Concepto de límite

5x − 1
R.1. a)Demuestra que lim =3.
3x →2
b) Encuentra el entorno del punto 2 para el que los valores de las imágenes
pertenezcan al entorno de centro 3 y de radio 0’01.

Solución:

a) Dado un ε > 0 cualquiera, debemos encontrar un δ > 0 que dependerá de cada ε tal
que para todos los x que verifiquen que x − 2 < δ sus imágenes cumplan f (x ) − 3 < ε ,
5x − 1 5 x − 10 5 x − 10
es decir: −3 <ε ⇔ < ε de donde − ε < < ε y operando
3 3 3
−3ε 3ε 3ε
− 3ε < 5 x − 10 < 3ε ⇔ < x−2< , que equivale a afirmar que x − 2 < , así que
5 5 5

tomando un δ = se cumple la condición de la definición de límite.
5

b) Sólo se trata de elegir ε = 0'01 con lo que δ = = 0'006 con lo que el entorno pedido
5
es x − 2 < 0'006 ⇔ x ∈ (2 − 0'006,2 + 0'006) ⇔ x ∈ (1'994,2'006) .

2x + 1
R.2. a) Demuestra que lim =2.
x → +∞ x + 2
b) Encuentra los valores de x para los que los valores de las imágenes pertenecen al
entorno de centro 2 y de radio 0’01.

Solución:

a) Dado un ε > 0 cualquiera, debemos encontrar un M ∈ R real de manera que ∀x > M se


2x + 1 −3
cumpla que f (x ) − 2 < ε ⇔ −2 < ε ⇔ < ε como x > M no se pierde rigor al
x +2 x +2
−3 3 3 − 2ε
considerar x + 2 > 0 con lo que <ε⇔ <ε⇔x > por lo tanto si se
x +2 x+2 ε
3 − 2ε
toma M = se cumple la condición de la definición.
ε
3 − 0' 02
b) Debemos considerar ε = 0'01 con lo que M = = 298 por tanto todos los
0' 01
números reales x > 298 tienen sus imágenes en el entorno pedido.

3
R.3. a) Demuestra que lim = +∞ .
+
x →2 (x − 2)3
b) Encuentra los valores de x ∈ (2, x0 ) para los que f (x ) > 3000 .

Solución:

3
a) Se trata de probar que ∀H ∈ R ∃δ > 0 : si x − 2 < δ y x > 2 ⇒ > H no hay
(x − 2)3
problema en considerar H > 0 y como también x − 2 > 0 se tiene

2-12
Límites de funciones Tema 2

3 3 3
> H ⇔ H (x − 2) < 3 ⇔ x − 2 < 3
3
, así que eligiendo δ = 3 aquellos x que cumplan
(x − 2) 3 H H
3
x − 2 < δ y x > 2 harán que >H.
(x − 2)3
3
b) Se considera H = 3000 con lo que δ = 3 = 0'1 por tanto los valores de x pedidos
3000
son los x ∈ (2,2'1) .


Indeterminación del tipo

R.4. Calcula los siguientes límites :

3
x2 + 3 x 2 + 2x − 3 x 2 + 2x − 3 x + 33 x
a) lim b) lim c) lim d) lim
x → +∞ x+2 x → +∞ x−2 x → −∞ x−2 x → +∞ 2 x + 53 x

Solución:

3
x2 + 3 ∞
a) lim = [indet.] dividimos numerador y denominador por x
x → +∞ x+2 ∞
3
1 3
3 +
x2 + 3 x x3 0
lim = lim = =0.
x → +∞ x+2 x → +∞ 2 1
1+
x
2 3
1+ −
x 2 + 2x − 3 ∞ x x2
b) lim = [indet.] = lim = 1.
x → +∞ x−2 ∞ x → +∞ 2
1−
x
x 2 + 2x − 3 ∞
c) lim = [indet.] como x → −∞ estamos considerando x < 0 y hay que actuar
x → −∞ x−2 ∞
con cautela puesto que un número negativo no puede introducirse dentro de una raíz cuadrada,
así que hay que dividir numerador y denominador por –x que es positivo y que pasará como
2 3
2 1+ −
x + 2x − 3 x x2
(− x )2 = x 2 con lo que tenemos: lim = lim
2
= −1 .
x → +∞ x−2 x → +∞
− 1+
x
3
1+
x + 33 x
∞ 6
x = 1 (hemos dividido por
d) lim = [indet.] = lim x ).
x → +∞ 2 x + 53 x ∞ x → +∞ 5 2
2+
6
x

Indeterminación del tipo ∞ − ∞

R.5. Calcula los siguientes límites :

 1 2 
a) lim  x 2 + 3 − 3 x 2 − 7  b) lim  x − x 2 + 3 x  c) lim+  − 2 
x → +∞  x → +∞  x →3  x − 3 x −9

d) lim  x + 3 x 2 − 2 x 
x → −∞ 

Solución:

2-13
Límites de funciones Tema 2

 x 2 + 3 − 3 x 2 − 7  x 2 + 3 + 3 x 2 − 7 
  
a) lim  x + 3 − 3 x − 7  = ∞ − ∞ [indet.] = lim 
 2 2   =
x → +∞  x → +∞  x 2 + 3 + 3x 2 − 7 
 
 
10
= lim
(
x 2 + 3 − 3x 2 − 7
= lim
) − 2 x 2 + 10 ∞
= [indet.] = lim
− 2x +
x = −∞
x → +∞ 2
x + 3 + 3x − 7 2 x → +∞ 2
x + 3 + 3x − 7 2 ∞ x → +∞ 3 7
1+ 2 + 3 − 2
x x

 x − x 2 + 3 x  x + x 2 + 3 x 
  
b) lim  x − x + 3 x  = ∞ − ∞ [indet.] = lim 
 2   =
x → +∞  x → +∞ 2
x + x + 3x
−3 x ∞ −3 3
= lim = [indet.] = lim =− .
x → +∞
x + x 2 + 3x ∞ x → +∞ 3 2
1+ 1+
x
 1 2  x+3−2 x +1
c) lim+  − 2  = ∞ − ∞ [indet.] = lim+ 2 = lim+ 2 = +∞ .
x →3  x − 3 x −9 x →3 x −9 x →3 x − 9

 x + 3 x 2 − 2 x  x − 3 x 2 − 2 x 
  
d) lim  x + 3 x − 2 x  = −∞ + ∞ [indet.] = lim 
 2   =
x → −∞  x → −∞  x − 3x 2 − 2x 
 
 
− 2x 2 + 2x −∞ 2x − 2
= lim = [indet.] = lim = +∞ (dado que x < 0 hemos dividido
x → −∞ 2
x − 3 x − 2x −∞ x → −∞ 2
− 1− 3 −
x
numerador y denominador por –x que pasa dentro de la raíz como x 2 ).

0
Indeterminación del tipo
0

R.6. Calcula los siguientes límites :

x 3 − 3 x 2 − 9 x + 27 x 3 − 3 x 2 − 9 x + 27 x+4 −2
a) lim 3 2
b) lim 2
c) lim
x →3 x − 5x + 3x + 9 x → −3 x + 6x + 9 x →0 2 x + 16 − 4
3
x +1− x +1
d) lim
x →0 x +1− 6 x +1

Solución:

x 3 − 3 x 2 − 9 x + 27 0
a) lim 3 2
= [indet.] factorizamos el numerador y denominador.
x →3 x − 5x + 3x + 9 0

lim
x 3 − 3 x 2 − 9 x + 27
= lim
(x + 3)(x − 3)2 = lim (x + 3) = 3 .
x →3 3 2
x − 5x + 3x + 9 x → 3 (x − 3)2 (x + 1) x → 3 (x + 1) 2

2-14
Límites de funciones Tema 2

b) lim
x 3 − 3 x 2 − 9 x + 27
=
0
[indet.] = xlim (x + 3)(x − 3) = lim (x − 3) = 36 no existe límite
2 2

x → −3 x 2 + 6x + 9 0 → −3 (x + 3)2 x → −3 (x + 3 ) 0

aunque sí que hay límites laterales lim +


x 3 − 3 x 2 − 9 x + 27
= lim +
(x − 3)2 = +∞ y
x → −3 x 2 + 6x + 9 x → −3 (x + 3)
lim −
x 3 − 3 x 2 − 9 x + 27
= lim −
(x − 3)2 = −∞ .
x → −3 x 2 + 6x + 9 x → −3 (x + 3)

x+4 −2 0
c) lim = [indet.] multiplicamos numerador y denominador por el conjugado de
x →0 2 x + 16 − 4 0

uno y de otro. lim


x+4 −2
= lim
(
)( x + 4 + 2)( 2x + 16 + 4) =
x+4 −2
2 x + 16 − 4
x →0 ( 2x + 16 − 4)( 2x + 16 + 4)( x + 4 + 2)
x →0

= lim
(x + 4 − 4)( 2x + 16 + 4) = lim x ( 2x + 16 + 4) = lim ( 2x + 16 + 4) = 1.
(2 x + 16 − 16)( x + 4 + 2)
x →0 2 x ( x + 4 + 2) 2( x + 4 + 2)
x →0 x →0

x +1− 3 x +1 0
d) lim
6
= [indet.] hacemos el cambio de variable 6
x + 1 = t ⇔ x + 1 = t 6 con lo
x →0 x +1− x +1 0
x +1− 3 x +1 t3 − t2 t 2 (t − 1) t 6
x +1
que se tiene = = = = y por tanto
x +1− x +1 6 3
t −t t (t + 1)(t − 1) t + 1 6
x +1+1
x +1− 3 x +1 6
x +1 1
lim = lim = .
6 x →0 6 2
x →0 x +1− x +1 x +1+1

Indeterminación del tipo 1∞

R.7. Calcula los siguientes límites :

x2 +2 x 2 +2 1
 3x 3 − 2x 2   3 x 3 − 2x 2   2x + b  x − b
a) lim   b) lim   c) lim  
x → −∞ 3 x 3 − 1  x → +∞ 3 x 3 − 1  x → b 2x − b 
   

Solución:

x 2 +2
 3 x 3 − 2x 2  g (x )
a) lim   = 1∞ [indet.] . Aplicamos la fórmula lim [f (x )] = 1∞ = lim e [f (x )−1]⋅ g (x ) y
x → −∞ 3 x 3 − 1  x →a x →a
 
 3x 3 − 2x 2  2  −2 x 2 + 1  2  −2 x 4 − 3 x 2 + 2 
 3 x 3 − 2x 2 
x 2 +2 lim 
x → −∞  3
−1 x + 2

( ) lim 
x → −∞  3
 x +2

( ) lim 
x → −∞  3 x 3 −1


lim   =e  3 x −1 
=e  3 x −1 
=e  
= e + ∞ = +∞ .
x → −∞ 3 x 3 − 1 
 

b) Todos los pasos del apartado a) son iguales, pero al llegar al final se tiene que:
x2 + 2  −2 x 4 − 3 x 2 + 2 
lim  
 3x 3 − 2x 2  x → +∞  3 x3 −1 
lim   =e  
= e−∞ = 0 .
x → +∞  3 x 3 − 1 
 

1
 2x + b  x − b
c) lim   Si b = 0 el límite es trivial pues se trata de la función f (x ) = 1 ∀x ≠ 0 y su
x → b 2x − b 

límite cuando x → 0 es 1.

2-15
Límites de funciones Tema 2

1  2x + b  1  2b  1
lim  −1 lim  
 2x + b  x − b
= 1∞ [indet.] = e  2 x − b  x − b = e  2 x − b  x − b =
x →b x →b
Nos centramos en el caso b ≠ 0 lim  
x → b 2x − b 

2 2
lim lim
x −b
= e + ∞ = +∞ y
+
= e x→b x − b no existe límite, sin embargo sí hay límites laterales pues e x →b
2
lim
x −b
= e−∞ = 0 .

e x →b

Infinitésimos equivalentes

R.8. Calcula utilizando la equivalencia de infinitésimos los siguientes límites :

x ⋅ arcsenx 1 − cos ax ax − 1
a) lim b) lim c) lim
x →0 senx ⋅ cos x 2 x → 0 x ⋅ (2 − x )tgbx x →0 1+ x − 1

senx − senx ⋅ cos x


d) lim
x →0 3x 3

Solución:

x ⋅ arcsenx 0 i .e x⋅x x
a) lim = [indet.] = xlim = lim =0.
x →0 senx ⋅ cos 2 x 0 → 0 x ⋅ cos 2 x x → 0 cos 2 x

(ax )2
1 − cos ax 0 i .e
2 a2 a2
b) lim = [indet.] = lim = lim = .
x → 0 x ⋅ (2 − x )tgbx 0 x → 0 x ⋅ (2 − x ) ⋅ bx x → 0 2 ⋅ (2 − x )b 4b
ax − 1 0 i .e x ln a
c) lim = [indet.] = xlim
→0 x
= 2 ln a .
x →0 1+ x − 1 0
2
x2
x⋅
senx − senx ⋅ cos x 0 senx ⋅ (1 − cos x ) i .e 2 = 1.
d) lim 3
= [indet.] = lim 3
= lim
x →0 3x 0 x →0 3x x →0 3 x 3 6

2-16
Límites de funciones Tema 2

Ejercicios propuestos

P.1. Calcula los límites siguientes:

3x 3 + 2x − 1 4x 2 − 5x  2x 2 − 3x  x 3 + 2x − 1
a ) lim b ) lim c ) lim  − 2 x  d ) lim
3 2 2  2x 5 − x 2 + 5
x → +∞ 3x − x x → −∞ x( x + 1) x → +∞
 x−4  x → −∞

4x tgx  2 2-3x 
e) lim f ) lim g ) lim  x - 2 x  h) lim x ⋅ L( x + 1)
x →0 − 7x 2 − 6 x x→
π x x →0
 x − 4  x →0
2

P.2. Estudia la existencia de los siguientes límites y calcula su valor, o los valores de los límites
laterales correspondientes, cuando existan.

x
x2 + x − 6 x x x x − x + x −1
a) lim 2
b) lim 2
c ) lim e d ) lim
x→2 x −4 x →0 x +x x →0 x →1 x −1

x 2 − 3 x + 2 − 2x − 4 2x 2 − 3 x 3− 5+x
e) lim f ) lim g ) lim
x →3
x 3 − 6x 2 + 9 x x → +∞ 2x − 1 x→4 1− 5 − x

P.3. Calcula:

3
x -8 x −1
a) lim b) lim  x + 3 1 − x 3  c) lim d ) lim  4 x 4 + 1 − x 
x → 64 3 x -4 x → +∞   x →1 x −1 x → +∞  

3
1+ x − 4 1+ x x 2 − 23 x + 1
e) lim f ) lim
x →0 3 1+ x − 5 1+ x x →1 (x − 1)2
P.4. Calcula:

 2x 2 − 3x  1 + 2x − 3  2 1  x 3 + 2x + 1
a) lim  − 2x  b ) lim c ) lim  2 −  d) lim
x → −∞   x →1  x − 1 x − 1 
 x−4  x→4 x −2 x → +∞ 2 x x + 2 x + 3

P.5. Calcula:

1 1
x 2 +3
 x +1  2x + 1  x − 7  x + 2  (x − 3 )5
a) lim   b) lim   c) lim  
x → +∞ x − 2  x → 7 x + 8  x → 3 3 x − 5 

P.6. Calcula utilizando infinitésimos equivalentes:

x ⋅ ex − x x − x cos x 3x − 1
a) lim b) lim c) lim
x → 0 1 − cos x x →0 (arctgx )3 x →0 2x − 1

2-17
Límites de funciones Tema 2

Soluciones

P.1. a) 1 (se divide numerador y denominador por x 3 ). b) 0 (se divide numerador y


4
denominador por x 2 ). c) 5 (se opera). d) 0. e) − (se simplifica el factor x). f) no existe
6
tgx tgx
lim + = −∞ y lim − = +∞ . g) 0 (sin indeterminación). h) 0 (sin indeterminación).
π x π x
x→ x→
2 2

5
P.2. a) ( simplificar por x-2). b) no existe (1 con x → 0 + y –1 con x → 0 − ). c) no existe (e si
4
1
x → 0+ y si x → 0 − ). d) 1 (factorizar y multiplicar y dividir por x + 1 , o bien hacer x = t 2 ).
e
1 1
e) no existe ( si x → 3 + y − si x → 3 − multiplicar y dividir por el conjugado del
2 6 2 6
2
numerador y sacar factor del denominador x − 3 en x → 3 + y 3 − x en x → 3 − ). f) (dividir
2
1
por x). g) − ( multiplicar el numerador y denominador por los conjugados de ambos).
3

3
6 x + 26 x + 4 - 8
P.3. a) 3 (hacer el cambio x = t y simplificando queda lim ). b) 0 (hacer
6
x → 64 x +2
x3 + y 3
y = 3 1− x3 (
y considerar que x 3 + y 3 = (x + y ) x 2 − xy + y 2 ⇒ x + y = ) x − xy + y 2
2
). c)
1
3
(

cambio y = 3 x y simplificar). d) 0 (multiplicar y dividir por el conjugado dos veces sucesivas).

e)
15
( Cambio t = 60 1 + x al simplificar queda lim
t 3 t 14 + t 13 + L + t + 1 (
). f)
1
( cambio
)
8 7 6 9
t →1 t + t +L+ t +1
y = 3 x y simplificar).

4
P.4. a) 5 (se opera). b)
( multiplicar el numerador y denominador por los conjugados de
3
1 1
ambos). c) − (se opera y se simplifica). d) (dividir numerador y denominador por x 3 ).
2 2

 3x 2 +9 
lim  
g (x ) x → +∞  x − 2 
P.5. a) +∞ si se aplica lim [f (x )] = 1∞ = lim e [f (x )−1]⋅ g (x ) se llega a e   = +∞ b) igual
x →a x →a
 x −7  1 1
lim  
x →7 x +8  x −7
y se llega a e = e 15 c) no existe ( +∞ si x → 3 + y 0 si x → 3 − ).

x2 1 x2 ln 3
P.6. a) 2 ( e x − 1 ⇒ x y 1 − cos x ⇒ ) b) ( 1 − cos x ⇒ y arctgx ⇒ x ) c)
2 2 2 ln 2
( a x − 1 ⇒ x ln a ).

2-18
Continuidad de funciones Tema 3

Tema 3
Continuidad de funciones
Empezando con la definición rigurosa de continuidad de una función
en un punto, introducimos una clasificación de discontinuidades que permiten
el estudio de la continuidad de funciones reales de variable real. Acaba el
tema con la exposición de algunos teoremas de continuidad.

3.1 Definición de función continua en un punto

Sea la función f : A → R , donde A ⊆ R , y un punto a ∈A .

Definición Diremos que f es continua en el punto a , cuando lim f ( x ) = f (a ) . Esto es,


x →a

cuando ∀ ε > 0 , ∃ δ > 0 tal que si x − a < δ , entonces f (x ) − f (a ) <ε .

Nota Implícitamente, esta definición afirma que para que f sea continua en a, se deben cumplir
tres cosas:
• Que exista f (a ) .
• Que exista lim f (x) .
x→a

• Que f (a ) y lim f (x) coincidan.


x→a

Ejemplos

x2 −1

Ejemplo 3.1 La función f (x ) =  x + 1 ,si x ≠ −1 no es continua en x = -1, porque f (− 1) = 2 ,
 2,si x = −1

x 2 − 1  0 2x
lim =   = lim = −2 , pero no coinciden.
x→−1 x + 1  0  H x→ −1 1

x2 −1

Ejemplo 3.2 La función f (x ) =  x + 1 ,si x ≠ −1 sí es continua en x = -1, porque f (− 1) = −2 ,
 − 2,si x = −1

x 2 − 1  0 2x
lim =   = lim = −2 , y coinciden.
x→−1 x + 1  0  H x→ −1 1

3-1
Continuidad de funciones Tema 3

3.2 Definición de función continua en un subconjunto de R

Definición Diremos que f es continua en un intervalo abierto J ⊆ A , cuando lo sea en cada


uno de los puntos de J.

x2 −1
Ejemplo 3.3 La función f (x ) =
x +1
es continua en el intervalo (0, 2 ) , porque para

x 2 − 1 a2 − 1
cualquier punto a de él se cumple que lim = = f (a ) .
x →a x +1 a +1

Nota la definición anterior también puede extenderse a subconjuntos de R que contienen


x2 −1
entornos de cada uno de sus puntos. Así, diremos que f (x ) = es continua en R − {− 1} .
x +1

Definición Diremos que f es continua en un intervalo cerrado [a,b ]⊆ A cuando


• f es continua en (a,b ) ,
• lim f ( x ) = f (a ) ,
x →a +

• lim f ( x ) = f (b ) .
x →b −

Ejemplo 3.4 La función f (x ) = ln (x ) es continua en el intervalo [1,2 ] , también lo es en el


intervalo (0,2 ) , pero no lo es en el [0,2 ] , porque no existe f (0 ) .

3.3 Tipos de discontinuidades

Si una función f , definida en un entorno reducido de un punto b, no verifica alguna de


las tres condiciones de la continuidad, diremos que f es discontinua en b . En este caso, para
clasificarla, utilizaremos la siguiente terminología:

Definición Diremos que f tiene una discontinuidad evitable en el punto b cuando existe
lim f ( x ) , pero no existe f (b ) o el límite no coincide con f (b ) .
x→b

Nota Se dice evitable porque, definiendo f (b ) = lim f ( x ) , siempre se puede definir una nueva
x→b

función que extiende a la anterior y que sea continua también en b . Esto es, la función
 f (x ),si x ≠ b
g (x ) =  que coincide con la anterior, salvo en b, ya es continua en b .
xlim f (x ),si x = b
→b

3-2
Continuidad de funciones Tema 3

 1
Ejemplo 3.5 La función f (x ) = x ⋅ sen  tiene una discontinuidad evitable en x = 0, porque
x
 1
existe lim x ⋅ sen   = 0 , pero es distinto de f(0) que no existe. Se evita la discontinuidad
x→0 x
  1
 x ⋅ sen ,si x ≠ 0
definiendo la función g (x ) =  x .
 0,si x = 0

Definición Diremos que f tiene una discontinuidad no evitable en el punto b cuando no


existe lim f (x ) .
x→b

Distinguimos los siguientes casos:


• Que los límites laterales existan, pero no coincidan. La llamaremos discontinuidad no
evitable de salto finito.
• Que alguno o ambos de los límites laterales sean infinito. La llamaremos discontinuidad
no evitable de salto infinito.
• Que no exista algún límite lateral. La llamaremos discontinuidad no evitable de
segunda especie.

Nota A los dos primeros tipos de discontinuidad no evitable se les llama también
discontinuidades de primera especie.

Ejemplos

x−2
Ejemplo 3.6 La función f (x ) = tiene una discontinuidad de salto finito en x = 2, porque
x−2
x−2 x−2
lim+ = 1 y lim− = −1 . El salto es 1 − (− 1) = 2 .
x →2 x−2 x →2 x−2

π
Ejemplo 3.7 La función f (x ) = tgx tiene una discontinuidad de salto infinito en x = , porque
2
sus límite laterales son infinitos (uno negativo y otro positivo).

1
Ejemplo 3.8 La función f (x ) = tiene una discontinuidad de salto infinito en x = 0 , porque
x2
sus límite laterales son infinitos (ambos positivos).

 1
Ejemplo 3.9 La función f (x ) = sen   tiene una discontinuidad de segunda especie en x = 0 ,
x
porque no existen sus límites laterales (la función está acotada en un entorno del cero).

1  1
Ejemplo 3.10 La función f (x ) = sen  tiene una discontinuidad de segunda especie en
x x
x = 0 , porque no existen sus límites laterales (la función no está acotada en un entorno del
cero).

3-3
Continuidad de funciones Tema 3

0,si x∈ Q
Ejemplo 3.11 La función de Dirichlet, f (x ) =  , es discontinua de segunda especie en
 1,si x∈ I
todo R, porque no existen los límites laterales en ningún punto de R.

3.4 Continuidad de las funciones elementales

Las funciones elementales, esto es, las afines, polinómicas, raíces de índice natural,
exponenciales, logarítmicas y trigonométricas, son continuas en todos los puntos de su
dominio.

3.5 Propiedades algebraicas de las funciones continuas

Puesto que la continuidad está definida en términos de límites, no es sorprendente que


las funciones continuas y los límites tengan muchas propiedades en común.

Proposición Sean f y g funciones continuas en A, entonces:


• Las funciones f ± g , f ⋅g son continuas en A.
f
• La función es continua en cualquier punto de A en el que g no se anule.
g

Ejemplos

2x 2 + x
Ejemplo 3.12 La función f (x ) = es continua en R − {0} , ya que si llamamos
x
f1 + f2
f1(x ) = 2x 2 , f 2 (x ) = x y f 3 (x ) = x , se tiene que f = . Como las tres funciones son
f3
continuas en R, f será continua en R salvo en los puntos que anulen el denominador, esto es,
en x = 0 .

 x 3 − 3,si x ≤ 0
Ejemplo 3.13 La función f (x ) =  es continua en R − {0} , porque las funciones
 senx ,si x > 0

g (x ) = x 3 − 3 y h(x ) = senx son continuas en R y, en particular en los trozos que definen a f,

x →0 x →0
( )
pero f no es continua en x = 0 porque lim− f (x ) = lim− x 3 − 3 = −3 ≠ lim+ f (x ) = lim senx = 0 .
x →0 x →0 +

Así que en x = 0 tiene discontinuidad no evitable de salto finito, igual a 3.

senx
Ejemplo 3.14 La función f (x ) = tgx = es continua en todos los puntos que no anulen el
cos x
denominador, ya que es cociente de dos funciones continuas en R. Así pues f es discontinua
 (2k + 1)π 
de salto infinito en {x∈R:cos x = 0} =  ,k∈Z  .
 2 

3-4
Continuidad de funciones Tema 3

3.6 Continuidad de la composición de funciones

Proposición Sean A, B, C subconjuntos de R y f : A → B , g : B → C dos funciones. Si f


es continua en un punto a de A y g es continua en el punto b = f (a ) , entonces la función
compuesta h = g o f : A → C es continua en el punto a . En consecuencia,

lim g (f (x )) = g (f (a )) = g  lim f (x ) .


x →a  x →a 

3.7 Propiedades de las funciones continuas


Las siguientes proposiciones son necesarias para demostrar con rigor el Teorema de
Bolzano.

Proposición Si una función f es continua en un punto a, con f (a ) ≠ 0 , entonces existe un


entorno de a en el que la función tiene el mismo signo que f (a ) .

Proposición Toda función continua en un punto está acotada en algún entorno de dicho
punto.

3.8 Propiedades de las funciones continuas en un intervalo cerrado

Teorema de Bolzano Sea f :[a, b ]→ R una función continua en [a, b ] tal que el signo de
f (a ) es distinto al signo de f (b ) , entonces ∃c∈ (a, b ) tal que f (c ) = 0 .

Nota Con palabras, este Teorema dice que una función continua en un intervalo cerrado, con
imágenes de distinto signo en los extremos del mismo, necesariamente se anula en algún
punto interior del intervalo.

Interpretación geométrica: Todo arco de curva continua, con distinto signo en los extremos,
necesariamente corta al eje de las equis (eje OX).

Nota El teorema afirma que la corta en un punto, pero no necesariamente es único. Pueden
haber varios puntos en los que corte al eje de las equis (eje OX).

Nota Antes de aplicar la tesis del teorema hay que verificar que se cumple la hipótesis.

Demostración.- Demostraremos el teorema por reducción al absurdo.

Previamente enunciamos el Axioma de los intervalos encajados, que dice: Si se considera


una sucesión In de intervalos cerrados In = [an , bn ] en los que I1 ⊃ I 2 ⊃ I3 ⊃ ....In ⊃ ... como en
la figura

y en los que además


lim (bn − an ) = 0 , es decir que
n→∞
su longitud tiende a cero entonces existe siempre un y sólo un número real α tal que
α ∈ In ∀n , o sea que pertenece a todos ellos.

3-5
Continuidad de funciones Tema 3

En la función dada construimos una sucesión de intervalos encajados dividiendo el


 a + b  a + b 
intervalo [a, b ] cada vez por la mitad a, , , b  y quedándonos con aquel de los dos
 2   2 
a+b
en los que f (x ) cambie de signo. ( Si en la función se anulase el teorema estaría
2
demostrado).

Dicha sucesión define un número real α en el que necesariamente f (α ) = 0 .

En efecto, si consideramos que f (α ) ≠ 0 por ser la función continua en α existe un


entorno de α en el que la función tiene el mismo signo que f (α )
Por otra parte sabemos que podemos encontrar un intervalo de la sucesión de
intervalos encajados totalmente contenido en el entorno, ya que la longitud de dichos intervalos
es tan pequeña como queramos.

En los extremos de dicho intervalo la función cambiaría de signo lo cual es una


contradicción, por lo tanto f (α ) = 0 necesariamente con lo que el teorema queda demostrado.

Ejemplos

− 1,si x < 4
Ejemplo 3.15 El Teorema de Bolzano no se puede aplicar a la función f (x ) =  , en
 1,si x ≥ 4
el intervalo [2,5] , porque f no es continua en el punto x = 4 de dicho intervalo.

Ejemplo 3.16 (Aplicación del Teorema de Bolzano para demostrar que una ecuación tiene
alguna solución real)
Para demostrar que la ecuación cos x − 2x + 1 = 0 tiene alguna solución, empezaremos
definiendo la función f (x ) = cos x − 2 x + 1 , que sabemos que es continua en todo R por ser
suma de funciones continuas.
A continuación, buscamos un intervalo cerrado en el que la función tome distinto signo
π 
en los extremos. Como f (0 ) = 2 > 0 y f   = 0 − π + 1 < 0 , un intervalo en el que se cumple la
2
 π
hipótesis de Bolzano es el 0,  .
 2
Por lo tanto, podemos asegurar que en ese intervalo la función se anula y, en
consecuencia, ahí hay una raíz de la ecuación dada.

Ejemplo 3.17 (Aplicación del Teorema de Bolzano para aproximar raíces de una ecuación
dada) Siguiendo con el ejemplo anterior, del que sabemos que la ecuación tiene solución en el
 π
intervalo 0,  , veamos cómo podemos hallar una aproximación de una de sus raíces.
 2
π
Tomemos un punto intermedio entre 0 y , por ejemplo el 1, y veamos el signo de
2
f (1) . (Ojo con la calculadora, debes trabajar en radianes). f (1) = cos 1 − 2 + 1 < 0 . Como
f (0 ) = 2 > 0 , el Teorema de Bolzano asegura que:
Hay una raíz entre 0 y 1.
Tomamos otro punto intermedio del (0,1) , por ejemplo el 0,5 ,
f (0,5 ) = cos 0,5 − 2 ⋅ 0,5 + 1 = 0,87 > 0 . Como f (1) < 0 , el Teorema de Bolzano asegura que:
Hay una raíz entre 0,5 y 1.

3-6
Continuidad de funciones Tema 3

Tomamos otro punto intermedio, por ejemplo el 0,7 , f (0,7 ) = cos 0,7 − 2 ⋅ 0,7 + 1 > 0 .
Como f (1) < 0 , el Teorema de Bolzano asegura que:
Hay una raíz entre 0,7 y 1.

Tomamos otro punto intermedio, por ejemplo el 0,8, f (0,8 ) = cos 0,8 − 2 ⋅ 0,8 + 1 > 0 .
Como f (1) < 0 , el Teorema de Bolzano asegura que:
Hay una raíz entre 0,8 y 1.

Tomamos otro punto intermedio, por ejemplo el 0,9 , f (0,9 ) = cos 0,9 − 2 ⋅ 0,9 + 1 < 0 .
Como f (0,8 ) > 0 , el Teorema de Bolzano asegura que:
Hay una raíz entre 0,8 y 0,9.

Así pues, una raíz con un error menor a 0,1 de la ecuación cos x − 2x + 1 = 0 es 0,8 .

Ejemplo 3.18 Demostremos que la función polinómica f (x ) = 3 x 5 − 4 x 2 + 3 x + 2 tiene al


menos un cero y hallemos una aproximación de él hasta las centésimas.
Para probar la existencia de un cero, ya que f es continua en todo R, bastará encontrar
dos valores de x en los que la función tenga distinto signo.
Para x = 0, es evidente que f (0 ) > 0 .
Como la función tiende a menos infinito cuando x tiende a menos infinito, buscaremos
un valor negativo. Es fácil ver que f (− 1) < 0 .
Luego la función tiene una raíz entre –1 y 0.

Tomemos un punto intermedio entre -1 y 0, por ejemplo el -0,5 , y veamos el signo de


f (− 0,5 ) . Como f (− 0,5 ) < 0 , el cero está entre –0,5 y 0.

Tomemos otro punto intermedio entre –0,5 y 0, por ejemplo el –0,2. Como f (− 0,2) > 0
y f (− 0,5 ) < 0 , el cero está entre –0,5 y –0,2 .

Como f (− 0,3 ) > 0 , el cero está entre –0,5 y –0,3 .


Como f (− 0,4 ) > 0 , el cero está entre –0,5 y –0,4 .
Como f (− 0,45 ) < 0 , el cero está entre –0,45 y –0,4 .
Como f (− 0,43 ) < 0 , el cero está entre –0,43 y –0,4 .
Como f (− 0,41) > 0 , el cero está entre –0,43 y –0,41 .

Por último, como f (− 0,42 ) < 0 , el cero está entre –0,42 y –0,41; así que –0,41 es una
aproximación hasta las centésimas.

n
Ejemplo 3.19 Para demostrar que todo número positivo c tiene una raíz enésima c , basta
n
considerar que equivale a demostrar que la ecuación x − c = 0 tiene solución, pues si
n
llamamos x = c , despejando c se tiene la ecuación de arriba .

3-7
Continuidad de funciones Tema 3

Llamando f (x ) = x n − c , como es continua en todo R, buscamos dos valores en los que


tome distinto signo y aplicamos el Teorema de Bolzano.
Evidentemente, f (0 ) < 0 . Y, como (c + 1)n > c n , entonces f (c + 1) > 0 , por lo que el
Teorema de Bolzano asegura que hay una raíz entre 0 y c + 1 .

Ejemplo 3.20 Apliquemos el Teorema de Bolzano para probar que las gráficas de f (x ) = −e x y
g (x ) = x 3 se cortan en algún punto.
Consideremos la función h(x ) = g (x ) − f (x ) . Evidentemente es continua en R, por ser
resta de funciones continuas. Además, h(0 ) > 0 y h(− 1) < 0 . Por tanto, el Teorema de Bolzano
asegura que existe un punto del ]− 1,0 [ en el que h se anula.
Para aproximarlo a una décima, tomamos -0,5 . Como h(− 0,5 ) > 0 y h(− 1) < 0 , el punto
está en ]−1,−0.5 [ . Como h(− 0,7 ) > 0 y h(− 1) < 0 , está en ]−1,−0.7 [ . Como h(− 0.8 ) < 0 y
h(− 0,7 ) > 0 , está en ]− 0.8,−0.7 [ . Luego el punto es –0,7 , con una décima de aproximación.

El siguiente teorema viene a ser una generalización del de Bolzano.

Teorema de los valores intermedios Sea f :[a, b ]→ R una función continua en [a, b ] tal que
f (a ) ≠ f (b ) , entonces f toma todos los valores comprendidos entre f (a ) y f (b ) .

Nota Este teorema afirma que para cualquier valor M comprendido entre f (a ) y f (b ) , siempre
existe un c del intervalo (a, b ) tal que f (c ) = M . Con palabras, viene a decir que si f es continua
en un intervalo cerrado no se puede pasar de un valor de la variable dependiente a otro sin
pasar por los intermedios. Esto es, la función asume (al menos una vez) cualquier número que
esté entre f (a ) y f (b ) .

 π π 
Ejemplo 3.21 La función f (x ) = senx es continua en el intervalo 0,  y f (0 ) = 0 ≠ f   = 1 ,
 2  2
por lo que el teorema anterior afirma que para cualquier valor M del (0,1) existe al menos un
 π
punto c del  0,  tal que f (c ) = M .
 2

Teorema de Weierstrass. Sea f :[a, b ]→ R una función continua en [a, b ] , entonces f está
acotada en [a, b ] .

Notas

- Que una función f esté acotada en un intervalo [a, b ] significa que existen dos números
reales m y M , con m < M , tales que m ≤ f (x ) ≤ M para todo x del intervalo [a, b ] .

3-8
Continuidad de funciones Tema 3

- Basta que la hipótesis no se verifique para un punto, para que no se cumpla el Teorema. Por
1
 ,si x ≠ 0
ejemplo, la función f (x ) =  x no es continua en [−4,4] . En consecuencia no podemos
 0,si x = 0
aplicar el teorema anterior a [−4,4] . De hecho, como se ve en el dibujo, la función no está
acotada en [−4,4] .
 x 2 ,si x < 1
- El recíproco del teorema no es cierto. Por ejemplo, la función f (x ) =  está acotada
 0,si x ≥ 1
en [0,2] , pero no es continua en [0,2] . Esto es, hay funciones acotadas en un intervalo cerrado
que no son continuas en él.

x +2
- La función f (x ) = es continua en (0,2) , pero no lo es en [0,2] , por lo que no podemos
x 2 − 2x
aplicarle el teorema. De hecho, ésta función no está acotada en [0,2] .

3.9 Extremos absolutos (máximos y mínimos absolutos)

Definición Se dice que f tiene un máximo absoluto en a ∈A si f (x ) ≤ f (a ),∀x∈A .

Ejemplo 3.22 La función f (x ) = − x 2 , definida en R, tiene un máximo absoluto en x = 0 .

Definición Se dice que f tiene un mínimo absoluto en b ∈A si f (b ) ≤ f (x ),∀x∈A .

Ejemplos

Ejemplo 3.23 La función f (x ) = x , definida en R, tiene un mínimo absoluto en x = 0 .

Ejemplo 3.24 La función f (x ) = x 3 , definida en R, no tiene extremos absolutos.

Ejemplo 3.25 La función f (x ) = x 2 ⋅ e − x , definida en R, tiene mínimo absoluto en x = 0, pero


no tiene máximo absoluto, pues se va a más
infinito.

3-9
Continuidad de funciones Tema 3

Nota Al máximo o mínimo absoluto se les llama extremos absolutos.

Teorema de Bolzano – Weierstrass Sea f :[a, b ]→ R una función continua en [a, b ] ,


entonces f alcanza máximo y mínimo absolutos en [a, b ] .

Nota El teorema asegura que existen c , d pertenecientes al intervalo [a, b ] tales que
f (x ) ≤ f (c ) y f (d ) ≤ f (x ) para todo x del intervalo [a, b ] .

Ejemplos

Ejemplo 3.26 La función f (x ) = x 3 , definida en [− 1,2] , tiene su máximo absoluto en x = 2, y su


mínimo absoluto en x = -1 .

Ejemplo 3.27 La función f (x ) = x 3 − 4 x 2 + x + 6 ,


definida en R, no tiene extremos absolutos; pero
definida en [− 2,3] si que los tiene (véase la gráfica).

3-10
Continuidad de funciones Tema 3

Ejercicios resueltos

Continuidad en un punto.

 1

R.1. Estudia la continuidad en x = 0 de la función f ( x ) = e x si x ≠ 0 .
0 si x = 0
Solución:
Por definición, para que la función sea continua se debe cumplir que lim f ( x ) = f ( 0 ) ;
x →0

f (0) = 0 , para calcular el lim f ( x ) es preciso calcular los límites laterales.


x →0
1 1
lim+ f ( x ) = lim+ e x = e + ∞ = +∞ y lim− f ( x ) = lim− e x = e −∞ = 0 . Por lo tanto, en x = 0 , la
x →0 x →0 x →0 x →0
función es discontinua de salto infinito.

 x −43

R.2. Estudia la continuidad en x = 0 de la función f ( x ) = e x si x ≠ 0 .
 0 si x = 0
Solución:
Veamos si lim f ( x ) = f ( 0 ) ; f ( 0 ) = 0 , para calcular el lim f ( x ) calculamos los límites
x →0 x →0
x −3 x −3
laterales. lim− f ( x ) = lim− e x4 = e −∞ = 0 y lim+ f ( x ) = lim+ e x4 = e −∞ = 0 .
x →0 x →0 x →0 x →0

La función es continua en x = 0 .

 x

R.3. Estudia la continuidad en x = 0 de la función f ( x ) = e x si x≠0 .
0 si x=0

Solución:
− x si x<0
Lo primero es escribir la función de forma más detallada. Como x =
x si x>0
e −1 si x < 0

se tiene que f ( x ) =  0 si x = 0 . Evidentemente lim f ( x ) = lim+ e = e , mientras que
x →0 +
 e si x > 0 x →0

1
lim f ( x ) = lim− e −1 = . Por lo tanto, la función presenta una discontinuidad de salto finito.
x →0 − x →0 e


 2 x − 1 si x ≤ 0

R.4. Dada la función f ( x ) =  k si 0 < x ≤ 1 se pide.
x2 − 3
 si x > 1
 x +1
a) Calcula k para que la función sea continua en x = 0 .
b) Para ese valor de k ¿es una función continua en todo ℜ ?.

Solución:

3-11
Continuidad de funciones Tema 3

a) Para que sea continua f ( 0 ) = lim− f ( x ) = lim+ f ( x ) . f ( 0 ) = 2 ⋅ 0 − 1 = −1 .


x →0 x →0

lim f ( x ) = lim− (2 x − 1) = −1 y lim f ( x ) = lim+ k = k , así pues, para que la función sea
x →0 − x →0 x →0 + x →0

continua en x = 0 el valor de k debe ser -1.


b) Para todos los x < 0 la función es polinómica y por lo tanto continua.
Para 0 < x < 1 f ( x ) = −1 función constante que es continua.
x2 − 3
Para x > 1 f ( x ) = como en esta zona el denominador x + 1 ≠ 0 la función es también
x +1
continua. Por lo tanto el único punto posible de discontinuidad es x = 1 .
x2 − 3
Veamos si f ( 1) = lim− f ( x ) = lim+ f ( x ) ; f ( 1) = −1 , lim− f ( x ) = −1 y lim+ f ( x ) = lim+
= −1 ,
x →1 x →1 x →1 x →1 x →1 x + 1

luego la función también es continua en x = 1 y en definitiva f ( x ) es una función continua


∀x ∈ ℜ .

Continuidad de una función y tipos de discontinuidades.

R.5. Estudia la continuidad y clasifica las discontinuidades de las siguientes funciones:


 2 x − 3 si x ≤ −1  3
si x≤3
 
a) f ( x ) = − x + 2 si − 1 < x < 3 b) f ( x ) =  x − 3
1
 x2  si x >3
 − 2 si x ≥ 3  x + 3
 3
x +1
c) f ( x ) = x − 2 + x − 3 d) f ( x ) = x 2 − 1 +
x2 − 1

Solución:
a) En cada trozo de definición f (x ) es continua por tratarse de funciones polinómicas,
así que los únicos punto posibles de discontinuidad son x = −1 y x = 3 . Estudiaremos f (x ) en
ambos valores
En x = −1 : f ( −1) = 2 ⋅ ( −1) − 3 = −5 = lim − (2 x − 3 ) y lim + f ( x ) = lim + (− x + 2) = 3 luego
x → −1 x → −1 x → −1

la función presenta una discontinuidad de salto finito en x = −1 .


32  x2 
En x = 3 f (3) = − 2 = 1 = lim+  − 2  , como lim f ( x ) = lim− ( − x + 2) = −1 por
3 x →3  3  x →3 −
  x →3

tanto f (x ) presenta una discontinuidad de salto finito en x = 3 .


b) Para los valores x < 3 y para x > 3 la función es continua por ser cociente de
funciones continuas y porque el denominador en ambos casos es distinto de cero. Estudiamos
la continuidad de f (x ) en x = 3 para lo cual es preciso calcular los límites laterales.
3 1 1
No existe f (3) , lim− f ( x ) = lim− = −∞ y lim f ( x ) = lim =
x →3 x →3 x −3 x →3 + x →3 − x + 3 6
La función presenta una discontinuidad de salto infinito en x = 3 y es continua en todos
los demás valores de x, obsérvese que f (x ) es continua por la derecha en x = 3 , es decir f (x )
sería considerada continua por ejemplo en el intervalo [3,5] .
c) f (x ) será continua cuando lo sean a la vez y = x − 2 e y = x − 3 por tanto
estudiaremos cada una de ellas por separado.

3-12
Continuidad de funciones Tema 3

− x + 2 si x ≤ 2
y = x−2 = , el único punto donde puede no ser continua es x = 2 ,
x − 2 si x > 2
aplicamos la condición de continuidad, f (2 ) = 0 , lim+ f ( x ) = lim+ ( x − 2) = 0 y
x →2 x →2

lim f ( x ) = lim ( − x + 2) = 0 por tanto y = x − 2 es continua ∀x ∈ ℜ .


x →2− x →2−

y = x − 3 es continua en todo su dominio D = {x ∈ ℜ : x ≥ 3} = [3,+∞ ) .


La función f (x ) es continua ∀x ∈ [3,+∞ ) en los demás valores de ℜ presenta es
discontinua con discontinuidades de 2ª especie.
x +1
d) Al igual que en el apartado anterior f ( x ) = x 2 − 1 + será continua cuando lo
x2 − 1
x +1
sean a la vez g ( x ) = x 2 − 1 y h( x ) = cuya continuidad estudiaremos por separado.
x2 − 1
 x 2 − 1 si x ≤ −1

g ( x ) = x 2 − 1 = − x 2 + 1 si − 1 < x < 1 g (x ) es continua en todos los puntos excepto quizás
 x 2 − 1 si x ≥1

en x = −1 y x = 1. Estudiamos estos valores.
( ) (
En x = −1 : f (− 1) = (− 1) − 1 = 0 , lim − x 2 − 1 = 0 y lim + − x 2 + 1 = 0 . Continua.
2
x → −1 x → −1
)
En x = 1: f (1) = −(1) + 1 = 0 , lim (x − 1) = 0 y ( )
2 2
lim − x 2 + 1 = 0 . Continua.
x →1+ x → −1−

x +1
h( x ) = es cociente de dos funciones continuas en todo ℜ y por tanto es continua
x2 − 1
excepto cuando el denominador se anule es decir en x = −1 y x = 1 donde será discontinua.
Estudiamos el tipo de discontinuidad.
x +1 0 x +1 1 1
En x = −1 : lim = [Inde.] = lim = lim =− Discontinuidad
x → −1 x 2 − 1 0 x → −1 (x − 1)(x + 1) x → −1 x − 1 2
evitable.
x +1 2 x +1
En x = 1: lim
x →1 x 2
= [Inde.] estudiamos límites laterales, lim− = −∞ y
−1 0 x →1 x2 − 1
x +1 x +1
lim = +∞ . Discontinuidad de salto infinito. En conclusión f ( x ) = x 2 − 1 + presenta
x →1+x −1 2
x2 − 1
dos discontinuidades: En x = −1 evitable y en x = 1 de salto infinito.

2 + senx
R.6. Estudia la continuidad y clasifica las discontinuidades de f (x ) =
cos 2 x − senx
Solución:
Al ser f (x ) cociente de dos funciones continuas, los puntos de discontinuidad son las

soluciones de cos 2 x − senx = 0 . Aplicamos la fórmula del ángulo doble y ponemos cosx en

−1
función de senx con lo que queda 2sen 2 x + senx − 1 = 0 ⇒ senx =  .
1 2

Si senx = −1 ⇒ x = ± 2kπ con k ∈ N .
2

3-13
Continuidad de funciones Tema 3

 π
 x = 6 ± 2kπ
Si senx = 1 2 ⇒  con k ∈ N .

x = ± 2kπ
 6
Clasificamos las discontinuidades:
3π 2 + senx 1
En x = : lim f ( x ) = lim = [indet.] independientemente del signo
2 x→

x→
3π cos 2 x − senx 0
2 2

de los límites laterales tendremos una discontinuidad de salto infinito, igual ocurre en todos los

valores de la serie x = ± 2kπ .
2
5
π 2 + senx
En x = lim = 2 [indet.] . Al igual que en el caso anterior la función
6 x→
π cos 2 x − senx 0
6

π
presenta una discontinuidad de salto infinito en todos los valores de la serie x = ± 2kπ .
6
3
5π 2 + senx
En x = lim = 2 [indet.] . También hay discontinuidad de salto
6 5π cos 2 x − senx 0
x→
6


infinito en todos los valores de la serie x = ± 2kπ .
6

x −2
R.7. Dada la función f (x ) = se pide:
x +5 −3
a) Estudia la continuidad de f (x ) y clasifica sus discontinuidades.

b) Redefine f (x ) de modo que la nueva función sea continua en x = 4 .

Solución:
a) La función es continua en todos los puntos que pertenezcan a la vez al dominio de

y = x y de y = x + 5 excepto para x = 4 donde el denominador es cero. El dominio de

y = x es el intervalo [0,+∞ ) y el de y = x + 5 es el intervalo [− 5,+∞ ) , por tanto el dominio

de f (x ) es el intervalo [0,+∞) con x ≠ 4 . En el intervalo (− ∞,0) la función presenta

discontinuidades de segunda especie. Estudiemos qué ocurre en x = 4 .

x −2 0
lim f (x ) = lim = [ind.] multiplicamos numerador y denominador por sus conjugados
x→4 x→4 x +5 −3 0

lim f (x ) = lim
( x − 2)( x +2 )( x+5 +3 ) = lim
(x − 4)( x + 5 + 3) = 6 = 3
x→4 x→4 ( x +5 −3 )( x + 2)( x +5 +3 ) x → 4 (x − 4 )( x + 2) 4 2

La función tiene una discontinuidad evitable en x = 4

3-14
Continuidad de funciones Tema 3

 x −2
 si x≠4
b) Definimos la función g (x ) =  x + 5 − 3 que en definitiva equivale a la
3 si x=4
 2

x+5 +3
función g (x ) = .
x +2
R.8. Calcula los valores de los parámetros a y b para que las siguientes funciones sean
continuas. Dibuja las gráficas de dichas funciones para dichos valores.
 x + a si x ≤ −2 3 x − 2 si x < −1
 
a) f ( x ) =  x 2 + 1 si −2< x<3 b) g ( x ) = ax + b si −1≤ x ≤ 4
bx + 3 si x≥3  5 si x>4
 
Solución:
a) Los únicos puntos donde la función puede ser discontinua son x = −2 y x = 3 por tanto
exigimos que la función sea continua en estos puntos.
Para que sea continua en x = −2 f (− 2) = −2 + a = lim (x + a ) y
x → −2 −

( )
lim f (x ) = lim + x 2 + 1 = 5 luego a − 2 = 5 ⇒ a = 7
x → −2 + x → −2

Exigimos la continuidad en el punto x =3, f (3 ) = 3b + 3 = lim+ f (x ) y como


x →3

( )
lim− x 2 + 1 = 10 3b + 3 = 10 ⇒ b =
x →3
7
3
. La gráfica


 x + 7 si x ≤ −2

de la función f ( x ) =  x 2 + 1 si − 2 < x < 3 es:
 7 x + 3 si x≥3
 3

b) Igual que en el apartado anterior exigimos la continuidad en x = −1 y x = 4


En x = −1 , g (− 1) = lim + (ax + b ) = −a + b y como lim − (3 x − 2) = −5 debe ser
x → −1 x → −1

−a + b = −5
En x = 4, g (4 ) = lim− (ax + b ) = 4a + b , por otra parte lim g (x ) = lim+ 5 = 5 y
x→4 x→4+ x →4

− a + b = −5 
4a + b = 5 debemos resolver el sistema 
4a + b = 5 
restando la 2ª con la 1ª ⇒ 5a = 10 ⇒ a = 2 y
sustituyendo en una de las dos se obtiene b = −3 . La
función continua resultante es
3 x − 2 si x < −1

g ( x ) = 2 x − 3 si − 1 ≤ x ≤ 4 y su gráfica:
 5 si x>4

3-15
Continuidad de funciones Tema 3

R.9. Estudia en función de los valores de los parámetros la continuidad de las siguientes
funciones:

 tgx
1 + senx si x ≤π e x si x<0
a) f ( x ) =  b) f ( x ) =  2 x + a
 2ax + cos x si x >π  si x≥0
 3
 e ax + a
 x si x≤0
3 x + a si x≤0  e + 1
c) f ( x ) =  sec x d) f ( x ) =  x 2 + 1 si 0< x <1
e si x>0  x −b si x ≥1


Soluciones:

a) Las funciónes y = 1 + senx e y = 2ax + cos x son continuas independientemente del


valor de a siendo x = π el único punto posible de discontinuidad. Para este valor se tiene:
f (π ) = 1 + senπ = 1 , lim − f (x ) = lim − (1 + senx ) = 1 y lim + f (x ) = lim + (2ax + cos x ) = 2πa − 1 si
x →π x →π x →π x →π

1
hacemos 2πa − 1 = 1 ⇒ a = por lo tanto se tiene:
π
1
Si a = f (x ) es continua en todo ℜ
π
1
Si a≠ f (x ) presenta una discontinuidad de salto finito en x = π .
π

b) f (x ) en el intervalo x < 0 será discontinua donde lo sea la función y = tgx , es decir


π
en x=− − kπ con k ∈ N y esto independientemente del valor de a, además son
2
discontinuidades de salto infinito.
Para x > 0 f (x ) es continua ∀a ∈ ℜ por ser una función polinómica.
Estudiamos lo que ocurre en x = 0 .
tgx
tgx
 2x + a  a a lim x
lim+ f (x ) = lim+   = , f (0 ) = y lim f (x ) = lim e x =e x →0 −
= e ( recuérdese que x y
x →0 x →0  3  3 3 x →0 − x →0 −

a
tgx son infinitésimos equivalentes). Si igualamos = e ⇒ a = 3e y en definitiva:
3
Si a = 3e la función es continua en x = 0 .
Si a ≠ 3e la función presenta en x = 0 una discontinuidad de salto finito.
En ambos casos la función es discontinua de salto infinito en los puntos
π
x=− − kπ con x ∈ N como ya se ha dicho antes.
2

c) Para el intervalo x > 0 e independientemente del valor de a, f (x ) será discontinua


π
donde lo sea la función y = sec x , es decir cuando cos x = 0 ⇒ x = + kπ con k ∈ N . Son
2
discontinuidades de salto infinito como es fácil comprobar. En x < 0 la función es polinómica y
por tanto continua ∀a ∈ ℜ . Sólo falta ver qué pasa cuando x = 0 .

3-16
Continuidad de funciones Tema 3

1
lim cos x
f (0 ) = a , lim− f (x ) = lim− (3 x + a ) = a , y lim+ f (x ) = lim+ e sec x
=e x →0 +
= e por lo tanto:
x →0 x →0 x →0 x →0

Si a = e la función es continua en x = 0 .
Si a ≠ e la función presenta una discontinuidad de salto finito en x = 0 .
Independientemente del valor de a, la función presenta las discontinuidades expresadas con
anterioridad.

d) La función sólo puede ser discontinua en x = 0 y x = 1 .


Estudiamos la continuidad en x = 0 :
1+ a e ax + a 1 + a
f (0 ) =
2
, lim f (x ) = lim
x →0 −
x →0 e + 1
+ x
=
2 x →0 x →0
( )
y lim+ f (x ) = lim+ x 2 + 1 = 1 , por tanto para que la

1+ a
función sea continua = 1 ⇒ a = 1.
2
Estudiamos ahora la continuidad en x = 1 :
f (1) = 1 − b lim+ f (x ) = lim+ (x − b ) = 1 − b
x →1 x →1
y
x →1− x →1
(
lim f (x ) = lim− x 2 + 1 = 2 ) así que si hacemos

1 − b = 2 ⇒ b = −1 y en conclusión se tiene:
Si a = 1 y b = −1 f (x ) es continua ∀x ∈ ℜ .
Si a ≠ 1 f (x ) presenta en x = 0 una discontinuidad de salto finito.
Si b ≠ −1 f (x ) presenta en x = 1 una discontinuidad de salto finito.

Teorema de Bolzano

3  π π
R.10. La función f (x ) = cambia de signo en los extremos del intervalo − ,  ya
1 − 2senx  2 2

 π π   π π
que f  −  = 1 y f   = −3 ¿puede asegurarse que en algún punto c del intervalo  − , 
 2   
2  2 2
f (c ) = 0 ?.
Solución:

π  π π
No. La función no es continua en x = que pertenece al intervalo − 2 , 2  y por tanto
6  
el Teorema de Bolzano no es aplicable. (De hecho la función no se anula nunca ya que el
numerador es constante).
x
R.11. La función f (x ) = no cumple las condiciones de la hipótesis del T. De Bolzano en
(x − 1)2
[− 3,3] dado que no es continua en x = 1 que pertenece al intervalo, sin embargo sí se anula en
x = 0 y 0 ∈ (− 3,3 ) . ¿Contradice este hecho el Teorema de Bolzano?.
Solución:
No. Si la función no cumple la hipótesis del teorema, la tesis puede cumplirse o no sin
que esto contradiga el teorema, es decir el teorema en este caso no afirma nada y la función
puede anularse o no en algún punto del intervalo.

3-17
Continuidad de funciones Tema 3

R.12. Demuestra utilizando el Teorema de Bolzano que la ecuación 5 x + cos x − 2 = 0 tiene


solución real.
Solución:
Construimos la función f (x ) = 5 x + cos x − 2 .

Si demostramos que f (x ) = 0 habremos demostrado que la ecuación tiene solución.

Esta función es continua ∀x ∈ ℜ y por lo tanto en cualquier intervalo cerrado que se


considere por lo tanto todo se reduce a encontrar dos valores donde la función cambie de
signo, el Teorema de Bolzano nos asegura que entre esos valores la función se anula.

 π  5π π 5π
Probamos f (0 ) = 5 ⋅ 0 + cos 0 − 2 = −1 < 0 y f   = + cos − 2 = −2>0
2 2 2 2

 π
por tanto como f (x ) = 5 x + cos x − 2 es continua en el intervalo 0,  y cambia de signo en
 2

 π
los extremos del intervalo el Teorema de Bolzano asegura que ∃ c ∈  0,  tal que f (c ) = 0 , es
 2
decir, c es solución de la ecuación.

75
R.13. Comprueba que la ecuación x 3 + = 81 tiene solución real.
3 + 2senx

Solución:
75
Consideramos la función f (x ) = x 3 + − 81 . Esta función es continua ya que
3 + 2senx
75 75
3 + 2senx ≠ 0 , además f (0 ) = − 81 < 0 y f (2π ) = (2π ) + − 81 = (2π ) − 56 > 0
3 3
3 3 + 2sen 2π
El teorema de Bolzano asegura que ∃ c ∈ (0,2π ) tal que f (c ) = 0 , es decir, c es solución

de la ecuación dada.
x
R.14. Demuestra mediante el T. de Bolzano que las gráficas de f (x ) = ln x y g (x ) = se cortan.
4
Solución:
x
Construimos la función diferencia de las dadas h (x ) = ln x − si demostramos que
4
h (x ) = 0 en algún punto habremos demostrado que en ese punto f (x ) = g (x ) y por lo tanto que

las gráficas se cortan.


h (x ) es continua ∀x ∈ (0, ∞ ) , buscaremos en esa zona un intervalo donde cambie de signo

1 1 e 4−e
h (1) = ln1 − = − < 0 y h (e ) = ln e − = > 0 como h (x ) es continua en [1, e ] el T.B.
4 4 4 4
asegura la existencia de un c ∈ (1, e ) tal que h(c ) = 0 , es decir f (c ) = g (c ) .

3-18
Continuidad de funciones Tema 3

 a
 x − 3 si x≤2

R.15. Calcula los valores de a y b para que a la función f (x ) =  x 2 + b si 2 < x < 4 le sea
 x si x≥4


aplicable el Teorema de Bolzano en el intervalo [0,4] . Calcula el valor o valores vaticinados por
el teorema.
Solución:
Exigimos que f (x ) sea continua en x = 2 y x = 4 , pues en los demás puntos del

intervalo es continua.
En x = 2 f (2 ) = lim− f (x ) = −a , lim+ f (x ) = 4 + b luego 4 + b = −a
x →2 x→2

En x = 4 f (4 ) = lim+ f (x ) = 4 , lim− f (x ) = 16 + b ⇒ 16 + b = 4 ⇒ b = −12 ⇒ a = 8


x→4 x→4

falta comprobar que para estos valores la función cambia de signo en los extremos de [0,4]

8
f (0 ) = − < 0 y f (4 ) = 4 > 0 lo que en efecto ocurre.
3
Buscamos el valor de x donde f (x ) = 0 con x ∈ (0,4 ) , es evidente que la función sólo

puede anularse en el intervalo 2 < x < 4 , hacemos x 2 − 12 = 0 ⇒ x = ± 12 como x = − 12

está fuera del intervalo (0,4) la solución vaticinada por el teorema es únicamente x = 12 .

R.16. Encuentra una solución de la ecuación x + senx = 1 con un error menor de una décima.
Solución:
Construimos la función f (x ) = x + senx − 1 , utilizaremos el T.B. para demostrar que se

anula y después para acotar el valor donde f (x ) = 0 en un intervalo de longitud menor que una

décima.
f (x ) es continua en cualquier intervalo cerrado que nos convenga considerar, por lo tanto

buscaremos intervalos cada vez mas pequeños donde la función cambie de signo.
f (0 ) = −1 < 0 , f (1) = sen1 > 0 elegimos [0,1]

f (0'5 ) = 0'5 + sen0'5 − 1 ≈ −0'02 < 0 elegimos [0'5,1]

f (0'6 ) = 0'6 + sen0'6 − 1 ≈ 0'16 > 0 elegimos [0'5,0'6]

En el intervalo [0'5,0'6] f (x ) es continua y cambia de signo en los extremos luego

según el Teorema de Bolzano ∃c ∈ (0'5,0'6 ) : f (c ) = 0 luego cualquier número de ese intervalo

es una solución aproximada de la ecuación dada con error menor de una décima.

R.17. Dada la ecuación x 3 − 5 x 2 + 2 = 0 se pide:


a) Separa sus tres soluciones reales.
b) Acota las soluciones con un error menor de una décima.

3-19
Continuidad de funciones Tema 3

Solución:

a) Construimos la función f (x ) = x 3 − 5 x 2 + 2 que es continua en todo ℜ . Separar las

soluciones es encontrar intervalos en los que esté sólo una solución; buscando tres intervalos
disjuntos donde la función cambie de signo por el T.B. habremos encontrado tres ceros de la
función como pretendemos.
f (0 ) = 2 > 0 y f (− 1) − 4 < 0 luego existe un x1 ∈ (− 1,0 ) tal que f (x1 ) = 0 y x1 es solución.

f (1) = −2 < 0 luego existe un x2 ∈ (0,1) tal que f (x2 ) = 0 y x2 es otra solución.

f (5 ) = 2 > 0 luego existe un x3 ∈ (1,5 ) tal que f (x3 ) = 0 y x3 es la tercera solución.

b) Se va probando con la calculadora dividiendo cada uno de los intervalos anteriores


por la mitad y comprobando el signo hasta que se encuentran aproximaciones de las tres
soluciones con la precisión pedida, así encontramos x1 ∈ (− 0'6,−0'5 ) , x2 ∈ (0'6,0'7 ) y

x3 ∈ (4'9,5 ) .

3-20
Continuidad de funciones Tema 3

Ejercicios propuestos

Continuidad en un punto

1 + x
 , si x ≠ 1
P.1. Dada f (x ) =  x − 1 , ¿existe algún valor de k para el que la función f (x) sea
 k , si x = 1

continua en x =1?

 2x
 , si x ≠ 0
P.2. La función f (x ) =  x , ¿ es continua en el origen ?
 0, si x = 0

 senx , si x ≤ π
P.3. Siendo f (x ) =  , halla b para que sea continua en x = π .
2 x + b, si x > π

P.4. Estudia la continuidad en x = 0 de las siguientes funciones:

 x −22  x  x
  5 − , si x ≠ 0 
f (x ) =  e x , si x ≠ 0 g (x ) =  x h(x ) =  senx , si x ≠ 0
 0, si x = 0  5, si x = 0  1, si x = 0

 e tgx − 1 π
π  tgx , si x ≠
P.5. Estudia la continuidad en x = de la función f (x ) =  e + 1 2
2  1, si x = π
 2

Continuidad de una función y tipos de discontinuidades


P.6. Estudia la continuidad de las siguientes funciones:

4 x + sen 2 x − 4 cos x
a )f (x ) = b )f (x ) = (x − 4 )(x + 3)
4senx − 2

P.7. Estudia la continuidad de las siguientes funciones y clasifica sus discontinuidades:


 x + 2, si x ≤ 0  1
  , si x ≤ 2
a ) f (x ) = x − 1 b ) f (x ) =  2, si 0 < x < 2 c ) f (x ) =  x − 1
 x 2 − 1, si x ≥ 2  2, si x > 2

1
d ) f (x ) = 2 x + 1 + e ) f (x ) = 2 x −1
x −1

1 x −1  x 2 + 1 , si x ≤ 1
f) f (x ) = − 2 g ) f (x ) = 
x −3 x + x −2  2 x − 2, si x > 1

3-21
Continuidad de funciones Tema 3

 3, si x ≤ −1
2 x + 3, si − 1 < x ≤ 0


P.8. Dibuja f (x ) =  1 , si 0 < x ≤ 6 , estudia su continuidad y clasifica sus
 x −3
 1
 x − 5 , si x > 6

discontinuidades.

P.9. Estudia la continuidad de las siguientes funciones y clasifica sus discontinuidades:

 ex
 x 2 − x + 1, si x < 0  , si x ≤ 0 1 x
a ) f (x ) =  b ) f (x ) =  e x + 1 c ) f (x ) = x − − 2
 x − 1 , si x ≥ 0  x 2 + 1, si x > 0 x−2 x −x

P.10. Estudia la continuidad de las siguientes funciones y clasifica sus discontinuidades:


 1
2  tgx sen 
3 x + cos x − 1  , si x ≠ 0 x
a ) f (x ) = b ) f (x ) =  x c ) f (x ) =
sen 2 x  1, si x = 0
1
1+ e x

P.11. Estudia, según el valor de k, la continuidad de las siguientes funciones:

2 x 2 − 4, si x ≠ −3 3 − x + 6

a ) f (x ) =  b ) f (x ) =  x − 3 , si x ≠ 3
 k , si x = −3  k , si x = 3

x3 + x 2 + x + a
P.12. La función f (x ) = , no está definida en x =1. Halla a para que sea posible
x −1
redefinir f (x ) como una función continua en R.

x2 − x
P.13. Dada la función f (x ) = , con x∈]0,1[ , define f(0) y f(1) de forma que f sea
sen (π x )
continua en [0,1]. (Sugerencia: para x = 1 , haz el cambio de variable t = x − 1 ).

 x 2 + 2 x − 1, si x < 0

P.14. Dada la función f (x ) =  ax + b, si 0 ≤ x < 1 , halla a y b para que la función sea
 2, si x ≥ 1

continua y dibuja la gráfica de la función que resulta.

P.15. Calcula el valor de los parámetros para que las siguientes funciones sean continuas en
toda la recta real.
 ax + 2, si x ≤ −1
 e ax , si x ≤ 0  x 2 + ax , si x ≤ 2 
a ) f (x ) =  b ) f (x ) =  c ) f (x ) =  x 2 − 1, si − 1 < x < 1
 x + 2a, si x > 0  a − x 2 , si x > 2  2 x + b, si x ≥ 1

3-22
Continuidad de funciones Tema 3

 ax 2 + c , si x ≤ −1
 x + 1, si x ≤ −2  2
 2 − bx + ax − c − 1, si − 1 < x < 1
d ) f (x ) = ax + b, si − 2 < x < 1 e ) f (x ) =  2
 − 4 x , si x ≥ 1  ax − cx + b + 1, si 1 ≤ x ≤ 2
 
 − cx 2 − 4b, si x > 2

 x 2 − 2, si x∈Q
P.16. Razona por qué la función f (x ) =  sólo es continua en {1−
, 2} .
− x , si x∈R − Q

Teorema de Bolzano
P.17. Si f(x) es continua en [1,9] y es tal que f (1) = −5 y f (9 ) > 0 . ¿Podemos asegurar que en

estas condiciones la función g(x) = f(x) + 3 tiene, al menos, un cero en el [1,9]? Razona la
respuesta.

P.18. Sabiendo que f(x) es tal que f(-2) < 0 y f(0) > 0 ¿Podemos asegurar que existe un punto
c del intervalo ]-2,0[ tal que f(c) = 0 ? Razona la respuesta.

1
 , si x ≠ 0
P.19. Sea f (x ) =  x que verifica f(-1) < 0 y f(1) > 0 y, sin embargo, la función no se
 2, si x = 0

anula en el intervalo ]-1,1[ . ¿Contradice ésto el Teorema de Bolzano?

P.20. ¿Es cierto que si una función f(x) es tal que f (a ) ⋅ f (b ) < 0 y es continua en [a,b],

entonces existe una solución única en [a,b] de la ecuación f(x) =0 ? Razona la respuesta.

tgx , si x ≠ π 2
P.21. La función f (x ) =  toma valores de distinto signo en los extremos del
 1, si x = π 2
 π 3π 
intervalo  ,  y, sin embargo, no se anula en él , ¿contradice ésto el T. De Bolzano?
4 4 

4 1
P.22. ¿Podemos aplicar el Teorema de Bolzano para asegurar que la ecuación x + =0
x −2
tiene alguna solución en el intervalo ]0,3[ . ¿ Y en el ]-2,0[ ? Razona la respuesta.

P.23. Sea f continua en [0,1], tal que 0 < f(x) < 1 en dicho intervalo, demuestra que tiene que
haber un c ∈ ] 0,1 [ tal que f(c) = c. (Un Teorema del punto fijo).

P.24. Determina los valores del número real k para los cuales la función f (x ) = x 3 − 3 x + k se

anula en algún punto del intervalo [-1,1].

P.25. Demuestra que la ecuación π x = e tiene una solución en ]0,1[.

3-23
Continuidad de funciones Tema 3

P.26. Sean f y g dos funciones continuas en [a,b], () ()


tales que f (a ) > g (a ) y f b < g b .
Demuestra que sus gráficas se cortan.

P.27. Sea f :[0,2π ]→R continua, tal que f (0 ) = f (2π ) . Demuestra que existe un punto x del

intervalo (0, π ) tal que f (x ) = f (x + π ) .

4 2
P.28. Encuentra cuatro intervalos disjuntos en los que la ecuación 2 x − 14x + 14x − 1 = 0
tenga una raíz.

P.29. ¿Se puede aplicar el Teorema de Bolzano a la función f (x ) = cosec (x ) , en el intervalo

 π 3π 
 , ?
2 2 

Resolución aproximada de ecuaciones


P.30. Halla, con error menor de una décima, la raíz real de cada una de las siguientes
3 2 3
ecuaciones: a) x + 4 x − 6 = 0 ; b) x + x + 1 = 0 .

5
P.31. Justifica que la ecuación x = 5 − 3x tiene una solución entre 1 y 2. Halla dicha solución
con dos cifras decimales.

P.32. Halla la menor solución positiva de tg x = x , con dos decimales.

5
P.33. Utiliza el Teorema de Bolzano para hallar 37 con un error menor que una décima.

P.34. Utiliza el Teorema de Bolzano para demostrar que la ecuación x + senx = 8 tiene, al
menos, una raíz y calcula ésta con error menor que una centésima.

P.35. Comprueba que f (x ) = ln (x ) y g (x ) = e − x se cortan en un solo punto. Calcula ese punto


de forma aproximada.

Propiedades de las funciones continuas


x +x x −x
P.36. Dadas las funciones f (x ) = y g (x ) = , estudia la continuidad de las
2 2
f
funciones: f , g , g o f , f o g , f ⋅ g , .
g

P.37. Si f es continua en x = a y g discontinua en x = a , ¿qué puede decirse de f + g ? ¿ Y de


f ⋅g?

P.38. Una función f : R -> R es tal que vale 4 en todos los números racionales. Si f es continua,
¿qué función es f ?

3-24
Continuidad de funciones Tema 3

P.39. Si f(x) es continua en [5,8] y además f(5) = 1; f(8) = 3 entonces: a) todos los valores de f
están en el [1,3]; b) existe un valor c del ]5,8[ tal que f(c) = 1.4 ; c) la función se anula en un
punto; d) se verifica siempre que f(x) > 1 en este intervalo. Razona la respuesta correcta.

Soluciones
P.1. No. Tiene una discontinuidad de salto infinito ∀k ∈ ℜ . P.2. No. Tiene una discontinuidad

 x −2 
de salto finito. P.3. b = −2π P.4. f (x ) continua  f (0 ) = lim e x 2 = e − ∞ = 0  ; g (x )
 x →0 
 
discontinuidad de salto finito en x = 0 ; h(x ) discontinuidad de salto finito en x = 0 P.5.

 
 
Discontinua de salto finito  lim + f (x ) = −1, lim − f (x ) = 1 . P.6. a) Discontinua de salto infinito en
 x→ π x→
π 
 2 2 
π 5π
+ 2 kπ , + 2 kπ , ∀k∈Z ; b) continua en todo su dominio, que es R − (− 3,4 ) . P.7. a)
6 6
Continua en todo R ; b) discontinuidad de salto finito en x = 2 ; c) discontinuidad de salto
infinito en x = 1 y discontinuidad de salto finito en x = 2 ; d) continua ∀x ∈ (1,+∞ ) ; e) continua

∀x ∈ [1,+∞ ) ; f) discontinuidad de salto infinito en x = 3 y x = −2 , evitable en x = 1 ; g)

discontinuidad de salto finito en x = 1 . P.8. Discontinuidad de salto infinito en x = 3 , de salto


finito en x = −1 , x = 0 , x = 6 . P.9. a) Continua; b) discontinuidad de salto finito en x = 0 ; c)
discontinuidad de salto infinito en x = 1 y x = 2 , evitable en x = 0 . P.10. a) Discontinuidad
evitable en x = 0 , discontinuidad de salto infinito en x = kπ con k ∈ Z − {0} , b) Continua en

π
x = 0 , discontinuidad de salto infinito en x = + kπ con k ∈ Z , c) discontinuidad de 2º
2
especie en x = 0 (no existe lim f (x ) ). P.11. a) Si k = 14 continua, si k ≠ 14 discontinua
x →0 −

1
evitable en x = −3 ; b) discontinua si x < −6 ∀k , además si k = − continua en x = 3 y si
6
1 −1
k ≠− discontinua evitable en x = 3 . P.12. a = −3 . P.13. f (0 ) = f (1) = . P.14. a = 3 , b = -1.
6 π
P.15. a) a = 1/2 ; b) a = -8 ; c) a = 2 , b = −2 ; d) a = 1 , b = −5 ; e) para que sea continua en:

 x = −1 ⇒ a + c = − b − a − c − 1  a = 2
  
 x = 1 ⇒ −b + a − c − 1 = a − c + b + 1 ⇒  b = −1 . P.16. Que sólo puede ser continua en x = 1 y
 x = 2 ⇒ 4a − 2c + b + 1 = −4c − 4b   c = −2
  
x = − 2 podemos probarlo por medio de la caracterización del límite por sucesiones. Que es
continua en x = 1 y x = − 2 lo probamos por la definición de continuidad. (Con menos rigor,
podríamos servirnos de su gráfica para justificarlo). P.17. Sí, por el Teorema de Bolzano. P.18.
No lo podemos asegurar porque no dice nada sobre la continuidad. P.19. No lo contradice,
porque es discontinua en x = 0. P.20. Es cierto que existe solución, pero no tiene porqué ser

3-25
Continuidad de funciones Tema 3

π
única. P.21. No, porque es discontinua en . P.22. En el intervalo ]0,3[ no, porque es
2
discontinua en 2. En el ]-2,0[ sí, porque satiface la hipótesis del Teorema. P.23. Define
g (x ) = f (x ) − x y aplica el Teorema de Bolzano en el intervalo [0,1] . P.24. k∈[−2,2] . P.25.

Aplica el Teorema de Bolzano en el intervalo [0,1] a la función f (x ) = π x − e .

P.26. Aplica el Teorema de Bolzano en el intervalo [a,b] a la función h(x ) = f (x ) − g (x ) .

P.27. Define h(x ) = f (x ) − f (x + π ) y aplica en [0, π ] , el Teorema de Bolzano.

P.28. ]− 4,−3 [], 0,1[], 1,1.5 [], 1.8,1.9 [ . P.29. No, porque es discontinua en π.

P.30. a) 1,0 ; b) –0,6 . P.31. Define f (x ) = x 5 − 5 + 3 x y aplica el T. De Bolzano en [1,2] . Sale

1,10 . P.32 4,49 . P.33. Define f (x ) = x 5 − 37 y aplica el T. De Bolzano en [2,3] . Sale 2,0

P.34. Aplícalo a la función f (x ) = x + senx − 8 en el intervalo [0,8] .

P.35. Define h(x ) = f (x ) − g (x ) y aplica el Teorema de Bolzano a algún intervalo . Sale 1,309 .

f
P.36. f , g , g o f , f o g y f ⋅ g continuas, discontinua si x ≥ 0 (g = 0 ) . P.37. f + g es discontinua
g
en a . f ⋅ g no se puede asegurar, porque podría ser continua; por ejemplo, si

 x − 1, si x ≠ 1  3
 , si x ≠ 1
f (x ) =  y g (x ) = x −1 , su producto es una función continua. P.38.
 1, si x = 1  3, si x = 1

f(x) es la función constante igual a 4. P.39. La opción b) .

Gráfica P.8 Gráfica P.14

3-26
Derivabilidad Tema 4

Tema 4

Derivabilidad

4.1 Derivada de una función en un punto

Definición Se llama derivada de f en el punto a, y se denota por f ′(a ) al valor del límite
f (a + h ) − f (a ) f (a + h ) − f (a )
lim , cuando este límite es finito. Escribiremos f ′(a ) = lim .
h →0 h h →0 h

Si una función f tiene derivada en un punto a, se dice que es derivable en a.

Algunas otras notaciones para calcular o designar la derivada son:


f (x ) − f (a )
y ′(a ) = y& (a ) = D(f (a )) = lim
x →a x −a

Nota Algebraicamente, la derivada de una función en un punto es un número real.

Ejemplos

Ejemplo 4.1 Calculemos la derivada de f (x ) = x 2 + 3 en el punto x = 5.

f ′(5 ) = lim
f (5 + h ) − f (5 )
= lim
( )
(5 + h )2 + 3 − 5 2 + 3 = lim 25 + 10h + h 2 + 3 − 28 =
h →0 h h →0 h h →0 h

10 h + h 2 h (10 + h )
= lim = lim = 10
h →0 h h →0 h

Ejemplo 4.2 Veamos que no existe la derivada de f (x ) = x + 1 en el punto x = -1.

f (− 1 + h ) − f (− 1) (− 1 + h ) + 1 − − 1 + 1 h
f ′(− 1) = lim = lim = lim , que no existe, ya que los
h →0 h h →0 h h →0 h
límites laterales no coinciden.
h −h
En efecto, lim+ = 1 y lim− = −1 .
h →0 h h →0 h

4-1
Derivabilidad Tema 4

4.2 Derivadas laterales

Definiciones Llamamos derivada lateral por la derecha de la función f en el punto a, y la


f (a + h ) − f (a )
denotamos por f +′ (a ) , al valor del límite lim+ , cuando es finito.
h →0 h
Análogamente, llamamos derivada lateral por la izquierda de la función f en el punto
f (a + h ) − f (a )
a, y la denotamos por f −′ (a ) , al valor del límite lim− , cuando es finito.
h →0 h

Ejemplos

Ejemplo 4.3 Calculemos las derivadas laterales de f (x ) = x en x = 0.


f (0 + h ) − f (0 ) h h
La derivada por la derecha será f +′ (0 ) = lim+ = lim+ = lim = 1.
h →0 h h →0 h h →0 h

f (0 + h ) − f (0 ) h −h
La derivada por la izquierda será f −′ (0 ) = lim− = lim− = lim = −1 .
h →0 h h →0 h h →0 h

 x 2 , si x ≤ 1

Ejemplo 4.4 Calculemos las derivadas laterales de f (x ) =  1 en el punto de abcisa
 x , si x > 1
1.
1
−1
f (1 + h ) − f (1) 1 + h −h
La derivada por la derecha será f +′ (1) = lim+ = lim+ = lim = −1 .
h →0 h h →0 h h →0 h (1 + h )

La derivada por la izquierda será f −′ (1) = lim−


f (1 + h ) − f (1)
= lim−
(1 + h )2 − 1 = lim h 2 + 2h = 2 .
h →0 h h →0 h h →0 h

Proposición Una función f es derivable en un punto a si, y sólo si, existen sus derivadas
laterales y coinciden.

Ejemplo 4.5 La función f (x ) = x no es derivable en x = 0, porque sus derivadas laterales no


coinciden (ver Ejemplo 4.3).

Nota Si f es continua en a y sus derivadas laterales en a son distintas (pudiendo ser una de
ellas +∞ o −∞ , y la otra finita), se dice que en a hay un punto anguloso. Si una derivada
lateral es +∞ y la otra −∞ se dice que en a hay un punto de retroceso.

Punto de retroceso Punto anguloso

4-2
Derivabilidad Tema 4

4.3 Interpretación geométrica de la derivada de una función en un punto


Sea f una función derivable en un punto a y sea (x n ) una sucesión convergente a a,
cuyos términos están en un entorno reducido de a.

Consideremos, por un lado, la recta tangente a f en el punto P de coordenadas (a, f (a ))


y, por otro lado, las rectas secantes a f que pasan por los puntos P y Q n , de coordenadas
(x n , f (x n )) . Como la pendiente de una recta es la tangente trigonométrica del ángulo que forma
la recta con el semieje positivo de las X, las pendientes de estas rectas secantes son
f (x n ) − f (a )
mQn = .
xn − a

Como x n tiende a a, conforme x n se acerca a a sobre el eje X, también los puntos


Q n se acercan a P sobre la curva, definiendo un número infinito de rectas secantes que
tienden a una posición límite, que no es otra que la recta tangente a la curva en P.
Esta idea intuitiva nos lleva a definir la pendiente de la recta tangente a f en P como el
límite de las pendientes de las rectas secantes al tender Q n a P. Esto es, se tiene que
f (x n ) − f (a ) f (x ) − f (a )
mT = lim mQn = lim = lim = f ′(a ) , donde mT denota la pendiente de
Qn →P n →∞ xn − a x →a x −a
la recta tangente.
Geométricamente, por lo tanto, la derivada de una función en un punto es la pendiente
de la recta tangente a la función en ese punto.

Nota Como la ecuación de una recta que pasa por el punto (x0,y0) y tiene de pendiente m, es
y = f (x 0 ) + m (x − x 0 ) , la recta tangente a f (x ) en el punto (a, f (a )) , será:
y = f (a ) + f ′(a )(x − a ) .

Ejemplos

Ejemplo 4.6 Para hallar la ecuación de la recta tangente a la función f (x ) = senx en el punto
x = 0 , calculamos primero f (0 ) = 0 ; después, f ′(x ) = cos x , f ′(0 ) = 1 y, por último, aplicamos la
ecuación de la nota anterior, por lo que la ecuación de la recta tangente es
y = f (0 ) + f ′(0 )(x − 0 ) ⇔ y = x .

4-3
Derivabilidad Tema 4

Ejemplo 4.7 Para hallar en qué punto la recta tangente a la función f (x ) = x + 1 tiene de
1
pendiente m = 2 , hacemos f ′(x ) = ; después, resolvemos:
2 x +1
1 −15
2= ⇔4 x + 1 = 1 ⇔ 16 (x + 1) = 1 ⇔ x = .
2 x +1 16
−15
Por lo tanto, en el punto de abcisa x = , la función f (x ) = x + 1 tiene de pendiente m = 2 .
16

Derivadas infinitas
Al exigir, por definición, que f ′(a ) sea finita es evidente que la interpretación
geométrica de la derivada no es válida en el caso de que la recta tangente sea vertical. Para
evitar esto se habla de derivadas infinitas en un punto, siempre que f sea continua en él. De
modo que, si f es continua en a, siendo a un punto interior de D, diremos que f posee derivada
+∞ en a, y escribiremos f ′(a ) = +∞ , si ambas derivadas laterales en a valen +∞ . (La derivada
f ′(a ) = −∞ , se define análogamente).

4.4 Derivabilidad y continuidad

Proposición Si una función es derivable en un punto, entonces es continua en ese punto.

Nota Es importante tener claro que el recíproco no se cumple.

Por ejemplo, la función f (x ) = x es continua en x = 0, pero no es derivable en x = 0 .

Nota Una consecuencia geométrica de esta proposición es que las gráficas de las funciones
derivables son curvas suaves; esto es, sin cambios bruscos ni picos. Ya que si en algún punto
de ellas existiera cambio brusco es fácil probar que en ese punto existen dos rectas tangentes
(una a la izquierda y otra a la derecha) con distintas pendientes y, por consiguiente, las
derivadas laterales son distintas resultando que la función no sería derivable en ese punto.

Son continuas en cero, pero no tienen derivada en cero.

Se debe a Weierstrass un ejemplo, que impresionó a los matemáticos de su época, de


función continua sin derivada en ninguno de sus puntos.

Ejemplo 4.8 Si se nos pide que estudiemos, en todo R, la continuidad y derivabilidad de la


función
 3 , si x < −2

f ( x ) =  x + 5 , si − 2 ≤ x ≤ 2 ,
 x 2 − 3 x , si x >2

4-4
Derivabilidad Tema 4

comenzamos con la continuidad. Evidentemente, si x ∈ R − {−2,2} , la función es continua por


tratarse de funciones polinómicas.

En x = −2 , como f (−2 ) = lim − 3 = 3 y lim (x + 5 ) = 3 , la función es continua en


x → −2 x → −2 +
x = −2 . Para el estudio de la derivabilidad se puede utilizar la definición de derivada o bien la
función derivada para calcular derivadas laterales, esto último sólo porque la función es
continua en el punto.
 0 , si x < −2

f ′( x ) =  1 , si − 2 < x < 2 . Como f −′ (−2) = 0 , f +′ (−2) = 1 , la función no es derivable
2 x − 3 , si x>2

en x = −2 .
En x = 2 , como f (2) = lim− (2) = 7 ≠ lim+ (2) = −2 , la función no es continua y, por lo
x →2 x→
tanto, tampoco es derivable. Para todo x ∈ R − {−2,2} la función es derivable por tratarse de
funciones polinómicas.

4.5 Propiedades de las funciones derivables

Proposición (Propiedades algebraicas) Sean f y g funciones derivables en a, entonces:

1. f ± g es derivable en a y (f ± g )′ (a ) = f ′(a ) + g ′(a ) .


2. f ⋅ g es derivable en a y (f ⋅ g )′ (a ) = f ′(a ) g (a ) + f (a )g ′(a ) .

f f  f ′(a ) g (a ) − f (a )g ′(a )
3. Si g ′(a ) ≠ 0 , es derivable en a y   (a ) = .
g g  (g (a ))2

Ejemplos

Ejemplo 4.9 La función f (x ) = x 2 + sen x es derivable en todo R, por ser suma de funciones
derivables en todo R.

1
Ejemplo 4.10 La función f (x ) =
+ sen x es derivable en R − {0} , porque es suma de
x
funciones derivables en R − {0} (la segunda es derivable en todo R). En x = 0 no es derivable
porque no es continua, ya que no existe f (0 ) .

Ejemplo 4.11 La función f (x ) = cos x ⋅ x + 1 es derivable en (−1,+∞ ) porque es producto de


funciones derivables en (−1,+∞ ) . La función coseno es derivable en todo R. El dominio de f es
[−1,+∞] y h (x ) = x + 1 , cuyo dominio es [−1,+∞] , sólo es derivable en (−1,+∞ ) , pues
h
h ′(− 1) = lim que no existe cuando h tiende a cero por la derecha.
h →0 h

Derivada de la composición de funciones (Regla de la cadena) Sean f : A → R y


g : B → R funciones tales que f es derivable en a de A y g es derivable en f (a ) de B,
entonces: la composición g o f es derivable en a y se cumple (g o f )′ (a ) = g ′(f (a )) ⋅ f ′(a ) .

4-5
Derivabilidad Tema 4

Ejemplos

Ejemplo 4.12 La función h(x ) = x 2 + 1 es la composición de f (x ) = x 2 + 1 y g (x ) = x ; esto


es, h(x ) = g (f (x )) .
Por tanto, la derivada de h en x = 2 será h ′(2) = g ′(f (2)) ⋅ f ′(2) .
1 1 4 2
Como g ′(x ) = , f ′(x ) = 2 x y f (2) = 5 , se tiene que h ′(2) = ⋅ f ′(2) = = .
2 x 2 f (2) 2 5 5

Ejemplo 4.13 Para la función del ejemplo anterior, como la composición está definida para
1 2x x
todo x de R, para cualquier punto x se tiene que h ′(x ) = ⋅ f ′(x ) = =
2 f (x ) 2 x2 +1 x2 +1

4.6 Función derivada. Derivadas de orden superior

Definiciones Si una función f , definida en un


dominio D, tiene derivada en cada punto de D , la
correspondencia que a cada x de D le asocia el
valor f ′(x ) se llama función derivada de f , y se
denota por f ′ o, también, f (1) .
Si la función f ′ es derivable en D, la
función derivada se representa por (f ′)′ = f ′′ = f (2 ) ,
y recibe el nombre de derivada segunda de f.
Del mismo modo se definen la derivada
tercera f ′′′ = f (3 ) , la derivada cuarta f (4 ) ,y en
general, la derivada de orden n o derivada enésima
f (n ) .

Ejemplos

sen x x cos x − sen x


Ejemplo 4.14 La función derivada de f (x ) = es f ′(x ) = .
x x2
 x + 1 , si x ≤ 0
2
Ejemplo 4.15 Para hallar la función derivada de f (x ) =  , derivamos
 cos x , x > 0

( )
primeramente los trozos: x 2 + 1 = 2 x y (cos x )′ = −sen x .
Ahora queda ver qué pasa en x = 0 . La derivada por la izquierda será
f (0 + h ) − f (0 ) 2
h + 1− 1 h2
f −′ (0 ) = lim− = lim− = lim = 0 . La derivada por la derecha será
h →0 h h →0 h h →0 h

h2

f (0 + h ) − f (0 ) cos h − 1  0  i .e 2 = 0.
f +′ (0 ) = lim+ = lim+ =   = lim
h →0 h h →0 h  0  h →0 h
 2 x , si x ≤ 0
Luego, f ′(x ) =  .
− sen x , x > 0

4-6
Derivabilidad Tema 4

Ejemplo 4.16 (Una forma de hallar las derivadas laterales en un punto siempre que la función
sea continua en ese punto). Como la función del ejemplo anterior es continua en x = 0 , sus
derivadas laterales se hubieran podido calcular más fácilmente haciendo:

( )
f −′ (0 ) = 2 ⋅ 0 = 0 , pues a la izquierda de cero se tiene x 2 + 1 = 2 x ; y f +′ (0 ) = −sen 0 = 0 , pues
a la derecha de cero se tiene (cos x )′ = −sen x .

Ejemplo 4.17 Algunos ejemplos de cálculo de derivadas sucesivas son:

• f (x ) = 4 x 3 − 2 x ; f ′(x ) = 12 x 2 − 2 ; f ′′(x ) = 24 x ; f (3 ) (x ) = 24 ; f (4 ) (x ) = 0 .
• f (x ) = e 3 x ; f ′(x ) = 3 e 3 x ; f ′′(x ) = 3 2 e 3 x ; f (3 ) (x ) = 3 3 e 3 x ; K
1
• f (x ) = = (x + 2)− 1 ; f ′(x ) = −(x + 2)− 2 ; f ′′(x ) = 2 (x + 2 )− 3 ; f (3 ) (x ) = −2 ⋅ 3 (x + 2)− 4 ;
x+2
f (4 ) (x ) = 2 ⋅ 3 ⋅ 4 (x + 2)−5 ; f (5 ) (x ) = −2 ⋅ 3 ⋅ 4 ⋅ 5 (x + 2)−6 ; K

Nota Las derivadas sucesivas son el primer paso para el cálculo de la derivada enésima de
una función. Aunque en los ejemplos no utilizamos el Principio de Inducción, la demostración
rigurosa de lo que obtenemos se basa en él. Como dijimos, la derivada enésima es la derivada
de orden n de una determinada función.

Ejemplo 4.18 Si siguiéramos derivando las funciones del ejemplo anterior, obtendríamos por
inducción los siguientes resultados:

• f (x ) = 4 x 3 − 2 x ; f ′(x ) = 12 x 2 − 2 ; K; f (n ) (x ) = 0 , ∀ n ≥ 4 .
• f (x ) = e 3 x ; f ′(x ) = 3 e 3 x ; K; f (n ) (x ) = 3 n e 3 x , ∀ n ≥ 0 .
1 − (n +1)
• f (x ) = = (x + 2) ; f ′(x ) = −(x + 2 ) ; K ; f (n ) (x ) = (− 1) ⋅ (n!)(x + 2 )
−1 −2 n
x+2

Nota En general, para calcular la derivada enésima de una función racional se debe
descomponer ésta en suma de fracciones simples y, separadamente, calcular las derivadas
enésimas de las fracciones simples que aparecen.

x +1
Ejemplo 4.19 Calculemos la derivada enésima de f (x ) = . Como las raíces del
2
x +x −2
denominador son 1 y –2, la descomposición en suma de fracciones simples es:
x +1 A B
2
= + .
x +x−2 x − 1 x +2
Para hallar A y B, hacemos operaciones en el segundo miembro e igualamos los numeradores:
x + 1 = A (x + 2) + B (x − 1) ⇔ x + 1 = (A + B ) x + 2 A − B . Las incógnitas se obtienen identificando
2 1
coeficientes y resolviendo el sistema: 1 = A + B ; 1 = 2 A − B . De donde, A = ,B = . Así
3 3
x +1 2 1  1 B 
pues, derivar f (x ) = 2 es lo mismo que derivar f (x ) =  +  .
x +x −2 3  x − 1 3  x + 2 
Derivando por separado y teniendo presente el resultado del ejemplo 18, resulta
2 1 
f (n ) (x ) = (− 1)n ⋅ (n ! )  (x − 1)−(n +1) + (x + 2)−(n +1)  .
 3 3 

4-7
Derivabilidad Tema 4

4.7 Interpretación física de la derivada


Ya hemos visto cómo la derivada se puede interpretar como la pendiente de una curva.
Trataremos ahora, mediante el problema de la velocidad instantánea, dar un ejemplo de
interpretación física.

Galileo estableció experimentalmente que la distancia recorrida en el tiempo t por un


cuerpo que cae libremente en el vacío viene dada por la siguiente ecuación.

Si consideramos un intervalo de tiempo [ t , t + h ] , la velocidad media con la que cae el


s (t + h ) − s (t )
cuerpo es v m = . Es decir, la velocidad media expresa la razón de cambio de s
h
respecto a t en un intervalo de tiempo.

Supóngase que no nos interesa la velocidad media, sino la razón de cambio exacta de
s para un valor particular de t. A esta razón de cambio de s respecto a t en un instante
determinado se denomina velocidad instantánea. En concreto, lo que interesa es saber cuál es
la velocidad en el instante t0 .

Tal velocidad parece lógico se pueda calcular considerando la velocidad media en


cortos intervalos de tiempo alrededor de t0. Esto es, en intervalos del tipo [t0,t0+h] donde h
tiende a cero. Evidentemente, si empleamos intervalos de tiempo más cortos, y más cortos, las
velocidades medias resultan, cada vez, mejores aproximaciones de la velocidad instantánea en
t0; así pues, la velocidad en el instante t0 será
s (t 0 +h ) − s (t 0 )
v i = lim = s ′(t 0 ) .
h →0 h
Podemos decir, entonces, que la derivada de una función en un punto expresa la razón
de cambio (o variación) de la función, en un instante determinado de la variable independiente.

Ejemplo 4.20 Un punto se mueve sobre el eje X con velocidad constante 10 cm/s, y otro
punto se mueve sobre el eje Y con velocidad constante 30 cm/s. ¿Con qué velocidad se
separan los puntos cuando la abcisa vale 12 cm y la ordenada 20?
Solución: Nos piden la velocidad en el instante en el que x = 12 e y = 20 . Como la distancia
que separa dos puntos cualesquiera, por ejemplo ( x , 0 ) e ( y , 0 ) , en un instante t viene dada

por la expresión d (t ) = x (t )2 + y (t )2 , para calcular la velocidad pedida derivamos d,


2 x ′(t ) x (t ) + 2 y ′(t ) y ′(t ) x ′(t ) x (t ) + y ′(t ) y (t )
obteniendo d ′(t ) = = . Como x ′(t ) = 10 e y ′(t ) = 30
2 x (t )2 + y (t )2 x (t )2 + y (t )2
para todo t , en el instante en el que x (t ) = 12 e y (t ) = 20 , se tiene que
10 ⋅ 12 + 30 ⋅ 20 720
d ′(t ) = = = 30,86 cm/s .
2 2 544
12 + 20

4-8
Derivabilidad Tema 4

4.8 Resumen de las reglas de derivación


Se resumen a continuación en forma de tabla las reglas de derivación y las derivadas
de las funciones más importantes.

Tabla de derivadas

Función Derivada
k 0
x 1
f ( x ) + g( x ) f ′( x ) + g′( x )
f ( x ) ⋅ g( x ) f ′( x ) ⋅ g( x ) + f ( x ) ⋅ g′( x )
f ( x) f ′( x ) ⋅ g( x ) − f ( x ) ⋅ g′( x )
g( x ) (g( x ))2
f ( x )a con a real a ⋅ f ( x )a −1 ⋅ f ′( x )
f( x) f ′( x )
2⋅ f( x)
n f( x) f ′( x )
n⋅ n
(f ( x ))n−1
ln f ( x ) , con f(x)>0 ⋅f ′( x )
f (x)
loga f ( x ) , con f(x)>0 ⋅f ′( x )
⋅ loga e
f (x)
ef ( x ) e f ( x ) ⋅ f ′( x )
af ( x ) a f ( x ) ln a ⋅ f ′( x )
f ( x )g( x ) f g ⋅ ln f ⋅ g′ + g ⋅ f g−1 ⋅ f ′
senf ( x ) cos f ( x ) ⋅ ⋅f ′( x )
cos( f ( x )) −senf ( x ) ⋅ f ′( x )
tgf ( x )
(1+ tg f (x))⋅ f ′( x) = cos 1f (x) ⋅ f ′(x) = sec
2
2
2
f ( x ) ⋅ f ′( x )

cot gf ( x ) −1
( )
− 1 + cot g2 f ( x ) ⋅ f ′( x ) = 2
sen f ( x )
⋅ f ′( x ) = − cos ec 2 f ( x ) ⋅ f ′( x )

sec f ( x ) senf ( x ) tgf ( x )


sec f ( x ) ⋅ tgf ( x ).f ′( x ) = .f ′( x ) = .f ′( x )
cos2 f ( x ) cos f ( x )
cos ec( f ( x )) cos f ( x ) cot gf ( x )
− cos ecf ( x ) ⋅ cot gf ( x ).f ′( x ) = − 2
.f ′( x ) = − .f ′( x )
sen f ( x ) senf ( x )
arcsenf ( x ) o –arccos f(x) f ′( x )
1 − f ( x )2
arctgf ( x ) o –arccotg (x) f ′( x )
1 + f ( x )2
arc sec f ( x ) 0 – arccosec f(x) f ′( x )
f ( x ) ⋅ f ( x )2 − 1

4-9
Derivabilidad Tema 4

Ejercicios resueltos
Cálculo de la función derivada

R.1. Productos, cocientes, potencias y raíces..

1) f ( x ) = ( x 2 + x + 3 ) ⋅ (1 − 2 x )
Solución: Derivada del producto. f ′( x ) = (2 x + 1) ⋅ (1 − 2 x ) + ( x 2 + x + 3 ) ⋅ (− 2) . Operando se
llega a f ′( x ) = −6 x 2 − 2 x − 5 .

(
2) f ( x ) = ax 2 + b ) 4

Solución: Derivada de una potencia. f ′( x ) = 4 ⋅ ax 2 + b 2ax . ( )


3

x
3) f ( x ) = 2
x +1
x 2 + 1 − x ⋅ 2x 1− x 2
Solución: Derivada el cociente f ′( x ) = =
(x 2
+1 ) 2
(x 2
+1 )
2

4) f ( x ) = 4
(1 + x )3
3
Solución: Se escribe la raíz en forma de potencia y se deriva. f ( x ) = 4
(1 + x )3 = (1 + x ) 4
3 1
f ′( x ) = (1 + x )− 4 = 4 3 .
4 4 1+ x

x2 −1
5) f ( x ) =
x +1
Solución:
2x
(x + 1) − x2 −1
x ⋅ (x + 1) − x 2 − 1 ( )
2 x2 −1 x2 −1 x +1 1
f ′( x ) = 2
= 2
= =
(x + 1) (x + 1) x − 1 ⋅ (x + 1)
2 2
x − 1 ⋅ (x + 1)
2

R.2. Funciones logarítmicas y exponenciales.

(
1) f (x ) = ln 1 − x 2 )2

Solución: f ′(x ) =
( )
2 ⋅ 1 − x 2 ⋅ (− 2 x )
=−
4x
=
4x
.
(1 − x ) 2 2 1− x 2
x −12

1− x
2) f (x ) = log
1+ x
Solución:
1 1 −1 ⋅ (1 + x ) − (1 − x ) 1+ x −2 log e
f ′(x ) = ⋅ ⋅ ⋅ log e = ⋅ ⋅ log e = − .
1− x 1− x (1 + x ) 2 2 ⋅ (1 − x ) (1 + x )2 1− x 2
2
1+ x 1+ x

1+ x 2
3) f (x ) = L
1− x 2

4-10
Derivabilidad Tema 4

Solución: f ′(x ) = ⋅
(
1 − x 2 2x ⋅ 1 − x 2 + 2x ⋅ 1 + x 2
=
4x) =
4x( .
)
1+ x 2
1− x 2
2
(
2
1− x ⋅ 1+ x 2
) 1− x 4 ( )( )
ln x
4) f (x ) =
2x
1 x 1
⋅ 2 − ln x ⋅ 2 x ln 2 − ln x ⋅ ln 2
x x 1 − x ⋅ ln x ⋅ ln 2
Solución: f ′(x ) = = = .
2 x 2
( )
2 x
x ⋅ 2x

5) Calcula la derivada enésima de f (x ) = e 2 x − e −2 x


( ) (
Solución: f ′(x ) = 2 ⋅ e 2 x + e −2 x , f ′′(x ) = 2 2 ⋅ e 2 x − e −2 x , f ′′′(x ) = 2 3 ⋅ e 2 x + e −2 x ) ( ) con lo que
se observa que f (n ) (x ) = 2 n ⋅ e 2 x + (− 1)n +1 e −2 x . ( )
R.3. Funciones trigonométricas.

(
1) f ( x ) = 1 + cos 2 (2 x ) ) 2

Solución:
( )
f ′( x ) = 2 ⋅ 1 + cos 2 (2 x ) ⋅ 2 cos (2 x ) ⋅ (− sen (2 x )) ⋅ 2 = −4 ⋅ 1 + cos 2 (2 x ) ⋅ 2sen (2 x ) cos (2 x ) =( )
( )
= −4 ⋅ 1 + cos (2 x ) ⋅ sen (4 x ) .
2

2) f (x ) = sen 3 x
1 3 cos 3 x
Solución: f ′(x ) = 3 cos 3 x = .
2 sen 3 x 2 sen 3 x

1 − senx
3) f ( x ) =
1 + senx
− cos x ⋅ (1 + senx ) − cos x ⋅ (1 − senx ) − 2 cos x
Solución: f ′( x ) = = .
(1 + senx ) 2
(1 + senx )2

4) f ( x ) = sen 2 x ⋅ tgx
1
Solución: f ′( x ) = 2 ⋅ senx ⋅ cos x ⋅ tgx + sen 2 x ⋅ = 2 ⋅ sen 2 x + tg 2 x .
cos 2 x

5) f ( x ) = tgx ⋅ sec x
1
(
Solución: f ′( x ) = 1 + tg 2 x ⋅ sec x + tgx ⋅ ) 2 ⋅ sec x
⋅ tgx ⋅ sec x =

 1   3 
= sec x ⋅ 1 + tg 2 x + ⋅ tg 2 x  = sec x ⋅ 1 + ⋅ tg 2 x  .
 2   2 

R.4. Funciones recíprocas de las trigonométricas.

(
1) f ( x ) = arcsen e x − 1 )
Solución: f ′( x ) =
1
⋅ ex =
ex
=
(e ) x 2
=
ex
.
1− e − 1( x
)
2
( )
− e x 2
+ 2e x 2e − (e )
x x 2 2 − ex

(
2) f ( x ) = arccos 1 + e x )

4-11
Derivabilidad Tema 4

Solución: Aunque nada nos impide aplicar las reglas de derivación, es absurdo hacerlo en este
caso, ya que la función dada no existe para ningún valor de x real y en consecuencia tampoco
existe f ′(x ) . Si de todos modos decidimos aplicar las reglas de derivación se llega a
ex ex
f ′( x ) = − =− donde el radicando es siempre negativo lo que nos
1− 1+ e ( )
x 2 − e 2 x − 2e x
vuelve a confirmar lo dicho anteriormente.

3) f ( x ) = arctg (1 + cos x )
−sen x
Solución: f ′( x ) = .
1 + (1 + cos x )2

x +1
4) f ( x ) = arc cot
x −1
1 1 x − 1 − (x + 1) 1 1
Solución: f ′(x ) = − 2
⋅ ⋅ 2
= ⋅ =
 x +1 2 x +1 (x − 1) 2x x +1
1+   ⋅ (x − 1)2
 x −1 x −1 x −1 x −1
 
1 1 1
= = = .
x +1 2 x (x + 1) ⋅ (x − 1) 2 x x 2 − 1
2x ⋅ (x − 1)
x −1

R.5. Potencias de funciones y técnica de derivación logarítmica.

1) Deriva la función f ( x ) = (sen x )2 x +1


a) Aplicando la regla de derivación.
b) Utilizando la técnica de derivación logarítmica.

Solución: a) f ′( x ) = (sen x )2 x +1 ⋅ ln (senx ) ⋅ 2 + (2 x + 1)(sen x )2 x cos x

b) Se toman logaritmos ln[f ( x )] = (2 x + 1) ⋅ ln (sen x ) . 2. Se derivan los dos miembros


f ′( x ) cos x
= 2 ⋅ ln (sen x ) + (2 x + 1) ⋅ y se despeja la derivada:
f( x) sen x
 cos x 
f ′( x ) = (sen x )(2 x +1) 2 ⋅ ln (sen x ) + (2 x + 1) ⋅  . Ambas expresiones coinciden como se puede
 sen x
comprobar con facilidad.

2) Deriva la función f ( x ) = 5
(x + 1)2
x −1
a) Aplicando las reglas de derivación correspondientes.
b) Utilizando la técnica de derivación logarítmica.
c) Comprueba que se llega al mismo resultado.

Solución
1 2(x + 1) ⋅ (x − 1) − (x + 1)2 1 1 (x + 1) ⋅ (x − 3) =
a) f ′( x ) = ⋅ 2
=
 (x + 1)2 
4 (x − 1) 5 (x + 1)3 (x − 1)2
5⋅5   (x + 1) ⋅ 5

 x − 1 
(x − 1)4
=
1 (x − 3 ) , o bien f ′( x ) =
(x − 3 ) .
5 (x − 1) ⋅ (x + 1)3 (x − 1)
5
5⋅ 5
(x + 1)3 (x − 1)6

4-12
Derivabilidad Tema 4

b) Tomamos logaritmos y aplicamos las propiedades de las operaciones con logaritmos


1 f ′( x ) 1  2 1 
ln[f ( x )] = [2 ⋅ ln (x + 1) − ln (x − 1)] . Derivamos =  − . Despejamos f ′( x ) .
5 f ( x ) 5  x + 1 x − 1

f ′( x ) = 5
(x + 1)2

1 2

1 
.
x − 1 5  x + 1 x − 1
c) Comprobamos que esta expresión es equivalente a la obtenida anteriormente:
1 5 (x + 1)2
 x −3 
Sumamos f ′( x ) = ⋅
⋅  . Introducimos (x + 1) ⋅ (x − 1) dentro de la raíz,
5  (x + 1) ⋅ (x − 1) 
x −1

finalmente simplificando se tiene f ′( x ) =


(x − 3 ) .
5 ⋅ (x + 1)3 (x − 1)6
5

Definición e interpretación geométrica de la derivada

3 x 2 + 3 si x ≤1
R.6. Calcula f ′(x ) siendo f (x ) =  .
 6x si x >1

Solución: Utilizaremos las reglas de derivación para calcular f ′(x ) con x ≠ 1:


6 x si x < 1
f ′(x ) =  .
 6 si x > 1
Para f ′(1) calculamos las derivadas laterales a partir de la definición:

f (1 + h ) − f (1) 3(1 + h ) + 3 − 6
2
3h 2 + 6h
f−′ (1) = lim = lim = lim = lim (3h + 6 ) = 6
h →0 − h h →0 − h h →0 − h h →0 −
f (1 + h ) − f (1) 6(1 + h ) − 6
f+′ (1) = lim+ = lim+ = lim+ 6 = 6 como las derivadas laterales coinciden
h →0 h h →0 h h →0
f ′(1) = 6
Nota: Como f es continua en x = 1, se pueden utilizar las expresiones de f ′(x ) para calcular
las derivadas laterales y así afirmar que f−′ (1) = 6 ⋅ 1 = 6 y f+′ (1) = 6 . Pero, insistimos, esto
sólo es válido cuando la función es continua en el punto donde pretendemos calcular la
derivada. Como se verá en el siguiente ejercicio, ya no es válido en el caso de no haber
continuidad.

3 x 2 + 2 si x ≤1
R.7. Calcula f ′(x ) siendo f (x ) =  .
 6 x + 1 si x >1
Solución: Actuaremos como en el caso anterior para calcular f ′(x ) con x ≠ 1 y f ′(1) .
6 x si x <1
f ′(x ) = 
 6 si x >1
f (1 + h ) − f (1) 3(1 + h ) + 2 − 5
2
3h 2 + 6h
f−′ (1) = lim = lim = lim = lim (3h + 6 ) = 6
h →0 − h h →0 − h h →0 − h h →0 −
f (1 + h ) − f (1) 6(1 + h ) + 1 − 5 6h + 2
f+′ (1) = lim+ = lim+ = lim+ = +∞ ∉ ℜ . No hay derivada por la
h →0 h h →0 h h →0 h
derecha y por lo tanto no existe f ′(1) .

Obsérvese que de haber sustituido en las expresiones de f ′(x ) a izquierda y a derecha


de 1 hubiéramos obtenido bien f−′ (1) pero de ninguna manera f+′ (1) , de hecho la función no
puede ser derivable en x = 1 ya que no es continua en ese punto, como se puede comprobar.

4-13
Derivabilidad Tema 4

En ejercicios posteriores veremos que puede utilizarse la expresión de f ′(x ) en


funciones definidas a trozos para calcular cómodamente su derivada en el punto en el que
cambia la definición, pero para ello habrá que estudiar previamente su continuidad.

R.8. Calcula las ecuaciones de las rectas tangente y normal a la gráfica de la función
2
f (x ) = e x −1
en x = 1.

Solución: Como la pendiente de la recta tangente a una curva en un punto coincide con la
derivada de la función en ese punto, para hallar la recta tangente utilizaremos la ecuación de la
recta en forma punto-pendiente: y − f (x0 ) = f ' (x0 ) ⋅ (x − x0 ) . Por lo tanto, la ecuación de la recta
tangente en x = 1 es y − f (1) = f ′(1) ⋅ (x − 1) .
2
Calculamos f ′(x ) y de ahí f ′(1) , f ′(x ) = 2 x ⋅ e x −1
⇒ f ′(1) = 2 que es la pendiente, por otra parte
f (1) = 1 . Luego la recta tangente tiene de ecuación: y − 1 = 2 ⋅ (x − 1) ⇔ y = 2 x − 1 .
La recta normal es la perpendicular a la recta tangente en su punto de tangencia y, por
lo tanto, su pendiente es la opuesta de la inversa de la pendiente de la recta tangente, su
1 1 3
ecuación es: y − f (1) = − ⋅ (x − 1) ⇒ y = − x + .
f (1)
′ 2 2

R.9. Calcula el punto en el que la recta tangente a y = x 2 + 2 forma un ángulo de 30º


con la horizontal.

Solución: La pendiente de una recta es la tangente trigonométrica de su inclinación, así se trata


3 2x
de localizar el punto en el que f ′( x ) = tg 30º = . Como f ′( x ) = buscamos las
3 2 x2 + 2
x 3 x2 1
soluciones de = , elevando al cuadrado 2
= ⇒ 3x 2 = x 2 + 2 , y
x +2 2 3 x +2 3
despejando se llega a x = 1 y x = −1 . Como se ha elevado al cuadrado hay que comprobar las
soluciones porque alguna de ellas puede ser falsa:
1 3 −1 3
si x = 1 ⇒ = correcta; para x = −1 ⇒ ≠ solución falsa, que se ha
2
1 +2 3 (− 1) + 2
2 3
introducido debido al método de resolución y por lo tanto la única solución es x = 1,
sustituyendo ese valor en f (x ) se llega al punto P 1, 3 . ( )
Continuidad y derivabilidad.

 2−4 si x≤5
R.10. Estudia dónde es continua y calcula la derivada de f (x ) =  x
 2x + 7 si x>5
− a si a ≤ 0
Solución: Lo primero es definir de un modo claro x 2 − 4 , recordando que a =  y
 a si a ≥ 0
como x 2 − 4 > 0 cuando x ∈ (− ∞,−2) U (2, ∞ ) y x 2 − 4 < 0 para x ∈ (− 2,2) y se tiene que
 x2 − 4 si x ≤ −2
 2
− x + 4 si −2< x<2
f (x ) =  2
 x −4 si 2< x ≤5
 2 x + 7 si x>5
Continuidad: Los únicos puntos posibles de discontinuidad son x = −2 , x = 2 y x = 5 ,
estudiamos cada uno de esos puntos:

x → −2
( ) x → −2
( )
x = −2 ⇒ f (− 2) = lim + − x 2 + 4 = lim − x 2 − 4 = 0 continua en ese punto.

4-14
Derivabilidad Tema 4

x→2
( ) x →2
( )
x = 2 ⇒ f (2) = lim− − x 2 + 4 = lim+ x 2 − 4 = 0 continua en ese punto.

x = 5 ⇒ f (5 ) = lim (x 2
)
− 4 = 21 , lim+ (2 x + 7 ) = 17 discontinuidad de salto finito.
x →5 − x →5
Derivabilidad:
 2 x si x < −2
− 2 x si − 2 < x < 2

f ′(x ) =  . Calculamos las derivadas en x = −2 , x = 2 y x = 5
 2 x si 2 < x < 5
 2 si x>5

Como se dijo en los ejercicios R.6 y R.7, en los puntos donde la función es continua no
es obligado recurrir a la definición de derivada.

Por lo tanto:

Para x = −2 ⇒ como la función es continua, hacemos f−′ (− 2) = 2 ⋅ (− 2 ) = −4 y


f+′ (− 2) = −2 ⋅ (− 2) = 4 . No es derivable, presenta un punto anguloso.
Para x = 2 ⇒ la función es continua y se tiene: f−′ (2) = −2 ⋅ 2 = −4 y f+′ (2) = 2 ⋅ 2 = 4 .
No es derivable, presenta un punto anguloso.
En x = 5 la función no es derivable porque no es continua.

 sen 2 x si x≤0
R.11. Estudia dónde es continua y calcula la derivada de f (x ) =  2
3 x + 2 x si x>0
Solución: Continuidad: el único punto posible de discontinuidad es x = 0

( )
f (0 ) = lim− (sen 2 x ) = lim+ 3 x 2 + 2 x = 0 es continua.
x →0 x →0

Derivabilidad:
2 cos 2 x si x < 0
f ′(x ) =  calculamos ahora f ′(0 ) .
 6 x + 2 si x > 0
Como la función es continua en x = 0 se puede asegurar que f−′ (0 ) = 2 ⋅ cos 0 = 2 y
f+′ (0 ) = 6 ⋅ 0 + 2 = 2 . Por tanto, como las derivadas laterales coinciden, f ′(0 ) = 2 .

2 x 2 x 2 − 1 si x <1

R.12. Estudia dónde es continua y calcula la derivada de f (x ) =  4
 3 − x si x ≥1

 2 2 
Solución: Continuidad: lo es si 2 x 2 − 1 < 0 y x < 1 ; es decir, para x ∈  − , no hay valor
 2 2 
 
de la función y evidentemente ésta es discontinua.

En x = 3 la función presenta una discontinuidad de salto infinito.


 4 
En x = 1 se tiene: f (1) = lim−  2 x 2 x 2 − 1  = lim+   = 2 la función es continua.
x →1   x →1  3 − x 
Derivabilidad:
En los puntos donde la función es discontinua no hay derivada. En lo que sigue no
 2 2 
consideramos los valores x ∈  − , , ni tampoco x = 3 .
 2 2 
 

4-15
Derivabilidad Tema 4

 8x 2 − 2
 si x <1
 2x 2 − 1
f ′(x ) =  En x = 1 puesto que se sabe que la función es continua
 4
si x >1
 (3 − x )2

 8x 2 − 2   4 
f−′ (1) = lim−   = 6 y f+′ (1) = lim  = 1 , no es derivable y la función tiene un
x →1  2 x 2 − 1  x →1  (3 − x ) 
+ 2
   
punto anguloso.
2 2
En x = − y x= sólo se podría hablar de derivadas laterales y como es
2 2
 2   8x 2 − 2 
continua f−′  − = lim −   = ∞ la función no es derivable. Del mismo modo
 2   2 
  x→− 2  2 x − 1 
2

 2   8x 2 − 2 
f+′  = lim +  =∞, por lo que tampoco es derivable. Resumiendo:
 2  
2  2x 2 − 1 

 x→
2

 8x 2 − 2
 si x <1
 2x 2 − 1  2 2
f ′(x ) =  excepto x ∈  − , .
 4 x > 1  2 2 

 (3 − x )2
si 
 x ≠ 3

R.13. Estudia la continuidad y la derivabilidad de las siguientes funciones.


a) f (x ) = x ⋅ ln(x − 1) b) g (x ) = x − 1 en x = 1.

Solución: a) Lo primero es definir la función de forma clara.

Si se recuerda el dominio de la función logarítmica se tiene que el dominio de f (x ) son las


soluciones de x − 1 > 0 ⇒ x > 1 , por otra parte y teniendo en cuenta la definición de valor
absoluto se tiene que ln(x − 1) = ln(x − 1) cuando ln(x − 1) > 0 ⇒ x − 1 > 1 ⇒ x > 2 y
ln(x − 1) = − ln(x − 1) si ln(x − 1) < 0 ⇒ x < 2 , por último ln(x − 1) = 0 si x = 2 valor que encaja
en las dos expresiones anteriores, por tanto.

− x ⋅ ln(x − 1) si 1 < x < 2


f (x ) =  La función es continua en todo su dominio excepto quizás en
 x ⋅ ln(x − 1) si x≥2
x = 2 por ser producto de dos funciones continuas.

En x = 2 se tiene: f (2 ) = lim (− x ⋅ ln(x − 1)) = lim (x ⋅ ln(x − 1)) = 0 luego es continua.


x →2− x →2+

 x
− ln(x − 1) − x − 1 si 1 < x < 2
f ′(x ) =  . Calculamos f ′(2) .
x
 ln(x − 1) + si x>2
 x −1

 x   x 
f−′ (2) = lim−  − ln(x − 1) −  = −2 y f+′ (2) = lim+  ln(x − 1) +  = 2 . En x = 2 hay un punto
x →2  x − 1 x →2  x − 1
anguloso, en todos los demás valores del dominio la función es continua y derivable.

 1 − x si x <1
b) Definimos g (x ) = x − 1 de forma más manejable g (x ) =  .
 x − 1 si x ≥1

4-16
Derivabilidad Tema 4

Continuidad en x = 1 ⇒ g (1) = lim


x →1−
( 1− x ) = lim (
x →1+
)
x − 1 = 0 . Continua.

 1
− si x <1
Calculamos g′(1) ⇒ g ′(x ) =  2 1− x como es una función continua
1
 si x >1
 2 x −1
 1   1 
g −′ (1) = lim  −  = − ∞ y g +′ (1) = lim   = + ∞ . La función tiene un punto de
x →1−  2 1 − x  x →1+
 2 x −1
retroceso en x = 1.

Continuidad y derivabilidad para funciones dependientes de parámetros.

 3x - 2 si x ≤1
 2
R.14. Calcula los valores de a y b para que la función f (x ) = ax + bx si 1 < x < 2
 − x + 5 si x≥2

sea continua en x = 1 y x = 2 . Para los valores calculados ¿es derivable en dichos
puntos?.

Solución: Para que f (x ) sea continua en x =1 f (1) = lim− f (x ) = lim+ f (x ) y como


x →1 x →1

f (1) = lim (3 x − 2) = 1 se debe cumplir que


x →1−
( )
lim ax 2 + bx = 1 luego a + b = 1 .
x →1+

( )
Para que f (x ) sea continua en x = 2 f (2 ) = lim− ax 2 + bx = lim+ (- x + 5 ) , de donde
x →2 x →2

 1
a + b = 1  a = 2.
4a + 2b = 3 , así que resolvemos el sistema ⇒ 1
4a + 2b = 3 b =
 2
 3x − 2 si x ≤1
1 2 1
La función, con esos valores, queda f (x ) =  x + x si 1 < x < 2 , la función derivada para
2 2
 −x+5 si x≥2
todos los valores x excepto para x = 1 y x = 2 la obtenemos con las reglas de derivación,
 3 si x <1
 1
f ′(x ) =  x + si 1 < x < 2 . Calculamos ahora f ′(1) y f ′(2) .
 2
 − 1 si x>2
 1 3
f ′(1) : como la función es continua f−′ (1) = lim (3 ) = 3 y f+′ (1) = lim+  x +  = como las
x →1 −
x →1  2 2
derivadas laterales no coinciden no hay f ′(1) . Se trata de un punto anguloso.
 1 5
f ′(2) : como la función es continua f−′ (2) = lim−  x +  = y f+′ (2) = lim (− 1) = −1 que tampoco
x →2  2 2 x →2+

coinciden, no hay f ′(2) y en x = 2 hay también un punto anguloso.

R.15. Estudia según los valores de a y b la continuidad y la derivabilidad de


 x 2 + ax + b si x ≤ 0
f (x ) =  .
 3x + 1 si x > 0
Solución:

Para todos los x ≠ 0 , f (x ) es continua y derivable independientemente del valor de a y b ya


que están definidas mediante expresiones polinómicas.

4-17
Derivabilidad Tema 4

Para que f (x ) sea continua en x = 0 ( )


f (0 ) = lim− x 2 + ax + b = b = lim+ (3 x + 1) = 1
x →0 x →0
luego si b = 1 la función es continua.
2 x + a si x < 0
Calculamos f ′(x ) ⇒ f ′(x ) =  , para el cálculo de f ′(0 ) y considerando que la
 3 si x > 0
f−′ (0 ) = lim− (2 x + a ) = a
por lo que para que exista f ′(0 ) , a = 3 . En
x →0
función es continua se tiene
f+′ (0 ) = lim+ (3 ) = 3
x →0
resumen:
Para x = 0
si b ≠ 1 ⇒ f (x ) es discontinua de salto finito en x = 0 independientemente del valor de a.
si b = 1 y a = 3 ⇒ f (x ) es continua y derivable
si b = 1 y a ≠ 3 ⇒ f (x ) tiene un punto anguloso .
∀ x ≠ 0 ⇒ f (x ) es continua y derivable independientemente de los valores de a y b.

 3 x 2 + k si x≤2
R.16. Calcula el valor de k para que f (x ) =  sea derivable en x = 2 .
12x − 14 si x>2

Solución: Para que ∃ f ′(2) la función tiene que ser continua en x = 2 , es decir:
( )
f (2 ) = lim− 3 x 2 + k = lim+ (12 x − 14 ) ⇒ 12 + k = 10 ⇒ k = −2 . Si para este valor de k la función
x →2 x →2
no fuese derivable en x = 2 el ejercicio no tendría solución. Calculamos f ′(2)
6 x si x < 2
f ′(x ) =  y lim (6 x ) = lim+ 12 = f ′(2) = 12 , luego la función es derivable en
12 si x > 2 x →2− x →2

x = 2 si k = −2 .

 x 2 + mx + 1 si x ≤1
 2m
R.17. Dada la función f (x ) =  si 1 < x < 2 calcula los valores de m y n que
 x
n − ln(x − 1) si x≥2
hacen que la función sea continua, para esos valores calcula f ′(x ) .

Solución:

Los únicos puntos donde la función puede ser discontinua son x = 1 y x = 2 , exigimos la
continuidad en dichos puntos.
 2m 
x →1
( )
Continuidad en x = 1 ⇒ f (1) = lim− x 2 + mx + 1 = m + 2 y lim+ 
x →1  x 
 = 2m luego m = 2 .

 2m  4
Continuidad en x = 2 ⇒ f (2) = lim+ (n − ln(x − 1)) = n y lim−   = lim−   = 2
x →2 x →2  x  x →2  x 
 x 2 + 2x + 1 si x ≤1
 4
luego n = 2 . La función queda f (x ) =  si 1 < x < 2 y
 x
2 − ln(x − 1) si x≥2

 2x + 2 si x <1
 4
f ′(x ) =  − 2 si 1 < x < 2 calculamos las derivadas en x = 1 y en x = 2 .
 x
− 1 si x>2
 x − 1

4-18
Derivabilidad Tema 4

 4 
f−′ (1) = lim− (2 x + 2) = 4 , f+′ (1) = lim+  − 2  = −4 . Punto anguloso, no hay derivada.
x →1 x →1  x 

 4   1 
f−′ (2 ) = lim−  − 2  = −1 , f+′ (2) = lim+  −  = −1 . La función es derivable y f ′(2) = −1 .
x →2  x  x →2  x − 1


 1 + sen x si x≤0

R.18. Dada la función f (x ) = − 2 x + x + a si 0 < x < 2 calcula los valores de a y b para
2

 b
si x≥2
 x
que la función sea continua, para esos valores calcula f ′(0 ) y f ′(2) . Dibuja la gráfica de
la función.

Solución:

Los únicos puntos donde la función puede ser discontinua son x = 0 y x = 2 , exigimos la
continuidad en dichos puntos.
( )
En x = 0 ⇒ f (0 ) = lim− (1 + sen x ) = 1 y lim+ − 2 x 2 + x + a = a luego a = 1 .
x →0 x →0

b b
En x = 2 ⇒ f (2 ) = lim+   =
x →2  x  2
( )
y lim− − 2 x 2 + x + a = −5 ⇒ b = −10 .
x →2

Para calcular f ′(0 ) y f ′(2) conviene calcular f ′(x ) en


general .

 cos x si x<0

f ′(x ) = − 4 x + 1 si 0 < x < 2 como la función es
 10 si x>2
 x 2
continua se tiene:
f−′ (0 ) = lim− (cos x ) = 1 , f+′ (0 ) = lim+ (− 4 x + 1) = 1 .
x →0 x →0
La función es derivable con f ′(0 ) = 1 .
 10  10
f−′ (2 ) = lim− (− 4 x + 1) = −7 , f+′ (2) = lim+  2  = .
x →2 x →2  x  4
La función no es derivable, presenta un punto anguloso en x = 2 .

 2 x + b si x <1
R.19. Estudia la continuidad y la derivabilidad de f (x ) =  2 según los
ax + 1 si x ≥1
valores de a y b. Representa la función en estas dos situaciones:
a) a = −1 y b = −2 b) a = −1 y b = 0 .

Solución: Para todos los x ≠ 1, f (x ) es continua y derivable independientemente del valor de a


y b ya que está definida mediante expresiones polinómicas.
( )
Para que f (x ) sea continua en x = 1 lim+ ax 2 + 1 = f (1) = a + 1 = lim− (2 x + b ) = b + 2 luego si
x →1 x →1
a − b = 1 la función es continua.
 2 si x < 1
Calculamos f ′(x ) ⇒ f ′(x ) =  , calculamos de f ′(1) como la función es
2ax si x > 1
f−′ (1) = lim− (2 ) = 2
así que para que exista f ′(1) , a = 1 . Resumiendo:
x →1
continua se tiene
f+′ (1) = lim+ (2ax ) = 2a
x →1

si a − b ≠ 1 ⇒ f (x ) es discontinua de salto finito en x = 1.

4-19
Derivabilidad Tema 4

si a − b = 1 ⇒ f (x ) es continua en x = 1, en este caso se dan dos situaciones:


si a = 1 y b = 0 ⇒ f (x ) es continua y derivable
si a ≠ 1 ⇒ f (x ) tiene un punto anguloso .
f (x ) es continua y derivable ∀ x ≠ 1 independientemente de los valores de a y b.
Gráficas:
a) con los valores a = −1 y b = −2 como a − b = 1 la función es continua pero al ser
a ≠ 1 tendrá un punto anguloso como puede verse.
b) si a = −1 y b = 0 como a − b ≠ 1 la función es discontinua de salto finito.


 x 2 + ax + b si x ≤ 0

R.20. Calcula los valores de a y b sabiendo que la función f (x ) = 
( )
 2 ⋅ ex − 1
 si x > 0
 x
es derivable en x = 0 .
Solución: Para que la función sea derivable en x = 0 , ha de ser continua en dicho punto así
f (0 ) = lim− f (x ) = lim+ f (x ) ( )
f (0 ) = lim− x 2 + ax + b = b
que x →0 x →0 , como x →0 y

lim+ 
( )
 2⋅ e −1  0
x L' H
 = [in det .] = lim 2 ⋅ e = 2
x

 x  0 x →0 + 1
x →0
  necesariamente b = 2 . Para ese valor de b
exigimos que la función sea derivable en x = 0 . Calculamos f ′(x ) para x ≠ 0

 2x + a si x < 0

f ′(x ) =  y como la función es continua f−′ (0 ) = lim− (2 x + a ) = a y
 e x (x − 1) + 1 x →0
2 ⋅ si x > 0
 x2
 e x (x − 1) + 1  0 L' H x
f+′ (0 ) = lim+  2 ⋅  = [in det .] = lim e ⋅ x = 0 [in det .] = lim e x = 1 y en
x →0  x2  0 x →0 + x 0 x →0 +
 
a = 1
consecuencia a = 1, en conclusión  .
b = 2

Aplicación a casos prácticos de la regla de la cadena.

R.21. Un el recipiente como el de la figura


se está llenando con un grifo de caudal
constante de 12 l/min.

4-20
Derivabilidad Tema 4

Se quiere saber con qué velocidad asciende el nivel del agua en el instante en el que la
altura es de 20 cm .

Solución:
Debemos buscar la función que nos da el volumen de agua dependiendo de la altura. La figura
que forma la parte del recipiente con agua es un prisma recto de base trapezoidal variable (en
las caras laterales) y de altura constante 15 dm . Su volumen es V = Sb ⋅ 15 dm 3 .
Establecemos la base del prisma de agua en un
instante cualquiera en el que la altura del agua es h = AB
si llamamos x = BE , como CD = 1' 5 dm y CA = 6 dm , por
semejanza de los triángulos ABE y ACD de la figura se
BE CD x 1' 5 1' 5h h
tiene que = ⇒ = luego x = = y el
BA CA h 6 6 4
área del trapecio que marca la altura del agua es
 h
4 + 4 + 
Sb =
(
4 + 4 + 2x ) h=  2
h=
h2
+ 4h con lo que el
2 2 4
 h2 
volumen del prisma de agua dependiendo sólo de la altura es V (h ) = 15 + 4h  . V es función
 4 
 
de h que es función del tiempo, es decir V = V (h (t )) . Aplicando la regla de derivación de una
dV dV dh dV dV h 
función de función se tiene que = ⋅ (1). = 12 dm 3 / min , = 15 + 4  . Si
dt dh dt dt dh 2 
dV 2 
h = 2 dm = 15 + 4  = 75 dm 2 , así que despejando de (1)
dh  2 
dV
dh 12 dh
= dt = = 0'16 dm / min , o si se prefiere = 1'6 cm / min .
dt dV 75 dt
dh

R.22. Una pompa de jabón se está formando introduciendo un caudal de aire de


12 cm 3 / s . ¿Con qué velocidad aumenta el radio de la pompa en el momento en el que
éste es de 3 cm ?.
4π 3
Solución: El volumen de una esfera de radio r es de una esfera V = r V es función de el
3
radio que a su vez es función de t V = V (r (t )) y aplicando la regla de derivación de una función
dV dV dr dV dr
de función se tiene que = ⋅ ⇒ = 4πr 2 ⋅ si sustituimos r por 3 y despejamos se
dt dr dt dt dt
dV
dr 12 1
tiene = 4πr 2 ⋅ = dt = = ≈ 0' 1 cm / s .
dt 4πr 2 36 π 3π

Ejercicios propuestos
Cálculo de derivadas

P.1. Calcula las funciones derivadas de las funciones siguientes:

2 1 3
( )
1) f ( x ) = ( 2 x + 1) ⋅ x 2 + 1 (
2) f ( x ) = x 2 + 3 )
−4
3) f ( x ) = x 3 +x 6 −x 4

4-21
Derivabilidad Tema 4

4) f ( x ) =
x5
5) f ( x ) =
x2 + 4
6) f ( x ) =
(1 − x )3
(1 − x )5 x2 + 3x (1 + x )4
8) f ( x ) = 3 (1 − x )
2
7) f ( x ) = x 2 + 3 x 9) f ( x ) = x ⋅ 3 x 2 − 1
1− x x
10) f ( x ) = 11) f ( x ) = (1 − x ) ⋅ 1 + x 12) f ( x ) =
1+ x 1 − x2
Lx 1+ x
13) f ( x ) = 14) f ( x ) = ln 1 + x 15) f ( x ) = L
x 1− x
2−x 1− x
16) f ( x ) = loga (1 + 2x ) 17) f ( x ) = log 18) f ( x ) = L
2+ x 1+ x
Lx 2  1
19) f ( x ) = x
20) f ( x ) = 5 x −1
21) f ( x ) = e 2 x ln 
e x
3 + ex 2
22) f ( x ) = L x
23) f ( x ) = 3 x +6x
¿dónde es f’ (x) =0?
3−e
24) f ( x ) = 3 ⋅ sen3 x 25) f ( x ) = sen(7 x − b ) 26) f ( x ) = cos(7 − ax )
(
27) f ( x ) = cos3 3 x 2 + 5 x − 8 ) 28) f ( x ) = tg3 (5 x ) 29) f ( x ) = sen L(1 − 3 x )
2
1 + senx sec x
30) f ( x ) = ln 31) f ( x ) = 32) f ( x ) = ln senx
1 − senx 1 − tg2 x
33) f ( x ) = ln( senx ) 34) f ( x ) = cos ec 2 (1 − x ) (
35) f ( x ) = cot g3 e2 x + 8 )
x +1
36) f ( x ) = arcsen (2x 2 ) 37) f ( x ) = arcsen 38) f ( x ) = arc sec x
x −1
x2 − 1 1+ x
39) f ( x ) = arctg 2
x +1
40) f (x) = arcsen x 2 + 1 ( ) 41) f ( x ) = arctg
1− x
1 − x2 1+ ex 1 − cos x
42) f ( x ) = arcsen 2
43) f ( x ) = arctg x
44) f ( x ) = arctg
1+ x 1− e 1 + cos x
1 x
45) f ( x ) = x x 46) f ( x ) = x tgx
47) f ( x ) = (arctgx )

48) f ( x ) = x π
+ xe + ex + ( π) x

Derivada en un punto y función derivada

P.2. Calcula, si existen, las derivadas de las funciones siguientes en x = 0.


 2 1   1 
 sen  si x ≠ 0  x ⋅ sen  si x ≠ 0
a ) f(x) =  x x b) g(x) =  x
 0 si x = 0  0 si x = 0

P.3. Estudia la derivada de la función f (x ) = sen x en x = 0 .

P.4. Calcula la función derivada de las siguientes funciones:

| x | arctan( x 2 − 1) si x < −1
 si x≠0 
f(x) =  x g(x) =  x4 − 1 si −1≤ x ≤ 0
 0 si x=0  2
 x cosx si x>0

P.5. Calcula las derivadas enésimas de:


1 5 x−3 ex − e− x 7−x
a) y = b) y = c) y = 3
d) y = e) y = 2
x x−3 x −x 2 x +x−2

4-22
Derivabilidad Tema 4

Interpretación geométrica de la derivada

P.6. Obtén las ecuaciones de las rectas tangente y normal, en el punto (2,1), a la curva:
y = (x − 1) ⋅ 3 (3 − x )

P.7. ¿En qué puntos de la curva y = x 2 + 8 la tangente forma un ángulo de 135° con la parte
positiva del eje OX?

Continuidad y derivabilidad

P.8. Estudia dónde son continuas y dónde derivables las funciones siguientes:
 1 
 3 si x < −2  x ⋅ cos   si x<0
  x
f ( x ) = x + 5 si − 2 ≤ x ≤ 2 g( x ) =  x si 0 ≤ x < 1
 x 2 − 3 x si x>2  x 2
si x ≥1
 

ln( x − 1) si 1 < x ≤ 2

P.9. Estudia la continuidad y derivabilidad de la función f ( x ) =  x 2 − 4 si 2 < x ≤ 4
 − x + 1 si 4<x

− x
P.10. Se considera la función f : R → R definida por: f (x) = x ⋅ e ∀x ∈R . Se pide: 1)
Estudia si es continua y derivable en 0 y, en su caso, halla f ’(0); 2) Estudia si es continua y
derivable en todo R.

P.11. Determina los puntos angulosos de:


 (2 x − 5 ) ⋅ x 6 ⋅ cos 5 x si x ≥ 0
7
a) f ( x ) = 
( ) ( )
ln x 2 + 1 ⋅ arctan x 2 + 1 si x < 0
( )
b) f ( x ) = x 2 + x + 1
x

2 2
c) f (x) = x ln x d) f (x) = x ⋅ ln x
ln( e + senx ) si x < 0
P.12. Indica los valores que han de tener a y b para que la función f ( x ) =  3
 x + ax + b si x ≥ 0
sea derivable en x = 0 .

Ejercicios de profundización
3
P.13. Sobre un montón cónico de arena está cayendo arena a razón de 10 dm por minuto. El
radio de la base siempre es la mitad de la altura. ¿ A qué velocidad se está incrementando la
altura del montón cuando su altura es de 0'5m?.

P.14. Si el lado de un cuadrado se aumenta a velocidad constante, ¿cómo varía su área?. Estudia
un caso particular.

P.15. Una escalera de mano de 4m de longitud se desliza apoyada en una pared de modo que el
extremo del suelo se mueve a velocidad constante de 0'75m/s. ¿ A qué velocidad se mueve el
extremo apoyado en la pared en el momento en que la distancia del otro extremo a la pared es de
2m?.

P.16. Un globo está a 200 m sobre el suelo y se eleva a razón de 15 mIs. Un automóvil pasa
bajo el globo con velocidad de 45 Km/h. ¿Con qué velocidad se separan el coche y el globo un
segundo después?

4-23
Derivabilidad Tema 4

P.17. Halla el ángulo que forman las rectas tangentes a las curvas de ecuaciones:
x ⋅ y = 1, x 2 − 2y 2 = 1 en los puntos de intersección de dichas curvas.

Soluciones:

P.1.
2 − 13 1 − 5 6 3 − 14
1) f ′( x ) = 6 x 2 + 2x + 2 2) f ′( x ) = −8( x 2 + 3)−5 x 3) f ′( x ) = x + x − x
3 6 4

4) f ′( x ) =
5x 4
5) f ′( x ) =
3 x 2 − 8 x − 12
6) f ′( x) =
(1 − x )2 ⋅ (x − 7 )
(1 − x )6 ( x 2 + 3 x )2 (1 + x )5
2x + 3 −2 6x2 − 1
7) f ′( x ) = 8) f ′( x ) = 9) f ′( x ) =
2 x2 + 3x 33 (1 − x ) 3x2 − 1
−1 3x + 1 1
10) f ′( x ) = 11) f ′( x ) = − 12) f ′( x ) =
(1 − x ) ⋅ (1 + x )3 2 1+ x (1 − x )2 3

1 − Lx 1 2
13) f ′( x ) = 2
14) f ′( x ) = 15) f ′( x ) =
x 2x + 2 1 − x2
2 loga e −4 log e 1
16) f ′( x ) = 17) f ′( x ) = 18) f ′( x ) =
1 + 2x 4 − x2 x ⋅ (x − 1)
1 − xLx 2  1 1
19) f ′( x ) = x
20) f ′( x ) = 5 x −1
⋅ 2x ln 5 21) f ( x ) = e2 x  2 ln − 
x⋅e  x x
3e x 2
22) f ′( x ) = 2x
23) f ′( x ) = 3 x +6x
⋅ (2x + 6 ) ⋅ L3 , en x =-3
9−e
24) f ′( x ) = 9 ⋅ sen2 x ⋅ cos x 25) f ′( x ) = 7 cos(7 x − b ) 26) f ′( x ) = a ⋅ sen( 7 − ax )
2
( 2
) ( 2
)
27) f ( x ) = −3 ⋅ ( 6 x + 5 ) ⋅ cos 3 x + 5 x − 8 ⋅ sen 3 x + 5 x − 8 28) f ′( x ) = 15tg2 (5 x ) 1 + tg2 (5 x ) ( )
29) f ′( x ) =
− 3 cos L(1 − 3 x )
30) f ′( x ) =
2
31) f ′( x ) =
4tgx ⋅ 1 + tg 2 x
=
(
4senx cos x )
(2 − 6 x ) L(1 − 3 x ) cos x 2 2
1 − tg x ( )
1 − 2sen 2 x
2
( )
cot gx cos x 2 cos(1 − x )
32) f ′( x ) = 33) f ′( x ) = 34) f ′( x ) =
2 2senx ⋅ ln( senx ) sen3 (1 − x )
4x
( )(
35) f ′( x ) = −6 cot g 2 e 2 x + 8 ⋅ 1 + cot g 2 e 2 x + 8 ⋅ e 2 x ( )) 36) f ′( x ) =
1 − 4x 4
2 1 1
37) f ′( x ) = − 38) f ′( x ) = 39) f ′( x ) =
(x − 1) ⋅ − 4x 2x ⋅ x − 1 x ⋅ x4 − 1
2x 1 −2
40) f ′( x ) = 41) f ′( x ) = 42) f ′( x ) =
1+ x2 1+ x2
(
1− x + 1 2
)
2

ex senx 1
x 1 − ln x 
43) f ′( x ) = 2x
44) f ′( x ) = 2
45) f ′( x ) = x ⋅ 2 
1+ e 1 + cos x  x 
 tgx   1 
(
46) f ′( x ) = x tgx ⋅  1 + tg2 x ⋅ Lx + )
x 
47) f ′( x ) = (arctgx )x ⋅  L(arctgx ) +
x
⋅ 
1 + x arctgx 
2
 
48) f ′( x ) = π x π −1
+ ex e −1 + e x + ( π) Lx
π

4-24
Derivabilidad Tema 4

P.2. a) 0, b) no es derivable. P.3. Hay un punto anguloso en x = 0. P.4. f ′( x ) = 0,si x ≠ 0, no


 2x
 si x < −1


1 + x 2
− 1
2
( )
existe derivada en x = 0, g ′( x ) =  4x 3 si − 1 < x < 0 , no existe derivada en
2 x cos x − x 2 sen x si x>0



( −1) n ⋅ n! ( −1)n ⋅ 5 ⋅ n! 3 ⋅ ( −1)n ⋅ n! ( −1)n +1 n! 2 ⋅ ( −1)n +1 n!


x = 0 y x = −1 . P.5. a) , b) ,c) + + , d)
x n +1 ( x − 3)n +1 x n +1 ( x − 1)n +1 ( x + 1)n +1
e x − e −x e x + e −x ( −1)n ⋅ 2 ⋅ n! ( −1)n +1 ⋅ 3 ⋅ n!
y (n ) = con n par e y (n ) = con n impar , e) + .
2 2 ( x − 1)n +1 ( x + 2 )n +1
2 3
P.6. t : y − 1 = (x − 2) , n : y − 1 = − (x − 2) . P.7. x = -1/2. P.8. f no derivable en x = 2 no
3 2
es continua y x = −2 punto anguloso, g no es derivable en x = 0 ni en x = 1 . P.9 En x = 4
discontinuidad de salto finito, en x = 2 continua pero no derivable, punto anguloso. P.10. 1)
f´(0) =1; 2) continua y derivable en todo R. P.11. a) y b) continuas y derivables, c) x = 1 , d) x = 1
1 dh 4 dV dA dl
y x = −1 . P.12. a = , b = 1 . P.13. 8/5π dm/min. = 2
⋅ . P.14. = 2⋅l⋅ = k ⋅l .
e dt π ⋅ h dt dt dt
3 dy x dx
P.15. − m/s =− ⋅ . P.16. 15´7 m/s. P.17 arctag 3.
4 dt 16 − x 2 dt

4-25
Propiedades de las funciones derivables Tema 5

Tema 5

Propiedades de las funciones derivables

5.1 Extremos relativos (Máximos y mínimos relativos)

Definiciones Diremos que f tiene un mínimo local o


relativo en x0 si existe un entorno de x0 de radio δ tal que
para todo x del entorno ( x 0 − δ, x 0 + δ ) se tiene que
f (x ) ≥ f (x 0 ) .

Diremos que f tiene un máximo local o relativo en


x0 si existe un entorno de x0 de radio δ tal que para todo x
del entorno ( x 0 − δ, x 0 + δ ) se tiene que f (x ) ≤ f (x 0 )

Nota A los máximos y mínimos relativos se les llama extremos relativos o puntos extremos de f.

Teorema Sea f definida en ( a , b ) , derivable en x 0 ∈ ( a , b ) . Si en x0 tiene f un máximo o

un mínimo, entonces f ′(x 0 ) = 0 .

Demostración.- Veamos la demostración para el caso en el que en x0 haya un máximo; para


el otro caso es análogo. Como en x0 hay un máximo local, existirá un entorno de x0 de radio δ
tal que para todo x del entorno ( x 0 − δ, x 0 + δ ) se tiene que f (x ) ≤ f (x 0 ) . Y, por lo tanto,

f (x 0 + h ) − f (x 0 ) ′ f (x 0 + h ) − f ( x 0 )
f + ′ (x 0 ) = lim+ ≤0 y f − (x 0 ) = lim− ≥0 . Pero como f es
h →0 h h →0 h

derivable en x0 las derivadas laterales coinciden, lo que sólo es posible si f + ′ (x 0 ) = f − ′ (x 0 ) = 0 .

Por lo que f ′(x 0 ) = 0 .

5-1
Propiedades de las funciones derivables Tema 5

Nota Si una función derivable en un punto tiene un extremo relativo en ese punto, la recta
tangente a la función en ese punto es horizontal (paralela al eje de la equis).

5.2 Teorema de Rolle


Teorema de Rolle Sea f una función continua en [ a , b ] y derivable en ( a , b ) tal que

f (a ) = f (b ) . Entonces, existe al menos un punto c ∈ ( a , b ) tal que f ′(c ) = 0 .

Demostración.- Demostraremos el teorema por reducción al absurdo. Supongamos que


f ′(x ) ≠ 0 para todo x de ( a , b ) .
Como f es continua en [ a , b ], por el Teorema de Bolzano-Weierstrass (Tema 2),
sabemos que alcanza su mínimo m y su máximo M en [ a , b ] .
Evidentemente, m y M no pueden ser alcanzados en un punto interior; ya que, por el
Teorema 1, f ′(x ) se anularía en ellos, pero la posibilidad de que f ′(x ) se anule en el interior ha
sido descartada. Luego m y M se alcanzan en los extremos de [ a , b ] .
Ahora bien, como f (a ) = f (b ) , entonces m = f (a ) = f (b ) = M . Esto es, los valores mínimo
y máximo de la función coinciden, por lo que f sólo puede ser una función constante en [ a , b ] ,
lo que contradice que f ′(x ) no se anula en ningún punto interior, ya que toda función constante
tiene derivada nula.

Interpretación geométrica del Teorema de Rolle

Si los extremos de un arco de curva continua y derivable (es decir, con tangente en todos los
puntos) tienen la misma ordenada, entonces en algún punto del arco la tangente es paralela al
eje OX.

Nota En las siguientes figuras aparecen curvas que no satisfacen algún apartado de la
hipótesis y, en consecuencia, puede verse que no tienen por qué verificar la tesis.

f no es continua en [ a ,b ] f no es derivable en ( a , b ) f (a ) ≠ f (b )

5-2
Propiedades de las funciones derivables Tema 5

Ejemplos

Ejemplo 5.1 La función f : [ 1, 3 ] → R tal que f (x ) = x 2 − 4 x + 11 , satisface el Teorema de


Rolle porque es continua en [ 1, 3 ] , derivable en ( 1, 3 ) y f (1) = f (3 ) = 8 .

3
Ejemplo 5.2 La función f : [ − 1,1 ] → R tal que f (x ) = x 2 , es continua en [ − 1,1 ] y
f (−1) = f (1) = 1 , pero no existe ningún punto de ( − 1,1 ) en el que tenga derivada cero y, sin
embargo, no contradice el Teorema de Rolle, porque no es derivable en x = 0.

Nota Si una función satisface la hipótesis del Teorema de Rolle en un intervalo [ a ,b ] y,


además, f (a ) = f (b ) = 0 , el Teorema asegura que entre a y b hay un punto que anula la
derivada. Esto es, entre dos raíces de una función continua y derivable siempre hay una raíz de
la función derivada.

Ejemplos

Ejemplo 5.3 Si se nos pide que demostremos que la ecuación e x + 2 x − 1 = 0 tiene una única
solución real, debemos advertir que la pregunta es doble, pues primeramente hay que
demostrar que existe, al menos, una solución real y, después, que no hay otra (unicidad de la
solución).

La existencia, generalmente, se prueba mediante el Teorema de Bolzano, aunque en


este caso es evidente que x = 0 es una solución.

Para la unicidad podemos hacer uso del Teorema de Rolle en la versión de la Nota
anterior.

En efecto, definimos f (x ) = e x + 2 x − 1, que es continua y derivable en todo R.

Si tuviera dos raíces, c1 y c 2 , entonces f (c 1 ) = f (c 2 ) = 0 y como f es continua en


[ c1 , c 2 ] ( c1 , c 2 ) , el Teorema de Rolle asegura que la derivada se anula en
y derivable en
algún punto. Sin embargo, f ′(x ) = e x + 2 no se anula nunca, luego no puede tener dos raíces
reales y, como consecuencia, la ecuación inicial tiene una única solución.

Ejemplo 5.4 Si queremos demostrar que, para cualquier valor de m, la ecuación


2 x + x + m = 0 no tiene dos soluciones, definimos f (x ) = 2 x 5 + x + m , que es continua y
5

derivable en todo R. Si tuviera dos raíces, c1 y c 2 , entonces f (c 1 ) = f (c 2 ) = 0 y como f es


continua en [ c1 , c 2 ] y derivable en ( c 1 , c 2 ) , el Teorema de Rolle asegura que la derivada se
anula en algún punto. Sin embargo, f ′(x ) = 10 x 4 + 1 no se anula nunca y es independiente del
valor inicial de m, luego no puede tener dos raíces reales y, como consecuencia, la ecuación
inicial no puede tener dos soluciones reales distintas.

El siguiente corolario es una propiedad recíproca de la nota anterior.

5-3
Propiedades de las funciones derivables Tema 5

Sea f una función derivable y sean las ecuaciones: (A) f (x ) = 0 y (B) f ′(x ) = 0 . Entonces,
entre dos soluciones consecutivas, c1 y c 2 , de (B), hay a lo sumo una de (A). En
concreto:
- Si f (c1 ) y f (c 2 ) son del mismo signo, no hay soluciones de (A) en ( c 1 , c 2 ) .

- Si f (c1 ) y f (c 2 ) son de distinto signo, existe una única solución de (A) en ( c 1 , c 2 )

5.3 Teoremas de Cauchy y de Lagrange

Teorema de Cauchy (o del valor medio generalizado) Sean f y g dos funciones


continuas en [ a , b ] y derivables en ( a , b ) , entonces existe un punto c ∈ ( a , b ) tal que
g ′(c ) [f (b ) − f (a )] = f ′(c ) [g (b ) − g (a )] .

Demostración.- Basta aplicar el Teorema de Rolle a la función


F (x ) = g (x ) [f (b ) − f (a )] − f (x ) [g (b ) − g (a )] en el intervalo [ a , b ] .

Si g (a ) ≠ g (b ) y g ′(x ) no se nula en ( a , b ) , se tiene la fórmula de Cauchy:


f ′(c ) f (b ) − f (a )
= para algún c ∈ ( a , b ) .
g ′(c ) g (b ) − g (a )

Ejemplo 5.5 Las funciones f (x ) = x 3 + 1 y g (x ) = x 2 + 3 satisfacen la hipótesis del Teorema


de Cauchy en el intervalo [ 0 , 2 ] , luego existe un punto c del correspondiente intervalo abierto
tal que g ′(c ) [f (2 ) − f (0 )] = f ′(c ) [g (2 ) − g (0 )] .
Como f ′(x ) = 3 x 2 , g ′(x ) = 2 x , f (2) = 9 , f (0 ) = 1 , g (2) = 7 , g (0 ) = 3 , sustituyendo en
4
la expresión anterior, resulta 16 c = 12 c 2 , luego c = .
3

Teorema de Lagrange (del valor medio o de los incremento finitos) Sea f una función
continua en [ a , b ] y derivable en ( a , b ) , entonces existe un punto c ∈ ( a , b ) tal que
f (b ) − f (a )
f ′(c ) = .
b−a

Demostración.- Basta aplicar el Teorema de Cauchy, tomando g (x ) = x .

f (b ) − f (a )
Nota Teniendo en cuenta que es la pendiente de
b−a
la recta que une los puntos (a, f (a )) y (b, f (b )) del plano, el
Teorema de Lagrange tiene la siguiente interpretación
geométrica: en todo arco de curva continua y derivable
siempre hay al menos un punto en el que la recta tangente a
la curva en ese punto es paralela al segmento que une los
extremos del arco.

5-4
Propiedades de las funciones derivables Tema 5

Notas Observa que si f (a ) = f (b ) , tenemos el Teorema de Rolle. Y, es que, lo que sucede es


que los tres teoremas anteriores son equivalentes.

Desde un punto de vista físico, el teorema viene a decir, por ejemplo, que si hemos
recorrido 60 Km en 1 hora (velocidad media), entonces en algún momento hemos estado
viajando a 60 Km/h (velocidad instantánea).

Ejemplos

Ejemplo 5.6 Evidentemente la función f (x ) = 3 x 2 + 4 x + 1 satisface la hipótesis del Teorema


de Lagrange en el intervalo [ 3 , 5 ] . Para obtener el punto (o los puntos) de ese intervalo en el
que su recta tangente tiene la misma pendiente que el segmento que une los extremos (3, f (3 ))
f (5 ) − f (3 )
y (5, f (5)) , planteamos f ′(c ) = y, entonces, 6 c + 4 = 28 ⇔ c = 4 . Por lo tanto, el
5−3
punto buscado es el (4, f (4 )) .

x 3 + 2x
Ejemplo 5.7 La función f (x ) = no satisface la hipótesis del Teorema de Lagrange en
x +1
el intervalo [−2,5] porque no es continua en x = −1∈ (−2,5 ) .

Cálculo aproximado del valor de una función en un punto.

Si deseamos obtener el valor de una función f en un punto x1 , buscamos un punto


próximo a él del que sí conocemos el valor de la función y el de su derivada primera, por
ejemplo x 0 , y si f satisface la hipótesis del Teorema de Lagrange en el intervalo [x 0 , x 1 ] se
tiene
f (x 1 ) − f (x 0 )
f ′(c ) = ⇔ f (x1 ) = f (x 0 ) + f ′(c )(x 1 − x 0 ) .
x1 − x 0
Entonces, sustituyendo c por x 0 , ya que el intervalo (x 0 , x 1 ) tiene longitud pequeña,
podemos obtener la siguiente aproximación de f (x 1 )
f (x 1 ) ≅ f (x 0 ) + f ′(x 0 ) (x 1 − x 0 ) .

Ejemplo 5.8 Si queremos conocer el seno de 31°, como conocemos el seno y el coseno de
30° y teniendo en cuenta que la diferencia es de 1 grado que equivale a 0,017 radianes,
aplicamos el resultado de la nota anterior, esto es, sen 31o ≅ sen 30 o + cos 30 o ⋅ 0,017 ,
obteniendo que sen 31o ≅ 0,5147 , cuando el resultado que da la calculadora es 0,5150 .

Aplicación del Teorema de Lagrange al cálculo de límites

Si x tiende a infinito es posible utilizar el Teorema de Lagrange para calcular el límite.


Basta con trabajar en un intervalo de la forma [x , x + k ] , con k un entero, porque también el c
del correspondiente intervalo abierto tenderá a infinito.

Ejemplo 5.9 Apliquemos el Teorema de Lagrange (o del valor medio) para calcular
lim  x 2 + 2 x + 5 − x 2 + 4  . Consideramos la función f (x ) = x 2 + 4 en el intervalo
3 3 3
x →∞  
3 3
[x , x + 1] , ya que f (x + 1) = (x + 1)2 + 4 = x 2 + 2 x + 5 en el que satisface la hipótesis del
Teorema de Lagrange. Existe, por lo tanto, un punto c del intervalo abierto tal que

5-5
Propiedades de las funciones derivables Tema 5

f (x + 1) − f (x ) 3 3
f ′(c ) = ⇔ f ′(c ) = f (x + 1) − f (x ) = x 2 + 2x + 5 − x2 + 4 .
x + 1− x
2c 2x
Por lo tanto, lim  x 2 + 2 x + 5 − x 2 + 4  = lim
3 3
= lim que fácilmente
x →∞   c →∞ 3
3 c +4 ( 2
)2 x →∞ 3
3 x +4( 2
)
2

se comprueba que es igual a cero.

5.5 Regla de L’Hôpital. Aplicaciones


El siguiente teorema es consecuencia del Teorema de Cauchy.

Teorema de L’Hôpital Sean f y g dos funciones derivables en un entorno reducido V de


a , tales que g y g' no se anulan en V y que lim f (x ) = lim g (x ) = 0 , entonces
x →a x →a
f (x ) f ′(x )
lim = lim , cuando este último límite existe (pertenece a R ).
x → a g (x ) x →a g ′(x )

Demostración.- Por el T. de Cauchy aplicado al intervalo [x , a ] , existe un c comprendido entre


f ′(c ) f (a ) − f (x )
x y a, tal que = .
g ′(c ) g (a ) − g (x )

f (x ) f (a ) − f (x ) f ′(c ) f ′(x )
Luego: lim = lim = lim = lim ,
g (x ) x →a g (a ) − g (x ) x →a g ′(c ) x →a g ′(x )
x →a

como c está entre x y a, cuando x tiende a a arrastra a c, por lo que también c tiende a a,
pudiéndose cambiar c por x .

El resultado también es válido para límites laterales, cuando a es infinito (positivo o


negativo) o cuando los límites de f y g tienden a infinito (positivo o negativo).

La "regla de L'Hôpital" aparece por primera vez en el libro que publicó el marqués de
L'Hôpital en 1696; regla cuya paternidad reivindicó Bernoulli (Johann) después de la muerte del
marqués.

5.6 Aplicación de la Regla de L’Hôpital

0 ∞
Indeterminación ,
0 ∞

0 ∞
La regla de L'Hôpital resuelve directamente las indeterminaciones del tipo , .
0 ∞
Consiste en sustituir el numerador y el denominador por sus respectivas derivadas, tal como
indica el esquema siguiente:

f (x )  0 ∞  f ′(x )  0 ∞  f ′′(x )
lim = ,  = lim = ,  = lim = ...
x →a g (x )  0 ∞  x → a g (x )  0 ∞ 
′ x → a g ′′(x )

Nota Observa que es el cociente de las derivadas, no la derivada del cociente.

5-6
Propiedades de las funciones derivables Tema 5

Como se ve, esta regla se puede aplicar sucesivamente; en tal caso, cada vez y antes
de la sustitución directa, hay que recomponer.

Ejemplos

2x 3 − 8x 2 + 4x + 6  0 6x 2 − 16x + 4 10 5
Ejemplo 5.10 lim 2
=   = lim = =
x→3 x −9  0  H x→ 3 2x 6 3

2cos 2 x − e x − 1 0 −4 cos x senx − e x  −1


Ejemplo 5.11 lim =   = lim =   = −∞
x→0 3 + x senx − 3 cos x 0
  x →0
H senx + x cos x + 3senx  0

e x − 3x 2  ∞ e x − 6x  ∞ ex −6
Ejemplo 5.12 lim 2
=   = lim =   = lim = +∞
x→+∞ x +1  ∞  H x→+∞ 2x  ∞  H x→ +∞ 2

Indeterminación 0 ⋅ ∞

0 ∞
Se reduce, haciendo operaciones, a una indeterminación del tipo o , y se aplica
0 ∞
la regla de L`Hôpital.

Ejemplo 5.13
1  1 
ln  + 1 ln  + 1
1  x  x   0
lim e
x→+∞
2x +3
ln  +1 = 0 ⋅∞
x 
( )= lim
x→ +∞ 1
= lim
x→ ∞ e
−2x− 3
=   =
 0
2 x+3
e
1

x + x 2 = lim e
2x +3
∞ e 2 x +3  ∞  2e 2 x + 3
= lim =   = lim =   = lim = +∞ .
H x →∞ ( )
− 2e − 2 x −3 x →∞ 2 x + x 2  ∞  H x →∞ 1 + 2 x  ∞  x →∞ 2

Indeterminación ∞ − ∞

0 ∞
Haciendo operaciones se reduce a una indeterminación del tipo o .
0 ∞

 1 1 x − senx  0 1− cos x
Ejemplo 5.14 lim 
x→0  senx
−  = ∞ − ∞ = lim
x  x →0 x
(
⋅senx
)
=   = lim
0
  x →0
H senx + x cos x
=

 0 senx 0
=   = lim = = 0.
0
  x →0
H cos x + cos x − x senx 2

5-7
Propiedades de las funciones derivables Tema 5

0 0
Indeterminaciones 0 , ∞

Se resuelven tomando logaritmos, con lo que se transforman en indeterminaciones de


alguno de los tipos anteriores.

Supongamos que lim (f (x ))g


x →a
(x )
( )
= 0 0 , con lim f (x ) = 0 y lim g (x ) = 0 .
x →a x →a

(x )
Llamemos A = lim (f (x ))g y tomemos logaritmos:
x →a

 x →a ( 1) x →a (2 )x →a
( )
ln (A) = ln  lim (f (x ))g (x )  = lim ln (f (x ))g (x ) = lim [g (x ) ⋅ ln (f (x ))] ,

donde (1) se justificará en la lección de continuidad, y (2) es consecuencia de las propiedades


de los logaritmos. Entonces, por la definición de logaritmo, se tiene

lim g (x )⋅ln(f (x ))
A = e x→a .

Ejemplos

Ejemplo 5.15 Calculemos lim (senx )tgx . Aplicando el resultado anterior se tiene
x →0

lim tgx ⋅ln (senx )


lim (senx )tgx = 0 0 = e x →0
x →0
( ) = eB ,

donde B = lim (tgx ln(senx )) . Veamos cuánto vale B :


x →0

cos x
ln (senx )  ∞ 
lim (tgx ln (senx )) = (0 ⋅ ∞ ) = lim =   = lim senx = lim ( −senx cos x ) = 0 . Por lo
x →0 x →0 cot x  ∞  H x →0 − 1 x →0
2
sen x

tanto, lim (senx )tgx = eB = e0 = 1 .


x →0

Ejemplo 5.16 Calculemos lim 5 x + 3 x ( 2


) (
ln x 2 − 8 ) . Aplicando el resultado anterior se tiene
x →∞

1 1
)⋅ln (5 x )
2

lim (5 x 2
+ 3x ) (
ln x 2 − 8 ) = (∞ 0 ) = e lim ln (x
x →∞ 2
−8
+3 x
= eB ,
x →∞

1
donde B = lim ( 2
)
+ 3 x . Veamos cuánto vale B :
x →∞ ( 2
ln x − 8 ) ⋅ ln 5 x
10 x + 3

lim
1
( )
⋅ ln 5 x 2 + 3 x = lim
(
ln 5 x 2 + 3 x ) =  ∞  = lim 5 x 2 + 3 x = lim (10 x + 3 ) x − 8 = 1
2
( )
x →∞ (
ln x 2 − 8 ) x →∞ (
ln x 2 − 8 )  ∞  H x →∞ 2x x →∞ 2x 5 x 2 + 3 x ( )
x2 − 8
1

(
Por lo tanto, lim 5 x + 3 x 2
) (
ln x 2 − 8 )= eB = e1 = e .
x →∞

5-8
Propiedades de las funciones derivables Tema 5


Indeterminación 1

Esta indeterminación se deshace del mismo modo que las anteriores, pero aquí
podemos utilizar infinitésimos equivalentes y resulta la fórmula siguiente:
lim g ( x )⋅ (f ( x ) −1)
A = lim (f (x ))g (x ) = e x →a
x →a

1
 π  senx
Ejemplo 5.17 Calculemos lim tg  x +  . Aplicando el resultado anterior se tiene
x →0  4

 π
1 tg  x +  −1
lim 
4
 π
lim tg  x + 
senx
( )
= 1∞ =e x →0 senx = eB ,
x →0  4

 π
tg  x +  − 1
 4
donde B = lim . Veamos cuánto vale B :
x→0 senx
 π 2 π
tg  x +  − 1 1+ tg  x + 
 4  0  4 2
lim =   = lim = = 2 .
x→0 senx  0  H x→0 cos x 1

1
 π  senx
Por lo tanto, lim tg  x +  = eB = e 2 .
x →0  4 

Aplicación de la regla de L'Hôpital al cálculo del orden de infinitésimos


en x = 0
f (x )
Para ver el orden de un infinitésimo f, bastará determinar n para que lim =a ≠0.
x →0 xn

Ejemplo 5.18 Calculemos el orden del infinitésimo f (x ) = 1 − cos x .


1 − cos x 0 senx 0 cos x
lim n
= [ind .] = xlim n −1
= [ind .] = lim
x →0 n (n − 1)x n − 2
, como el numerador ya no es un
x →0 x 0 →0 nx 0
infinitésimo no hay indeterminación, es decir n − 2 = 0 ⇒ n = 2 , se trata de un infinitésimo de
orden 2.

Ejemplo 5.19 Calculemos el orden del infinitésimo f (x ) = tg x − x .

lim
tgx − x
=
0
[ind .] = xlim tg 2 x 0
= [ind .] = lim
(
2tgx 1 + tg 2 x 0)
= [ind .] = lim
[ (
2 1 + tg 2 x + 1 + tg 2 x 3tg 2 x ) ]
x →0 x n 0 →0 nx n −1 0 x →0 n (n − 1)x n − 2 0 x →0 n (n − 1)(n − 2)x n −3

El numerador ya no es infinitésimo y razonando como en el ejemplo anterior el orden es 3.

5-9
Propiedades de las funciones derivables Tema 5

Ejercicios resueltos

Teoremas de las funciones derivables

R.1. a) Comprueba que no es aplicable el Teorema de Rolle en [− 2,2] a la función:


(x + 1)2 si x ≤ 0
f (x) = 
 x − 1 si x > 0
b) En el caso anterior se observa que f ′(− 1) = 0 con − 1∈ (− 2,2) . ¿Contradice esto al
Teorema de Rolle?.

Solución:
a) El Teorema de Rolle aplicado a nuestra función en [− 2,2] exige que f (x ) sea continua en
[− 2,2] , derivable en (− 2,2) y que f (− 2 ) = f (2) ; veamos si se cumplen las tres condiciones:

1. f (− 2) = f (2) = 1 . Se cumple.
(x + 1)2 si x≤0

2. Continuidad. Definimos más detalladamente la función f ( x ) =  − x + 1 si 0 < x < 1 . Los
 x − 1 si x >1

únicos puntos de discontinuidad son x = 0 y x = 1.
En x = 0 : f (0 ) = 1 , lim (x + 1) = 1 y lim+ (− x + 1) = 1 . La función es continua.
2

x →0 x →0
En x = 1 : f (1) = 0 , lim (− x + 1) = 0 y lim+ (x − 1) = 0 . La función es continua.
x →0 − x →0

2(x + 1) si x<0

3. Derivabilidad: f ( x ) =  − 1 si 0 < x < 1 , veamos qué pasa en x = 0 y x = 1.
 1 si x >1

En x = 0 : Como la función es continua f−′ (0 ) = lim 2(x + 1) = 2 y f+′ (0 ) = lim+ (− 1) = −1 .
x →0 − x →0
No hay derivada.
No se cumplen las condiciones de la hipótesis del teorema.
Se puede comprobar que tampoco es derivable en x = 1, aunque lo fuese no cambiaría
nuestra conclusión.

b) No. Si la función no cumple las condiciones de la hipótesis el teorema no es aplicable, es decir,


no afirmamos ni negamos que exista algún punto de c ∈ (− 2,2) para el que f ′(c ) = 0 , tal punto
puede existir o no sin que una u otra cosa contradiga al teorema.

R.2. La función f (x ) = 3 (x − 1) toma valores iguales en los extremos de [0,2] . ¿Puede


2

aplicarse el Teorema de Rolle?.

Solución:
2
No. La función debe ser derivable en (0,2) si derivamos f ′(x ) = , se ve que no existe f ′(1) .
3
3 x −1

 π π
R.3. Comprueba si es aplicable el Teorema de Rolle a y = ctgx en − ,  .
 2 2
Solución:

No es aplicable. La función y = ctgx presenta una discontinuidad de salto infinito en x = 0 y


 π π
0∈− ,  .
 2 2

5-10
Propiedades de las funciones derivables Tema 5

R.4. Estudia si es aplicable el Teorema de Rolle a las siguientes funciones en los intervalos
que se indican:
x2
a) f (x ) = en [− 6,2] b) g (x ) = x − 2 + x − 3 en [1,4] .
x −3
En caso afirmativo calcula el valor o los valores pronosticados por el teorema.

Solución:

a) Comprobamos si se cumplen las tres condiciones de la hipótesis.


f (− 6) = −4 , f (2 ) = −4 . Se cumple.
La función es continua en [− 6,2] puesto que el único punto de discontinuidad es x = 3 .
x 2 − 6x
f ′(x ) = , f (x ) no es derivable en x = 3 y 3 ∉ (− 6,2) . Se puede aplicar el Teorema
(x − 3)2
de Rolle que asegura la existencia de un c ∈ (− 6,2) / f ′(c ) = 0 ; hacemos f ′(x ) = 0 ⇒ x 2 − 6 x = 0 lo
que lleva a dos soluciones x = 0 y x = 6 , 0 ∈ (− 6,2) es el pronosticado por el teorema.

− x + 2 si x ≤ 2
b) En primer lugar definiremos la función a trozos en [1,4] . Dado que x − 2 =  y
 x − 2 si x ≥ 2
− x + 3 si x ≤ 3
del mismo modo x − 3 =  tenemos que g (x ) = − x + 2 + (− x + 3 ) = −2 x + 5 si
 x − 3 si x ≥ 3
1 ≤ x ≤ 2 ; g (x ) = x + 2 − x + 3 = 1 si 2 < x < 3 y g (x ) = x + 2 + x − 3 = 2 x − 5 si 3 ≤ x ≤ 4 , en
− 2 x + 5 si 1 ≤ x ≤ 2

definitiva g (x ) =  1 si 2 < x < 3 , comprobamos las condiciones de la hipótesis:
 2 x − 5 si 3 ≤ x ≤ 4

f (1) = f (4 ) = 3 . Se cumple.

Continuidad en [1,4] .

Los únicos punto posibles de discontinuidad son x = 2 y x = 3 .


Estudiamos x = 2 : f (2) = 1 , lim− (− 2 x + 5 ) = 1 y lim+ 1 = 1 . La función es continua.
x →2 x →2
En x = 3 : f (3 ) = 1 , lim− 1 = 1 y lim (2 x − 5 ) = 1 . La función es continua. Se cumple.
x →3 x →3 +

Derivabilidad en (1,4 ) .
− 2 si 1 < x < 2

Calculamos g (x ) =  0 si 2 < x < 3 . Calculamos g ′(2) : Como es continua en x = 2 ,

 2 si 3 < x < 4

f−′ (2) = lim− − 2 = −2 y f+′ (2 ) = lim+ 0 = 0 , las derivadas laterales no coinciden por lo tanto no
x→2 x →2
existe g ′(2) y ya no importa lo que ocurra en x = 3 , el Teorema de Rolle no es aplicable a g (x )
en [1,4] .

R.5. Calcula el valor de a para que sea aplicable el Teorema de Rolle a la función
f (x ) = ax 2 + 2 x − 2 en el intervalo [− 1,3] . ¿Dónde se cumple la tesis?.

Solución:

La función es continua y derivable en todo R independientemente del valor de a, por lo tanto


solamente hay que exigir que f (− 1) = f (3 ) , f (− 1) = a − 4 y f (3 ) = 9a + 4 , luego a − 4 = 9a + 4 ,

5-11
Propiedades de las funciones derivables Tema 5

a = −1 . La función queda f (x ) = − x 2 + 2 x − 2 ; veamos dónde se cumple la tesis. f ′(x ) = −2 x + 2 ,


haciendo f ′(x ) = 0 ⇒ −2 x + 2 = 0 ⇒ x = 1 que efectivamente pertenece al intervalo (− 1,3 ) .

R.6. Calcula el valor de x0 para que a la función f (x ) = x 2 + 9 le sea aplicable el Teorema de


Rolle en [− 1, x0 ] .

Solución:

La función es continua y derivable ∀ x ∈ ℜ , así que sólo hay que exigir que f (− 1) = f (x0 ) ,
 x = −1
, x0 = −1 no tiene sentido luego el intervalo pedido es [− 1,1] .
2
es decir x0 + 9 = 10 ⇒  0
 x0 = 1

R.7. Calcula, si existen, los valores de a, b y c para que la función


ax 2 + b si x ≤ 1
f (x) =  cumpla las condiciones de la hipótesis del Teorema de Rolle en
x + 4 si x > 1
[0,c ] con c > 1.

Solución:

Se debe exigir que la función sea continua en [0, c ] y derivable en (0, c ) , el único punto
problemático es x = 1.
(
Para que la función sea continua en x = 1 f (1) = lim f (x ) ; f (1) = lim− ax 2 + b = a + b y
x →1 x →1
)
lim (x + 4 ) = 5 , luego a + b = 5 .
x →1+

2ax si x <1
Calculamos f ′( x ) =  f−′ (1) = lim− 2ax = 2a y f+′ (1) = lim 1 = 1, luego para
1 si x >1 x →1 x →1+

1 2 9
1 9  x + si x ≤1
que sea derivable a = y entonces b = , la función queda f ( x ) = 2 2 .
2 2  x + 4 si x >1
9
Para que sea aplicable el teorema debe cumplirse que f (0 ) = = f (c ) = c + 4 con c > 1 , es decir
2
9 1
c+4= cuya única solución c = < 1 por lo tanto los valores pedidos no existen.
2 2

R.8. Calcula, si existen, los valores de a, b y c para que cumpla las condiciones de la
ax 2 + bx + 3 si x ≤ 2
hipótesis del Teorema de Rolle f ( x ) =  en [0, c ] con c > 2 .
x si x > 2
Solución:

Se debe exigir que la función sea continua en [0, c ] y derivable en (0, c ) , el único punto donde
podría no serlo dependiendo de los valores de a y b es x = 2 .

x →2
( )
Continuidad en x = 2 : f (2 ) = lim− ax 2 + bx + 3 = 4a + 2b + 3 , lim (x ) = 2 ⇒ 4a + 2b + 3 = 2 .
x →2+
⇒ 4a + 2b = −1 .
2ax + b si x<2
Derivabilidad en x = 2 : Calculamos f ′( x ) =  como la función es continua en
1 si x>2
4a + 2b = −1
x = 2 f−′ (2 ) = lim− 2ax + b = 4a + b y f+′ (2 ) = lim 1 = 1 ⇒ 4a + b = 1 , resolvemos 
x →2 x →2 +
4a + b = 1 

5-12
Propiedades de las funciones derivables Tema 5

3 2
3  x − 2 x + 3 si x≤2
⇒ b = −2 y a = . La función es necesariamente f ( x ) = 4 .
4  x si x>2
f (0 ) = f (c )
Para que se cumpla la hipótesis debe ser  ⇒ c = 3 y el intervalo es [0,3] .
c>2 

R.9. Estudia si es posible aplicar el Teorema de Lagrange a las funciones siguientes en el


intervalo [− 1,5] , en caso afirmativo calcula el valor donde se cumple la tesis.
a) f (x ) = 3 x b) f (x ) = x + e x

Solución:

a) La función ha de ser continua en [− 1,5] y derivable en (− 1,5 ) . Es continua para todo R luego
1
no hay problema con la continuidad, pero si derivamos llegamos a f ′(x ) = y como se
3
3 x2
observa no hay derivada en x = 0 y 0 ∈ (− 1,5 ) , luego el teorema no es aplicable en ese
intervalo.

b) La función f (x ) = x + e x es continua y derivable ∀ x ∈ ℜ y en consecuencia lo será en los


intervalos requeridos. El teorema sí es aplicable.
f (5 ) − f (− 1)
En estas condiciones el Teorema de Lagrange afirma que ∃ c ∈ (− 1,5 ) : f ′(c ) = y se
5 − (− 1)
6 + e 5 − e −1 e 5 − e −1
tiene: ∃f ′(c ) = 1 + e c = ⇒ c = ln ≈ 3'2 ∈ (− 1,5 ) .
6 6

R.10. Calcula los valores de a y b para que sea aplicable el Teorema del Valor Medio a la
ax + b si x ≤ 0
función f ( x ) =  3 en [− 2,2] . Calcula el valor o valores vaticinados por el
si x > 0
 x + 1
teorema.

Solución:

La función debe ser continua en [− 2,2] y derivable en (− 2,2) , el único punto donde podría no
serlo dependiendo de los valores de a y b es x = 0 .
 3 
Para que sea continua en x = 0 : f (0 ) = lim− (ax + b ) = b , lim  =3 ⇒ b =3.
x →0 x →0 + 
x + 1
a si x < 0
 3
Para que sea derivable en x = 0 : Calculamos f ′( x ) = − si x > 0 como la función es
 (x + 1)2
3
continua en x = 0 f−′ (0 ) = lim a = a y f+′ (0 ) = lim+ − = −3 ⇒ a = −3 . La función queda
x →0 − x →0 (x + 1)2
− 3 x + 3 si x≤0 − 3 si x≤0
 3
f (x) =  3 y su derivada f ′( x ) = − si x>0
si x>0
 x + 1  (x + 1)2

5-13
Propiedades de las funciones derivables Tema 5

f (2) − f (− 2)
Buscamos dónde se cumple la tesis f ′(c ) = = −2 ; evidentemente para x ≤ 0 no hay
4
3 6
solución. Buscamos en x > 0 − = −2 ecuación que tiene dos soluciones c1 = −1 −
(c + 1)2 2
6
que cae fuera de (− 2,2) y c1 = −1 + ≈ 0'22 ∈ (− 2,2) que es donde se satisface el teorema.
2

x si x ≤ 2
R.11. Dada la función f ( x ) =  2 se pide:
− x − 7 si x > 2
f (3 ) − f (0 )
a) Comprueba que ∃ c ∈ (0,3 ) : f ′(c ) = .
3−0
b) La función no cumple las condiciones del Teorema de los Incrementos Finitos o de
Lagrange en el intervalo [0,3] . ¿No hay en esto una contradicción?.

Solución:

f (3 ) − f (0 ) − 16 1 si x<2 −16 8
a) = y f ′( x ) =  si hacemos − 2c = ⇒ c = ∈ (0,3 ) .
3−0 3 − 2 x si x>2 3 3

b) Efectivamente la función no es continua en todos los puntos de [0,3] , tiene una discontinuidad
de salto finito en x = 2 , pues
x →2 x→2
( )
f (2 ) = lim− x = 2 y lim+ − x 2 − 7 = −11 . No hay contradicción con

el Teorema de Lagrange ya que al no cumplirse la hipótesis el teorema no afirma ni niega la


f (3 ) − f (0 )
existencia de una valor donde f ′(c ) = , tal valor puede existir o no sin que halla
3−0
contradicción.

R.12. Comprueba que es posible aplicar el Teorema del Valor Medio o de Lagrange a la
 x 2 − 8 si x ≤ 2

función f ( x ) =  4 en [0,3] . Calcula el valor, o valores, donde se cumple la
1 − x si x > 2

tesis.

Solución:

La función debe ser continua en [0,3] y derivable en (0,3 ) , el único punto donde podría no serlo
es x = 2 , lo comprobamos.
 4 
x →2
( )
Continuidad: f (2) = lim− x 2 − 8 = −4 , lim+   = −4 . Es continua.
x →2  1 − x 

2 x si x<2

Derivabilidad: Calculamos f ′( x ) =  4 si x > 2 como la función es continua en x=2
 (1 − x )2
4
f−′ (2) = lim− 2 x = 4 y f+′ (2) = lim+ = 4 . Es derivable.
x →2 x →2 (1 − x )2
f (3 ) − f (0 )
El Teorema del Valor Medio afirma que ∃ c ∈ (0,3 ) : f ′(c ) = = 2 si buscamos soluciones
3−0
para x ≤ 2 se tiene 2c = 2 ⇒ c = 1 en este punto se cumple la tesis, veamos si hay más
4 c = 1 + 2
soluciones. Buscamos en la zona x > 2 , se tiene 2
=2⇒ la única solución de
(1 − c ) c = 1 − 2

5-14
Propiedades de las funciones derivables Tema 5

esta ecuación que tiene sentido es c = 1+ 2 , hay por tanto dos lugares donde se cumple la
c = 1
tesis del teorema  1 .
c 2 = 1 + 2

R.13. Comprueba que se puede aplicar el Teorema de Cauchy a las funciones:


f ( x ) = x 2 + 2 x + 1 y g ( x ) = x + 3 en el intervalo [0,2] . Calcula el valor c pronosticado por el
teorema.

Solución:

El enunciado del teorema exige que las dos funciones sean continuas en [0,2] y derivables en
(0,2) , como son funciones polinómicas que son continuas y derivables ∀ x ∈ ℜ , evidentemente se
cumplen las condiciones del teorema.
Buscamos el valor c ∈ (0,2) donde se cumple f ′(c )[g (2) − g (0 )] = g ′(c )[f (2) − f (0 )] , como
g (2) − g (0 ) = 2 y f (2) − f (0 ) = 8 , se tiene (2c + 2) ⋅ 2 = 8 ⇒ c = 1 que es valor pedido.

 π
R.14. Comprueba que la ecuación e senx + x − 3 = 0 tiene una sola raíz real en 0,  .
 2

Solución:

Consideramos la función f (x ) = e senx + x − 3 . Es equivalente afirmar que la ecuación dada tiene


solución a decir que f (x ) = 0 .
 π
Demostraremos en primer lugar que hay algún punto c ∈ 0,  donde f (c ) = 0 y para ello
 2
utilizaremos el Teorema de Bolzano.
La función es continua siempre por lo tanto si en un intervalo [a, b ] cambia de signo es seguro que
∃ c ∈ (a, b ) : f (c ) = 0 .
π
π  π  π
f (0 ) = −2 < 0 y f   = e 2 + − 3 > 0 luego ∃ c ∈  0,  : f (c ) = 0 .
2 2  2
Demostraremos por reducción al absurdo que no puede haber más soluciones en ese eintervalo y
para ello haremos uso del Teorema de Rolle.
Calculamos f ′(x ) = e senx ⋅ cos x + 1. La función es derivable ∀ x ∈ ℜ .
 π
Supongamos que existiese otro valor cualquiera del intervalo  0,  d > c con f (d ) = 0 ,
 2
entonces en el intervalo [c, d ] la función cumple las condiciones de la hipótesis del Teorema de
Rolle y en consecuencia debería existir un valor x0 ∈ (c, d ) : f ′(x0 ) = 0 , pero tal cosa es imposible
 π
porque f ′(x ) = e senx ⋅ cos x + 1 > 0 en todos los puntos de  0,  , luego nuestra suposición no
 2
puede ser cierta. Igual se razonaría para cualquier d < c .

R.15. Comprueba que la ecuación x 3 + 3 x + 2 = 0 tiene una sola solución real.

Solución:

Consideramos la función f (x ) = x 3 + 3 x + 2 . Afirmar que la ecuación dada tiene solución es lo


mismo que decir que f (x ) = 0 .
Demostraremos utilizando el Teorema de Bolzano que hay algún punto c tal que f (c ) = 0 .
La función es continua siempre por lo tanto si encontramos un intervalo [a, b ] donde cambie de
signo es seguro que ∃ c ∈ (a, b ) : f (c ) = 0 .

5-15
Propiedades de las funciones derivables Tema 5

f (0 ) = 2 > 0 y f (− 1) = −2 < 0 luego ∃ c ∈ (− 1,0 ) : f (c ) = 0 .


Demostraremos con del Teorema de Rolle que no puede haber más soluciones.
Calculamos f ′(x ) = 3 x 2 + 3 , su dominio es ℜ , luego la función es derivable en cualquier intervalo.
Supongamos que existiese otro valor d > c tal que f (d ) = 0 también, entonces en el intervalo
[c, d ] la función cumple todas las condiciones de la hipótesis del Teorema de Rolle y en
consecuencia debería existir un valor x0 ∈ (c, d ) : f ′(x0 ) = 0 , pero tal cosa es imposible porque
f ′(x ) = 3 x 2 + 3 > 0 ∀x , luego suponer que f (d ) = 0 contradice el Teorema de Rolle y por lo tanto
no hay más soluciones a la derecha de c. De manera idéntica se razonaría para cualquier d < c .

x
R.16. Dada la ecuación ln x − = 0 demuestra que tiene solamente dos soluciones reales
3
x1 < 3 y x2 > 3 .

Solución:
x
Construimos la función f (x ) = ln x − . Afirmar que la ecuación dada tiene solución es lo mismo
3
que decir que f (x ) = 0 .
Como la función es continua en todo su dominio D = (0,+∞ ) si encontramos un intervalo
[a, b] donde cambie de signo el Teorema de Bolzano afirma que ∃ c ∈ (a, b ) : f (c ) = 0 . Buscamos
dos intervalos donde cambie de signo uno a la izquierda y otro a la derecha de 3.
1 1
f (3 ) = ln 3 − 1 > 0 , f (1) = ln 1 − = − < 0 y finalmente f (6 ) = ln 6 − 2 < 0 , por lo tanto hay al
3 3
menos dos soluciones x1 ∈ (1,3 ) y x2 ∈ (3,6 ) .
Demostraremos que no hay mas soluciones menores que 3, lo haremos por el método de
1 1
contradicción. Calculamos f ′(x ) = − con x > 0 , la función es derivable en todo su dominio.
x 3
Supongamos que hay otro x3 < 3 con f (x3 ) = 0 , en el intervalo [x1, x3 ] o bien [x3 , x1] (según si el
valor es mayor o menor que x1 ) la función cumple las condiciones del Teorema de Rolle y por lo
tanto debería haber un punto c ∈ (x1, x3 ) o c ∈ (x3 , x1 ) (según corresponda) de manera que
1 1
f ′(c ) = 0 tal cosa es imposible pues f ′(x ) = 0 ⇒
− = 0 ⇒ x = 3 luego la derivada no se anula
x 3
en ningún punto del intervalo y en consecuencia nuestra suposición es falsa. No puede haber
soluciones menores de 3.
De manera análoga se prueba que tampoco hay soluciones mayores de 3.

R.17. Comprueba que la Regla de L’Hôpital no es aplicable para calcular los límites
siguientes y resuélvelos por otro procedimiento.
1
x 3sen
x − senx x
a) lim b) lim
x → ∞ x + senx x → 0 sen 2 x

Solución:
x − senx ∞ R. L'H 1 − cos x
a) lim = [indet.] = lim y este límite no existe. Sin embargo sí existe límite
x → ∞ x + senx ∞ x → ∞ 1 + cos x
pues el Teorema Regla de L’Hôpital no asegura la existencia del límite del cociente de las
derivadas, lo que afirma es que si tal límite existe entonces éste es el límite buscado.
Resolvemos la indeterminación dividiendo numerador y denominador por x, se tiene:
senx
1−
lim
x − senx
= lim x = 1− 0 = 1.
x → ∞ x + senx x →∞ senx 1 + 0
1+
x

5-16
Propiedades de las funciones derivables Tema 5

1 1 1
x 3sen R. L'H 3 x 2sen − x cos R. L'H
x 0 x x 0
b) lim 2
= [indet.] = lim = [indet.] =
x → 0 sen x 0 x →0 2senx cos x 0
1 1 1 1
6 xsen − 4 cos + sen
= lim x x x x , como no existe lim cos 1 el límite anterior no existe y por
2 2
x →0 2 cos x − 2sen x x →0 x
lo tanto la regla de L’Hôpital no es aplicable.

Si lo hacemos por el método de infinitésimos equivalentes sustituyendo senx por x se


1 1
x 3sen i.e. x 3sen
x x 1
tiene: lim = lim = lim xsen = 0 .
x → 0 sen 2 x x →0 x 2 x → 0 x

5-17
Propiedades de las funciones derivables Tema 5

Ejercicios propuestos

Teoremas de las funciones derivables

P.1. Estudia si es aplicable el teorema de Rolle en [1,5] a la función:


x + 2 1≤ x < 3
f(x) = 
7 − x 3 ≤ x ≤ 5

P.2. Estudia si es posible aplicar el Teorema de Rolle a las funciones siguientes en los intervalos
que se indican.
1 x 2 − 4x 1
a) f (x ) = 1 − 2
en [0 ,2] ; b) g ( x ) = en [0,4] c) y = en [− a, a]
(x − 1) x+2 x

2
d) f (x ) = x en [− a, a ] e) f (x ) = (2 − x ) 3 en [0,4]

P.3. Justifica si es o no aplicable el Teorema de Rolle a la función f ( x ) = 2 x − 1 en [0,1] .

P.4. Calcula b para que la función f ( x ) = x 3 − 4 x + b cumpla las condiciones de la hipótesis


del Teorema de Rolle en el intervalo [0, b ] . ¿Dónde se cumple la tesis?

P.5. Enuncia e interpreta geométricamente el T. de Rolle. Aplica, si es posible, el Teorema de


2
Rolle a la función: f ( x ) = x 2 + en [−2,1] .
x

P.6. Enuncia el Teorema de Rolle. Interpretación geométrica. ¿Se puede aplicar el teorema
anterior a f ( x ) = 4 x − x 2 en [2,6]?

P.7. Enuncia el Teorema del Valor Medio o de los incrementos finitos o de Lagrange.
ax − 3 si x < 4
Interpretación geométrica. Determina a, b de R para que f ( x ) =  2
− x + 10 x − b si x ≥ 4
cumpla las hipótesis del Teorema de Lagrange en el intervalo [2,6].

P.8. Di por qué la función y = x − 3 senx , cumple el Teorema del Valor Medio en [0, π] y
averigua dónde cumple la tesis.

P.9. Halla el punto c al que se refiere el Teorema de Lagrange, siendo la función y = e3 x , en el


intervalo [2,5] .

( )
P.10. Sea f (x ) = L 5 − x 2 y el intervalo [-2,2]. ¿Son aplicables los teoremas de Rolle y de
Lagrange? En caso afirmativo, hallar el valor intermedio para el que se cumple el Teorema.

Interpretación geométrica del Teorema del Valor Medio

P.11. En la curva y = x 3 se considera la cuerda AB que une los puntos de abcisa 0 y 1. Hallar
las coordenadas del punto C del arco AB cuya tangente es paralela al segmento AB. Hallar la
ecuación de dicha tangente. Efectuar la representación gráfica. ¿Qué teorema llega a esta
misma conclusión?

P.12. Prueba que existe un punto del intervalo [1,e] donde la gráfica de la función f ( x ) = ln x tiene
tangente paralela a la cuerda que une los puntos de dicha función, en los extremos de ese
intervalo. Calcula dicho punto.

5-18
Propiedades de las funciones derivables Tema 5

P.13. Sea f ( x ) = x 2 + 3 L(x + 3 ) . Halla la ecuación de la recta tangente en el punto de abcisa


x = 0 . Demuestra que la curva tiene otra tangente paralela a la anterior y determínala.

P.14. Dada y = x 2 , halla las coordenadas de un punto de la curva cuya tangente en él sea
paralela a la cuerda de extremos ( 1,1 ) y (2,4 ). Representa la figura. ¿ Tiene esto algo que ver
con algún teorema conocido?

Utilización conjunta de los teoremas de Bolzano y Rolle

P.15. Demuestra que la ecuación e x + 2 x − 1 = 0 tiene una sola raíz real.

P.16. Demuestra que la ecuación x 2 = 18 ln x , tiene una única solución en el intervalo [1, e ] .

P.17. Demuestra que la ecuación x cos x − senx = 0 tiene una única solución en el intervalo
 π π
− 2 , 2 
 

Teoremas de Cauchy y L´Hôpital

P.18. Estudia si se puede aplicar el Teorema de Cauchy a las funciones: f ( x ) = x 2 − 2 x + 3 y


g ( x ) = x 3 − 7 x 2 + 20 x − 5 en el intervalo [1,4]. En caso afirmativo aplícalo.

Regla de L´Hôpital

P.19. Encuentra el error en la siguiente aplicación de la regla de L'Hôpital:


x 3 − x 2 − 8x + 7 3 x 2 − 2x − 8 6 x − 2 10
lim 3 2
= lim 2
= lim =
x →2 x − 3x + 4 x →2 3x − 6x x →2 6 x − 6 6

P.20. Calcula:

a) lim
tgx
b) lim
( )
ln 2 x 2
c) lim
e x −1
d) lim x ⋅ e 2 x
x→0 x x → +∞ x −3 x → +∞ 2x 2 − 3 x → −∞

2 ⋅ senx x 1− x  1
e) lim f) lim g) lim h) lim  cot x − 
x →0 2 x − 1 x →0 x + senx x →1 ln x x →0  x

P.21. Calcula:

x  1 
a) lim+ (π − x ) ⋅ tg   b) lim (cosec ( x ) − cot (x )) c) lim+ x ⋅  e x − 1
x →π 2 x →0 x →0  
 

 1 x  x cos x − senx  1 1 
d) lim  −  e) lim 3
f) lim  − x −1 
x →1  ln x x − 1 x →0 x x →1  x − 1 e − 1

e x − e − x − 2x ( 1 − cos x ) ⋅ senx
g) lim h) lim i) lim+ (tgx ⋅ ln x )
x →0 x − senx x →0 x2 x →0

P.22. Calcula:

5-19
Propiedades de las funciones derivables Tema 5

tgx − 8 x − senx 1 − cos x  1 1


a) lim b) lim c) lim d) lim  − 
x → sec x + 10
π
2
x →0 tgx − senx x →0
(e x
−1 ) 2 x → 0  ln(1 + x ) x

P.23. Calcula:

a) lim
e x − esenx
b) lim
(2 − x )e x − x − 2   1 
c) lim x cos   − 1 d) lim
e x − cos x
x →0 x3 x →0 x2 x →∞  x  x →∞ e x + cos x

 1 1    π  ln(cos 3 x )
e) lim  −  f) lim  cos x ⋅ ln x −   g) lim
x →0 2 x
 (
x e πx + 1  ) x→ 
π
2
 2  x →0 ln(cos 2 x )

P.24. Calcula:
1 3 1
a) lim (cot(x − π ))
x →π
sen ( x − π )
b) lim− (1 + cos 2 x )cos x c) lim+ (2 + x )x
π x →0
d) lim e x + x
x →0
( )x

x→
2

tgx
1
 1  cot 2 3 x
e) lim + (1 + 2 cos x ) cos x
π
f) lim  2 
x →0  x 
g) lim+ x x
x →0
h) lim sec 2 2 x
x →0
( )
x→
2

P.25. Calcula, usando infinitésimos equivalentes:

1 − cos (ax ) ax − 1 1 − cos (1 − cos x ) ax − bx


a) lim b) lim c) lim d) lim
x →0 x(2 − x )tg (bx ) x →0 1+ x − 1 x →0 x4 x →0 x

P.26. Calcula:
x − tgx
a ) lim ((x − 1)ln(x − 1)) b ) lim− (1 − cos x )tgx c ) lim d ) lim− (tgx )cos x
x →1
x→
π x →0 x − senx x→
π
2 2

 1
sen  
3
x − xsen x 2
x 1 + ln(cos x ) 2
e) lim f) lim g) lim h) lim (1 − tgx )sec x
x →0 x x → +∞  1 π 1 + tgx π
sen 3   arctg   x→
2
x→
4
2 x
x ⋅ arcsenx 2 x − senx
π
(
i) lim− sec 2 x⋅ e − tgx ) j) lim
x →0 senx ⋅ cos x
k) lim (senx⋅ ln x ) l) lim
x → +∞ x →0 1 − cos x
x→
2

EJERCICIOS DE PROFUNDIZACIÓN

1 1
P.27. Demuestra aplicando el teorema del valor medio que < 66 − 8 <
9 8

 1 π
P.28. Sea f (x ) = arctg   + arctgx . Compruébese que f ′( x ) = 0 . Sin embargo, f (−1) = − , y
x 2
π
f (1) = , luego f no es constante. ¿Cómo se explica eso?
2

5-20
Propiedades de las funciones derivables Tema 5

P.29. Demuestra que una función derivable en cualquier punto que cumple que la ecuación
f ′(x ) = 0 no tiene solución, es inyectiva.

P.30. Razónese que, sea cual sea el número real c, la ecuación: x5 − 5 x + c = 0 no puede
tener dos soluciones positivas menores que 1.

5-21
Propiedades de las funciones derivables Tema 5

Soluciones:

P.1. No, no es continua en x = 3 , no vale lo mismo en los extremos del intervalo. P.2. a) No,
no es continua en x = 1 . b) Si. c) No, no es continua ni derivable en 0. d) No, no es derivable
1
en 0. e) No, no es derivable en 2. P.3. No, no es derivable en x = . P.4. b = 2 , se cumple
2
2 3
en x = . P.5. No es posible porque no es continua en x = 0 . P.6. No es derivable en
3
π
x = 4 . P.7. a = 2, b = 19 . P.8. Se cumple en x = . P.9. c = 4' 26 . P.10. Sí son aplicables,
2
3 2
c = 0 . P.11. x = , tangente y = x − , secante y = x . P.12. x = e − 1 . P.13.
3 3 3
3 9
y = x + 3′29 ; y − f ( 5 / 2 ) = x + 5 / 2 . P.14.  ,  , si con el T.V.M. P.15. Una solución es
2 4
x = 0 , no hay más soluciones porque siendo f (x ) = e x + 2 x − 1 f ′(x ) = e x + 2 > 0 ∀x y
contradice el T. de Rolle. P.16. Si f (x ) = x − 18 ⋅ ln x , se tiene f (0 ) = 1 > 0 y f (e ) = e 2 − 18 < 0 ,
2

18
por el Teorema de Bolzano hay alguna solución en [1, e ] ; como f ′(x ) = 2 x − con x > 0 y
x
f ′(x ) = 0 si x = 3 ∉ [1, e ] otra solución de f (x ) = 0 en [1, e ] contradice el Teorema de Rolle.
P.17. Una solución es x = 0 , si se considera f (x ) = x cos x − senx como f ′(x ) = − x ⋅ senx y la
 π π  π
única solución en − ,  de f ′(x ) = 0 es x = 0 no puede haber otra solución en 0, 2 
 2 2  
 π 
porque contradice el Teorema de Rolle. Se razona igual para − ,0  . P.18. Sí. En x =2
 2 
¡cuidado! en x = 4 no, pues 4 ∉ (1,4 ) . P.19. No hay indeterminación. P.20. a) 1; b) 0 ; c) +∞ ;
1 1 1 1
d) 0 ; e) 0; f) ; g) -1 ; h) 0. P.21. a) 2; b) 0 ; c) +∞ ; d) − ; e) − ; f) ; g) 2 ; h) 0 ;
2 2 3 2
1 1 1 1 π 9
i) 0. P.22. a) 1 ; b) ; c) ; d) . P.23. a) ; b) 0 ; c) 0 ; d) 1 ; e) ; f) 0 ; g) .
3 2 2 6 4 4
4
2 9 a2 1
P.24. a) 1 ; b) 0 ; c) ∞ ; d) e ; e) ∞ ; f) 1 ; g) 1 ; h) e . P.25. a) ; b) 2 lna ; c) ;
4b 8
 a 1
d) ln  . P.26. a) 0 ; b) ; c) -2 ; d) 1 ; e) 0 ; f) 1 ; g) 0 ; h) 0 ; i) 0 ; j) 0 ; k) ∃/ ; l) ∃/ .
b
  e
P.27. aplicar el TVM a f ( x ) = x , en [64,66]. P.28. No es continua en 0. P.29. Por reducción
al absurdo. P.30. Por reducción al absurdo.

Gráfica P.11. Gráfica P.14.

5-22
Monotonía y optimización de funciones Tema 6

Tema 6

Monotonía. Problemas de máximos y mínimos.


Optimización de funciones

6.1 Monotonía de una función (Crecimiento y decrecimiento)


Sea f : [a , b ] → R una función y x0 un punto de (a, b ) .

Definición Diremos que f es creciente en el punto x0 


si existe un entorno de x0 de radio δ tal que para todo
x de (x0 − δ, x0 ) entonces f (x ) ≤ f (x0 ) y para todo x de
(x0 , x0 + δ) entonces f (x ) ≥ f (x0 ) .

Definición Diremos que f es decreciente en el punto


x0  si existe un entorno de x0 de radio δ tal que para
todo x de (x0 − δ, x0 ) entonces f (x ) ≥ f (x0 ) y para todo
x de (x0 , x0 + δ ) entonces f (x ) ≤ f (x0 ) .

Nota En el caso de que las desigualdades sean estrictas, se dice que la función es
estrictamente creciente o estrictamente decreciente en x0 .

Nota A las funciones crecientes (o decrecientes) en un intervalo se les llama monótonas.

Proposición Sea f una función continua en [a, b ] y derivable en (a, b ) , tal que
f ′(x ) = 0 para todo x de (a, b ) , entonces f es constante en [a, b ] .

Demostración.- Sea h un número real positivo, tal que a + h ∈ [a, b ] , entonces por el Teorema
f (b ) − f (a )
de Lagrange se tiene que f ′(c ) = = 0 , con c de (a, a + h ) , luego f (a + h ) = f (a ) .
b−a

Proposición Sea f una función continua en [a, b ] y derivable en (a, b ) , entonces:


1. Si f ′(x ) ≥ 0 ,  ∀ x ∈ (a, b ) , entonces f es creciente en [a, b ] .
2. Si f ′(x ) > 0 ,  ∀ x ∈ (a, b ) , entonces f es estrictamente creciente en [a, b ] .
3. Si f ′(x ) ≤ 0 ,  ∀ x ∈ (a, b ) , entonces f es decreciente en [a, b ] .
4. Si f ′(x ) < 0 ,  ∀ x ∈ (a, b ) , entonces f es estrictamente decreciente en [a, b ] .

6-1
Monotonía y optimización de funciones Tema 6

Demostración.- Es análoga a la demostración de la Proposición anterior.

Ejemplos

Ejemplo 6.1 Veamos cómo se hallan los intervalos de crecimiento y decrecimiento de la


función f (x ) = x 3 − 4 x 2 − 5 x . En primer lugar siempre estudiamos su dominio, que en este
caso es R, por tratarse de una función polinómica. Después, hallamos la derivada,
4 ± 31
f ′(x ) = 3 x 2 − 8 x − 5 . Estudiamos en qué puntos se anula: f ′(x ) = 0 ⇔ x = . Y, por
3
último, estudiamos el signo de la derivada en los tres intervalos de la recta que resultan:

   4 − 31 4 + 31   4 + 31 
 − ∞, 4 − 31   ,   , + ∞
 3   3 3   3 
     
Signo de f ′ positivo negativo positivo
Monotonía creciente decreciente creciente

Ejemplo 6.2 Hallemos los intervalos de crecimiento y decrecimiento de la función


( )
f (x ) = ln x 2 − 3 x + 2 . El dominio de la función son aquellos puntos en los que x 2 − 3 x + 2 > 0 .
2
Como x − 3 x + 2 = 0 ⇔ x = 1 , x = 2 , el dominio vendrá dado por
(− ∞,1) (1,2) (2,+∞ )
2
Signo de x − 3 x + 2 positivo negativo positivo
Dominio sí no sí

2x − 3
Calculamos la derivada, f ′(x ) = 2
y la igualamos a cero. Es evidente que sólo
x − 3x + 2
3
se anula en x = = 1,5 . Pero 1,5 no forma parte del dominio. Estudiamos el signo en los
2
intervalos correspondientes:

(− ∞,1) (2,+∞ )
Signo de f ′ negativo positivo
Monotonía decreciente creciente

6.2 Curvatura de una función. Puntos de inflexión.

Definiciones
Sea f una función derivable en el punto x0 .
Diremos que f es cóncava en el punto x0  si existe un entorno de x0 de radio
δ tal que para todo x del entorno la tangente a la curva en x0 queda por debajo de
ésta, es decir, f (x ) ≥ f (x 0 ) + f ′(x0 )(x − x0 ) , ∀x ∈ Eδ (x 0 ) .
Diremos que f es convexa en el punto x0  si existe un entorno de x0 de radio
δ tal que para todo x del entorno la tangente a la curva en x0 queda por encima de
ésta, es decir, f (x ) ≤ f (x 0 ) + f ′(x0 )(x − x0 ) , ∀x ∈ Eδ (x 0 ) .
Diremos que f tiene un punto de inflexión en x0 si la función cambia de
curvatura en ese punto (pasa de cóncava a convexa o viceversa)

6-2
Monotonía y optimización de funciones Tema 6

6.3 Condiciones suficientes para la curvatura y el cálculo de extremos


relativos

Proposición (Criterios para la curvatura) Sea f una función definida en un entorno de un


punto a que admite derivada segunda en a , entonces:
• si f ′′(x 0 ) > 0 , entonces f es cóncava en x 0 .
• si f ′′(x 0 ) < 0 , entonces f es convexa en x 0 .

Si la f ′′(x0 ) = 0 no podemos determinar su curvatura sin hacer un estudio previo de lo


que sucede a su izquierda y a su derecha. De la proposición anterior se deduce que la
condición necesaria para que haya punto de inflexión es que la segunda derivada se anule en
ese punto.

Ya vimos (Tema 4) que si una función es derivable en un punto x 0 y en este punto la


función alcanza un máximo o un mínimo, entonces f ′(x 0 ) = 0 . Sin embargo, la condición no es
suficiente. Basta observar, por ejemplo, la función f (x ) = x 3 ; verifica que f ′(0 ) = 0 pero no tiene
extremo en 0 porque es creciente en todo entorno del origen. La siguiente proposición da una
condición suficiente para el cálculo de extremos.

Proposición (Criterios de la derivada segunda) Sea f una función definida en un entorno


de un punto a que admite derivada segunda en a , tal que f ′(x 0 ) = 0 , entonces:
• si f ′′(x 0 ) > 0 , entonces f tiene un mínimo relativo en x 0 .
• si f ′′(x 0 ) < 0 , entonces f tiene un máximo relativo en x 0 .

Ejemplo 6.3 Hallemos los extremos relativos de la función f (x ) = 2 x 3 − 3 x 2 − 12 x + 5 .

Si hay extremos relativos, éstos estarán entre los puntos que anulan la primera
derivada. Así pues, calculamos f ′(x ) = 6 x 2 − 6 x − 12 ; igualamos a cero, se tiene:
f ′(x ) = 0 ⇔ x = −1 , x = 2 .
Calculamos las segundas derivadas en estos puntos. Como f ′′(x ) = 12 x − 6 , f ′′(− 1) < 0
y f ′′(2 ) > 0 . Por lo tanto, según el criterio anterior, f tiene un máximo relativo en –1 y un mínimo
relativo en 2 .

Nota Este criterio funciona en la mayoría de los casos, sin embargo nos deja sin saber qué
pasa cuando f ′′(x 0 ) = 0 . Por ejemplo, la función f (x ) = x 4 tiene un mínimo en x = 0 y, sin
embargo, f ′′(0 ) = 0 . El siguiente teorema introduce un criterio más general que responde
también a esta necesidad.

Teorema Sea f una función definida en un entorno de x 0 y n veces derivable en ese punto.
Supongamos que f ′(x 0 ) = 0 , y sea n (mayor o igual que 2) el orden de la primera derivada
no nula en ese punto, entonces:
• Si n es par, f tiene un extremo relativo en x 0 , siendo
o mínimo , si f ( n ) (x 0 ) > 0 .
o máximo, si f ( n ) (x 0 ) < 0 .
• Si n es impar, f no tiene extremos relativos y tiene un punto de inflexión.

6-3
Monotonía y optimización de funciones Tema 6

Ejemplo 6.4 Comprobemos que la función f (x ) = cos (2 x ) + 2 x 2 − 4πx + 1 tiene un mínimo


relativo en x = π .
f ′(x ) = −2sen (2 x ) + 4 x − 4π , como f ′(π ) = 0 es posible que en x = π haya un extremo relativo.
Calculamos f ′′(x ) = −4 cos (2 x ) + 4 , como f ′′(π ) = 0 hay que seguir derivando. Calculamos
f (3 ) (x ) = 8sen (2 x ) , como f (3 ) (π ) = 0 hay que seguir derivando. Calculamos
f (4 ) (x ) = 16 cos(2 x ) , como f (4 ) (π ) = 16 > 0 , el teorema anterior asegura que en x = π hay un
mínimo relativo.

6.4 Problemas de optimización


Optimizar una función es hallar, según interese, los puntos donde la función alcanza su
máximo o su mínimo.
Así, por ejemplo, si con un determinado material queremos fabricar un recipiente de
volumen fijo, la optimización supondrá determinar la mínima cantidad de material que requiere
tal fabricación. Por el contrario, si fijamos la cantidad de material para construir un recipiente, la
optimización se conseguirá hallando el recipiente de volumen máximo que puede construirse
con dicha cantidad.
Por lo general, este tipo de problemas siempre impone a la función involucrada algún
tipo de restricción. Por ello, no es un simple cálculo de extremos absolutos en el campo de
existencia de la función, pues sabemos que algunas de ellas no los tienen, sino el cálculo de
los mismos al restringir la función a un dominio de definición concreto.
El primer paso, pues, es determinar la función a optimizar y, a continuación, establecer
el dominio en el que hay que determinar los extremos.
En lo que sigue, podemos suponer que la función a optimizar es siempre derivable y,
como todo extremo absoluto es relativo, empezamos siempre buscando éstos mediante los
criterios anteriormente expuestos.

En general, la estrategia a seguir ante estos problemas es:

• Encontrar la función que se ha de optimizar.


• Si la función depende de dos o más variables, encontrar las relaciones que las liga y
que nos permitan convertirla en función de una sola variable.
• Determinar el dominio que el enunciado del problema impone a la función.
• Hallar los extremos relativos de la función.
• Comparar los valores que alcanza la función en ellos o excluir aquellos que no tengan
sentido en nuestro problema.
• Expresar la solución.
Ejemplos

Ejemplo 6.5 Halla el máximo volumen que puede tener un cono de revolución cuya generatriz
mide 4 m.
1
Sabemos que el volumen de un cono viene dado por V = π r 2 h , función que depende de dos
3
variables, pero como el enunciado exige que la generatriz tenga 4 m y el Teorema de
Pitágoras asegura que h 2 + r 2 = 16 , podemos expresar V en función de h como
1 1
3
( ) 3
( )
V = π ⋅ 16 − h 2 ⋅ h = π ⋅ 16 ⋅ h − h 3 . Para que la solución tenga sentido, h y V deben ser
valores positivos, por lo que es fácil ver que 0 < h < 4 debe ser el dominio de definición de V.

6-4
Monotonía y optimización de funciones Tema 6

16
Derivando esta función e igualándola a cero se tiene que 16 − 3 ⋅ h 2 = 0 ⇔ h = ± .
3
Evidentemente, el valor negativo carece de sentido, por lo que sustituyendo el otro en
V ′′ = −6 ⋅ h se puede ver que es menor que 0 y, como consecuencia, el máximo volumen se da
16
para h = , ya que para 0 y 4 la función se anula. Sustituyendo en V se obtiene que la
3
128π
solución es V = 3.
27

Ejemplo 6.6 Dos postes de 20 y 28 metros de altura


respectivamente, se encuentran a 30 m de distancia. Se han de
sujetar con cables fijados en un solo punto, desde el suelo a los d d’
extremos de los postes. ¿Dónde se han de fijar los cables para
que la cantidad de cable a emplear sea mínima?

Si llamamos d y d’ a las longitudes de los cables (ver figura) y


aplicamos el Teorema de Pitágoras a los dos triángulos x 30 – x
rectángulos que resultan, la longitud total del cable, L, vendrá
dada en función de x por L( x ) = 28 2 + x 2 + 20 2 + (30 − x )2 . Evidentemente, para que L
exista y la solución tenga sentido, x debe estar comprendido entre 0 y 30.
Derivando la función e igualando a cero resulta
x

(30 − x ) = 0 , cuya
20 + (30 − x )
2 2 2 2
28 + x
solución es x = 17.5 . Sustituyendo este valor en la derivada segunda comprobamos que
L ′ ′(17.5) > 0 , lo que indica que L al canza un mínimo relativo si el cable se fija a 17’5 metros
del poste de 28 metros de altura. Evidentemente, este mínimo es absoluto en el intervalo de
definición.

Nota Si f es continua en un intervalo abierto (a, b ) y tiene un único extremo relativo en (a, b ) ,
entonces dicho extremo es también absoluto.

Ejemplos

Ejemplo 6.7 El perímetro de la ventana del dibujo mide 6 metros.


Los dos lados superiores forman entre sí un ángulo de 90º y son
iguales. Calcula la longitud de los lados a y b para que el área de la
ventana sea máxima.

Si llamamos l al lado superior, el área será la suma de la parte a a


l2
rectangular y la triangular que sería S = a ⋅ b + , por ser el b
2
2 2
triángulo rectángulo. Por el Teorema de Pitágoras 2 ⋅ l = b , luego
b2
S = a⋅b + (I).
4
6 − b − 2b 1+ 2
Por otro lado, como 2l + 2a + b = 6, 2 ⋅ b + 2a + b = 6 y a = =3− b,
2 2
1+ 2 2 2
sustituyendo en (I) queda S = 3b − b . Evidentemente 0 < b < 6 .
4
6 3 2
Derivando e igualando a cero se obtiene b = y, por lo tanto, a = .
1+ 2 2 1+ 2 2

6-5
Monotonía y optimización de funciones Tema 6

Como la expresión que nos da el área es un polinomio de grado 2 con coeficiente


principal negativo, es evidente que se trata de un máximo y no es necesario estudiar el signo
de la derivada segunda.

Ejemplo 6.8 En un semicírculo de radio 10 metros se


quiere inscribir un rectángulo, uno de cuyos lados esté
sobre el diámetro y el opuesto a él tenga sus extremos
en la parte curva. Calcula las dimensiones del
rectángulo para que el área sea máxima.

Llamando a y b a los lados horizontal y vertical del


rectángulo, el área del mismo es S = a ⋅ b .

Si consideramos un radio hasta uno de los vértices superiores del rectángulo, por el
2
 a 2 400 − a 2
Teorema De Pitágoras se tiene 100 =   + b , resultando S( a ) = a ⋅ , con
 2 2
0 < a < 20 .
200 − a 2
La derivada es S ′( a ) = , que es igual a cero para a = 10 2 y, por tanto,
400 − a 2
b=5 2.
(
− 2a ⋅ 400 − a 2 − 200 − a 2 ⋅) −a
400 − a 2
Como S′′(a ) = es negativa para ese valor de a,
400 − a 2
se trata de un máximo relativo (que es el máximo absoluto buscado).

Ejemplo 6.9 Se considera una ventana rectangular en la que el lado superior ha sido
sustituido por una semicircunferencia. Sabiendo que el perímetro de la ventana es 6 m, halla
las dimensiones a y b para que la superficie sea máxima.

π 2
El área será la suma de la del rectángulo y la del semicírculo, esto es, S = a ⋅ b +
⋅ b que es
8
una función de dos variables. Con la restricción de que el perímetro sea de 6 m, se tiene
b
2a + b + π ⋅ = 6 , que despejando a y sustituyendo su expresión en
2
b2 π ⋅ b2
la función inicial, tenemos S( b ) = 3b − − con 0 < b < 6 .
2 8
π⋅b
Derivando, S ′( b ) = 3 − b − , e igualado a cero se obtiene
4
12
b= , y como la función que nos da el área es polinómica de
4+π
grado dos con el coeficiente principal negativo es evidente que
6
corresponde a un máximo. El valor correspondiente para a es a = .
4+π

6-6
Monotonía y optimización de funciones Tema 6

Ejercicios resueltos

Funciones
2 x2
R.1. Calcula los extremos de las siguientes funciones: a) y = e − x +1
b) y = .
ln x
2
+1
Soluciones: a) La derivada es y ′ = −2 xe − x , se hará el ejercicio de dos modos diferentes.
2
+1
Método I. Estudiamos el signo de la derivada. Como el factor e −x es siempre positivo el
signo de la derivada vendrá dado por el factor − x y en consecuencia si x < 0 , y ′ > 0 y la

función es creciente; si x > 0 , y ′ < 0 y la función es decreciente y la función tiene un máximo

relativo en x = 0 .
2
Método II. Con el criterio de la derivada segunda. y ′ = 0 ⇒ −2 xe − x +1
= 0 ⇒ x = 0 , en este

punto puede haber máximo o mínimo relativo. Calculamos f ′′(x ) = 4 x 2 − 2 e − x ( ) 2


y como

f ′′(0 ) = −2 < 0 tenemos un máximo relativo en x = 0 .

x ⋅ (2 ln x − 1)
b) Calculamos y ′ = , hacemos y ′ = 0 ⇒ x ⋅ (2 ln x − 1) = 0 y se tiene x = 0 que no
(ln x )2
1
es del dominio de la función y 2lnx - 1 = 0 ⇒ ln x = ⇒ x = e que es el único punto donde
2
puede haber un extremo, para distinguir si es máximo o mínimo relativo se estudia el signo de

(2 ln x + 1) ⋅ (ln x )2 − x (2 ln x − 1)(2 ln x ) 1
la derivada segunda y ′′ = x . Al sustituir el valor e en y ′′
(ln x )4
se observa que el segundo sumando del numerador se anula, el denominador es positivo al

igual que el factor (ln x )2 y por tanto el signo de y ′′ vendrá dado por el de la expresión

(2 ln x + 1) , en consecuencia ( )
f ′′ e > 0 y la función presenta un mínimo relativo en x = e .

2
R.2. Halla el punto o los puntos de la curva y = e − x en los que la recta tangente tiene
pendiente máxima y calcula su valor.
2
Solución: La función derivada y ′ = −2 xe − x es la que nos da la pendiente de la curva dada en
2
cada punto, por lo tanto se trata de encontrar el máximo absoluto de f (x ) = −2 xe − x ,

evidentemente dicho máximo se encuentra en (− ∞,0 ) donde la pendiente es positiva.

Estudiamos el signo de f ′(x ) en ese intervalo ( )


f ′(x ) = 4 x 2 − 2 e − x ,
2

 1
si x<− → f ′(x ) > 0 ⇒ f (x ) es creciente
 1 2
 x = − 
2  1
f ′(x ) = 0 ⇒ 4 x − 2 = 0 ⇒ 
2
luego: si x>− → f ′(x ) < 0 ⇒ f (x ) es decrecient e
1 2
x= 
 2  1
si x=− → f (x ) tiene un máximo relativo
 2

6-7
Monotonía y optimización de funciones Tema 6

dicho máximo relativo es el máximo absoluto buscado ya que la función es creciente a la


izquierda y decreciente a la derecha en el intervalo estudiado. Por último el valor máximo de
1
2 −
2 2
dicha pendiente es m = e = .
2 e

R.3. Aplica el criterio general para saber si las gráficas de las funciones que se indican
presentan máximo, mínimo o un punto de inflexión en los puntos especificados.
a) y = (x − 1) + 3 en x = 1 b) y = (x − 1) e x en x = 1 c) y = (x − 3 ) en x = 3
5 3 4

Soluciones: a) Calculamos las sucesivas derivadas de la función en x = 1 hasta encontrar una


distinta de cero:
y ′(x ) = 5(x − 1) ⇒ y ′(1) = 0
4

y ′′(x ) = 5 ⋅ 4(x − 1) ⇒ y ′′(1) = 0


3

y ′′′(x ) = 5 ⋅ 4 ⋅ 3(x − 1) ⇒ y ′′′(1) = 0


2

y (4 ) (x ) = 5 ⋅ 4 ⋅ 3 ⋅ 2(x − 1) ⇒ y (4 ) (1) = 0
y (5 ) (x ) = 5 ⋅ 4 ⋅ 3 ⋅ 2 ⇒ y (5 ) (1) = 5! Como la primera derivada distinta de cero es de orden impar en
x = 1 hay un punto de inflexión.

b) Derivamos sucesivamente como en el caso anterior.


y ′(x ) = (x − 1) (x + 2)e x ⇒ y ′(1) = 0
2

( )
y ′′(x ) = (x − 1) ⋅ x 2 + 4 x − 1 e x ⇒ y ′′(1) = 0
( 2
) x
( )
y ′′′(x ) = x + 4 x − 1 e + (x − 1) ⋅ x 2 + 6 x + 3 e x ⇒ y ′′(1) = 4e Como es de orden impar, en x = 1
hay un punto de inflexión.

c) Actuamos como en los dos casos anteriores


y ′(x ) = 4 ⋅ (x − 3 ) ⇒ y ′(3 ) = 0
3

y ′′(x ) = 4 ⋅ 3 ⋅ (x − 3 ) ⇒ y ′′(3 ) = 0
2

y ′′′(x ) = 4 ⋅ 3 ⋅ 2 ⋅ (x − 3 ) ⇒ y ′′′(3 ) = 0
y (4 ) (x ) = 4 ⋅ 3 ⋅ 2 ⇒ y (4 ) (3 ) = 4! > 0 . Como la primera derivada distinta de cero en x = 3 es de
orden par hay un extremo, como además es positiva se trata de un mínimo relativo.

Problemas de optimización

R.4. Descompón 150 en dos sumandos ninguno de ellos negativo de forma que el triple
del primero por el cuadrado del segundo sea máximo.

Solución: Construimos que función que queremos optimizar.


Conviene llamar x al segundo número, así el primero será 150 − x y la función
f (x ) = 3 ⋅ (150 − x )x 2 con dominio D = [0,150] , evidentemente el máximo no está en ninguno de
los extremos del dominio donde la función se anula, por lo que el máximo absoluto, que es lo
que realmente buscamos, es también un máximo relativo y en ese punto la derivada de la
función se anula. Buscamos los puntos donde f ′(x ) = 0 y elegiremos el que nos conduzca a un
máximo relativo.
x = 0
( )
f ′(x ) = 3 ⋅ − 3 x 2 + 300x , f ′(x ) = 0 ⇒ −3 x 2 + 300 x ⇒  . Calculamos la derivada
 x = 100
segunda. f ′′(x ) = 3 ⋅ (− 6 x + 300) ⇒ f ′′(100) < 0 luego en x = 100 hay un máximo relativo, que es
el máximo absoluto pedido, x = 0 ni siquiera lo consideramos por razones obvias.
En conclusión los números pedidos son 50 y 100 .

6-8
Monotonía y optimización de funciones Tema 6

R.5. Descompón el número 4 en dos sumandos positivos tales que la diferencia de uno
de ellos con el inverso del otro sea máxima.

1
Solución: Llamamos a los números 4 − x y x respectivamente. La función es f (x ) = 4 − x −
x
con dominio D = (0,4 ) , como en el caso anterior derivamos e igualamos a cero.
1 1− x2
f ′(x ) = −1 + = ⇒ x = 1 como única solución de f ′(x ) = 0 ya que x = −1 queda fuera del
x2 x2
2
dominio. Calculamos f ′′(x ) = −⇒ f ′′(1) < 0 luego es un máximo relativo que es el máximo
x3
absoluto buscado. Los números buscados son respectivamente 3 y 1.

Nota: Se podría también haber estudiado el signo de f ′(x ) encontrando que la función es
creciente para x ∈ (0,1) y decreciente para x ∈ (1,4 ) con lo que se llega a idéntica conclusión.

R.6. Se desea vallar un recinto rectangular de 100 m2 pegado a un muro ya existente


como se indica en la figura. La valla perpendicular al muro tiene un coste de 6 €/m y la
parte paralela al muro 15 €/m. ¿Qué dimensiones debe tener el recinto para que el coste
sea mínimo?.

Solución: La función que nos da el coste es


C (x, y ) = 6 ⋅ 2 x + 15y = 12x + 15 y que depende de dos variables,
pero que está sujeta a la restricción x ⋅ y = 100 en la que
100 1500
despejando y = y sustituyendo C (x ) = 12 x + , función
x x
de una variable que somos capaces de manejar y en la que buscaremos el mínimo. Es fácil ver
que el mínimo absoluto no se da para valores de x muy grandes , pues el primer sumando hace
que el coste sea grande y tampoco para valores de x próximos a cero pues el segundo
sumando es ahora grande, así el mínimo que buscamos es también mínimo relativo.
1500
Derivamos C ′(x ) = 12 − 2 , haciendo C ′(x ) = 0 se tiene x 2 = 125 ⇒ x = 125 = 5 5
x
como única solución dentro del dominio de la función. Calculamos
3000
( )
C ′′(x ) = 3 ⇒ C ′′ 125 > 0 luego es un mínimo. Las dimensiones del recinto de menor coste
x
100
son x = 5 5 , y = =4 5.
5 5

R.7. Un objeto se mueve verticalmente y hacia arriba, la función que nos da la altura en
metros dependiendo del tiempo en segundos es h (t ) = 10 + 15t − 5t 3 . Se pide:
a) ¿Desde qué altura se inicia el movimiento?.
b) Si consideramos que la altura no puede ser negativa, ¿cuál es el dominio de la
función?.
c) ¿Cuál es la velocidad inicial?.
d) ¿Cuál es la altura máxima?.

Solución:
a) Calculamos h (0 ) = 10 m , altura inicial.
b) Resolvemos la ecuación h (t ) = 0 ⇒ 10 + 15t − 5t 3 = 0 , las soluciones son t = 2 y t = −1
(doble), la única que en nuestra situación tiene sentido es t = 2 y el dominio será t ∈ [0,2] .
c) Calculamos v (t ) = h ′(t ) = 15 − 15t 2 , de donde v (0 ) = 15 m/s .
d) Hacemos h ′(t ) = 0 ⇒ 15 − 15t 2 = 0 de soluciones t = ±1 , sólo la solución t = 1 tiene sentido,
comprobamos que se trata de un máximo. Calculamos h ′′(t ) = −30t ⇒ h′′(1) = −30 < 0 , es un
máximo, por tanto la altura es máxima al cabo de 1 segundo y es de 20 m.

6-9
Monotonía y optimización de funciones Tema 6

R.8. Se desea construir un vaso cónico a partir de una placa circular de radio R al que se
le corta un sector circular y se unen los bordes, como en la figura.¿Qué sector hay que
cortar para que el volumen sea máximo?.

Solución: Si se corta un sector muy pequeño el cono


formado tendrá mas altura pero al ser la base muy
pequeña el volumen es pequeño, éste aumenta al
aumentar el sector cortado, pero en algún momento
empieza a disminuir tendiendo a cero cuando el sector
que quitamos es cada vez mayor, por lo tanto el máximo
se presenta en un valor intermedio y será además máximo
relativo.
Consideramos una situación intermedia cualquiera. Si llamamos h a la altura y r al
π
radio de la base del cono formado, la función que debemos optimizar es V = r 2 h que
3
depende de dos variables, pero se tiene que r 2 = R 2 − h 2 y queda
π
(
V = R 2 − h2 h =
3
) π 2
3
( ) π
R h − h 3 , derivamos e igualamos a cero V ′(h ) = R 2 − 3h 2
3
( )
3
(
π 2
)
R − 3h 2 = 0 ⇒ h = ±
R
3
, solo es del dominio h =
R
3
, comprobamos que conduce a un

máximo V ′′(h ) =
π
(− 6h ) ⇒ v ′′ R  < 0 luego es un máximo relativo que es el máximo absoluto
3  3
R 6R
buscado. Si h = se tiene que r = . Por otra parte como la longitud de la
3 3
circunferencia de la base del cono es igual al arco de la placa a la que se le ha quitado el
6R
sector de ángulo α y entonces se tiene 2πr = 2πR − αR , es decir 2π = 2πR − αR y
3
 6 
despejando el ángulo α = 2π  1 − rad luego el ángulo del sector que conduce al cono de
 3 

mayor volumen es independiente del radio de la placa, como por otra parte es previsible.

R.9. Se desea construir una balsa con forma de prisma recto de base cuadrada de
108 m3 de capacidad, el revestimiento de las paredes y del fondo cuestan lo mismo.
¿Qué dimensiones ha de tener para que el coste de dicho revestimiento sea mínimo?.

Solución: Si llamamos x al lado del fondo e y a la altura de la balsa, la función que debemos
optimizar es S (x, y ) = x 2 + 4 xy restringida por la expresión x 2 y = 108 ya que el volumen está
108 432 108
fijado. Despejando y = 2
y sustituyendo S (x ) = x 2 + 4 x = x2 +
, viendo la función es
2
x x x
fácil comprobar que el mínimo está en una situación intermedia, así que buscamos el mínimo
432 432
relativo para lo que derivamos e igualamos a cero S ′(x ) = 2 x − 2 ⇒ 2 x − 2 = 0 ⇒
x x
864
⇒ x = 216 ⇒ x = 6 . Comprobamos que hay un mínimo. Calculamos S ′(x ) = 2 + 3 S ′(6 ) > 0
3
x
luego es un mínimo. Las dimensiones pedidas son x = 6 m e y = 3 m .

R.10. Consideramos todas las rectas que pasan por el punto P (1,2) y que cortan a los
ejes en la parte positiva. Al girar el segmento de recta limitado por los ejes alrededor de
OY se forma un cono. ¿Cuál sería la ecuación de la recta que formase el cono de menor
volumen?.

6-10
Monotonía y optimización de funciones Tema 6

Solución: Como puede verse en la figura la función que se ha de


π
optimizar es el volumen del cono V = a 2b (1) donde a y b son
3
los segmentos determinados por la recta con los ejes X e Y
respectivamente, de la ecuación segmentaria de la recta
x y 1 2
+ = 1 y como ha de pasar por P (1,2) se tiene + =1
a b a b
2a 2π a3
de donde b = y sustituyendo en (1) V (a ) = con dominio los valores a > 0 . En esta
a −1 3 a −1
2π a 2 (2a − 3 )
función derivamos y simplificamos V ′(a ) = , el único valor del dominio que hace
3 (a − 1)2
3
V ′(a ) = 0 es a = además es inmediato comprobar que V ′(a ) < 0 y por lo tanto la función
2
3 3
decreciente si 0 < a < e igualmente que V ′(a ) > 0 y por lo tanto la función creciente si a >
2 2
luego la función presenta un mínimo relativo que es también el mínimo absoluto que buscamos.
Sustituyendo el valor de a se tiene b = 6 con lo que la ecuación de la recta pedida es
x y
+ = 1 , o bien 4 x + y = 6 .
3 6
2

π
R.11. De todos los conos de volumen dm3 ¿cuál es el tiene menor área lateral?.
3

Solución: El área lateral de un cono es S = πrg siendo r el radio de la base y g la generatiz, si


llamamos h a la altura del cono y por el Teorema de Pitágoras g = h 2 + r 2 y por tanto
π 2 π 1
S = πr h 2 + r 2 , pero como el volumen del cono es constante se tiene: r h= ⇒h= 2
3 3 r
1 1
que sustituida en el área queda S = πr 4
+r2 =π 2
+ r 4 con dominio r > 0 .
r r
2
− 3
+ 4r 3
Derivamos e igualamos a cero con lo que se tiene: S ′(r ) = π r =0⇒
1 4
2 +r
r2
2 1
⇒ 3
= 4r 3 ⇒ r = , estudiamos el signo de la derivada para comprobar que se trata de un
6
r 2
2r 6 − 1 1 1
mínimo S ′(r ) = π . Si 0 < r < → S ′(r ) < 0 y la función es decreciente, si r >
6 6
1 2 2
2r 3
2
+r4
r
S ′(r ) > 0 y la función es creciente luego es un mínimo. El cono pedido tiene de dimensiones
1 1
r= dm y sustituyendo h = dm .
6 3
2 2

R.12. De todos los cilindros inscritos en una esfera de radio R como


en la figura, calcula las dimensiones de aquel que tiene mayor área
lateral.

Solución: La función que se ha de optimizar es A = 2π ⋅ r ⋅ h (1) y como se


ve en la figura se debe cumplir siempre que (2R ) = (2r ) + h 2 de donde
2 2

despejando h y sustituyendo en (1) se tiene A(r ) = 4π ⋅ R 2 r 2 − r 4 con

6-11
Monotonía y optimización de funciones Tema 6

dominio D = (0, R ) .
Es evidente que para valores de r muy pequeños o próximos a R el área es pequeña,
así que el máximo buscado se da en una situación intermedia y es por tanto también un
R 2r − 2r 3
máximo relativo, buscamos dónde la derivada A′(r ) = 4π ⋅ se anula, A′(r ) = 0 nos
R 2r 2 − r 4
R
lleva a que R 2r = 2r 3 ⇒ r = como única solución dentro del dominio, estudiamos el signo
2
2 3
(R 2
− 6r 2 )Rr
2 2
(
− r 4 − R 2 r − 2r 3 ) R r − 2r
de la derivada segunda A′′(r ) = 4π ⋅ R 2r 2 − r 4 , el signo de
R 2r 2 − r 4
 R 
A′′  se puede ver fácilmente si nos damos cuenta de que el segundo sumando del
 2
numerador se anula y el denominador es positivo.

Por lo tanto el signo de la expresión viene dado por el de R 2 − 6r 2 que al sustituir


2
 R 
queda R 2 − 6  = −2R 2 < 0 y es un máximo.
 2
R
Las dimensiones del cilindro son r = y sustituyendo h = 2R (compruébese que
2
se trata de un cilindro de la misma altura que anchura).

R.13. En el recinto formado por la gráfica de la función y = 3 x los ejes de coordenadas


y la recta de ecuación x = 4 , se inscriben rectángulos con
un vértice situado sobre la curva, la base en el eje OX y un
lado sobre la recta x = 4 como se indica en la figura.
Calcula las coordenadas del punto P que forma el
rectángulo de mayor área.

Solución: Construimos la función de la que debemos encontrar


el máximo que es el área del rectángulo cuya base es
b = 4 − x y de altura y S (x, y ) = (4 − x )y pero como y = 3 x
se tiene S (x ) = (4 − x ) 3 x con dominio D = (0,4 ) , derivamos

e igualamos a cero S ′(x ) = − 3 x +


(4 − x )3 = 0 que lleva a − 2 ⋅ 3 x + (4 − x )3 = 0 con lo que
2 3x
4 3 (3 x + 4 )
x= Calculamos la derivada segunda y simplificamos y queda S ′′(x ) = − y
3 4x 4
4 4
S ′′  < 0 luego es un máximo y las coordenadas del punto son x = e y = 2 .
3 3

R.14. Dos focos luminosos A y B separados 50 metros tienen intensidades luminosas


respectivas de 135 y 40 candelas. Sabiendo que la intensidad luminosa es inversamente
proporcional al cuadrado de la distancia al foco, se quiere saber qué punto de la línea
entre ambos focos es el menos iluminado.

Solución: Si llamamos x a la distancia de un punto cualquiera al foco A, la distancia a B será


135 40
50 − x y la intensidad luminosa en ese punto será I (x ) = 2 + . Estudiamos dónde la
x (50 − x )2
135 ⋅ 2 40 ⋅ 2
función presenta el mínimo. Derivamos I ′(x ) = − + igualando a cero y
x3 (50 − x )3

6-12
Monotonía y optimización de funciones Tema 6

135 ⋅ 2 40 ⋅ 2 27 8 3 2
simplificando se tiene = ⇒ = ⇒ = ⇒ x = 30 Hacemos la
x 3
(50 − x ) 3
x 3
(50 − x )3
x 50 − x
240 810
derivada segunda y simplificamos I ′′(x ) =+ y para x = 30 es positiva con lo que
x4 (50 − x )4
tenemos un mínimo. En conclusión el punto menos iluminado se encuentra a 30 m del foco A y
20 m del foco B.

R.15. Calcula el área del mayor rectángulo que puede inscribirse en la elipse de ecuación
x2 y 2
+ = 1.
4 9

Solución: La función de la que buscamos el máximo es


S (x, y ) = 4 xy con x e y positivos y sujeta a la restricción
x2 y 2 3
+ = 1 de donde despejando y = 4 − x 2 y sustituyendo
4 9 2
en el área queda S (x ) = 6 x 4 − x 2 = 6 4 x 2 − x 4 con dominio
D = (0,2) . Para localizar el máximo derivamos e igualamos a
2 − x2
cero S ′(x ) = 12 , S ′(x ) = 0 ⇒ 2 − x 2 = 0 ⇒ x = 2
como única solución dentro del
2
4−x
dominio comprobamos que se trata de un máximo Calculando S ′′(x ) y simplificando queda

(
− 2x 4 − x 2 − 2 − x 2 ) −x

S ′′(x ) = 12
2
4 − x 2 = 12 x − 6 x ,( ) es inmediato comprobar que
2
4−x
4 − x2
3
( )
( )
S′′ 2 < 0 y que se trata de un máximo. Para x = 2 ⇒ y =
3 2
2
, el rectángulo tiene de base

2 2 y de altura 3 2 con lo que el área pedida es S = 6 u. a. .

R.16. En un rectángulo de base 1 m y altura 2 m se circunscriben triángulos isósceles tal


y como se indica en la figura. Calcula las dimensiones de aquel que tenga área mínima.

Solución: La función que nos da el área de cualquiera de esos


1
triángulos es A(b, h ) = b ⋅ h siendo b la longitud de la base y h
2
la altura del triángulo respectivamente , pero si nos fijamos en
los triángulos ABC y DEB, que son semejantes, se tiene que
h 2 2b b2
= de donde despejando h = y A(b ) =
b (b − 1) b −1 b −1
2 2
con dominio D = (1,+∞ ) , es fácil ver que el mínimo que buscamos es también mínimo relativo
b 2 − 2b
por lo que derivamos e igualamos a cero. A′(b ) = = 0 ⇒ b = 2 como única solución
(b − 1)2
dentro del dominio de la función. A′(b ) < 0 si 1 < b < 2 y la función decreciente. A′(b ) > 0 si
b > 2 y la función es creciente luego en b = 2 hay un mínimo. Las dimensiones del triángulo
de menor área son b = 2 m y h = 4 m .

6-13
Monotonía y optimización de funciones Tema 6

R.17. Calcula el punto de la hipérbola de ecuación x 2 − y 2 = 1 con x > 0 que está a la


menor distancia del punto P (0,3 ) .

Solución: Si llamamos x e y a las coordenadas del punto buscado,


la función que nos da la distancia entre los dos puntos es
d = x 2 + (y − 3 ) , como el punto pertenece a la hipérbola
2

Se cumple que x 2 − y 2 = 1 y despejando x 2 = 1 + y 2 con lo que


2
d (y ) = 1 + y 2 + (y − 3 ) y simplificando d (y ) = 2y 2 − 6 y + 10 .
2y − 3 3
Derivamos e igualamos a cero d ′(y ) = =0⇒y =
2
2y − 6 y + 10 2
3
es inmediato comprobar que si y < se tiene que d ′(y ) < 0 con lo que la función es
2
3 3
decreciente y si y > d ′(y ) > 0 y la función crece, luego el valor y = corresponde a un
2 2
 13 3 
mínimo como buscamos. El punto de la hipérbola es el de coordenadas  , .
 2 2
 

R.18. Halla el máximo volumen que puede tener un cono de revolución cuya generatriz
mide 4 metros.
1 2
Solución: Sabemos que el volumen de un cono viene dado por V = π r h , función que
3
depende de dos variables, pero como el enunciado exige que la generatriz tenga 4 m y el

Teorema de Pitágoras asegura que h 2 + r 2 = 16 , podemos expresar V en función de h como


1 1
V=
3
( ) ( )
π ⋅ 16 − h 2 ⋅ h = π ⋅ 16 ⋅ h − h 3 . Para que la solución tenga sentido, h y V deben ser
3
valores positivos, por lo que es fácil ver que 0 < h < 4 debe ser el dominio de definición de V.

16
Derivando esta función e igualándola a cero se tiene que 16 − 3 ⋅ h 2 = 0 ⇔ h = ± .
3
Evidentemente, el valor negativo carece de sentido, por lo que sustituyendo el otro en
V ′′ = −6 ⋅ h se puede ver que es menor que 0 y, como consecuencia, el máximo volumen se da

16
para h = , ya que para 0 y 4 la función se anula. Sustituyendo en V se obtiene que la
3
128π
solución es V = 3.
27

6-14
Monotonía y optimización de funciones Tema 6

Ejercicios propuestos

Problemas de máximos y mínimos relativos

P.1. Calcula los valores de a, b, c, y d para que la función f ( x ) = ax 3 + bx 2 + cx + d tenga el

máximo (0,4 ) y el mínimo (2,0 ) .

P.2. Calcula los coeficientes de una función polinómica de grado 3 que tiene un mínimo en P (1,1) y
1 4
un punto de inflexión en x = con tangente paralela a y = − x .
3 3

P.3. La función f ( x ) = x 3 + px 2 + q tiene en x = 2 un mínimo relativo igual a 3. Halla p y q.

Sabiendo que tiene también un máximo relativo, halla dónde se alcanza.

P.4. ¿Qué valores deben tomar a, b, c y d para que la función f ( x ) = ax 3 + bx 2 + cx + d tenga

un punto crítico en P (1,3 ) (su primera derivada es nula) y un punto de inflexión con tangente de

5
ecuación y = 2 x + en x = 0 ?
3
1
P.5. Halla los puntos de la curva y = en los que la recta tangente tiene pendiente
1+ x 2
máxima y el valor de esta pendiente.

P.6. Indica si las gráficas de las funciones que se indican presentan máximo, mínimo o un punto
de inflexión en los puntos que se especifican en cada caso.
a ) y = x 5 ; b) y = x 8 ; c ) y = x 4 . e x ; d ) y = x 3 . e2x ; e ) y = senx - x ⋅ cosx en x = 0
(x - 7)4 + 11 (x - 7)5 + 11
f)y= ; g) y = en x = 7
-3 2

Problemas de optimización

P.7. Descompón 30 en dos sumandos de manera que la suma de sus inversos sea mínima.

P.8. Descompón el número M en dos sumandos positivos de manera que su producto sea
máximo. Resuélvelo también para M = 40 .

P.9. La suma de las aristas de un prisma recto de base cuadrada es 24 cm. ¿Qué dimensiones
debe tener dicho prisma para que su volumen sea máximo?

P.10. Halla dos números cuya suma es 18 y el producto de uno de ellos por el cuadrado del otro
sea máximo.

P.11. Se quiere acotar un terreno rectangular que se encuentra a lo largo de un río y que tenga
una superficie de una hectárea, calcula sus dimensiones para que el gasto sea el menor posible
(recuerda que los márgenes de los ríos no se pueden vallar).

P.12. Queremos ir del punto A al C, separados por un


canal de 40 m de ancho. Si nadamos a la velocidad de
1 m/s y caminamos a 2 m/s. Sabiendo que el punto
C dista 100 m de A’, ¿cómo debe elegirse el camino

6-15
Monotonía y optimización de funciones Tema 6

ABC para llegar a C en el menor tiempo posible? ¿y si A’C = 200 m?. ¿Qué conclusión
obtienes?

P.13. De una cartulina rectangular de dimensiones a y b se recortan cuatro cuadrados iguales


(uno en cada esquina), y con la superficie resultante se construye una caja sin tapa. ¿Cómo
deben hacerse los recortes para conseguir que la caja tenga volumen máximo?. Resuélvelo
también para el caso concreto de una cartulina de 20 x 15 cm². ¿Y si la cartulina es cuadrada
de lado a?

P.14. Un fabricante es capaz de vender x artículos por semana a un precio por unidad marcado
x
por la ley de Pu = 2 − ; cada artículo le cuesta al fabricante 0’50 euros y el mantenimiento
10000
semanal, maquinaria y empleados, es de 200 euros. Calcular el nivel de producción semanal
para que el beneficio sea máximo.

P.15. Halla la distancia mínima del punto (0,5/2) a la parábola y = x 2 + 1.

Construcciones geométricas

P.16. De todos los sectores circulares que tienen de perímetro 18m, ¿qué ángulo corresponde al
de mayor área? ¿Y para un perímetro cualquiera P?.

P.17. Cuando un triángulo rectángulo de 6 cm de hipotenusa gira alrededor de ésta se forman dos
conos. Calcula su altura sobre esta hipotenusa con la condición de que la diferencia de los
volúmenes de los conos engendrados sea máxima.

P.18. Calcula las dimensiones del cono de volumen máximo inscrito en una esfera de radio R.
Particulariza el resultado si la el radio de la esfera es de 22'45cm.

P.19. Realiza un estudio similar al ejercicio anterior cuando la figura inscrita en la esfera sea un
cilindro. Resuelve un caso particular.

P.20. Queremos construir un depósito de chapa de 1500 litros de capacidad y con forma cilíndrica.
Calcula sus dimensiones para que la cantidad de chapa utilizada sea la menor posible. ¿Qué
proporciones observas? Resuélvelo para el caso general de un volumen V.
3
P.21. Se quiere construir un recipiente cilíndrico cerrado de base circular de 64 cm de
volumen. Hallar las dimensiones que debe tener para que la cantidad de metal sea mínima.

P.22. Con una cuerda de 30 m de larga se desea formar un triángulo isósceles de área
máxima, ¿cuánto medirán los lados de dicho triángulo, y cuál es el valor del área?.

P.23. De todos los rectángulos inscritos en un triángulo isósceles de base b y altura h, con un lado
del rectángulo en el lado desigual del triángulo. ¿Cuál es el de mayor área?. Resuélvelo también
para el caso de un triángulo de 10 cm de base y 15 de altura.

P.24. Halla las dimensiones del rectángulo de área máxima inscrito en una circunferencia de
5 m de radio.

P.25. De todos los triángulos isósceles inscritos en una circunferencia de radio r, ¿cuál es el
de mayor área?.

P.26. Se considera un cilindro de radio r cuya altura es igual al diámetro de la base; ¿cuál de
los conos circunscritos al mismo tiene menor volumen? (Se supone que la base del cono
contiene a la del cilindro).

P.27. Determina la ecuación de la recta que, pasando por el punto (2,3), forma con la parte
positiva de los ejes de coordenadas un triángulo de área mínima.

6-16
Monotonía y optimización de funciones Tema 6

P.28. Se desea construir un depósito de latón (con tapadera) con forma de cilindro de área total
2
igual a 54 m . Determina el radio de la base y la altura del cilindro para que el volumen sea
máximo.

P.29. El coste de producción de x unidades de un producto viene dado por la expresión


C = x 2 − 300 x + 100 euros y el precio de venta de una unidad es U = 1000 – x euros.
¿Cuántas unidades se deben fabricar para que al venderlas todas el beneficio sea máximo?

P.30. Halla las dimensiones de un depósito abierto superiormente, en forma de prisma recto de
3
base cuadrada, de 1000 m de capacidad que tenga un revestimiento interior de coste mínimo.
2 2
El precio del m de revestimiento lateral es de 100 €, El precio del m de revestimiento del
fondo es de 200 €. Halla también el coste mínimo.

Ejercicios de profundización

P.31. Una estatua de 4 m de altura se coloca sobre una base de 10 m. Hallar a qué distancia
de la base se observará la estatua bajo un ángulo máximo.

P.32. Sobre un edificio de 30 m de altura hay un cartel anunciador de 10 m de alto. ¿A qué


distancia del edificio verá el cartel bajo ángulo máximo un "diminuto" peatón que camina
perpendicularmente a la fachada? .

P.33. Sea una esfera de centro O y radio r, sea M un punto exterior a ella y d(O,M)=a. Sobre la
recta OM se toma un punto X, interior a la esfera, tal que OX=x. El plano que pasa por X
perpendicularmente a OM corta a la esfera en un círculo. Hállese el volumen máximo del cono
que tiene ese círculo por base y M por vértice.

P.34. Considérese un segmento AB sobre el semieje OX positivo. ¿Desde qué puntos de la


parábola y 2 = 2x se ve dicho segmento bajo ángulo máximo? (Supóngase que A(a,0) y B(b,0)).

P.35. Un barco está anclado a 3 Km de la costa y frente


a un punto distante 5 Km a lo largo de la costa rectilínea
está anclado otro barco 9 Km aguas afuera. Una lancha 3 Km 9 Km

procedente del primer barco ha de desembarcar a un


5 Km
pasajero en la orilla y continuar después hacia el
segundo barco. ¿Cuál será la distancia total más corta
que podrá recorrer la lancha?.

6-17
Monotonía y optimización de funciones Tema 6

Soluciones.

P.1. a = 1, b = −3, c = 0, d = 4 . P.2. a = 1, b = −1, c = −1, d = 2 P.3. p = −3, q = 7 ; en


2 5 1 3 3
x = 0 . P.4. a = − , b = 0, . P.5. x = −
c = 2, d= , M= ≅ 0′64 la función de la
3 3 3 8
−2 x
que hay que buscar el máximo es y’ o sea g ( x ) = y buscar g’(x) = 0. P.6. a) punto de
1+ x 2
2
( )
inflexión , b) mínimo, c) mínimo, d) punto de inflexión, e) punto de inflexión, f) máximo, g) punto de
1 1 1 1
inflexión. P.7. 15 y 15. La suma es S = + , S ′( x ) = − 2 + y
x 30 − x x (30 − x )2
2 2 M
S′′( x ) = + . P.8. 20 y 20. x = y = P.9. lado de la base x = 2 , altura h = 2 . El
x3 (30 − x ) 3 2
volumen V ( x ) = x 2 ⋅ h con la restricción 2 x + h = 6 , es V ( x ) = x 2 ⋅ (6 − 2 x ) . P.10. 12 y 6. P.11.
2 40 40
lado perpendicular ≅ 70'5 m lado paralelo 2 ≅ 141 m . P.12. A ′B = BC = 100 − , solo
2 3 3
a + b − a 2 + b 2 − ab
depende de la anchura del canal, pero no de A’C. P.13. x = ; x = 2’83 cm;
6
a 5
x= . P.14. 7500 artículos. P.15. d = que corresponde a los puntos (-1,2) y (1,2),
6 2
2
 3
d( x ) = x 2 +  x 2 −  d ′( x ) = 0 ⇒ x = 0, x = −1 ( mínimo ), x = 1 ( mínimo ) . P.16. 2 radianes;
 2
P ⋅r 2π
− r 2 . P.17. h = 6 , d =
igual (no depende del perímetro). A( r ) =
2 3
− x 3 + 9 x 2 − 18 x con( )
d diferencia de volúmenes y x uno de los segmentos de la altura con la hipotenusa. P.18.
4R 2 2 2 3R 6R
h= ;r = R; h = 29’93 cm y r = 21’17 cm. P.19. h= ;r = ;
3 3 3 3
 h 3  V
V ( h ) = π ⋅  R 2 h −  . P.20. r = 0' 62 m y h = 1' 24 m ; h = 2r . r = 3 . P.21.
 4  2π
32 64
r =3 ≅ 2'17 cm, h = 2 = 2r ≅ 4' 34 cm . P.22. l = 10 m , A = 25 3 m 2 . P.23.
π πr
b h
x = , y = (x, y base y altura del rectángulo); x = 5 cm , y = 7'5 cm . P.24. x = y = 50 m .
2 2
3r 3 3r πr 2 x3
P.25. h = , b= . P.26. Rc = , hc = 6r si hc = x , V ( x ) = . P.27.
2 2 2 3 (x − 2r )2
x y 3 6
+ = 1 . P.28. r = , h= V ( r ) = 27r − π ⋅ r 3 . P.29. x = 325 unid., el beneficio
4 6 π π
400000
B( x ) = −2x 2 + 1300 x − 100 y el coste C( x ) = 200 x 2 + . P.30. lado de la base x = 10 m
x
1
altura h = x . P.31. 11’832 m. P.32. 34’64 m. P.33. x = ⋅  a − a 2 + 3R 2  . P.34.
3  
1
x = ⋅  a + b − 2 ⋅ a 2 + b 2 + 8ab  . P.35. 13 km .
6  

6-18
Gráficas de funciones Tema 7

Tema 7

Representación gráfica de funciones explícitas


Representar gráficamente una función dada en forma explícita tal como y = f (x ) es
hallar el lugar geométrico de los puntos (x , f (x )) del plano real.
Para esta representación pueden usarse diversas estrategias. En esta lección,
desarrollamos aquellos conceptos que, conocidos, dan una buena información sobre la gráfica.
A la vista de la función a representar, el alumno elegirá los que convenga y, si no lo tiene claro,
puede estudiarlos todos. Tal estudio puede seguir el orden que se indica a continuación:

1. Dominio.
2. Puntos de corte con los ejes.
3. Regiones.
4. Asíntotas (horizontales, verticales y oblicuas).
5. Monotonía y extremos relativos.
6. Curvatura y puntos de inflexión.
7. Simetrías y periodicidad.

7.1 Dominio
Definición El dominio de una función es el conjunto de los valores de x que tienen
imagen; escribiremos: Dom (f ) = {x ∈ R : ∃ f ( x ) } .

En la siguiente tabla indicamos el dominio de las funciones más frecuentes.

Funciones Dominio
Polinómicas y = P( x ) R
P( x ) R excepto Q(x) =0
Racionales y =
Q( x )
Raíces y = f ( x ) o y = n f ( x ) con n par el de f (x ) y que además f ( x ) ≥ 0

Exponenciales y = a f ( x ) el de f (x )
Logarítmicas y = loga f ( x ) el de f (x ) y que además f ( x ) > 0
Trigonométricas y = sen f ( x ) , y = cos f ( x ) el de f (x )

Trigonométricas y = tgf ( x ) π
el de f (x ) y que además f ( x ) ≠ ± kπ
2
y = arcsen f ( x ) , y = arccos f ( x ) el de f (x ) con f ( x ) ∈ [− 1,1]

y = arctgf ( x ) el de f (x )

7-1
Gráficas de funciones Tema 7

Ejemplos

Ejemplo 7.1 Para f (x ) =


(x − 1)2 , no son del dominio los valores de x tales que x 2 − 4 = 0 .
x2 − 4
Por tanto, Dom(f ) = R − {− 2, 2 } .

Ejemplo 7.2 Para y = x 3 − 3 x 2 , sólo son del dominio las soluciones de la inecuación
x − 3 x ≥ 0 , Dom (f ) = {0 } U [3,+∞ ) .
3 2

Ejemplo 7.3 ( )
Para y = L x 2 − 3 x + 2 sólo son del dominio las soluciones de la inecuación
x 2 − 3 x + 2 > 0 , Dom(f ) = ( −∞,1) U ( 2, ∞ ) .

7.2 Puntos de corte con los ejes


La función corta al eje OY en el punto en el que x = 0 (si es del dominio). Corta al eje OX en los
puntos (si existen ) para los que y = 0. Estos puntos serán importantes para el estudio de las
regiones.
Ejemplos

Ejemplo 7.4 Sea y =


(x − 1)2 . El punto de corte con el eje OY , se obtiene haciendo x = 0, de
x2 − 4
1  1
donde y = − por lo que el punto es Py  0,−  . Con el eje OX, haciendo y = 0 se tiene x = 1,
4  4
luego el punto de corte es el Px (1,0 ) .

Ejemplo 7.5 Sea y = x 3 − 3 x 2 . El punto de corte con el eje OY , se obtiene haciendo x = 0,


de donde y = 0 por lo que el punto es Py (0,0 ) , que también corta a OX. Con el eje OX,
haciendo y = 0 se tiene x 3 − 3 x 2 = 0 , x = 0 y x = 3, luego Px1(0,0 ), Px 2 (3,0 ) .

( )
Ejemplo 7.6 Sea y = L x 2 − 3 x + 2 . El punto de corte con el eje OY , se obtiene haciendo x =
0, de donde y = L2 , por lo que Py (0, L2) . Con el eje OX, haciendo y = 0 se tiene
( 2
)
L x − 3x + 2 = 0 , lo que implica que x 2 − 3x + 2 = 1 cuyas soluciones son
3+ 5 3− 5 3− 5  3+ 5 
x1 = y x2 = , luego Px1 ,0 , Px 2  ,0  .
2 2  2   2 
   

7.3 Regiones
Se trata de determinar las regiones del plano en las que hay gráfica. Para ello se
estudia el signo de f (x ) . Puesto que f (x ) sólo puede cambiar de signo en los puntos de corte
con OX o en los puntos de discontinuidad, la idea inicial es localizar unos y otros. Una vez
calculados, el plano correspondiente al dominio queda dividido en regiones por encima o por
debajo del eje OX, según sea f positiva o negativa, lo que ayuda a situar su gráfica.

Ejemplo 16 Estudiemos las regiones de y =


(x − 1)2 .
x2 − 4

7-2
Gráficas de funciones Tema 7

Como el numerador es siempre positivo, el signo de la función vendrá dado por el del
denominador y la función tendrá su gráfica en las zonas no sombreadas.

Dominio (− ∞, − 2) (− 2, 2) (2,+∞ )
Signo de f positivo negativo positivo

7.4 Asíntotas
En el Tema 2 ya vimos que las asíntotas, son rectas a las cuales la función se va
aproximando indefinidamente sin llegar nunca a cortarlas. Se definen también como rectas
tangentes a la curva en el infinito. Como las rectas sólo pueden ser verticales, horizontales u
oblicuas, sólo hay tres tipos de asíntotas:

Verticales: Una recta x = a se dice asíntota vertical de f cuando verifica alguna de las
siguientes condiciones: lim+ f ( x ) = +∞ (− ∞ ) , lim− f ( x ) = +∞ (− ∞ ) o lim f ( x ) = +∞ (− ∞ ) .
x →a x →a x →a
f( x)
Nota En el caso de funciones del tipo y = deben buscarse en las soluciones de
g( x )
g (x ) = 0 . Para funciones del tipo y = ln (f ( x )) deben buscarse en las soluciones de f (x ) = 0 .

Horizontales: Una recta y = a se dice asíntota horizontal de f si verifica que lim f ( x ) = a o


x →+∞
bien lim f ( x ) = a .
x →−∞

Nota Por la definición se advierte que un función tiene, a lo sumo, dos asíntotas horizontales.

Oblicuas: Son rectas de la forma y = mx + n , con m ≠ 0 . El cálculo de m y n se realiza del


f( x)
modo siguiente: m = lim y n = lim (f(x) - mx ) , cuando estos límites existan y sean
x →∞ x x →∞
finitos.

Nota Por la definición se advierte que una función tiene, a lo sumo, dos asíntotas oblicuas.

Nota Cuando hay asíntota horizontal en un infinito, no hay asíntota oblicua en dicho infinito.

Nota En el caso de funciones racionales, la asíntota oblicua (si existe) coincide con el
cociente que resulte al efectuar la división polinómica.

Nota Las funciones polinómicas no tienen asíntotas.

Ejemplos

7-3
Gráficas de funciones Tema 7

Ejemplo 7.7 Calculemos las asíntotas de y =


(x − 1)2 .
x2 − 4

Asíntotas verticales: x = 2 y x = -2 , pues lim−


(x − 1)2 = −∞ ,
x →2 x2 − 4

lim+
(x − 1)2 = +∞ , lim −
(x − 1)2 = +∞ y lim +
(x − 1)2 = −∞ .
x →2 x2 − 4 x →−2 x2 − 4 x →−2 x2 − 4

Asíntotas horizontales: y = 1 , pues lim


(x − 1)2 = 1.
x →±∞ x2 − 4

Ejemplo 7.8 Calculemos las asíntotas de y = L x 2 − 3 x + 2 . ( )


Asíntotas verticales: x = 1 y x =2, porque lim− f ( x ) = −∞ y lim+ f ( x ) = −∞ .
x →1 x →2
Asíntotas horizontales: no tiene, pues lim f ( x ) = +∞ .
x →±∞
Asíntotas oblicuas: no tiene, pues

m = lim
(
L x 2 − 3x + 2 )
= (L' Hôpital ) = lim
2x − 3
=0.
x → ±∞ x x → ±∞ 1

1

Ejemplo 7.9 Calculemos las asíntotas de y = e x .
1

Asíntotas verticales: x = 0 por la izquierda, pues lim− e x = +∞ ,
x →0
1

pero lim+ e x =0.
x →0
1

Asíntotas horizontales: y = 1 , pues lim e x = 1.
x →±∞
No tiene asíntotas oblicuas porque tiene horizontales por los dos
infinitos.

Ejemplo 7.10 Calculemos las asíntotas de y =


(x − 1)2 .
3x − 4

Asíntotas verticales: x=
4
, ya que lim −
(x − 1)2 = −∞ y
3 x→
4 3x − 4
3

lim +
(x − 1)2 = +∞ .
x→
4 3x − 4
3

Asíntotas horizontales: no tiene, pues lim = ±∞ .


(x − 1)2
3x − 4 x →±∞

1
Asíntota oblicua: la recta de ecuación y = x −
2
, pues m = lim
(x − 1)2 = 1 y
3 9 x →±∞ (3 x − 4 )x 3
 (x − 1)2 x  − 2x + 3 2
n = lim  −  = lim =− .
x →±∞  3 x − 4 3  x → ±∞ 9 x − 12 9

7.5 Monotonía y extremos relativos


Como ya vimos con anterioridad (Tema 6), el estudio del signo de f ′(x ) nos permite
determinar los intervalos de crecimiento, decrecimiento, máximos y mínimos relativos de la
función.

7-4
Gráficas de funciones Tema 7

7.6 Curvatura y puntos de inflexión


Como ya vimos con anterioridad (Tema 6), el estudio del signo de f ′′(x ) nos permite
determinar los intervalos de concavidad, convexidad y los puntos de inflexión de la función.

7.7 Simetrías
Hay simetría respecto al eje OY, si f (− x ) = f (x ) ; esto es, si al cambiar x por -x la
función no varía. Cuando esto se cumple se dice de la función que es par.
Por ejemplo, son funciones pares todas las polinómicas cuyos términos son todos de
grado par y la función f (x ) = cos x .
Hay simetría respecto al origen, si f (− x ) = −f (x ) ; esto es, si al cambiar x por -x la
función cambia de signo. Cuando esto se cumple se dice de la función que es impar.
Por ejemplo, son funciones impares todas las polinómicas cuyos términos son todos de
grado impar, la función f (x ) = senx y la función f (x ) = tgx .

Ejemplos

Ejemplo 7.11 Estudiemos la simetría de la función y = 3 x 3 + 2 x .


Como f ( − x ) = 3(− x ) + 2(− x ) = −3 x3 − 2 x = −f ( x ) , es una función impar y, por tanto,
3

simétrica respecto del origen.

senx
Ejemplo 7.12 Estudiemos la simetría de la función f ( x ) = .
x
sen (− x ) − senx
Como f (− x ) = = = f (x ) , es una función par y, por tanto, simétrica
−x −x
respecto al eje OY.

Ejemplo 7.13 Estudiemos la simetría de la función y =


(x − 1)2 .
x2 − 4

Como f (− x ) =
(− x − 1)2
≠ f (x ) y, también, f (− x ) =
(− x − 1)2
≠ −f (x ) , no es ni par ni impar.
( − x )2 − 4 ( − x )2 − 4
Luego no tiene ninguna de las simetrías que podemos determinar.

x2
Ejemplo 7.14 Estudiemos la simetría de la función f (x ) = .
x2 −1
( − x )2 x2
Como f ( − x ) = = = f ( x ) , se trata de una función par y, por tanto, simétrica
( − x )2 − 1 x2 −1
respecto de OY .

7.8 Periodicidad
Fundamentalmente nos aparecerá en funciones trigonométricas. La gráfica de una
función periódica se repite. El período, p, es el número que da la amplitud del intervalo de
valores de x para el que la función se repite; es decir, f (x ) = f (x + p ) y, por tanto, basta con
realizar la gráfica en un periodo. Las funciones seno y coseno son periódicas de periodo 2π, la
función tangente es periódica de periodo π.

7-5
Gráficas de funciones Tema 7

Ejemplo 7.15 Estudiemos la periodicidad de y = sen 2x . Como la función seno es periódica


de periodo 2π , sen (2 x ) = sen (2 x + 2π ) = sen (2(x + π )) , esto es, f ( x + π ) = f (x ) , luego la
función es periódicas periódica de periodo π.

7.9 Otras informaciones


Opcionalmente se pueden estudiar otras características como puntos de corte de la
gráfica con sus asíntotas, puntos de discontinuidad, puntos angulosos, etc.

Es interesante saber que si conocemos la gráfica de la función y = f(x) entonces:

1. La gráfica de y = −f ( x ) es simétrica respecto de OX.

2. La gráfica de y = f ( x ) es simétrica respecto de OX en las regiones en que f ( x ) ≤ 0 e


igual a la de f(x) en las regiones en que f ( x ) ≥ 0 .

3. La gráfica de y = f ( − x ) es simétrica respecto de OY.

4. La gráfica de y = f ( x + b ) con b > 0 está desplazada b unidades hacia la izquierda;


esto es, la gráfica está adelantada b unidades respecto a y = f (x ) .

5. La gráfica de y = f ( x − b ) con b > 0 está desplazada b unidades hacia la derecha; esto


es, la gráfica está retrasada b unidades respecto a y = f (x ) .

6. La gráfica de y = f ( x ) + b está desplazada b unidades hacia arriba o hacia abajo según


b sea positivo o negativo.

7. La gráfica de y = kf ( x ) con k > 0 tiene la misma forma que f(x) pero los valores de y
multiplicados por k (máximos, mínimos, situación de las asíntotas horizontales, etc).

8. La gráfica de y = kf ( x ) con k < 0 es simétrica respecto de OX y con los valores de y


multiplicados por k (máximos se convierten en mínimos y viceversa).

Ejemplo 16 Representamos la función y = x 2 − 3 x + 2 y a partir de su gráfica representamos


y = x 2 − 3 x + 2 e indicamos sus características.
No es preciso hacer un estudio completo para dibujar un función polinómica de grado
dos, pues es una parábola con las ramas hacia arriba; basta con situar el vértice y los puntos
de corte con los ejes. En este caso el vértice es el
mínimo, pues está abierta hacia arriba porque su
coeficiente principal es positivo, es el valor de x tal
que y ′(x ) = 0 . Como y ′(x ) = 2 x − 3 , el mínimo se
3 1
alcanza en x = , cuya imagen es y = − . Los
2 4
puntos de corte con OX, esto es, los que hacen
x 2 − 3 x + 2 = 0 , son (1,0) y (2,0), la gráfica es:

7-6
Gráficas de funciones Tema 7

A partir de ella, sabemos que y = x 2 − 3 x + 2 será la


misma función en la parte en que f (x ) es positiva y
simétrica respecto del eje de las x en la parte en que
f (x ) sea negativa:

Resultando una función que tiene un máximo


3 1
relativo M  ,  y dos mínimos relativos m1(1,0) y
2 4
m2(2,0) , que son puntos angulosos.

7-7
Gráficas de funciones Tema 7

Ejercicios resueltos

R.1. Representa la función y = (x − 1)3 y a partir de su gráfica dibuja la de y = −(x − 1)3 .

Se trata de una función polinómica, por lo que su dominio es todo R.

Puntos de corte: con OY, haciendo x = 0, obtenemos y = -1, por lo que el punto es Py(0,-1).
Análogamente, con OX, haciendo y = 0 obtenemos x = 1, por lo que el punto es Px(1,0).

Asíntotas: Recordemos que las funciones polinómicas no tienen asíntotas.

2
Monotonía: la primera derivada es y′ = 3(x − 1) que es siempre mayor o igual que cero, la
función es siempre creciente .

Curvatura y puntos de inflexión: la segunda derivada es y′′ = 6 ⋅ (x − 1) , cuyo signo es:


x (− ∞,1) 1 ( 1,+∞ )
y’’ - 0 +
Curvatura convexa I(1,0) cóncava

Simetría: no hay pues f ( − x ) = (− x − 1)3 ≠ f ( x ) y f(-x) ≠ -f(x)

Regiones: Es fácil ver que f es positiva si x > 1 , es negativa si x < 1.

Resumiendo:

Dominio R
Puntos de corte con los ejes Py (0,−1) , Px (1,0 )

Asíntotas. No tiene
Monotonía ( signo de y’) Creciente siempre
Concavidad convexidad y P. I. (signo de y’’) Convexa en x ∈ (− ∞,1) , cóncava en
x ∈ (1, ∞ )
Punto de inflexión I(1,0)
Simetrías No tiene
Regiones Función positiva en x ∈ (− ∞,1) y negativa
en x ∈ (1, ∞ )

Con estos datos la gráfica es:

7-8
Gráficas de funciones Tema 7

Como y = −(x − 1)3 es la opuesta de la anterior, su gráfica


es su simétrica respecto del eje de las x :

R.2. Representa la función y = x − 1 + x − 2

Con este tipo de funciones lo que hacemos es quitar el valor absoluto. Para ello,
debemos tener en cuenta que cada sumando varía según los valores que los hacen cero.

Hacemos el estudio en la siguiente tabla:

Valores de x x <1 1 1< x < 2 2 x>2


x −1 −x + 1 0 x −1 1 x −1
x−2 −x + 2 1 −x + 2 0 x−2
y = x −1 + x − 2 −2 x + 3 1 1 1 2x − 3

Por lo que f se puede rescribir como:


− 2x + 3 si x ≤ 1

f ( x ) = 1 si 1 < x < 2 ,
 2x + 3 si x ≥ 2

cuya gráfica es elemental.

x2 +1 x2 +1
R.3. Representa la función y = y, a partir de ella, la de y = .
2
x −1 x2 −1
Dominio: no son del dominio los valores que anulan el denominador, luego D = R − {− 1,1 }
Puntos de corte: con OY, haciendo x = 0 obtenemos y = -1; el punto es Py(0,-1). Con OX,
haciendo y = 0 vemos que no tiene solución real, luego no hay puntos de corte con OX.
x2 + 1 x2 + 1
Asíntotas verticales: x = -1, x = 1 porque lim 2 = ∞ y lim 2 = ∞.
x → −1 x − 1 x →1 x − 1

x2 + 1
Asíntotas horizontales: y = 1 porque lim = 1.
x → ±∞ x2 − 1

Asíntotas oblicuas: no tiene porque hay horizontales por los dos infinitos.

Monotonía: la primera derivada es y′ =


( ) (
2x ⋅ x 2 − 1 − 2x ⋅ x 2 + 1 )= − 4x
, su signo es:
(x − 1)
2 2
(x − 1)
2 2

7-9
Gráficas de funciones Tema 7

x x<0 0 x>0
y’ + 0 -
monotonía creciente M(0,-1) decreciente

4x2 + 4 4x 2 + 4
Curvatura y puntos de inflexión: calculamos y′′ = = cuyo signo viene
(x − 1)
2 3
(x − 1)3 ⋅ (x + 1)3
determinado por el del denominador, pues el numerador es siempre positivo.

x x < −1 -1 −1 < x < 1 1 x >1


y’’ + Ninguno - Ninguno +
Curvatura Cóncava No existe convexa No existe Cóncava
( − x )2 + 1 x2 + 1
Simetrías: es una función par ya que f ( − x ) = = = f ( x ) , por tanto la función es
( − x )2 − 1 x2 − 1
simétrica respecto al eje OY.

Regiones: positiva si x < -1 y x > 1 , negativa si –1 < x < 1 .

Resumiendo:

Dominio D = R − {− 1,1}
Puntos de corte con los ejes Py (0,−1) , con OX no hay

Asíntotas. Verticales x = -1, x = 1


Horizontales y = 1
Oblicuas no tiene
Monotonía Creciente en x ∈ (− ∞,0 )
Decreciente en x ∈ (0, ∞ )
Máximo M(0,-1)
Concavidad convexidad y P. I. Cóncava x ∈ ( −∞,−1) U (1, ∞ )
Convexa x ∈ ( −1,1) sin puntos de inflexión.
Simetrías Respecto OY
Regiones Función positiva en (− ∞,−1) U (1, ∞ ) y negativa en
el resto del dominio.

x2 + 1
Con estos datos la gráfica es: La gráfica de y = será:
x2 − 1

7-10
Gráficas de funciones Tema 7

R.4. Representa de y =
(x − 1)2 .
x2 − 4

El dominio, puntos de corte con los ejes, asíntotas, simetría y regiones están
estudiados en los ejemplos 1, 4, 7, 13, y 16 respectivamente, debemos pues hacer un estudio
del signo de y’ y de y’’ para completar su estudio.
2 ⋅ (x − 4 )(x − 1)
Crecimiento, decrecimiento y extremos: Hacemos su derivada y′ = , el
x2 − 4
2
( )
denominador es siempre positivo y por tanto el signo de la derivada viene dado por el
numerador.
Encontramos que la función es creciente en x ∈ (− ∞,1) U (4, ∞ ) , decreciente en x ∈ (1,4 ) tiene un
 3
máximo relativo M(1,0) y un mínimo relativo m  4,  . La derivada segunda
 4
− 4 x 3 + 30 x 2 − 48 x + 40
y′′ = , el signo del numerador sólo se puede estudiar de forma
(x
−4
3 2
)
aproximada con el Teorema de Bolzano, por tanto renunciamos a su estudio y dibujamos la
función con los datos obtenidos que son los siguientes:

Dominio R- {-2,2}
Puntos de corte con los ejes  1
Py  0,−  , Px (1,0 )
 4 
Asíntotas. Verticales x = 2, x = -2
Horizontales y = 1
Monotonía ( signo de y’) Creciente en x ∈ (− ∞,1) U (4, ∞ )
Decreciente en x ∈ (1,4 )
 3
M(1,0) m  4, .
 4
Concavidad convexidad y P. I. (signo de y’’) Renunciamos a su estudio.
Simetrías No tiene
Regiones Véase el ejemplo 16

Con estos datos la gráfica es la siguiente:

x
R.5. Representa la función y =
1− x
 x
 si x≤0
Si recordamos la definición de valor absoluto queda la función : f ( x ) = 1 + x .
x
 si x≥0
1 − x

7-11
Gráficas de funciones Tema 7

A primera vista parece que requiere el estudio de dos expresiones algebraicas


diferentes lo que duplica el trabajo, sin embargo si estudiamos en primer lugar la simetría el
estudio se simplifica notablemente.
−x −x
Como f ( − x ) = =− = −f ( x ) , es una función impar y, por tanto, simétrica
1− − x 1− x
x
respecto del origen. Así que representaremos el primer trozo, y = cuando x ≤ 0 , y el otro
1+ x
saldrá por simetría. En adelante damos por supuesto la restricción del dominio indicada.

Dominio: D = R − {− 1} . Puntos de corte: P(0,0)


x
Asíntotas verticales: x = -1 , porque lim =∞ .
x → −1 1 + x
x
Asíntotas horizontales: )sólo la estudiamos cuando x → −∞ ) y = 1 , porque lim = 1.
x → −∞ 1 + x
1+ x − x 1
Monotonía: derivamos y ′ = 2
= > 0 y, por tanto, la función es creciente.
(x + 1) (x + 1)2
2
Curvatura y puntos de inflexión: calculamos y′′ = , cuyo signo :
(x + 1)3
x x < −1 -1 −1 < x < 0
y’’ + Ninguno -
Curvatura Cóncava No existe Convexa

Cuando consideremos la simetría se verá que P(0,0) es un punto de inflexión.

Regiones: positiva si x < -1 y negativa si –1 < x < 0 .

Considerando la simetría la función en su conjunto tiene las siguientes características:

Dominio D = R − {− 1,1}
Puntos de corte con los ejes P(0,0)
Asíntotas. Verticales x = -1, x = 1
Horizontales y = 1 si x → −∞
y = −1 si x → +∞
Monotonía Creciente siempre
Concavidad convexidad y P. I. Cóncava x ∈ ( −∞,−1) U (0,1)
Convexa x ∈ ( −1,0) U (1, ∞ )
Punto de inflexión I(0,0)
Simetrías Respecto del origen
Regiones Función positiva en (− ∞,−1) U (0,1) y
negativa en el resto del dominio.

Con estos datos la gráfica es:

7-12
Gráficas de funciones Tema 7

( )
R.6. Representa la función y = ln 1 + x 2 y, a partir de ésta, indica las características de

y = ln 1 + x 2 .

Dominio: D = R , pues 1+ x 2 es siempre positivo. Puntos de corte: P(0,0) .

Asíntotas: no hay.
2x
Monotonía: derivamos, y′ = , estudiamos el signo.
1+ x2

x x<0 0 x>0
y’ - 0 +
Crecimiento ↓ m(0,0) ↑

− 2x 2 + 2 2 ⋅ (1 − x ) ⋅ (1 + x )
Curvatura y puntos de inflexión: calculamos y′′ = = y estudiamos
(1 + x )2
(1 + x )
2

x x < −1 -1 −1 < x < 1 1 x >1


y’’ - 0 + 0 -
Curvatura Convexa I1 (− 1, ln 2) Cóncava I2 (1, ln 2) Convexa

( )
Simetrías: como f ( − x ) = ln 1 + (− x )2 = f ( x ) es una función par, simétrica respecto al eje OY.
Con estos datos la gráfica es:

1
La función y = ln 1 + x 2 =
2
( )
ln 1 + x 2 y, por tanto, es la anterior multiplicada por una
constante positiva. Los intervalos de crecimiento, decrecimiento, mínimo, concavidad y
 ln 2   ln 2 
convexidad son los mismos, los puntos de inflexión son ahora I1  − 1,  y I1  − 1,  siendo
 2   2 
su forma igual a la anterior con un cambio de escala en OY.

(
R.7. Representa y = L x 2 − 3 x + 2 . )
Al igual que en el caso anterior sólo nos falta hacer ver del signo de y’ y de y’’ para
completar su estudio.
2x − 3 3
f ′( x ) = 2 , se anularía en x = , pero ese punto no es del dominio pues recordamos
x − 3x + 2 2
que Dominio = ( −∞,1) U (2, ∞ ) en este dominio el signo de y’ es negativo para x<1 y positivo
para x>2 por tanto la función es decreciente en x ∈ (− ∞,1) y creciente en x ∈ (2, ∞ ) y no hay
extremos relativos.

7-13
Gráficas de funciones Tema 7

− 2x 2 + 6 x − 5
Derivamos nuevamente y tenemos f ′′( x ) = que es siempre negativa y
(x 2
− 3x + 2 )
2

por tanto la función es convexa en todo su dominio.

Los datos obtenidos de la función son:

Dominio ( −∞,1) U (2, ∞ )


Puntos de corte con los ejes 3− 5  3+ 5 
Py (0, L2) , Px1 ,0  , Px 2  ,0 
 2   2 
   

Asíntotas. Verticales x = 1, x =2
Horizontales no tiene
Oblicuas no tiene
Monotonía ( signo de y’) Creciente en x ∈ (2, ∞ )
Decreciente en x ∈ (− ∞,1)
Sin máximos ni mínimos
Concavidad convexidad y P. I. (signo de y’’) convexa en todo su dominio
Simetrías No tiene
Regiones  3 − 5   3 + 5 
Función positiva en  − ∞, U , ∞
 2   2 
   
y negativa en el resto del dominio.

Con estos datos la gráfica es:

R.8. Representa la función y = (x − 1) ⋅ e x y, a partir de la anterior, indicar las


x +1
características de y = x ⋅ e .

Dominio: D = R .

Puntos de corte: con el eje OY x = 0 y = -1 Py(0,-1) con OX y = 0 x = 1 Px(1,0) .

Asíntotas: verticales no hay, horizontales no hay con x → +∞ calculamos el límite con x → −∞

usaremos la Regla de L’Hôpital. lim (x − 1) ⋅ e x = ∞ ⋅ 0 (ind) = lim


(x − 1) = 1
= 0 , hay
−x lim −x
x → −∞ x →−∞ e x → −∞ − e
pues asíntota horizontal y = 0 con x → −∞ , oblicuas se buscan sólo con x → +∞ no hay pues
(x − 1) ⋅ e x = lim x ⋅ e x = +∞ .
lim
x →+∞ x x →+∞

Monotonía: derivamos y ′ = x ⋅ e x estudiamos el signo.

7-14
Gráficas de funciones Tema 7

x x<0 0 x>0
y’ - 0 +
y ↓ m(0,-1) ↑

Concavidad convexidad y puntos de inflexión: calculamos y ′′ = (x + 1) ⋅ e x . Estudiamos el signo.


x x < −1 -1 −1 < x
y’’ - 0 +
y Convexa  2 Cóncava
I  − 1,− 
 e

Simetría: no tiene.

Regiones: la función está por encima de OX en x ∈ (1, ∞ ) y por debajo en x ∈ ( −∞,1) .


Con estos datos la gráfica es la siguiente: La gráfica de y = x ⋅ e x +1 se encuentra adelantada
una unidad respecto de la anterior sería:

R.9. Representa la función y = x 2 − 4 . A partir de la anterior, dibuja y = 3 + x 2 − 4 .

Dominio: son del dominio las soluciones de x 2 − 4 ≥ 0 es decir D = (− ∞,−2] U (2, ∞ ] .

Puntos de corte: con el eje OY no puede tener, con OX hacemos x2 − 4 = 0


P1(2,0) y P2 ( −2,0) .

Asíntotas: verticales no hay, horizontales no hay, buscamos asíntotas oblicuas primero para
x2 − 4
x → +∞ m = lim = 1 (se ha resuelto la indeterminación dividiendo numerador y
x →+∞ x
−4
denominador por x) y n = lim  x 2 − 4 − x  = lim = 0 (hemos multiplicado y
x → +∞   x →+∞ x 2 − 4 + x
dividido por la expresión conjugada), buscamos ahora asíntota con x → −∞
x2 − 4
m = lim = −1 . ¡Atención! se ha resuelto la indeterminación dividiendo numerador y
x →−∞ x
denominador por –x, pues dentro de la raíz no puede entrarse un número negativo y ahora
−4
x → −∞ el valor de n será: n = lim  x 2 − 4 + x  = lim = 0 en resumen
x → −∞   x→+∞ x 2 − 4 − x
encontramos dos asíntotas oblicuas y = x cuando x → ∞ e y = −x cuando x → −∞
x
Monotonía: derivamos y ′ = estudiamos el signo.
x2 − 4

7-15
Gráficas de funciones Tema 7

x x < −2 −2 < x < 2 x>2


y’ - ninguno +
y ↓ No hay función ↑

En x = −2 y en x = 2 no hay derivada y ′ → ∞ si x → −2 + o si x → 2− .
−4
Concavidad convexidad y puntos de inflexión: calculamos y ′′ = que es
(x 2
)
− 4 x2 − 4
negativa en todo el dominio y por tanto la función es siempre convexa.

Simetría: f ( − x ) = (− x )2 − 4 = f ( x) la función es par y por tanto simétrica respecto al eje OY.

Regiones: la función está por encima de OX en todo el dominio.

Con estos datos la gráfica es la siguiente:

La gráfica de y = 3 + x 2 − 4 es similar a la anterior desplazada 3 unidades hacia arriba:

7-16
Gráficas de funciones Tema 7

Ejercicios propuestos

Valor absoluto

1. Representa y = x − 1 − x − 3

Polinómicas

2. Estudia las características y representa la gráfica de las siguientes funciones:


a) y = x − x 3 b) y = x 3 − x a partir dela anterior c) igual con y = x − x 3

d) y = x 4 − 2x 2

Fracciones algebraicas

3. Estudia las características y representa la gráfica de las siguientes funciones:


x +1 x2 − x + 2 x2 − 3 x2 − 3
a) y = b) y = c) y = y a partir de ella y =
x −1 x x−2 x−2
4 2
d) y = e) y = 2 (curva de Agnesi)
x2 − 4 x +1
1 2x
f) y = x 2 + (Tridente de Newton) g) y = 2 (Serpentín de Newton)
x x +1

Logarítmicas

4. Estudia las características y representa la gráfica de las siguientes funciones:


ln x 4
a) y =
4 ⋅ ln x
x
(
b) y = ln x 2 + x − 2 ) c) y =
x

Exponenciales

5. Estudia las características y representa la gráfica de las siguientes funciones:


1 x2
− 2x
a) y = x⋅ex b) y = e 2 (Gaussiana) c) y =
ex

Trigonométricas

6. Estudia las características y representa la gráfica de la función: y = senx + cos x

7. A partir de la gráfica de la función y = senx , representa:


a) y = sen (x + π) b) y = −3 ⋅ senx c) y = senx

Irracionales

8. Estudia las características y representa la gráfica de las siguientes funciones:


x2 −1 x2
a) y = b) y =
x2 − 4 x2 −1

7-17
Gráficas de funciones Tema 7

Soluciones:

1. y = x − 1 − x − 3

 − 2 si x < 1

y = 2 x − 4 si 1 ≤ x ≤ 3
 2 si x > 3

2. a) y = x − x 3 D = R
1 1
Creciente si − <x<
3 3
1 1
Decreciente si x < − o x>
3 3
 1 2   1 2 
m − ,−  , M
  , 

 3 3 3  3 3 3
cóncava en x ∈ ( −∞,0) , convexa en
x ∈ (0, ∞ ) , I(0,0). Función impar

2. b) y = x 3 − x Simétrica de la anterior respecto a OX (a la izquierda)


2. c) y = x − x 3 Simétrica de la del apartado a) en y<0 e igual en y>0 (a la derecha)

2. d) y = x 4 − 2x 2
D = R, Puntos de corte con los ejes
(− 2 ,0), (0,0) ( 2 ,0)
Creciente en x ∈ ( −1,0) U (1, ∞ ) decreciente
en x ∈ ( −∞,−1) U (0,1) M(0,0), m1(-1,-1),
m2(1,-1)
 1   1 
Cóncava en x ∈  − ∞,− U
  , ∞ 
 3  3 
 1 1 
convexa en x ∈  − , 
 3 3 
 1 5  1 5
I1  − ,−  I2  ,− 
 3 9  3 9
Función par .

7-18
Gráficas de funciones Tema 7

x +1
3. a) y = D = R – { -1} , p. corte (-1,0) y (0,-1)
x −1
2
A.H. y = 1 , A.V. x = 1 y ′ = − decreciente
( x − 1) 2
4
siempre, y ′′ = cóncava si x>1 convexa si
( x − 1)3
x<1. Simetría no

x2 − x + 2
3. b) y =
x
D = R – { 0} , puntos de corte con los ejes no tiene.
x2 − 2
A.V. x = 0 , A.O. y = x − 1 . y′ = decreciente
x2
si , (
x ∈ − 2, 2 ) creciente si
(
x ∈ − ∞,− 2 U ) ( 2, ∞ ) M( − 2,−3'83 ) y m ( 2,1'83 ) ;
4
y′′ = cóncava si x>0 convexa si x<0. Simetría no
x3

x2 − 3 x2 − 3
3. c) y = , y=
x−2 x−2
 3
D = R – { 2} , puntos de corte con los ejes ( − 3 ,0), ( 3 ,0) y  0,  A.V. x = 2 , A.O. y = x + 2 .
 2
x 2 − 4x + 3
y′ = decreciente si , x ∈ (1,3 ) creciente si x ∈ (− ∞,1) U (3, ∞ ) M(1,2) y m (3,6) ;
( x − 2)2
2
y′′ = cóncava si x>2 convexa si x<2. Simetría no ( gráfica de la izquierda)
( x − 2)3
x2 − 3
y= (gráfica de la derecha)
x−2

4
3. d) y = D = R – { -2,2} , puntos de corte con
2
x −4
los ejes (0,-1) A.V. x = 2 y x = −2 A.H. y = 0 .
−8 x
y′ = creciente si x ∈ (− ∞,0 ) decreciente si
(x 2
−4 ) 2

24 x 2 + 32
x ∈ (0, ∞ ) (con x ∈ D ), M(0,−1) ; y ′′ =
(x 2
−4 )
3

7-19
Gráficas de funciones Tema 7

cóncava si x ∈ (− ∞,−2) U (2, ∞ ) convexa si x ∈ (− 2,2) . Función par sim. OY

2
3. e) y = 2
D = R , punto de corte con OY
x +1
−4 x
P(0,2) A.H. y = 0 . y′ = creciente si
(x + 1) 2 2

x ∈ (− ∞,0 ) decreciente si x ∈ (0, ∞ ) M(0,2) ;


2
12 x − 4
y′′ = cóncava si
(x + 1)
2 3

 3   3 
x ∈  − ∞,− U ,∞ convexa
 3   3 
 
 3 3   3 3   3 3
si x ∈  − , I − , , I  ,  . Función par, simétrica respecto a OY
 3 3  1 3 2  2  3 2 
     

1
D = R − {0} , punto de
3. f) y = x 2 +
x
corte con OX P( −1,0) A.V. x = 0 .
2x 3 − 1  1 
y′ = 2
creciente si x ∈  , ∞ 
3
x  2 
 1 
decreciente si x ∈  − ∞, 
3
 2
 1 3  2x 3 + 2
m ,  ; y ′′ = cóncava si
3
 2 2 2
3
x3
x ∈ (− ∞,−1) U (0, ∞ ) convexa si x ∈ (− 1,0 ) ,
I (− 1,0 ) .
2x
3. g) y = 2
D = R , punto de corte con los ejes
x +1
2 − 2x 2
P(0,0) A.H. y = 0 . y ′ = creciente si
(x
+1
2 2
)
x ∈ (− 1,1) decreciente si x ∈ (− ∞,−1) U (1, ∞ ) M(1,1) ,

m( −1,−1) ; y ′′ =
(
4x ⋅ x 2 − 3 ) cóncava si
(x 2
+1 ) 3

(
x ∈ − 3 ,0 U ) ( 3,∞) convexa
 3   3 
si x ∈ (− ∞,− 3 ) U (0, 3 ) I1  − 3 ,− , I2 (0,0 ) , I3  3 , . Función impar, simétrica respecto
 2   2 
 
del origen.
4 ln x +
4. a) y = D = R , , punto de corte con OX P(1,0)
x
A.V. x = 0 con x → 0 + , A.H.
1 − ln x
y = 0 (sólo con x → +∞ ) . y ′ = 4 ⋅ creciente si
x2
4
x ∈ (0, e ) decreciente si x ∈ (e, ∞ ) M(e, ) ;
e

7-20
Gráficas de funciones Tema 7

2 ln x − 3  3   3   3 6 
y ′′ = 4 ⋅ cóncava si x ∈ e , ∞ convexa si x ∈ 0, e 2  ,
 2   I e 2 , 3 .
x3      
     e 2 
4. b) (
y = ln x 2 + x − 2 ) D = (− ∞,−2) U (1, ∞ )
puntos de corte con OX P1( −2'3,0 ) ,P2 (1'3,0 )
A.V.
x = −2 con x → −2 − , x = 1 con x → 1+ ,
2x + 1
y′ = 2
decreciente si x ∈ (− ∞,−2)
x +x−2
2x 2 + 2x + 5
creciente si x ∈ (1, ∞ ) , y ′′ = −
(x 2
+x−2 )
2

convexa en todo D.
ln x 4
4. c) y= la función es equivalente a
x
4 ln x 4 ln x 4 ln(− x )
y= es decir y =
si x > 0 y y =
x x x
si x < 0 y su gráfica se hace a partir de la del
apartado 4.a, función impar simétrica respecto del
origen.

1
5. a) y = x ⋅ e x D = R − {0} , A.V.
1
x=0 con x → 0+ lim x⋅ex =0 A.O.
x →0 −
1
x −1
y = x + 1. y′ = e x creciente si
x
x ∈ (− ∞,0 ) U (1, ∞ ) decreciente si x ∈ (0,1) m(1, e) ;
1
1
y ′′ = e x cóncava si x ∈ (0, ∞ ) convexa
x3
si x ∈ (− ∞,0 ) .

x2 − x2

5. b) y = e 2 D = R , A.H. y = 0 , y ′ = −x ⋅ e 2
creciente si x ∈ (− ∞,0 ) decreciente si x ∈ (0, ∞ )
2
−x
M(0,1) ; (
y ′′ = x 2 − 1 ⋅ e ) 2 cóncava si
 1 
x ∈ (− ∞,−1) U (1, ∞ ) convexa si x ∈ (− 1,1) I1  − 1, ,
 e 
 1 
I2 1,  , simétrica respecto al eje OY.

 e

2x
5. c) y = D = R , punto de corte con los ejes (0,0)
ex
A.H. y = 0 con x → +∞ , y ′ = 2e − x (1 − x ) creciente
 2
si x ∈ (− ∞,1) decreciente si x ∈ (1, ∞ ) M1,  ;
 e

7-21
Gráficas de funciones Tema 7

 4 
y ′′ = 2e − x (x − 2) cóncava si x ∈ (2, ∞ ) convexa si x ∈ (− ∞,2) I  2, 2  .
 e 

 π
6. y = senx + cos x = 2 ⋅ cos x −  a
 4
π
partir de la gráfica de cosx retrasada
4
unidades y multiplicados los valores de
y por 2 es periódica de periodo 2π

7. a) y = sen (x + π) 7. b) y = −3 ⋅ senx

7. c) y = senx

x2 −1
8. a) y= D = (− ∞,−1) U (1, ∞ ) excepto
x2 − 4
x=2 y x = −2 , A.V. x = −2
x2 −1 x2 −1
lim − 2
= +∞ lim + = −∞ x=2
x →−2 x −4 x →−2 x2 − 4
x2 −1 x2 −1
lim− = −∞ lim+ = +∞ A.H. y = 0 .
x →2 x2 − 4 x →2 x2 − 4

y′ =
(
− x ⋅ x2 + 2 )
creciente si x ∈ (− ∞,−1)
(x − 4)
2
x −1
2 2

decreciente si x ∈ (1, ∞ ) , y’’ es demasiado


complicada se renuncia a su estudio; simétrica respecto a OY.

7-22
Gráficas de funciones Tema 7

x2
8. b) y = D = (− ∞,−1) U (1, ∞ ) A.V.
x2 −1
x = −1 con x → −1− y x = 1 con x → 1+ ,
A.O. y = x si x → +∞ e y = − x cuando
x 3 − 2x
x → −∞ y′ = creciente si
(x − 1) x − 1 2 2

x ∈ (− 2,−1) U ( 2 , ∞ ) decreciente si
x ∈ (− ∞,− 2 ) U (1, 2 ), m (− 2,2) m ( 1 2 2,2 )
x2 + 2
, y ′′ = cóncava en todo su dominio, simétrica respecto a OY.
(x 2
)
−1
2
x2 −1

7-23
Primitivas. Métodos de integración Tema 8

Tema 8

Funciones Primitivas. Métodos de Integración

8.1 Función primitiva


Definición Sea f una función real definida en un intervalo J. Una primitiva de f en J es otra
función real F, definida y derivable en J, tal que F ′(x) = f (x) ∀ x ∈J .

Ejemplo 8.1 Para f ( x ) = 2 x , la función F ( x ) = x 2 es una función primitiva de f en cualquier


intervalo J, porque F ′(x ) = 2 x = f (x ) , ∀ x ∈ R . También lo es la función G( x ) = x 2 + 5 y, en
general, cualquier función de la forma H (x ) = x 2 + C , donde C es una constante real.

Nota Del ejemplo anterior se concluye que la función primitiva de una función no es única y,
aún más, que si una función tiene una primitiva entonces tiene infinitas; basta sumar una
constante distinta a una primitiva conocida.
La siguiente proposición deja claro que no pueden existir más primitivas que las que se
obtienen de esta forma.

Proposición Sean F y G dos primitivas de f en J, entonces existe una constante C tal que
G (x ) = F (x ) + C ∀ x∈J .

Demostración.- Si consideramos la función H (x ) = G(x ) − F (x ) definida en el intervalo J, como


H ′(x ) = G ′(x ) − F ′(x ) = 0 , ∀x ∈ J , H (x ) es una función constante en todo el intervalo; esto es,
H (x ) = C y, en consecuencia, G (x ) − F (x ) = C , ∀x ∈ J , de donde se sigue lo que queríamos
demostrar.

Nota Con palabras, la anterior proposición viene a decir que “dos primitivas cualesquiera de
una función difieren siempre en una constante”.

Definición Al conjunto de todas las primitivas de f le llamamos integral indefinida de f y la


denotamos por ∫f o, bien, ∫ f (x )dx .

Nota Dada una función f y una primitiva F de f , por lo visto en la proposición anterior se tiene
que ∫ f (x )dx = {F (x ) + C, C ∈ R} . Observa que la integral indefinida representa un conjunto de
funciones. Por abuso de lenguaje escribiremos tan solo ∫ f (x )dx = F (x ) + C .

8-1
Primitivas. Métodos de integración. Tema 8

Nota La justificación del símbolo utilizado para la integral indefinida se verá en el próximo
tema.

Linealidad de la integral

• ∫ [f ( x ) + g ( x )]dx = ∫ f ( x )dx + ∫ g( x )dx . Con palabras: la integral de la suma es igual


a la suma de las integrales.

• ∫ k ⋅ f ( x )dx = k ⋅ ∫ f ( x )dx , ∀k ∈ R . Con palabras: la integral de una constante por


una función es igual a la constante por la integral de la función, es decir los
números pueden entrar o salir del signo integral.

Ejemplos

2
x
Ejemplo 8.2 ∫ (x + 1)dx = ∫ x dx + ∫ 1dx = 2 + x +C .

Ejemplo 8.3 ∫ 7 cos x dx = 7∫ cos x dx = 7senx + C .


cos x 1 1 sen (x )
Ejemplo 8.4 ∫ 5 dx = 5 ∫ cos xdx = 5senx + C = 5 + C .

8.2 Métodos de Integración


Llamamos método de integración a todo procedimiento que permita obtener las
primitivas de una función.

Conviene dejar claro que no todas las funciones tienen primitivas y que, además, entre
las que sí tienen, hay algunas cuyas primitivas no pueden ser obtenidas por ninguno de éstos
x
métodos; por ejemplo, f (x ) = .
sen (ax )

8.2.1 Integrales inmediatas

Son aquellas en las que el integrando es la derivada de una función elemental.

Integrales inmediatas Generalización

x n +1 (f ( x ))n +1 + C
∫ (f ( x ))
n
∫x
n
dx = + C con n ≠ −1 f ′( x )dx = con n ≠ −1
n +1 n +1

dx f ′( x )
∫ x
= ln x + C ∫ f ( x ) dx = ln f ( x ) + C

ax af ( x )
∫a ∫ f ′( x )a
x f( x )
dx = +C dx = +C
ln a ln a

∫e ∫ f ′( x )e
x
dx = e x + C f(x)
dx = e f ( x ) + C

∫ senxdx = − cos x + C ∫ f ′( x )sen(f ( x ))dx = − cos(f ( x )) + C

∫ cos xdx = senx + C ∫ f ′( x ) cos(f ( x ))dx = sen (f ( x )) + C

8-2
Primitivas. Métodos de integración Tema 8

dx f ′( x )
∫ cos 2 x = tgx + C ∫ cos 2 (f ( x )) dx = tg (f ( x )) + C

dx f ′( x )
∫ sen 2 x = −ctgx + C ∫ sen 2 (f ( x )) dx = −ctg (f ( x )) + C

senxdx f ′( x )sen( f ( x ))
∫ cos 2 x = sec x + C ∫ dx = sec (f ( x )) + C
cos 2 (f ( x ))

cos xdx f ′( x ) cos( f ( x ))


∫ = − cos ecx + C ∫ dx = − cos ec (f ( x )) + C
sen x 2
sen 2 (f ( x ))

dx f ′( x )dx
∫ 1− x 2
= arcsenx + C = − arccos(x ) + C ∫ 1 − (f ( x ))2
= arcsenf ( x ) + C = − arccos(f ( x )) + C

dx f ′( x )dx
∫ 1+ x 2
= arctgx + C = −arc cot (x ) + C ∫ 1 + (f ( x )) 2
= arctgf ( x ) + C = −arc cot (f ( x )) + C

dx f ′( x )dx
∫ x⋅ x2 −1
= arc sec x + C = − arccos ec (x ) + C ∫ f (x ) (f ( x ))2 − 1
= arc sec f ( x ) + C = − arccos ec (f ( x )) + C

Para reconocer los tipos generales hay que identificar la función y tener en cuenta la
derivada de la función compuesta.

Ejemplos

Ejemplo 8.5.

x4 x3 x2
∫ (2x ) ∫ ∫ ∫ ∫
3
− 3 x 2 − 5 x + 2 dx = 2 x 3 dx − 3 x 2 dx − 5 xdx + 2dx = 2 −3 −5 + 2x + C y
4 3 2

x4
∫ (2x )
3 5
simplificando queda − 3 x 2 − 5 x + 2 dx = − x 3 − x 2 + 2x + C
2 2

3x 2 + 5 x − 4x + 2  −
1
2 3
Ejemplo 8.6 ∫ dx =  3 x + 5 x 2 − 4 + dx = x 2 + 10 x − 4 x + 2 ln x + C

x  x 2
 
1
∫ x (3 x ) ∫ ( )
2
Ejemplo 8.7 − 12 dx = ( multiplico y divido por 6 ) = 6 x 3 x 2 − 12 dx =
6
1
=
12
(
3x 2 − 12) + C
2

−1 −1 1
∫ ( ) ( ) ( )
x 1
Ejemplo 8.8 ∫ 2
x +3
dx = x ⋅ x 2 + 3 2 dx =
2 ∫
2 x ⋅ x 2 + 3 2 dx = x 2 + 3 2 = x2 + 3 +C

Ejemplo 8.9 ∫x 2
x
+3
dx =
1 2x

2 x2 + 3
1
(
dx = ln x 2 + 3 + C
2
)
arctgx ′ 1
∫ 1+ x ∫
1 2
Ejemplo 8.10 2
dx = (arctgx ) ⋅ (arctgx ) dx = (arctgx ) + C
2

8-3
Primitivas. Métodos de integración. Tema 8

∫ (2x ) ∫( )
2 3 8 7 36 5
Ejemplo 8.11 + 3 dx = 8 x 6 + 36 x 4 + 54 x 2 + 27 dx = x + x + 18 x 3 + 27 x + C
7 5
1 1 x 3 +2
Ejemplo 8.12 ∫x 5 2 x 3+2
dx =
3∫ 2 x 3 +2
3x 5 dx =
3ln5
5 +C

  x  2 
x x x
a x + 2 x +1 a 1  a 2 2 
Ejemplo 8.13 ∫ e
x ∫
dx =   + 2 ⋅    dx =
 e 
  e  
 
 a  e 
+   +C
 2  e 
L  L 
 e  e
′ 1
Ejemplo 8.14 ∫ cos x sen 3
xdx = ∫ (senx ) sen 3 4
xdx = sen x + C
4
senx cos −2 x 1
Ejemplo 8.15 ∫ cos 3
x ∫
dx = − (− senx)cos −3 xdx = −
−2
=
2cos 2 x
+ C . Otro modo de

senx senx 1 ′ tg2 x


hacerla es: ∫ cos 3
x
dx = ∫ cos x cos 2
x
dx = ∫ tgx (tg x ) dx = 2
+C

dx1 1 1 1 (Lx +1)′ dx = 1 L Lx + 1 + C


Ejemplo 8.16 ∫ ( =
2x Lx + 1 2 ) ∫ x Lx + 1
dx =
2 ∫ Lx +1 2

1
1 1 1 3 1  x 
Ejemplo 8.17 ∫x 2
+3
dx = ∫ x 2
dx =
3 ∫ x 
2
dx =
3
arctg 
 3
 +C

3 + 1   +1
 3   
   3

Nota El ejemplo 8.15 puede servir para ver que dos primitivas difieren en una constante. En
efecto, no lo parece a primera vista, pero si restamos:
tg 2 x1 1 sen 2 x 1 − sen 2 x cos 2 x 1
2
= 2
− − 2
= 2
= 2
= ,
2 cos x 2 2 cos x 2 cos x 2 cos x 2 cos x 2
se comprueba que, a pesar de las apariencias, sí difieren en una constante.

8.2.2 Integración por partes

La fórmula de integración por partes se deduce de la derivada de un producto de


funciones.

Como (u( x ) ⋅ v ( x )) = u ′( x ) ⋅ v ( x ) + u( x ) ⋅ v ′( x ) , integrando queda

∫ ∫
(u( x ) ⋅ v ( x )) dx = u ′( x )⋅ v ( x ) dx + u( x ) ⋅ v ′( x ) dx , ∫
despejando ∫ u( x ) ⋅ v ′( x ) dx , queda:

∫ u( x ) ⋅ v ′( x ) dx = ∫ (u( x ) ⋅ v ( x )) dx − ∫ u ′( x )⋅ v ( x ) dx ,

como ∫ (u( x ) ⋅ v ( x )) dx = u( x ) ⋅ v ( x ) y aplicando la notación diferencial para derivadas, resulta
∫ u ⋅ dv = u ⋅ v − ∫ v ⋅ du , o bien ∫ u( x ) ⋅ v ′( x ) dx = u( x ) ⋅ v ( x ) − ∫ v ( x ) ⋅ u ′( x ) dx
que es la fórmula llamada de integración por partes.

Nota El método consiste en descomponer la integral en producto de dos partes de manera


que, al aplicar la fórmula, la nueva integral sea más sencilla que la original. Suele ser el método
adecuado cuando al descomponer el integrando la parte que se deriva se simplifica: por

8-4
Primitivas. Métodos de integración Tema 8

ejemplo, en logarítmicas, exponenciales, trigonométricas y sus inversas, etc. Se puede aplicar


reiteradamente.

Aunque no existe una regla general que indique la manera adecuada de elegir los
factores del integrando, en la siguiente tabla se relacionan algunos casos típicos.

f(x) g(x) Elección de las partes


x n con n ∈ N o polinomio en x Exponencial (*) u = f ( x ) , dv = g ( x ) ⋅ dx

x n con n ∈ N o polinomio en x Trigonométrica (*) u = f ( x ) , dv = g ( x ) ⋅ dx

Logarítmica x n con n ∈ N o polinomio en u = f ( x ) , dv = g ( x ) ⋅ dx


x o una constante
Inversa de trigonométrica (*) La unidad o una constante u = f ( x ) , dv = g ( x ) ⋅ dx

Exponencial (*) Trigonométrica (*) Indistinto (aplicación reiterada)

(*) generalmente de integración inmediata

Ejemplos

u=x du = dx
∫ x⋅e
3x −1
Ejemplo 8.18 dx , elegimos las partes: ⇒ e 3 x −1 y
dv = e 3 x −1
dx v = ∫ e 3 x −1dx =
3
aplicando la fórmula
xe 3 x−1 1 3x −1 xe 3x−1 1 xe 3x−1 1 3x −1
∫ x ⋅ e 3x−1dx =
3

3
e ∫
dx =
3

9 ∫
3 ⋅e 3x −1dx =
3
− e
9
+C.
Ejemplo 8.19
u = x 
2 du = 2 xdx
∫ x 2 senxdx = 
dv = senxdx

v = ∫ sendx = − cos x 


= −x 2 cos x + 2 x cos xdx = (*),

reiteramos el método eligiendo ahora


u = x du = dx 
(*) = −x cos x + 2  x ⋅ senx − senxdx por tanto:

2
 ⇒
dv = cos xdx v = ∫ cos dx =senx   

∫x
2
senxdx = − x 2 cos x + 2x senx + 2cos x + C .
 1 
 u = ln x du = dx  dx
Ejemplo 8.20 ∫ ln xdx = 
dv = dx
⇒ x  = x ⋅ln x − x
v = x 
x ∫
= x ⋅ ln x − x +C .

 1 
u = arctgx du = dx 

Ejemplo 8.21 arctg xdx = 
dv = dx

v=x
1+ x 2 

xdx 1 2xdx
= x ⋅ arctgx −
1+ x ∫2
= x ⋅ arctgx −
2 1+ x 2 ∫
quedando:

1
∫ 2
arctg xdx =x ⋅ arctg x − ln 1+ x + C .
2 ( )
Nota Al aplicar la integración por partes puede darse el caso que muestran los dos siguientes
ejemplos:

∫e
x
Ejemplo 8.22 ⋅ senxdx llamaremos F ( x ) a la función integral y aplicaremos el proceso de
integración por partes dos veces, no importa cómo elijamos las partes pero si elegimos como
parte integral la exponencial nos cuidaremos de elegir la segunda vez otra vez la exponencial
como parte integral, así:

8-5
Primitivas. Métodos de integración. Tema 8

u = e x du = e x dx 

F(x ) = e x ⋅senxdx =  ∫
x x
⇒  = −e ⋅ cos x + e cos xdx =
dv = senxdx v = − cos x 
u = e x du = e x dx 
= ⇒ 
dv = cos xdx v = senx 


= −e x ⋅ cos x + e x senx − e x senxdx = − e x ⋅ cos x + e x senx − F ( x ) .

Resumiendo hasta aquí, tenemos: F ( x ) = −e x ⋅ cos x + e x senx − F ( x ) .


()
Pasando F x al primer miembro y despejando, tenemos finalmente
e x ⋅ (senx − cos x )
F( x ) =
+C .
2
Ejemplo 8.23 ∫ cos 2 xdx llamaremos también F ( x ) a la función integral F ( x ) = ∫ cos 2 xdx =
u = cos x du = −senxdx 
 = cos x ⋅ senx + ∫ sen xdx =
2
= ⇒
dv = cos xdx v = senx 
(
cos x ⋅ senx + ∫ 1 − cos 2 x ⋅ dx = )
= cos x ⋅ senx + x − ∫ cos x ⋅ dx = cos x ⋅ senx + x − F ( x ) .
2

()
Resumiendo hasta aquí, tenemos: F x = cos x ⋅ senx + x − F(x ) .
Pasando F(x) al primer miembro y despejando tenemos finalmente
cos x ⋅ senx + x
F ( x ) = ∫ cos 2 xdx = +C .
2

8.4 Integración por descomposición


Consiste en descomponer la integral dada en suma de integrales basándose en las
propiedades de la linealidad de la integral indefinida. Ya se han visto algunos ejemplos, pero en
este epígrafe daremos algunos más específicos.

8.4.1. Trucos algebraicos.

Al integrando se puede sumar y restar la misma cantidad o multiplicar y dividir por una
misma expresión si conviene.

Ejemplos

Ejemplo 8.24
1− x 1− x ⋅ 1− x (1 − x )2 1− x 1 −x
∫ 1+ x
dx = ∫ 1+ x ⋅ 1− x
dx = ∫ 1− x 2
dx = ∫ 1− x 2
dx = ∫ 1− x 2
dx + ∫ 1− x 2
dx =

= arcsenx + 1 − x 2 + C

∫ tg ∫ (1 + tg ) ∫( ) ∫
2 2
Ejemplo 8.25 xdx = x − 1 dx = 1 + tg 2 x dx − dx =tgx − x + C

8.4.2. Descomposición de funciones trigonométricas circulares.

La aplicación de fórmulas de trigonometría nos permiten descomponer una integral en


sumas de integrales más sencillas.

8-6
Primitivas. Métodos de integración Tema 8

∫ cos
2
Ejemplo 8.26 Para resolver xdx (resuelta por partes en el ejemplo 23) aplicamos la
1 + cos 2 x
identidad cos 2 x = y queda
2
1+ cos 2x 1 cos2x x sen2x
∫ cos 2
xdx = ∫ 2
dx =∫ 2
dx +
2 ∫ dx = +
2 4
+C

Resolución de las integrales del tipo

∫ sin(mx ) ⋅ cos(nx ) ⋅ dx , ∫ cos mx ⋅ cos nx ⋅ dx y ∫ senmx ⋅ sennx ⋅ dx


a) ∫ sen mx ⋅ cos nx ⋅ dx
1
Aplicamos la identidad sena. cos b = (sen (a + b ) + sen (a − b )) y se tiene:
2

∫ sen mx ⋅ cos nx ⋅ dx =
1
(sen(m + n )x + sen(m − n )x )dx = − cos(m + n )x − cos(m − n )x + C

2 2(m + n ) 2(m − n )
b) ∫ cos mx ⋅ cos nx ⋅ dx
1
Aplicamos cos a. cos b = (cos (a + b ) + cos (a − b )) y se tiene:
2

∫ cos mx ⋅ cos nx ⋅ dx =
1
(cos(m + n )x + cos(m − n )x )dx = sen(m + n )x + sen(m − n )x + C

2 2(m + n ) 2(m − n )

c) ∫ senmx ⋅ sennx ⋅ dx
1
Aplicamos sena.sen b = (cos (a − b ) − cos (a + b )) y se tiene:
2

∫ senmx ⋅ sennx ⋅ dx =
1
(cos(m − n )x − cos(m + n )x )dx = sen(m − n )x − sen(m + n )x + C

2 2(m − n ) 2(m + n )

Ejemplos

1 1 1
Ejemplo 8.27 ∫ sen5x ⋅ cos 4x ⋅dx = 2 ∫ (sen9x + senx )dx = − 18 cos 9x − 2 cos x + C
1 1 1
Ejemplo 8.28 ∫ cos 5x ⋅ cos3x ⋅ dx = 2 ∫ (cos8x + cos 2x )dx = 16 sen8x + 4 sen2x + C .
1 1 1
Ejemplo 8.29 ∫ sen 5x ⋅ sen3x ⋅ dx = 2 ∫ (cos2x − cos 8x )dx = 4 sen2x − 16 sen8x +C .
Nota Las integrales de estos últimos ejemplos también pueden ser resueltas aplicando
“partes” repetidamente.

8.5 Integración de funciones racionales


Una función racional es aquella que se puede escribir como cociente de dos
P( x )
polinomios. Se trata, pues, de integrar expresiones de la forma , donde tanto P(x) como
Q( x )
Q(x) son polinomios.

8-7
Primitivas. Métodos de integración. Tema 8

No obstante, antes de aplicar el método que vamos a explicar es aconsejable


comprobar que no se trata de una integral inmediata. Por ejemplo, la integral racional
2x + 1
∫ x2 + x + 8
dx es inmediata y no necesita de métodos.

Con todo, para integrar funciones racionales distinguiremos dos casos:

Caso I: grado de P(x) ≥grado de Q(X)

En este caso se efectúa primeramente la división polinómica. Sean C(x) y R(x) el


P (x ) R (x )
cociente y el resto, respectivamente. Teniendo en cuenta que = C (x ) + , la integral
Q (x ) Q (x )
P( x ) R( x )
puede ser descompuesta de la forma ∫ dx = ∫ C( x )dx + ∫ dx .
Q( x ) Q( x )
Si R (x ) = 0 , la descomposición se reduce sólo al primer sumando, que es la integral de
x 2 − 5x + 6
un polinomio y, por lo tanto, sencilla de resolver. Por ejemplo,
x −2
dx . ∫
Si R( x ) ≠ 0 , la integral se descompone en la integral de un polinomio y una integral
racional donde el grado del numerador es menor que el del denominador y este es el caso que
consideraremos a continuación.

Caso II: grado de P(x) <grado de Q(X)

Primero, se descompone factorialmente el denominador Q(x) y, a continuación, se


P( x )
descompone el cociente en suma de fracciones simples. Para su estudio, distinguiremos
Q( x )
los siguientes casos:

• Todas las raíces de Q(x ) son reales y simples.


• Hay raíces reales múltiples
• Hay raíces complejas simples (polinomio de 2º grado irreducible)
• Hay raíces complejas múltiples (polinomio de 2º grado irreducible múltiple)

8.5.1. Integración de funciones racionales con raíces reales simples en el denominador.

Supongamos que Q( x ) = (x − x1 ) ⋅ (x − x 2 ) ⋅ (x − x 3 ) ⋅ ⋅ ⋅ ⋅(x − x n ) , donde x1 , x 2 ,K x n ∈R


y distintos dos a dos.
P( x )
En tal caso siempre es posible descomponer de la forma:
Q( x )
P( x ) A1 A2 A3 An
= + + + ⋅⋅⋅ + ,
Q( x ) (x − x1 ) (x − x 2 ) (x − x 3 ) (x − x n )
P( x )
donde los valores A1 ,⋅ ⋅ ⋅ , An se obtienen por identificación de con su descomposición.
Q( x )
Una vez obtenidos los Ai , queda
P( x ) A1 A2 A3 An
∫ Q( x ) = ∫ (x − x ) + ∫ (x − x
1 2 )
+ ∫ (x − x 3 )
+ ⋅⋅⋅ + ∫ (x − x n )
Ai
y, como, ∫x−x i
dx = Ai ln x − x i + C , la integral inicial queda resuelta.

8-8
Primitivas. Métodos de integración Tema 8

x2 +1
Ejemplo 8.30. ∫ x 2 − 3 x + 2dx como numerador y denominador son del mismo grado dividimos

x2 +1  3x − 1  3x − 1
y descomponemos ∫ x 2 − 3 x + 2dx = ∫ 1 + x 2 − 3 x + 2 dx = ∫ dx + ∫ x 2 − 3 x + 2dx .
Las raíces del denominador son reales simples x = 1, x = 2 , la descomposición es:
3x − 1 3x − 1 A B A(x − 2) + B (x − 1)
= = + = .
x − 3 x + 2 (x − 1)(x − 2) x − 1 x − 2
2 (x − 1)(x − 2)
Identificando los numeradores: 3 x − 1 = A(x − 2 ) + B(x − 1) ; y dando valores a x se tiene
x = 1 → A = −2 y con x = 2 → B = 5 , por tanto:
x2 +1 3x − 1 −2 5
∫ x 2 − 3 x + 2dx = ∫ dx + ∫ x 2 − 3 x + 2dx = x + ∫ x − 1 dx + ∫ x − 2 dx = x − 2 ln x − 1 + 5 ln x − 2 + C

8.5.2. Integración de funciones racionales con raíces reales múltiples

Supongamos que Q( x ) = (x − x1 )a ⋅ (x − x 2 )b ⋅ (x − x 3 )c ⋅ ⋅ ⋅ ⋅(x − x n )d , donde las x i son


raíces reales múltiples distintas dos a dos.
En este caso la fórmula de la descomposición es:
P( x ) A1 A2 Aa B1 B2 Bb
= + + ⋅⋅⋅ + + + + ⋅⋅⋅ + + ⋅⋅⋅ ,
Q( x ) (x − x1 ) (x − x1 )2 (x − x 1 )a ( x − x 2 ) (x − x 2 )2
(x − x 2 )b
porque cada raíz genera tantas fracciones simples como indica su multiplicidad.
Una vez obtenidos los Ai ,Bi ,K , queda
P( x ) A1 A2 Aa B1 B2 Bb
∫ Q( x ) = ∫ (x − x ) + ∫ (x − x )
1 1
2
+ ⋅⋅⋅ + ∫ (x − x ) + ∫ (x − x
1
a
2 )
+ ∫ (x − x 2 ) 2
+ ⋅⋅⋅ + ∫ (x − x 2 )b
+ ⋅⋅⋅

Ai
Las integrales de los nuevos sumandos son del tipo ∫ (x − x ) i
n
dx , con n > 1 , que son

inmediatas del tipo potencial y cuya solución es:


Ai (x − x i )−n +1 = Ai
∫ (x − x ) ∫ i
( )−n
dx = Ai x − x dx = Ai +C .
i
n − n +1 (1 − n )(x − x i )n −1
Una vez calculadas todas, la integral queda resuelta.

Ejemplos

x +1 1 x +1
Ejemplo 8.31 ∫ 3 x 2 − 6 x + 3dx = 3 ∫ x 2 − 2 x + 1dx y x 2 − 2 x + 1 = (x − 1)2 , que es una raíz

doble, luego:
x +1 A1 A2x +1
2
+= . 2
=
x − 2 x + 1 (x − 1) x − 1 (x − 1)2
Sumando e identificando numeradores: x + 1 = A1 (x − 1) + A2 ;
y dando valores a x se tiene x = 1 → A2 = 2 y x = 0 → A1 = 1 , con lo que:
x +1 1 1 1 2 1 2
∫ 3 x 2 − 6 x + 3dx = 3 ∫ x − 1dx + 3 ∫ (x − 1)2 dx = 3 ln x − 1 − 3(x − 1) + C

3x 2 − 6x − 6
Ejemplo 8.32 Calculemos dx . Como x 3 − 3 x + 2 = (x − 1)2 (x + 2)
∫ , el
x 3 − 3x + 2
denominador tiene una raíz simple y una raíz doble, la descomposición es
3x 2 − 6x − 6 A B B2
= + 1 + .
(x − 1) (x + 2) x + 2 x − 1 (x − 1)2
2

Sumando e identificando los numeradores, resulta

8-9
Primitivas. Métodos de integración. Tema 8

3 x 2 − 6 x − 6 = A(x − 1)2 + B1 (x − 1)(x + 2) + B 2 (x + 2) ;


y dando a x los valores de las raíces del denominador, se tiene
• x = 1 → −9 = 3B 2 ⇒ B2 = −3
• x = −2 → 18 = 9 A ⇒ A = 2 .
Como el denominador no tiene otras raíces, se da a x un valor cualquiera para obtener la
tercera incógnita; en este caso, tomamos x = 0 y usando los valores de A y B2 ya calculados,
se obtiene x = 0 → −6 = 2 − 2B1 − 6 ⇒ B1 = 1 , por tanto:
3x 2 − 6x − 6 2 1 3 3
∫ 3
x − 3x + 2
dx = ∫ x + 2 dx + ∫ x − 1 dx − ∫ (x − 1)2 dx = 2 ln x + 2 + ln x − 1 + x − 1 + C

8.5.3. Integración de funciones racionales con raíces complejas simples

En este caso al factorizar el denominador aparece, además de lo ya visto, un factor de


segundo grado sin raíces reales. Sea ax 2 + bx + c este factor.
La descomposición en suma de fracciones simples se hace del modo habitual. Pero
aquí aparece un nuevo sumando, el correspondiente a ax 2 + bx + c , que es de la forma
Mx + N
.
ax 2 + bx + c
Mx + N N
Si M = 0 , la integral 2
ax + bx + c ∫ se convierte en 2
ax + bx + c ∫
, cuya primitiva

sabemos que es una arcotangente.


Mx + N
Si M ≠ 0 , la integral ∫
ax 2 + bx + c
se descompone en dos; una tendrá como primitiva

un logaritmo neperiano y la otra tendrá una arcotangente, tal y como se explica en los
siguientes ejemplos.

Ejemplos

x
Ejemplo 8.33 ∫ x 2 − 2x + 2dx es una integral en la que el denominador del integrando no

tiene raíces reales, así que procederemos del modo siguiente:

1º Se intenta que aparezca en el numerador la derivada del denominador tal y como sigue:
1 2x
2 ∫ x 2 − 2x + 2
- multiplicamos y dividimos por 2: dx

1 2x − 2 + 2
2 ∫ x 2 − 2x + 2
- restamos y sumamos 2: dx

1 2x − 2 1 2
2º La descomponemos en suma de dos integrales: ∫ 2 dx + ∫ 2 dx .
2 x − 2x + 2 2 x − 2x + 2
La primera es inmediata, se trata de un logaritmo neperiano; mientras que para la
segunda hay que buscar la arcotangente tal como sigue:
1 1
∫ x 2 − 2 x + 2dx = ∫ (x − 1)2 + 1 dx = arctg (x − 1) .
Resumiendo:
x 1 2x − 2 1 1
∫ x 2 − 2 x + 2dx = 2 ∫ x 2 − 2 x + 2dx + ∫ (x − 1)2 + 1dx = 2 L x − 2 x + 2 + arctg (x − 1) + C .
2

dx
Ejemplo 8.34 ∫ x4 + x2 ( )
. Como x 4 + x 2 = x 2 x 2 + 1 , tiene una raíz real doble, x = 0 , y dos

1 A1 A2 Mx + N
raíces complejas, x = ±i , la descomposición es: 4 2
= + 2 + 2 .
x +x x x x +1

8-10
Primitivas. Métodos de integración Tema 8

Poniendo el mínimo común múltiplo en el segundo miembro e igualando los numeradores:


( ) (
1 = A1 ⋅ x ⋅ x 2 + 1 + A2 ⋅ x 2 + 1 + (Mx + N ) ⋅ x 2 )
Para x = 0 ⇒ A2 = 1 . Como no tenemos más valores que sean cómodos para sustituir,
ordenamos el polinomio e identificamos coeficientes:
1 = (A1 + M )x 3 + (A2 + N )x 2 + A1x + A2
Igualando los coeficientes de ambos miembros se tiene:
grado 0 ⇒ A2 = 1 ; grado 1 ⇒ A1 = 0 ;
grado dos ⇒ A2 + N = 0 ⇒ N = −1 ; grado 3 ⇒ A1 + M = 0 ⇒ M = 0
dx 1 −1 1
Por lo tanto: ∫ 4 2
= ∫ 2 dx + ∫ 2 dx = − − arctgx + C .
x +x x x +1 x
5 x 2 + 8 x + 22
Ejemplo 8.35 ∫ (x 2 + 6 x + 13 ) ⋅ (x − 2)dx . Como x 2 + 6 x + 13 no tiene raíces reales, la

descomposición en suma de fracciones simples es:


5 x 2 + 8 x + 22
A Mx + N
+ 2 = .
(
x + 6 x + 13 ⋅ (x − 2)
2 x)− 2 x + 6 x + 13
Poniendo el mínimo común múltiplo en el segundo miembro e igualando los numeradores:
( )
5 x 2 + 8 x + 22 = A ⋅ x 2 + 6 x + 13 + (Mx + N ) ⋅ (x − 2) ,
Para x = 2 ⇒ 58 = 29 A ⇒ A = 2 .
Para x = 0 ⇒ 22 = 26 − 2N ⇒ N = 2 .
Para x = 1 ⇒ 35 = 40 + (M + 2) ⋅ (− 1) ⇒ M = 3 .
Por lo que podemos escribir:
5 x 2 + 8 x + 22 2 3x + 2
∫ x 2 + 6 x + 13 ⋅ (x − 2)dx = ∫ x − 2dx + ∫ x 2 + 6 x + 13 dx .
( )
La primera integral es 2 ln x − 2 , así que ahora nos ocupamos de la segunda:
4
2x +
3 3
2 ∫ x 2 + 6 x + 13
- sacamos 3 como factor y después multiplicamos y dividimos por 2: dx

4
2x + 6 + − 6
3 3
- sumamos y restamos 6: ∫ 2 dx
2 x + 6 x + 13
4
−6
3 2x + 6 3 3
- separamos en dos integrales: ∫ 2 dx + ∫ 2 dx
2 x + 6 x + 13 2 x + 6 x + 13
3
(
- la primera es ln x 2 + 6 x + 13
2
)
dx dx
- la segunda será un arco tangente: − 7 ∫ 2 = −7 ∫
x + 6 x + 13 (x + 3 )2 + 4
7 dx
- sacamos como factor 4 en el denominador − ∫ y multiplicamos y dividimos por
4  x + 3 2
  +1
 2 
1
dx
7 2 7  x +3
2: − ∫ 2
= − arctg   . Por lo tanto:
2 x +3 2  2 
  +1
 2 
5 x 2 + 8 x + 22 3 7  x +3
∫ (x 2 + 6 x + 13 )⋅ (x − 2)dx = 2 ln x − 2 + 2 ln (x )
2
+ 6 x + 13 − − arctg   +C
2  2 

8-11
Primitivas. Métodos de integración. Tema 8

Resumiendo: Integración de funciones racionales por descomposición:

1º Se divide si procede.
2º Se hallan las raíces del denominador y se descompone éste en factores.
3º Se descompone el integrando en suma de fracciones simples.
a) la integral con raíces reales simples es suma de logaritmos neperianos
b) la integral con raíces reales múltiples es suma de logaritmos neperianos y
potencias
c) la integral con raíces complejas simples es suma de logaritmo neperiano y
arcotangente.

8.6 Integración por cambio de variable o sustitución


Ante una integral que no encaje en ningún modelo de los vistos con anterioridad, se
suele recurrir al método de sustitución o de cambio de variable que es el más frecuente en la
resolución de integrales.

El método consiste en encontrar una función x = g ( t ) que nos transforme la integral en


otra más sencilla. La función x = g ( t ) tiene que cumplir dos condiciones:
1. Ser derivable, pues hay que calcular dx = g ′( t )dt
2. Admitir función inversa puesto que una vez resuelta

∫ f ( x )dx = ∫ f (g(t ) ⋅ g ′(t )dt = H (t ) + C


es preciso deshacer el cambio para dejar la primitiva en función de x.

Aunque no hay reglas fijas, sí que hay determinados cambios que funcionan bien en
diferentes tipos de funciones. La elección de un cambio inadecuado puede complicar más la
resolución de la integral.

∫x
2
Ejemplo 8.36 Para calcular 1 + 2 x dx se hace el cambio 1 + 2 x = t ; se diferencia
t −1
2 ⋅ dx = dt , se despeja la x, x = , y se pone la integral en función de t :
2
1 1 1  5 3 1  1 7 1 52 1 32
∫x
2
1 + 2 x dx = ∫ (t − 1)2 ⋅ t 2 dt = ∫  t 2 − 2t 2 + t 2 dt = t 2 − t + t .
8 8   28 10 12
Después, se deshace el cambio:
1 7 1 5 1 3
∫ x 1 + 2x dx = 28 (1 + 2x ) 2 − 10 (1 + 2x ) 2 + 12 (1 + 2x ) 2 + C .
2

8.6.1 Exponenciales y logarítmicas

Para integrales de fracciones racionales en a x y log(x ) , o de los tipos ∫ R (a )


x
, x ⋅ dx y

∫ R (log(x ), x ) ⋅ dx , se hace el cambio a = t o log(x ) = t .


x

Ejemplos

e 2 x +1
Ejemplo 8.37 ∫ 4 + e x dx .
e x = t  dt dt
Se hace el cambio  x
e dx = dt 
 y como e
2 x +1
= ex ( ) 2
⋅ e = e ⋅ t 2 y dx =
e x
=
t
se tiene:

8-12
Primitivas. Métodos de integración Tema 8

e 2 x +1 e ⋅ t 2 dt  4 
∫ 4 + ex dx = ∫ = e ∫ 1 − x
(
 ⋅dt = e ⋅ (t − 4 ln 4 + t ) = e ⋅ e − 4 ln 4 + e + C
x
( ))
4+t t  4+t 

5 ⋅ (ln x )2 − 2 ⋅ ln x + 25 ln x = t  5t 2 − 2t + 25
Ejemplo 8.38 ∫ x ⋅ (ln x )3 − 6 ⋅ (ln x )2 + 25 ⋅ ln x
( dx
⋅ = 1
 dx = dt 
 x ) 
=
t 3 − 6t 2 + 25t
⋅dt∫
Resultando una integral conocida que se resuelve por descomposición:
(
Como t 3 − 6t 2 + 25t = t ⋅ t 2 − 6t + 25 )
5t 2 − 2t + 25 A Mt + N
t 3 − 6t 2 + 25t
= + 2
t t − 6t + 25
( )
, 5t 2 − 2t + 25 = A t 2 − 6t + 25 + (Mt + N ) ⋅ t

Haciendo t = 0 e identificando coeficientes por grados se obtiene A = 1, M = 4 y N = 4 , por lo


que la integral queda:
5t 2 − 2t + 25 1 4t + 4
∫ t 3 − 6t 2 + 25t dt = ∫ t dt + ∫ t 2 − 6t + 25dt , la primera es inmediata y la segunda:
4t + 4 2t + 2 2t − 6 + 6 + 2 2t − 6 1
∫ t 2 − 6t + 25dt = 2∫ t 2 − 6t + 25dt = 2∫ t 2 − 6t + 25 dt = 2∫ t 2 − 6t + 25dt + 16 ∫ t 2 − 6t + 25dt ,
la primera es un logaritmo y la segunda un arcotangente:
1
1 1 1 4 t −3
16 ∫ 2 dt = 16 ∫ dt = 16 ∫ dt = 4 ∫ dt = 4arctg  
t − 6t + 25 2
(t − 3 ) + 16  t − 3  2   t − 3  2   4 
16   + 1   + 1
 4    4  
5t 2 − 2t + 25 t −3
Luego ∫ t 3 − 6t 2 + 25t dt = ln t ( )
+ 2 ln t 2 − 6t + 25 + 4arctg   y en definitiva:
 4 
5 ⋅ (ln x )2 − 2 ⋅ ln x + 25  (ln x ) − 3 
∫ x ⋅ ((ln x )3 − 6 ⋅ (ln x )2 + 25 ⋅ ln x ) ⋅dx = ln ln x + 2 ln ((ln x ) − 6(ln x ) + 25 ) + 4arctg  4  + C
2

8.6.2 Trigonométricas

Integrales del tipo ∫ R (senx ,cos x ) ⋅ dx función impar en senx , cambio cos x = t

Integrales del tipo ∫ R (senx , cos x ) ⋅ dx función impar en cos x , cambio senx = t

Integrales del tipo, ∫ R (senx ,cos x ) ⋅ dx función par en senx y en cos x , cambio tgx = t

x
Integrales del tipo, ∫ R (senx ,cos x ) ⋅ dx los demás casos, cambio general tg   = t
2

Ejemplos

∫ cos
2
Ejemplo 8.39 La integral x ⋅ sen 3 x ⋅ dx es impar en senx , ya que

{cos 2
}
x ⋅ (− senx )3 = − cos 2 x ⋅ sen 3 x . Por tanto se hace el cambio cos x = t ⇒ −senx ⋅ dx = dt

8-13
Primitivas. Métodos de integración. Tema 8

∫ cos
2
∫ ∫ ( )
x ⋅ sen 3 x ⋅ dx = cos 2 x ⋅ sen 2 x ⋅ senx ⋅ dx = cos 2 x ⋅ 1 − cos 2 x ⋅ senx ⋅ dx = − t 2 1 − t 2 ⋅ dt ∫ ( )
3 5
t t cos 3 x cos 5 x
que es inmediata así ∫ cos x ⋅ sen
2 3
x ⋅ dx == − ∫ t (1 − t )⋅ dt = − +
2 2
=− + +C
3 5 3 5

dx
Ejemplo 8.40 ∫ cos x , es impar en cos x ; se hace el cambio senx = t ⇒ cos x ⋅ dx = dt y
dx cos x cos x dt
resulta: ∫ cos x = ∫ cos 2 x dx =∫ 1 − sen 2 x dx =∫ 1 − t 2 , que hacemos por descomposición:
1 A B A(1 + t ) + B(1 − t )
= + = ⇒ 1 = A(1 + t ) + B (1 − t ) ,
1− t 2 1− t
1+ t 1− t 2
1 1
si t = 1 ⇒ A = , si t = −1 ⇒ B = . Por tanto:
2 2
dx dt 1 dt 1 dt 1 1 1 1
∫ cos x = ∫ 1 − t 2 = 2 ∫ 1 − t + 2 ∫ 1 + t = − 2 L 1 − t + 2 L 1 + t = − 2 L(1 − senx ) + 2 L(1 + senx ) + C

dx
Ejemplo 8.41 ∫ cos 2 x − sen 2 x par en senx y en cos x , pues

1 1
2 2
= .
(− cos x ) − (− senx ) cos x − sen 2 x
2

1
Por tanto, se hace el cambio tg (x ) = t ⇔ dx = dt , y dividiendo dentro de la integral
cos 2 x
1
dx
cos 2 x dx dt
numerador y denominador por cos 2 x ∫ cos 2 x − sen 2 x ∫ 1 − tg 2 x = ∫ 1 − t 2 ,
se tiene: =

integral que ha sido resuelta en el ejercicio anterior. Por tanto:


dx dt 1 1 1 1 + tgx
∫ cos 2 x − sen 2 x = ∫ 1 − t 2 = − 2 L 1 − tgx + 2 L 1 + tgx = 2 L 1 − tgx + C .
x
- Cambio general tg   = t . Aplicando las correspondientes fórmulas trigonométricas se
2
2t 1− t 2 2t x
tiene: senx = 2
, cos x = 2
, tgx = 2
. Por otra parte como tg   = t ⇔ x = 2arctgt ,
1+ t 1+ t 1− t 2
2dt
diferenciando se obtiene: dx = .
1+ t 2

dx x
Ejemplo 8.42 ∫ 1 + senx + cos x
. Aquí es obligado hacer el cambio general tg   = t , se tiene:
2
2dt 2dt
dx 1+ t 2
1+ t 2 2 ⋅ dt  x
∫ 1 + senx + cos x = ∫ 2t 1− t 2
= ∫ 2
1 + t + 2t + 1 − t 2
=∫
2 + 2t
= ln 1 + t = ln1 + tg  + C
 2
1+ 2
+ 2 2
1+ t 1+ t 1+ t

Nota Cualquiera de los tres casos anteriores a éste se podría haber resuelto con el cambio
x
general tg   = t aunque este cambio conduce habitualmente a integrales cuya resolución es
2
larga y por lo tanto lo utilizaremos cuando no sea factible ninguno de los anteriores.

8-14
Primitivas. Métodos de integración Tema 8

8.7 Integrales no resolubles por cuadraturas


Como dijimos, no siempre existe primitiva de una función. Cuando una función no tiene
primitiva la única opción es contentarse con funciones que sean aproximación de las dadas y,
entonces, integrar.

A modo de ejemplo se dan algunas funciones que no pueden integrarse.

senax x dx cos ax x dx dx
∫ x
dx , ∫
senax
dx , ∫
x ⋅ senax
, ∫
x
dx , ∫
cos ax
dx , ∫
x ⋅ cos ax
, ∫
x ⋅ cos ax
,

tgx x dx e ax dx x ⋅ dx
∫ x ∫ x ⋅ tgax ∫ ∫ x dx , ∫ e ⋅ ln x ⋅ dx , ∫ ln x , ∫ ln x ,
ax
dx , ∫ tgx dx , , x ⋅ tgax ⋅ dx ,

ax 2
∫ ln(cos x ) ⋅ dx , ∫ e ⋅ dx .

8-15
Primitivas. Métodos de integración. Tema 8

AMPLIACIÓN

I. Integración de funciones racionales con raíces complejas múltiples

Al factorizar el denominador aparece un factor de segundo grado del tipo

(ax 2
+ bx + c )
n
, donde ax 2 + bx + c no tiene raíces reales.

El método de Hermite resuelve este tipo de integrales, pero no lo veremos aquí; por lo
que damos un método que se puede aplicar en casos sencillos.
En la descomposición, las fracciones simples correspondientes a este factor son del
M 1 x + N1 Mn x + Nn
tipo + ⋅⋅⋅+ , la resolución de las integrales de este tipo de
2
ax + bx + c (ax 2
+ bx + c )n

fracciones se hace mediante un proceso de reducción aplicando la fórmula de integración por


partes. El cálculo es largo y supera nuestro propósito, no obstante y a modo de ilustración
resolveremos un caso de los más sencillos.

dx
Ejemplo 8.43 ∫
(x 2
+4 )
2

dx 1 4dx 1 x2 + 4 − x2 1 x2 + 4 1 − x2
∫ = ∫ 4 ∫ x2 + 4 2
dx==
4 ∫ x2 + 4 2
dx =
4 ∫ x2 + 4
dx =
(x 2
+4 ) 2 4 x2 + 4 ( 2
) ( ) ( ) ( ) 2

1
1 x2 + 4 1 1 1 2 1 x
4∫
la primera dx = ∫ 2 dx = ∫ dx = arctg
( 2
x +4
2 4 ) x + 4 8  2( 
  x  + 1
8 )
2
 2  
 
1 x2
la segunda − ∫ dx la hacemos por partes eligiendo:
(
4 x2 + 4 2 )
u = x du = dx 
 xdx xdx 1 2 xdx  1
dv = ∫ ⇒ v=  así que:
∫ = ∫ =−
( )


2
x +4 ( ) 2
(x 2
+4 )
2 2 x2 + 4( )2
2 x2 + 4 

1 x2 1 x 1 1 1 x 1 x
− ∫ dx = − ∫ 2 dx = − arctg ,obsérvese que esta
4 x2 + 4 (2
)
4 2⋅ x2 + 4 2 8 x +4 8 x2 + 4(2 16 2 ) ( )
última integral es la resuelta anteriormente.
dx 1 x 1 x 1 x 1 x 1 x
Luego, ∫ = arctg + − arctg = + arctg .
x2 + 4
2
(
8 )
2 8 x2 + 4 2 16 2 8 x2 + 4 2 16 2 ( ) ( )

II. Integración de funciones irracionales cuadráticas

Se trata de integrar funciones del tipo ax 2 + bx + c  dx . Dado que todo


 ∫  x ,
polinomio de grado 2 se puede escribir de una de estas tres formas:
(
K (x + b )2 − a 2 , K a 2 − (x + b )2 o bien K (x + b )2 + a 2 ) ( ) ( )
se presentan tres tipos de integrales:

8-16
Primitivas. Métodos de integración Tema 8

∫ a
2
− x 2 dx se resuelve con el cambio x = a ⋅ sent o x = a ⋅ cos t

∫ x
2
− a 2 dx se resuelve con el cambio x = a ⋅ sec t o x = a ⋅ cos ect

∫ x
2
+ a 2 dx se resuelve con el cambio x = a ⋅ tgt o x = a ⋅ cot gt

Ejemplos

Ejemplo 8.44 ∫ a 2 − x 2 dx .
Se hace el cambio x = a ⋅ sent → dx = a ⋅ cos t ⋅ dt
a 2 (sent ⋅ cos t + t ) a 2  x a2 − x 2 x
∫ a 2 − x 2 dx = ∫ a 2 cos 2 t ⋅ dt = = + arcsen  + C
2 2 a a a
 
2
x x x a2 − x 2
ya que t = arcsen , sent = y cos t = 1 − sen 2 t = 1 −   =
a a a a

dx
Ejemplo 8.45 ∫ 2
.
(x + 2 ) x 2 + 4 x − 1
Como x 2 + 4 x − 1 = (x + 2)2 − 5 , se tiene:
dx dx  x + 2 = 5 ⋅ sec t 
∫ =∫ ,   y se tiene:
(x + 2)2 x 2 + 4x − 1 (x + 2)2 (x + 2)2 − 5 dx = 5 ⋅ sec t ⋅ tgt ⋅ dt 
dx 5 ⋅ sec t ⋅ tgt ⋅ dt 5 ⋅ sec t ⋅ tgt ⋅ dt 1
∫ 2 2
=∫
2 2
=∫
5 sec 2 t ⋅ 5 ⋅ tgt
=
5∫
cos t ⋅ dt .
(x + 2 ) (x + 2 ) −5 5 sec t ⋅ 5 sec t − 5

dx 1 1 x 2 + 4x − 1
Por tanto: ∫ =
5
sent =
5 x +2
+ C ya que:
(x + 2)2 x 2 + 4x − 1
2
5  5  x 2 + 4x − 1
x + 2 = 5 ⋅ sec t → cos t = → sent = 1 −   = .
x +2  x +2 x+2
 

dx
Ejemplo 8.46 ∫ .
x ⋅ 2 + 3x 2
2 2 dt
Se hace el cambio x = ⋅ tgt → dx = .
3 3 cos 2 t
Realizando el cambio y simplificando se tiene:
dx 1 dt 1 1 + cos t
∫ 2
=
2
∫ sent = 2 ln 1 − cos t .
x ⋅ 2 + 3x
Deshacer el cambio de variable es trabajoso y a veces no es necesario, pero sí
conviene hacerlo:
1 + cos t 1 + tg 2 t + 1 2 + 3x 2 + 2
= =
1 − cos t 1 − tg 2 t − 1 2 + 3x 2 − 2
y queda:
dx 1 2 + 3x 2 + 2
∫ = ln +C
x ⋅ 2 + 3x 2 2 2 + 3x 2 − 2

8-17
Primitivas. Métodos de integración Tema 8

Ejercicios resueltos
Integrales sencillas (casi inmediatas):

R.1. Calcula las siguientes integrales:


dx x2
∫ (3 x ) (6 x − 7)dx
3
∫ ∫ 3x
2
a) − 7x b) c) 3
dx .
3x − 2 −2
Soluciones:

a) Inmediata ∫ (3 x
2
− 7x ) (6 x − 7)dx = 41 (3 x
3 2
− 7x ) 4
+C

dx 2 3dx 2
b) ∫ 3x − 2
= ∫ =
3 2 3x − 2 3
3x − 2 + C

x2 1 9x 2 1
c) ∫ 3x 3
−2
dx = ∫
9 3x 3 − 2
dx = ln 3 x 3 − 2 + C .
9

R.2. Calcula las siguientes integrales.

2x cos 3 x x dx
a) ∫ 1+ 4 x
dx b) ∫ sen3 x dx c) ∫ cos (x 2 2
+1 ) dx d) ∫x 2
+ a2
.

Soluciones:

2x 1 2 x ⋅ ln 2 1
a) ∫ 1+ 4 x
dx =
ln 2 1 + 2 x∫ ( )
2
dx =
ln 2
arctan 2 x + C ( )
cos (3 x ) 1 3 ⋅ cos (3 x ) 1
b) ∫ sen(3 x ) dx = 3 ∫ sen (3 x )
dx = ln sen (3 x ) + C
3

x 1 2x 1
c) ∫ cos (x 2 2
+1 ) dx = ∫ 2 2
2 cos x + 1
dx = tg x 2 + 1 + C
(2 ) ( )
1 1
dx dx
dx 2 1
d) Dividimos numerador y denominador por a 2 se tiene ∫x 2
+ a2
= ∫ a
x
2
= a
a  x 2 ∫
=
  +1   +1
a a

1 x
= arctg   + C
a a

R.3. Calcula:
 1 3 
∫ (1 − x ) ∫ (e )
5
∫  x ∫ tgx ⋅ dx
2 2 2x
a) x ⋅ dx b) − + x  ⋅ dx c) + 2 e 2 x dx d)
x3 
tgx dx dx
∫ e sen (e )⋅ dx ∫ cos ∫ x ⋅ sen ∫x
x x
e) f) dx g) h)
2
x 2
(ln x ) 2
+ 6x + 9

8-17
Primitivas. Métodos de integración Tema 8

Soluciones:

 1 5
2
3
2
7

∫ (1 − x ) x ⋅ dx =  x 2 − x 2  ⋅ dx = x 2 − x 2 + C

2
a)
  3 7
 
 2 1 3   1
1 3 1 −2 3
4
1 3 1 33 4
∫  x − 3 + x  ⋅ dx =  x − x + x 3  ⋅ dx = x − x + x 3 = x − 2 +

2 −3
b) x +C
 x    3 2 4 3 2x 4
 
1 1 2x
∫( ) ∫( ) ( )
5 5 6
c) e 2 x + 2 e 2 x dx = e 2 x + 2 2e 2 x dx = e + 2 +C
2 12
−senx
d) ∫ ∫
tgx ⋅ dx = − dx = − ln cos x + C
cos x
e) ∫ e sen (e ) ⋅ dx = − cos (e ) + C
x x x

tgx 1
f) ∫ dx = ∫ tgx ⋅ D(tgx ) dx = tg
2
2
x +C
cos x 2
dxD(ln x )
g) ∫ x ⋅ sen (ln x ) 22
(ln x )
= ∫ sen
dx = − cot g (ln x ) + C

dx 1
h) ∫ x 2 + 6x + 9 ∫
= (x + 3 )− 2 dx = −(x + 3 )−1 = −
x+3
+C

R.4. Calcula:
x ⋅ dx cos x 3
a) ∫ 1 + (x 2
−2 )2
b) ∫ 1 + sen x dx 2
c) ∫ (1 + cos x ) senx ⋅ dx
3x + 6x 2
x ⋅ dx 3x 2 + 2
d) ∫ x 3 + 3x 2
dx e) ∫ sen 2 x ⋅ cos x ⋅ dx f) ∫ 2 + x4
g) ∫ (x 3
)
+ 2x − 1
3
dx

x +1
∫2 ∫
x
h) ⋅ 3 x dx i) dx .
x

Soluciones:

x ⋅ dx 1 2 x ⋅ dx 1
a) ∫ = ∫ = arctg x 2 − 2 + C ( )
1+ x − 2 ( 2
)2 2 1+ x − 2
2
(
2 2 )
cos x d ( senx )
b) ∫ 2
dx = ∫ = arctg (senx )
1 + sen x 1 + sen 2 x
3 3 1
c) ∫ (1 + cos x ) senx ⋅ dx = − ∫ (1 + cos x ) ⋅ d (cos x ) = −
4
(1 + cos x )4 + C
3x 2 + 6x 3x 2 + 6x
d) ∫ 3 2
dx =2∫
3 2
dx =2 x 3 + 3 x 2 + C
x + 3x 2 ⋅ x + 3x
1
e) ∫ sen 2 x ⋅ cos x ⋅ dx = ∫ sen 2 x ⋅ d (senx ) = sen 3 x + C
3
x ⋅ dx x ⋅ dx 1 2 ⋅ x ⋅ dx 2  x2 
f) ∫ =∫ = ∫ = arctg   +C

2+ x 4
  2 2  2 2  x2 
2 4  2
 x  
1+  
2 ⋅ 1 +   2
  2    
 
3x 2 + 2 1 3 1
g) ∫ (
dx = ∫ x 3 + 2 x − 1 ) d (3 x
−3 2
)
+2 = − ( −2
x + 2x − 1 = −) +C
(x 3
+ 2x − 1 )3 2 3
2 x + 2x − 1
2
( )
x
6
∫2 ⋅ 3 x dx = ∫ 6 x dx =
x
h) +C
ln 6

8-18
Primitivas. Métodos de integración Tema 8

x +1  x 1   1 − 
1
2
3 1
2
i) ∫ dx = ∫  + dx = ∫  x 2 + x 2  ⋅ dx = x 2 + 2 x 2 = x3 + 2 x + C
x  x x   3 3
 

R.5. Integra por partes:

∫ ∫e ∫ d) ∫ cos (ln x ) ⋅ dx
2x
a) arccos x ⋅ dx b) sen 3 x ⋅ dx c) x ln x ⋅ dx

∫ (x ) ∫x
2
e) + 1 ⋅ e − x dx f) 2
⋅ e − x −1dx

Soluciones:

 1 
u = arccos x du = − dx  xdx
a) ∫ arccos x ⋅ dx =  ⇒ 1− x 2  = x ⋅ arccos x + ∫ =
dv = dx  1 − x 2
 v=x 
= x ⋅ arccos x − 1 − x 2 + C
 2x du = 2e 2 x dx  − e 2 x cos 3 x 2 2 x
u = e
b) ∫ e sen 3 x ⋅ dx = 
2x
⇒ + ∫ e cos 3 x ⋅ dx =
v = − cos 3 x 
1 =
dv = sen 3 x ⋅ dx 3 3
3 
 2x du = 2e 2 x dx  − e 2 x cos 3 x 2  e 2 x sen 3 x 2 2 x 
u = e
 ⇒ 1 = +  − ∫ e sen 3 x ⋅ dx  , llamando
dv = cos 3 x ⋅ dx v = sen 3 x  3 3  3 3 
3
F ( x ) = ∫ e 2 x sen 3 x ⋅ dx ,
− e 2 x cos 3 x 2 2 x 4 − 3e 2 x cos 3 x 2e 2 x sen3 x
F (x ) = + e sen 3 x − F (x ) ⇒ F (x ) = + +C
3 9 9 13 13

 1 
u = ln x du = dx  3 3
x  2 x ln x 4 x
c) ∫ x ln x ⋅ dx =  ⇒ 3  = − +C
dv = x ⋅ dx v= x
2 2  3 9
 3 

d) ∫ cos (ln x ) ⋅ dx antes de aplicar la fórmula multiplicamos y dividimos por x.

cos (ln x ) u = x du = dx 


∫ cos (ln x ) ⋅ dx = ∫ x ⋅ ⋅ dx =  cos (ln x ) ⇒ 
x 
dv = ⋅ dx v = sen (ln x )
x 
∫ cos (ln x ) ⋅ dx = xsen (ln x ) − ∫ sen (ln x ) ⋅ dx volvemos a multiplicar y dividir por x antes de aplicar
u = x du = dx 
la fórmula otra vez eligiendo ahora:  sen (ln x ) ⇒  con lo que:

dv = ⋅ dx v = − cos (ln x )
x 
∫ cos (ln x ) ⋅ dx = xsen(ln x ) + x cos (ln x ) − ∫ cos (ln x ) ⋅ dx pasamos la integral al primer término y
1
despejamos: ∫ cos (ln x ) ⋅ dx = [xsen (ln x ) + x cos (ln x )] + C
2

u = x 2 + 1 du = 2 xdx 
e) ∫ (x
2
)
+ 1 ⋅ e − x dx  −x
dv = e ⋅ dx

v = −e − x 
−x
(
 = − x + 1 e + ∫ 2 x ⋅ e dx =
2 −x
)

u = 2 x du = 2dx 

dv = e −x
⋅ dx

v = − e 
2
−x  = − x + 1 e
−x
( )
− 2 xe − x + ∫ 2 ⋅ e − x dx = − x 2 + 2 x + 3 e − x + C( )

8-19
Primitivas. Métodos de integración Tema 8

u = x 2 du = 2 xdx 
∫x + ∫ 2 x ⋅ e − x −1dx =
2
f) ⋅ e − x −1dx  ⇒  = −x e
2 − x −1
dv = e − x −1
⋅ dx v = −e − x −1 
u = 2 x du = 2dx 

dv = e
− x −1
⋅ dx
⇒ 2 − x −1
 = −x e
v = −e − x −1 
(
− 2 xe − x −1 + ∫ 2 ⋅ e − x −1dx = − x 2 + 2 x + 2 ⋅ e − x −1 + C )
R.6. Integra mediante trucos o formulas de descomposición:

dx dx
a) ∫ 1 − senx b) ∫ x +1 + x −1
c) ∫ senx ⋅ cos 2x ⋅ dx d) ∫ cos x ⋅ cos 4 x ⋅ dx
e) ∫ senx ⋅ sen9x ⋅ dx .
Soluciones:

dx (1 + senx )dx (1 + senx )dx


a) ∫ 1 − senx = (se multiplica y divide por 1 + senx ) = ∫ 1 − sen x2
= ∫ cos 2 x
=

dx senxdx
= ∫ cos 2
x
+ ∫ cos 2
x
= tgx + sec x + C

dx
b) ∫ x +1 + x −1
, multiplicamos y dividimos por el conjugado del denominador.


dx
=∫
( x +1 − x −1 ) 1 1 1 1
dx = ∫ (x + 1) 2 dx − ∫ (x − 1) 2 dx = 
1
(x + 1)3 − (x − 1)3  + C
x +1 + x −1 (x + 1) − (x − 1) 2 2 3 

1
c) ∫ senx ⋅ cos 2 x ⋅ dx = ∫ (sen3 x + sen (− x )) ⋅ dx = − cos 3 x + cos x + C
2 6 2

1
d) ∫ cos x ⋅ cos 4 x ⋅ dx = ∫ (cos 5 x + cos (− 3 x )) ⋅ dx = − sen5 x + sen3 x + C
2 10 6

1
e) ∫ senx ⋅ sen9 x ⋅ dx = (cos (− 8 x ) − cos 10 x ) ⋅ dx = sen8 x − sen10 x + C
2∫ 16 20

R.7. Integra las fracciones algebraicas.

6 x 2 − 10 x + 2 − 2x 2 + x − 2 2x 2 − 4x + 1 x2 + 4
a) ∫ x 3 − 3 x 2 + 2x dx b) ∫ x3 − x2
dx c) ∫ x ⋅ (x − 1)2
dx d) ∫ x 2 − 5 x + 4 dx

x2 +1 x +1 3x x2 −1
e) ∫ x 2 − 4 x + 13 dx f) ∫ x 3 + x 2 − 6 x dx g) ∫ x 2 + 2 x + 3 dx h) ∫ x ⋅ (x 2 + 1) dx
x+2
i) ∫ x 3 − 4 x 2 + 4 x dx .

Soluciones:

6 x 2 − 10 x + 2
a) ∫ x 3 − 3 x 2 + 2x dx , x 3 − 3 x 2 + 2 x = x ⋅ (x − 1) ⋅ (x − 2 ) raíces reales simples.

6 x 2 − 10 x + 2 A B C A ⋅ (x − 1) ⋅ (x − 2) + B ⋅ x ⋅ (x − 2 ) + C ⋅ x ⋅ (x − 1)
3 2
= + + = identificando:
x − 3x + 2x x x −1 x − 2 x 3 − 3 x 2 + 2x
x = 0 → A = 1; x = 1 → B = 2; x = 2 → C = 3 , por tanto:

8-20
Primitivas. Métodos de integración Tema 8

6 x 2 − 10 x + 2 1 2 3
∫ x 3 − 3 x 2 + 2 x dx = ∫ x dx + ∫ x − 1 dx + ∫ x − 2 dx = ln x + 2 ln x − 1 + 3 ln x − 2 + C

− 2x 2 + x − 2
b) ∫ 3 2
dx , x 3 − x 2 = x 2 ⋅ (x − 1) una raíz real simple y una doble.
x −x
2
− 2x + x − 2 A1 A2 B A1 ⋅ x ⋅ (x − 1) + A2 ⋅ (x − 1) + B ⋅ x 2
+ + = = identificando:
x3 − x2 x x2 x −1 x3 − x2
x = 0 → A2 = 2; x = 1 → B = −3; x = −1 → A1 = 1 , por tanto:
− 2x 2 + x − 2 1 2 −3 2
∫ 3
x −x 2
dx = ∫ dx + ∫ 2 dx + ∫
x x x − 1
dx = ln x − − 3 ln x − 1 + C
x

2x 2 − 4x + 1
c) ∫ x ⋅ (x − 1)2
dx , una raíz real simple x = 0 y una doble x = 1 .

2x 2 − 4x + 1 A B1 B2 A ⋅ (x − 1)2 + B1 ⋅ x ⋅ (x − 1) + B2 ⋅ x
+ + == identificando:
x ⋅ (x − 1)2 x x − 1 (x − 1)2 x ⋅ (x − 1)2
x = 0 → A = 1; x = 1 → B2 = −1; x = −1 → B1 = 1 , por tanto:
2x 2 − 4x + 1 dx dx dx 1
∫ x ⋅ (x − 1) 2
dx = ∫
x
+∫
x −1
−∫
(x − 1)2
= ln x + ln x − 1 +
x −1
+C

x2 + 4
d) ∫ x 2 − 5 x + 4 dx , x 2 − 5 x + 4 = (x − 1) ⋅ (x − 4 ) raíces reales simples.

x2 + 4 5x
2
= 1+ 2
x − 5x + 4 x − 5x + 4
5x A B A ⋅ (x − 4 ) + B ⋅ (x − 1) 5 20
2
= + = 2
; x = 1→ A = − ; x =4→B = , así:
x − 5x + 4 x − 1 x − 4 x − 5x + 4 3 3
x2 + 4 5 1 20 1 5 20
∫x 2
− 5x + 4 ∫
dx = dx −
3 ∫ x − 1 dx + 3 ∫ x − 4 dx = x − 3 ln x − 1 + 3
ln x − 4 + C

x2 +1 x2 +1 4 x − 12
e) ∫ x 2 − 4 x + 13 dx , dividiendo queda: 2
x − 4 x + 13
= 1+ 2
x − 4 x + 13
,por tanto

x2 +1 4 x − 12 4 x − 12
∫ x 2 − 4 x + 13 dx = ∫ dx + ∫ x 2 − 4 x + 13 dx , ∫ dx = x ,calculamos ∫ x 2 − 4 x + 13 dx , raíces
complejas en el denominador por tanto.
4 x − 12 2x − 4 − 2 2x − 4 −2
∫ x 2 − 4 x + 13 dx = 2∫ x 2 − 4 x + 13 dx = 2∫ x 2 − 4 x + 13 dx + 2∫ x 2 − 4 x + 13 dx ,
2x − 4
2∫ 2
x − 4 x + 13
dx = 2 ln x 2 − 4 x + 13 ( )
−4 1
calculamos ∫ 2 dx = −4 ∫ dx hacemos x − 2 = 3t → dx = 3dt y
x − 4 x + 13 (x − 2)2 + 9
1 3 4 4  x −2
− 4∫ 2
dx = −4 ∫ 2
dt = − arctgt = − arctg   en definitiva:
(x − 2 ) +9 9t + 9 3 3  3 
2
x +1 4  x −2
∫ x 2 − 4 x + 13 dx = x + 2 ln (x )
2
− 4 x + 13 − arctg   +C
3  3 

x +1
f) ∫ x 3 + x 2 − 6 x dx , x 3 + x 2 − 6 x = x ⋅ (x − 2) ⋅ (x + 3 ) raíces reales simples.

8-21
Primitivas. Métodos de integración Tema 8

x +1 A B C
+ = + , x + 1 = A ⋅ (x − 2) ⋅ (x + 3 ) + B ⋅ x ⋅ (x + 3 ) + C ⋅ x ⋅ (x − 2 ) dando
x 3 + x 2 − 6x x x −2 x +3
1 3 2
valores a x se tiene: x = 0 → A = − ; x = 2 → B = ; x = −3 → C = − , por tanto:
6 10 15
x +1 1 1 3 1 2 1 1 3 2
∫ x 3 + x 2 − 6 x dx = − 6 ∫ x dx + 10 ∫ x − 2 dx − 15 ∫ x + 3 dx = − 6 ln x + 10 ln x − 2 − 15 ln x + 3 + C

3x
g) ∫ x 2 + 2 x + 3 dx , el denominador no tiene raíces reales.
3x 3 2x 3 2x + 2 − 2 3 2x + 2 1
∫ x 2 + 2 x + 3 dx = 2 ∫ x 2 + 2x + 3 dx = 2 ∫ x 2 + 2 x + 3 dx = 2 ∫ x 2 + 2 x + 3 dx − 3∫ x 2 + 2x + 3 dx
3 2x + 2 3 1 dx
2 ∫ x 2 + 2x + 3
(
dx = ln x 2 + 2 x + 3 hacemos ahora − 3 ∫ 2 )
dx = −3 ∫ se
2 x + 2x + 3 (x + 1)2 + 2
hace el cambio x + 1 = 2t → dx = 2dt
dx 2dt 2 dt 3 2 3 2 x +1
− 3∫ 2
= −3 ∫ 2
=−3 ∫ 2
=− arctgt = − arctg
(x + 1) +2 2t + 2 2 t +1 2 2 2
3x 3 3 2 x +1
∫ x 2 + 2 x + 3 dx = 2 ln (x )
2
+ 2x + 3 − arctg +C
2 2

x2 −1 x2 −1 A Mx + N
h) ∫ x ⋅ (x 2 + 1) dx una raíz real y dos complejas
(
x ⋅ x +1 2
) =
x
+ 2
x +1
( )
x − 1 = A ⋅ x + 1 + (Mx + N ) ⋅ x , si x = 0 → A = −1 ; x = 1 → M + N = 2 ; x = −1 → M − N = 2
2 2

x2 −1 −1 2x
por tanto M = 2 y N = 0 y ∫ x ⋅ (x 2 + 1) dx = ∫ x
dx + ∫ 2
x +1
dx = − ln x + ln x 2 + 1 + C ( )
x+2
i) ∫ x 3 − 4 x 2 + 4 x dx , x 3 − 4 x 2 + 4 x = x ⋅ (x − 2)2 , una raíz real simple x = 0 y una doble x = 2 .

x +2 A B1x+2 B2 A ⋅ (x − 2)2 + B1 ⋅ x ⋅ (x − 2) + B2 ⋅ x
=+ + ==
x 3 − 4 x 2 + 4 x x ⋅ (x − 2)2 x x − 2 (x − 2)2 x ⋅ (x − 2)2
1 1
identificando: x = 0 → A = ; x = 2 → B2 = 2; x = 3 → B1 = − , así que:
2 2
x+2 1 dx 1 dx dx 1 1 2
∫ x 3 − 4 x 2 + 4 x dx = 2 ∫ x − 2 ∫ x − 2 + 2∫ (x − 2)2 = 2 ln x − 2 ln x − 2 − x − 2 +C

Integración por cambio de variable

Trigonométricas

R.8. Integra:
a) ∫ cos 3 x ⋅ dx ∫ sen ∫ cos x ⋅ sen ∫ sec
2
b) x ⋅ cos 2 x ⋅ dx c) 4
x ⋅ dx d) 6
x ⋅ dx

e) ∫ sen 3 x ⋅ cos 7 x ⋅ dx f)
cos 3 x (cos 2
)
x + 4 ⋅ senx 3 cos x
∫ senx dx g) ∫ cos 2 x − 5 cos x + 4 dx h) ∫ (1 + senx )3 dx

Soluciones:

a) ∫ cos 3 x ⋅ dx cambio senx = t → cos x ⋅ dx = dt


t3 sen 3 x
∫ cos
3
( )
x ⋅ dx = ∫ cos 2 x ⋅ cos x ⋅ dx = ∫ 1 − sen 2 x cos x ⋅ dx = ∫ 1 − t 2 ⋅ dt = t − ( ) 3
= senx −
3
+C

8-22
Primitivas. Métodos de integración Tema 8

1 − cos 2 x 1 + cos 2 x
∫ sen
2
b) x ⋅ cos 2 x ⋅ dx aplicamos las fórmulas sen 2 x = , cos 2 x =
2 2
2 2
1 − cos 2 x 1 + cos 2 x 1 − cos 2 x x cos 2 x
∫ sen x ⋅ cos x ⋅ dx = ∫ 2 dx = ∫ dx = − ∫
2 2
dx repitiendo la
2 4 4 4
cos 2 2 x 1 1 + cos 4 x x sen 4 x
fórmula ∫ dx = ∫ dx = + así que:
4 4 2 8 32
x x sen 4 x
∫ sen x ⋅ cos x ⋅ dx = 4 − 8 − 32 + C
2 2

∫ cos x ⋅ sen
4
c) x ⋅ dx cambio senx = t → cos x ⋅ dx = dt
t5 1
∫ cos x ⋅ sen x ⋅ dx = ∫ t dt =
4 4
= sen 5 x + C
5 5

d) ∫ sec 6 x ⋅ dx = ∫ sec 2 x ⋅ sec 4 x ⋅ dx = ∫ sec 2 x ⋅ 1 + tg 2 x ( ) 2


⋅ dx cambio tgx = t → sec 2 x ⋅ dx = dt
2t 3 t 4 2tg 3 x tg 4 x
6 2
(
2 4
)2
∫ sec x ⋅ dx = ∫ 1 + t dt = ∫ 1 + 2t + t ⋅ dt = t + ( ) 3
+
4
= tgx +
3
+
4
+C

e) ∫ sen 3 x ⋅ cos 7 x ⋅ dx cambio senx = t o bien cos x = t , hacemos cos x = t → −senx ⋅ dx = dt

t 8 t 10
∫ sen
3
( )
x ⋅ cos 7 x ⋅ dx = ∫ 1 − cos 2 x ⋅ cos 7 x ⋅ senxdx = − ∫ 1 − t 2 ⋅ t 7 dt = − ( ) +
8 10
deshaciendo el

cos 10 x cos 8 x
∫ sen
3
cambio x ⋅ cos 7 x ⋅ dx = − +C
10 8

cos 3 x
f)∫ senx dx cambio senx = t → cos x ⋅ dx = dt
cos 3 x 1 − sen 2 x cos x ( 1− t 2 1 ) 1( )
∫ senx ∫ ∫ t dt = ∫ t dt − ∫ t ⋅ dt = ln senx − 2 sen x + C
2
dx = dx =
senx

(cos 2
x + 4 ⋅ senx )
g) ∫ cos 2 x − 5 cos x + 4 dx , cos x = t → −senx ⋅ dx = dt

(cos 2
)
x + 4 ⋅ senx t2 + 4
∫ cos 2 x − 5 cos x + 4 dx = ∫
t 2 − 5t + 4
dt resuelta en el ejercicio 4. d) y por tanto

(cos 2
)
x + 4 ⋅ senx 5 20
∫ cos 2 x − 5 cos x + 4 dx = − 3 ln cos x − 1 + 3
ln cos x − 4 + C

3 cos x
h) ∫ (1 + senx )3 dx , senx = t → cos x ⋅ dx = dt

3 cos x −3 3 3
∫ (1 + senx )3 dx = 3 ∫ (1 + t ) dt = − (1 + t )−2 = − +C
2 2(1 + senx )2

Otros cambios de variable

R.9. Integra:
x 2 + 2x dx dx x3
a) ∫3 dx b) ∫ 1+ x
c) ∫ 1+ e x d) ∫ dx
x 3 + 3x 2 x −1

Soluciones:

8-23
Primitivas. Métodos de integración Tema 8

x 2 + 2x dt
a) ∫3 ( )
dx cambio x 3 + 3 x 2 = t → 3 x 2 + 6 x ⋅ dx = dt → x 2 + 2 x ⋅ dx = ( ) 2
x 3 + 3x 2
2
x 2 + 2x 1 dt 1 3 3 3 3 3
∫3 dx =
2∫3t 22
= t =
4
x + 3x 2 ( )
2
+C
x 3 + 3x 2

dx
b) ∫ 1+ x
, x = t → x = t 2 → dx = 2tdt

∫ 1+
dx
x
=∫
2tdt
1+ t
(se hace la división) = ∫ 2dt − 2∫
dt
1+ t
= 2t − 2 ln (1 + t ) = 2 x − 2 ln 1 + x + C ( )
dx dt
c) ∫ 1+ e x , cambio e x = t → e x dx = dt → dx =
t
dx dt
∫ 1 + e x = ∫ t ⋅ (1 + t )
1 A B
= + , 1 = A ⋅ (1 + t ) + B ⋅ t si t = 0 → A = 1 si t = −1 → B = −1 por tanto:
t ⋅ (1 + t ) t 1 + t
dx dt
∫ 1 + e x = ∫ t ⋅ (1 + t ) = ln t − ln 1 + t = ln e − ln 1 + e = x − ln 1 + e + C
x x x

Otro modo de hacerla:


dx (1 + e x
− ex )dx = ex
∫ 1+ e x = ∫ ∫e dx − ∫
x
x x
dx la primera es inmediata e x ,para la segunda se
1+ e 1+ e
x
e dt
hace 1 + e x = t → e x dx = dt ∫ 1 + e x dx = ∫ t
( )
= ln t = ln 1 + e x llegando al mismo resultado.

x3
d) ∫ x −1
dx , cambio x − 1 = t → x = t 2 + 1 → dx = t ⋅ dt

x3 2t 7 6t 5 2 (x − 1)7 6 (x − 1)5
∫ (
dx = ∫ t 2 + 1 2dt = )
3

7
+
5
+ 2t 3 + 2t =
7
+
5
+2 (x − 1)3 + 2 x −1 +C
x −1

Para ampliar

Irracionales cuadráticas

R.10. Integra:

dx dx x2 +1
a) ∫ b) ∫ c) ∫ x
dx
x 2 1− x 2 x2 − 2

Soluciones:

dx
a) ∫ 2
cambio x = sent → dx = cos t ⋅ dt
x 1− x 2
dx cos t cos t 1− x 2
∫ =∫
sen 2 t ⋅ cos t
dt = − cot gt = −
sent
=−
x
+C
x 2 1− x 2

8-24
Primitivas. Métodos de integración Tema 8

dx cos t
b) ∫ 2
cambio x = 2 cos ec t → dx = − 2
sen 2 t
dt
x −2
dx 2 cos t 2 cos t dt
∫ 2
= −∫
2 2
dt = − ∫
cos t
dt = − ∫
sent
para resolver esta
x −2 sen t 2 cos ec t − 2 sen 2 t 2
sent
integral hacemos cos t = u → −sent ⋅ dt = du y se tiene:
dt sent ⋅ dt sent ⋅ dt du
∫ sent = ∫ sen 2 t = ∫ 1 − cos 2 t = − ∫ 1 − u 2 racional que por descomposición nos da:
( )
1 1− u dx 1 1− u 1 1 − cos t x
ln y por tanto ∫ = − ln = − ln , sent = → cos t = 1 − sen 2 t
2 1+ u 2
x −2 2 1 + u 2 1 + cos t 2

x2 − 2
2 1−
x −2 dx 1 x 1 x − x2 − 2
sustituyendo cos t =
x
y por tanto ∫ = − ln
2
= − ln
2 x + x2 − 2
+C
x2 − 2 x2 − 2
1+
x
x2 +1 dt
c) ∫ x
dx cambio x = tgt → dx =
cos 2 t
x2 +1 dt sen 2 tdt cos 2 tdt sentdt dt
∫ x
dx = ∫ 2
sent ⋅ cos t
= ∫ sent ⋅ cos t2
+ ∫ 2
sent ⋅ cos t
=∫ 2
cos t
+∫
sent
la primera es

inmediata sec t la segunda es la del ejercicio anterior deshaciendo el cambio queda:


x2 +1 1 1− x 2 + 1
∫ x
dx = x 2 + 1 + ln
2 1+ x 2 + 1
+C

8-25
Primitivas. Métodos de integración Tema 8

Ejercicios propuestos

P.1. Integra:

8x 2 3x 2 − 4x
∫ c) ∫ ctgx ⋅ dx ∫ cos xsen
3
a) dx b) ∫ x 3 − 2x 2 + 1 dx d) x ⋅ dx
(x 3
−2 )
3

 1  5(x + 1)dx x2
∫  senx − 2 cos 2x  ⋅ dx ∫ ∫ cos 2 x 3 dx h) ∫ x ⋅ x 5 ⋅ dx
4
e) f) −3
g)
x
e x dx 3x 2 − 2 ex
i) ∫ 1+ e 2x j) ∫ dx k) ∫ 3x ⋅ dx
(
1− x 3 − 2x )2

2
l) ∫ senx cos x (senx + cos x ) ⋅ dx m) ∫ 3 5 x ⋅ xdx n) ∫ 3x ⋅ 1 + 5 x 2 ⋅ dx

cos (ln x ) e −2 x dx
∫ x dx ∫ x sen (x )⋅ dx
2 3
ñ) o) ∫ 1 + e −4 x p)

P.2. Integra por partes:

a) ∫ arcsenx ⋅ dx b) ∫ arc cot x ⋅ dx c) ∫ x ⋅ cos xdx


2
d) ∫ x ln x ⋅ dx 2

∫ cos x ⋅ e f) ∫ cos 3 x ⋅ dx ∫ x ⋅ e ⋅ dx h) ∫ senx ⋅ ln(1 + senx ) ⋅ dx


x 3 x
e) ⋅ dx g)
2
i) ∫x
3
⋅ e x ⋅ dx j) ∫ x ⋅ (ln x )2 ⋅ dx k) ∫ (2 x + 4 ) ⋅ e dx
−5 x

P.3. Integra:

8x 3 + 8x 2 + x + 1 dx
a) ∫ x +1
dx b) ∫ , con a > 0 c) ∫ sen3 x ⋅ cos 5 x ⋅ dx
a − x2
2

d) ∫ cos 7 x ⋅ cos 5 x ⋅ dx e) ∫ sen 2 x ⋅ sen7 x ⋅ dx


P.4. Integra las siguientes fracciones racionales:

dx x2 6x + 8 x2
a) ∫ 1− x 2 b) ∫ 81 − x 4 dx c) ∫ x 2 + 2x + 5 dx d) ∫ 1 + 2x 3 dx
x4 − x2 −1 x +5 2x + 3 x3 +1
e) ∫ x +1
dx f) ∫ x 2 + x − 2 dx g) ∫ 3 x + 4 dx h) ∫ x 2 − 5 x + 4 dx
dx x4 dx (x + 1)dx
j)∫ x 5 + 3 x 4 + 3 x 3 + x 2 k) ∫ x 4 − 1 dx l) ∫ (x + 1) ⋅ (x 2 + 1) m) ∫ x 2 + 4x − 5
n) ∫
(x + 5)dx ñ) ∫
xdx
3
(x + 1) ⋅ (x + 2) ⋅ (x + 3) (x − 2)2
P.5. Integra:

e −tgx senx dx
∫ sen ∫ senx
5
a) ∫ cos 2 x dx b) ∫ 3 + cos 2 x dx c) x ⋅ dx d)

senx
e) ∫ 1 + sen 2 x ⋅ dx f) ∫ cos 4 x ⋅ dx g) ∫ cos 3 x + cos 2 x + 2 cos x dx
sen 3 x
∫ x ⋅ ctg (x ) ∫ (sen2x + tg 2x ) dx
2

2
h) + 1 ⋅ dx i) dx j)
cos x

8-26
Primitivas. Métodos de integración Tema 8

cos x dx dx
∫ cos ∫ 2 + cos x ∫ 3 − 2 cos x
2
k) x ⋅ sen 4 x ⋅ dx l) ∫ sen 2 x − 6senx + 12 dx m) n)

sen 2 xdx dx 1 dx
ñ) ∫ 1 + cos 4 x o) ∫
cos ec 2 x − ctg 2 x
p) ∫ 1+ x +1
dx q) ∫3 x + x
x 2
(2 + e )dx x 2

∫ cos ∫x
3
r) ∫ dx s) ∫ x
t) 3 x dx u) e −4 x dx
x+2 3e − 1
dx e 3 x dx
v) ∫ x (1 + (ln x ) ) 2
w) ∫ 2+e 6x

P.6. Integra

dx − x2 x −1
∫ (ln x )
3
a) dx b) ∫ (1 − x )2 c) ∫ x ⋅e dx d) ∫ 3 x 2 − 6 x + 5 dx
dx dx 2x 4
x3 + 3 x4
e) ∫ 1 + 2x 2
f) ∫ 1+ e x g) ∫
1+ 4 ⋅ 2x
dx h) ∫ x
dx

ex +1
∫ (2x )
dx 2 xdx
i) ∫ x3 −1 j) − 2 x − 3 ln x ⋅ dx k) ∫3 l) ∫ 1 − e x dx
(x − 2)2 − x −2
dx arcsenx x 3 + x 2 + 2x + 1
m) ∫ x[(ln x ) 3
− 2(ln x ) − ln x + 2
2
] n) ∫
1− x 2
dx ñ) ∫ x2 +1
dx

3x 2 + 5x + 1 x 2 + 3x − 4
o) ∫ x3 − x2 − x +1 dx p) ∫ x 2 − 2 x + 8 dx

Para ampliar

P.7. Integra

dx
∫ 9 − x 2 dx ∫ x 2 − 4 ⋅dx ∫ x 2 + 4 ⋅dx ∫x
2
a) b) c) d) 1 − x 2 dx e) ∫
(1+ x ) 2 2

8-27
Primitivas. Métodos de integración Tema 8

Soluciones:

8 1
P.1.a) − + C , b) ln x 3 − 2 x 2 + 1 + C , c) ln senx + C , d) sen 4 x + C ,
(
6 x −2 3
)
2 4

1 5x 4 tgx 3 44 x 15
e) − cos x − sen 2 x + C , f) x 5 + + C , g) + C , h) + C , i) arctge x + C
4 4 3 15
x
e
  2
 3 sen 3 x − cos 3 x 35x
3
(
j) arcsen x − 2 x + C , k) )
1 − ln 3
+ C , l)
3
+ C , m)
10 ln 3
+C

1 1 1
n)
5
( 3
)
1 + 5 x 2 + C , ñ) sen (ln x ) + C , o) − arctge −2 x + C , p) − cos x 3 + C
2 3

P.2. a) x ⋅ arcsenx + 1 − x 2 + C , b) x ⋅ arc cot x +


(
ln 1 + x 2)+ C , c)
2
x 2 ⋅ senx + 2 x cos x − 2senx + C
x 3 ln x x 3 senx + cos x sen 3 x
d) − + C , e) e x + C , f) senx − +C ,
3 9 2 3
g) x 3 ⋅ e x − 3 x 2 ⋅ e x + 6 x ⋅ e x − 6 ⋅ e x + C , h) − cos x ⋅ ln (1 + senx ) + cos x + x + C
2
2
ex x2 −1( )  ln x 1  x
2
10 x + 22
∫ x ⋅ e ⋅ dx =
3
i) x
+ C , j) x 2 ln x  − + + C , k) − e −5 x +C
2  2 2 4 25

8x 3 x cos 2 x cos 8 x sen12 x sen 2 x


P.3. a) + x + C , b) arcsen + C , c) − + C , d) + +C
3 a 4 16 24 4
sen 5 x sen9 x
e) − +C
10 18

1 1+ x 1 3+x 1 x  x + 1
P.4. a) ln + C , b) ln − arctg   + C , c) 3 ln x 2 + 2 x + 5 + arctg   + C d)
2 1− x 12 3 − x 6 3  2 
1 x4 x3 2x 1
6
( )
ln 1 + 2 x 3 + C , e)
4

3
− ln x + 1 + C , f) 2 ln x − 1 − ln x + 2 + C , g) + ln 3 x + 4 + C ,
3 9
x2 65 2 1 2 1
h) + 5x + ln x − 4 − ln x − 1 + C , j) 3 ln x + 1 − 3 ln x − − − +C
2 3 3 x x + 1 2(x + 1)2
1 1 1 1 1 1
k) x +
4
ln x − 1 − ln x + 1 − arctgx + C , l) ln x + 1 − ln x 2 + 1 + arctgx + C
4 2 2 4 2
( )
1 2 11 5 1 1
m) ln x − 1 + ln x + 5 + C , n) ln x + 1 + − − 3 ln x + 2 + ln x + 3 + C
3 3 4 2(x + 1) (x + 1)2 4
2
ñ) ln x − 2 − +C
x−2

3 cos x 2 cos 3 x cos 5 x


P.5. a) − e −tgx + C , b) − arctg + C , c) − cos x + − +C
3 3 3 5
1 1 − cos x 3 x sen (4 x ) sen (2 x )
d) ln + C e) − cos x + senx + C , f) + + +C
2 1 + cos x 8 32 4

8-28
Primitivas. Métodos de integración Tema 8

ln cos x ln cos 2 x + cos x + 2  2 cos x + 1 


1
g) − + arctan
+  + C
2 4 2 7  7 
1 1
2
( )
h) ln sen x + 1 + C , i) − ln cos x + cos x + C
2 2
2

tg (2 x ) x sin(4 x ) 1 1 + sin(2 x ) x sen (4 x ) sen 3 (2 x )


j) − − − sin(2 x ) + ln + C , k) − − +C ,
2 2 8 2 1 − sin(2 x ) 8 32 48
 x
 senx − 3   tg 
3 2 3 2 5  x
l) arctg   + C m) arctg  2  + C , n) arctg  5tg  + C ,
3  3  3  3  5  2 
 
 
1 1
ñ) +C , o) ln 1 − cos 2 x + C p) 2 x + 1 − 2 ln 1 + x + 1 + C ,
4 cos(2 x ) 2
q) 2 x − 33 x + 66 x − 6 ln 6 x + 1 + C (cambio 6 x = t → x = t 6 )
2 7
r) (x + 2)5 − 8 (x + 2)3 + 8 x + 2 + C , s) − 2 x + ln 3e x − 1 + C
5 3 3
t)
2
3
[
3 x ⋅ sen 3 x ]
+ cos 3 x + C , u) −
1
32
( 2
) 2
( )
4 x 2 ⋅ e − 4 x + e − 4 x + C , v) arctg ln x + C

1  e3x 
w) arctg   +C
3 2  2 
 

2
1 e−x
P.6. a) x (ln x )3 − 3(ln x )2 + 6 x ln x − 6 x + C , b) − + C , c) − +C
1− x 2

d)
1
ln 3 x 2 − 6 x + 5 + C , e)
( )
2arctg 2 x
+C ( )
, f) − ln 1 + e x + x + C , g)
1+ 4 ⋅ 2 x
+C
6 2 2 ln 2
3 4
4 3 1 1 2 x + 1
3 4 3 2
(
h) x 4 + x 3 + C , i) ln x − 1 − ln x 2 + x + 1 − 3arctg )
 +C ,
3 
 2x 3  2x 3 x 2
j)  − x 2 − 3 x  ⋅ ln x − + + 3x + C
 3  9 2
 
3t 8 63t 7 6t 5 3t 4
k) + +t6 + + + 2t 3 + 9t 2 + 18t + 18 ln t − 1 + C t 6 = x − 2 , l) x − 2 ln 1 − e x + C
4 7 5 2
1 1 1
m) ln x − 2 + ln x + 1 − ln x − 1 + C , n)
(arcsenx )2 + C , ñ) x 2 + x + 1 ln x 2 + 1 + C , o) ( )
3 6 2 2 2 2
13 9 1 5 x − 1
ln x − 1 + − ln x + 1 + C , p) x + ln x 2 − 2 x + 8 − 7arctg
4 2(x − 1) 4 2 7
9 9 1 + cos t 2cost 2
P 7. a) t + sen 2t con x = 3sent , b) ln + con x =
2 4 1 − cos t sen 2t sent
1 + sent 2 t sen 4t arctgx x
c) ln − con x = 2tgt , d) − (x = sent ) , e) + +C
1 − sent cos 2 t 8 32 2 2( x 2 + 1)

8-29
Integral definida Tema 9

Tema 9

Integral definida

9.1 Introducción
El concepto de Integral Definida viene a resolver el problema del cálculo del área de
una región plana limitada, al menos, por una curva. Aunque, con los siglos, se multiplicarán sus
aplicaciones.

Desde tiempo inmemorial se conocen buenas aproximaciones del área del círculo de
radio conocido, pero el problema surgía cada vez que la línea curva en cuestión no era un arco
de circunferencia.

Fue Arquímedes (siglo III a. de C.) quien, mejorando el método de exhaución de


Eudoxo (siglo IV a. de C.), dio las primeras ideas para resolver este problema. De manera
análoga a como se hizo con el círculo, aproximar su área mediante las áreas de polígonos
regulares inscritos y circunscritos al mismo con un número de lados cada vez mayor,
Arquímedes aproximó la superficie de una región limitada por una curva parabólica mediante
rectángulos inferiores y superiores a la curva en cuestión.

Pero como los griegos sentían “horror por el infinito”, Arquímedes llegó a este resultado
por su propio método en el que evitaba el uso del infinito. Pero el cómo llegó al resultado es
cuestión que se puede leer en otros libros. Ahora, basta saber que el método es similar a la
definición que posteriormente se dará.

A partir del siglo XVII, la idea de Arquímedes fue retomada y mejorada por los
matemáticos. Pero, cuando descubrieron la relación entre el problema del área y el de la recta
tangente, la integración abandonó su origen geométrico y tan sólo fue considerada como la
operación inversa de la diferenciación.

Fue Cauchy, en el siglo XIX, quien recuperó el sentido geométrico de la integral como
área. Pero fue Riemann quien, posteriormente, dio una definición más general de integral
basada en el método de Cauchy de aproximación por sumas sucesivas que, en último término,
se remonta a Arquímedes.

No obstante, la generalización del concepto de integral siguió durante la primera mitad


del siglo XX, hasta el punto de que se ha llegado a decir que “aunque la integración es tan
antigua como la época de Arquímedes, la teoría de la integral ha sido una creación del siglo
XX”.

9-1
Integral definida Tema 9

9.2 Definición de Integral de Riemann


Sea f (x ) una función continua en [a, b ] . Si se divide el intervalo [a, b ] en n partes
iguales, mediante los puntos a = x 0 < x1 < x2 <KK < x n = b , se obtienen n subintervalos de la
forma [x i −1 , x i ] con i = 1,2,K, n .

b
Definición Llamamos integral definida de f (x ) entre a y b, y la denotaremos por ∫ f (x )dx ,
a
al límite
b
∫ f (x )dx =
a
lim [f (c1 )(x1 − x 0 ) + f (c 2 )(x 2 − x1 ) + K + f (c n )(x n − x n −1 )] =
n →∞

 n 
n → ∞

= lim  f (c i )(x i − x i −1 ) , con c i ∈[x i −1 , x i ] ,

 i =1 
cuando este límite exista y sea el mismo para cualquier elección de los puntos c i . En tal
caso, se dice que f es integrable en [a, b ] .

a xi- 1 ci xi b

Notas

- A f (x ) se le llama integrando y los valores a y b reciben el nombre de límites de integración.

- El segundo miembro es una suma "infinita" de áreas de rectángulos, con signos positivos o
negativos según sea el signo de f. Si f (x ) es positiva en todo el intervalo [a, b ] es fácil ver que,
al tender n a infinito, la suma de éstas áreas coincidirá con el área limitada por la gráfica de la
curva y = f (x ) , el eje OX y las rectas x = a , x = b .

- El símbolo ∫ viene a ser una deformación de la inicial de la palabra suma. Y con f (x )dx se
da a entender el área de un rectángulo de altura f (x ) y base dx . Esto es, un diferencial de
área.

a b
- Por convenio, si a < b y f es integrable en [a, b ] , se define ∫ f = −∫ f . Por otro lado, para
b a
a
cualquier f definida en a se considera ∫a
f =0 .

9-2
Integral definida Tema 9

Proposición Sean f una función real integrable en [a, b] , m = inf {f (x ) : x ∈[a, b ]} y


M = sup {f (x ) : x ∈[a, b ]} , entonces:
b

m ⋅ (b − a ) ≤ f ≤ M ⋅ (b − a ) .
a

(Demostración en el anexo final).

Una ilustración gráfica de la proposición anterior puede verse en las figuras siguientes:

a b a b

Dada la definición de integral definida, se plantean dos cuestiones esenciales:


• ¿Para qué funciones existe la integral definida?
• En caso de que exista, ¿cómo obtenerla?
La primera cuestión se responde en la siguiente proposición. Mientras que la otra
requiere, en ocasiones, del concepto de primitiva (ver Tema 8) y de la Regla de Barrow que se
verá más adelante.

9.3 Propiedades de las funciones integrables

Proposición Sea f : [a, b ]→R una función.


• Si f es continua en [a, b ] , entonces f es integrable en [a, b ] .
• Si f es acotada y monótona a trozos en [a, b ] , entonces f es integrable en [a, b ] .

Las siguientes propiedades se han trabajado ya en la integral indefinida.

Linealidad de la integral

1. Sean f y g funciones reales integrables en [a, b ] , entonces la función f + g es integrable


en [a, b ] y se cumple
b b b
∫a (f + g ) = ∫af +∫a g .
2. Sea f una función real integrable en [a, b ] , entonces para todo α∈R , la función α ⋅ f es
integrable en [a, b ] y se cumple
b b

a
α⋅f = α⋅ f . ∫
a

9-3
Integral definida Tema 9

Proposición ( Aditividad respecto del intervalo de integración) Sea f una función real
integrable en [a, b ] y sea c∈(a, b ) , entonces f es integrable en [a , c ] y [c , b ] y, además, se
cerifica
b c b
∫ ∫ ∫
f = f+ f .
a a c

Nota El recíproco también es cierto; esto es, si f es integrable en [a , c ] y [c , b ] , entonces f es


integrable en [a, b ] y se verifica la igualdad.

2 1 2
∫x ∫x ∫
2 2
Ejemplo 9.1 La proposición anterior afirma que dx = dx + x 2 dx . En el ejemplo
0 0 1
2 8
9.17 de “para ampliar” se ve que ∫0
x 2 dx =
3
. Utilizando el mismo método se puede
comprobar que
1 1 2 7
∫x ∫
2
dx = y x 2 dx = ,
0 3 1 3
de donde surge inmediata la igualdad.

Propiedades de la monotonía Sean f y g funciones reales integrables en [a, b ] .


b
• Si f (x ) ≥ 0 , ∀ x ∈ [a, b ] , entonces ∫f ≥ 0 .
a
b b
• Si f (x ) ≤ g (x ) , ∀ x ∈ [a, b ] , entonces ∫ ∫ g.
a
f ≤
a

Teorema del valor medio para integrales o Teorema del valor intermedio Sea f
una función real continua en [a, b ] , entonces existe un c∈(a, b ) tal que
b
∫ f (x )dx = f (c )(b − a ) .
a

Demostración.- Como f es continua en


[a, b] , por el Teorema de Bolzano-
Weierstrass (ver tema 3), f alcanza el
máximo y el mínimo en el intervalo [a, b ] .
Sean m y M los valores mínimo y máximo,
respectivamente, de f en dicho intervalo.
Aplicando la proposición del
epígrafe 9.2, se tiene que
b

m ⋅ (b − a ) ≤ f ≤ M ⋅ (b − a ) .
a
a c c b
Dividiendo las anteriores
desigualdades entre (b − a ) queda
1 b
m≤
b−a a ∫
f ≤M .
Ahora bien, como f es continua, por el teorema de los valores intermedios (ver tema 3)
se sabe que f alcanza todos los valores comprendidos entre m y M. En particular, el valor
1 b 1 b

b−a a ∫f se alcanzará para algún c ∈(a, b ) ; esto es, f (c ) =


b−a a
f , de donde se sigue lo que ∫
se quería demostrar.

9-4
Integral definida Tema 9

Nota Gráficamente, si f (x ) ≥ 0 , este teorema expresa que existe un punto c del intervalo [a, b ]
tal que el área limitada por la curva, el eje OX y las rectas x = a , x = b , es igual a la del
rectángulo de base (b − a ) y altura f (c ) . Esto es, se compensan zonas de área.

9.4 Teoremas fundamentales del Cálculo


Los teoremas de este epígrafe se llaman fundamentales porque prueban que la
integración es la operación inversa de la derivación y, a la vez, dan un método para resolver la
integral definida de aquellas funciones que admiten primitivas.

Sea f una función real integrable en [a, b ] , y S la función definida en [a, b ] del siguiente
x
modo S (x ) = ∫ f (t ) dt .
a

Nota A la función S se le suele llamar función área, porque en el caso de que f sea no
negativa en el intervalo [a, b ] , el valor S (x 0 ) determina, para cada x 0 del intervalo, el área
limitada por la gráfica de f, el eje OX y las rectas x = a , x = x 0 .

a x b

Primer teorema fundamental del Cálculo Integral


Sea f una función continua en [a, b ] , y S la función definida anteriormente.
Entonces:
• S es derivable en [a, b ] .
• S es una primitiva de f en [a, b ] .

Demostración.- En un mismo paso se prueba que S es derivable y que su derivada es f. En


efecto, para todo x ∈ [a, b ] se tiene:
x +h x x +h a

S ′(x ) = lim
S (x + h ) − S (x )
= lim

a
f (t ) dt − ∫
a
f (t ) dt
= lim
∫ a
f (t ) dt + ∫ x
f (t ) dt
=
h →0 h h →0 h h →0 h
x +h

= lim
∫ x
f (t ) dt
f (c ) ⋅ h
= lim= lim f (c ) = f (x ) , con c ∈ (x, x + h ) (como h → 0 , entonces
h →0 h h h →o c →x
c → x ) y, como f es continua, se sigue el teorema.

Nota Por un lado, este teorema afirma que toda función continua admite una primitiva en
 x 
 a  ∫
forma de integral definida y, por otro, como D  f (t ) dt  = f (x ) , permite afirmar que en cierto

sentido la derivación es la operación contraria de la integración.

9-5
Integral definida Tema 9

Regla de Barrow (Segundo teorema fundamental del Cálculo Integral)


Sea f una función continua en [a, b ] , y F una primitiva de f en [a, b ] .
Entonces:
b
∫ f = F( b ) − F( a ) = F( x )
a
a b

x
Demostración.- Por el primer teorema fundamental se sabe que S (x ) = ∫ f (t ) dt
a
es una
primitiva de f.
Como dos primitivas difieren en una constante F ( x ) − S( x ) = k , con k una constante.
Ahora bien, como S (a ) = 0 y F ( a ) − S( a ) = k , se deduce que k = F ( a ) .
Por otro lado, de F ( b ) − S( b ) = k y k = F (a ) , se tiene que F (b ) − S (b ) = F (a ) .
b
Despejando S (b ) , se tiene S (b ) = F (b ) − F (a ) y como, por definición, S (b ) = ∫ f (x ) dx
a
,
se sigue de manera inmediata lo que se quería demostrar.

Nota La importancia del Teorema de Barrow es también doble:


• por una parte, permite calcular una integral definida a partir de una primitiva del
integrando, con lo que hace innecesario hallar el límite de las sumas indicado en la
definición de integral de Riemann,
• por otro lado, muestra la conexión existente entre el Cálculo Diferencial y el Cálculo
b
Integral ya que ∫ a
F ′(x ) dx = F (b ) − F (a ) .

Ejemplo 9.2 Ahora se pueden realizar con mayor comodidad los cálculos efectuados en el
ejemplo anterior.
2
2  x3 
 = 8 −0 = 8 .
∫ x dx = 
2

0
 3 0 3 3
1
1  x4 
 = 1 − 1 = 0.
∫ x dx = 
3
 4 
−1
  −1 4 4

9.5 Cambio de variable


Las siguientes proposiciones vienen a decir que si se obtiene la primitiva por cambio de
variable también cambian los límites de integración.

Proposición (Primer tipo de cambio de variable) Sea g una función con derivada
continua en [a, b ] y f una función continua definida al menos en Im(g). Entonces, si se
efectúa el cambio de variable g (x ) = t , se tiene que
b g (b )
∫ f (g (x )) dx = ∫ ( ) f (t ) dt .
a g a

π2
Ejemplo 9.3 Para calcular ∫ 0
sen 2 x ⋅ cos x dx se hace el cambio cos x = t , por lo que
0
π2 0 1 0 1  1

2
sen x ⋅ cos x dx = ∫ − t 1 − t dt = 2
2 ∫
2
3
(
− 2t 1 − t dt =  1 − t 2 )
3
 =
3
.
0 1 1  1

9-6
Integral definida Tema 9

Proposición (Segundo tipo de cambio de variable) Sea f una función continua en


[a, b] y g : [c , d ] → [a, b] una función continua y biyectiva en [c , d ] , derivable en (c , d )
tal que g ′(x ) ≠ 0 para todo x ∈ (c , d ) . Entonces, si se realiza el cambio de variable
x = g (t ) , se tiene que
b g −1 (b )
∫ a
f (x ) dx = ∫
g −1 (a )
f (g (t )) dt

1
Ejemplo 9.4 Para calcular ∫0
1 − x 2 dx se hace el cambio x = sen t , por lo que
π2
1 π2 π2 1 + cos (2t )  t sen (2t )  π
∫0
1 − x 2 dx = ∫
0
cos 2 t dt = ∫
0 2
dt =  +
2 4 

0
=
4
.

9.6 Aplicaciones al cálculo de áreas

Si f es una función integrable en [a , b ] , de la definición se deduce:


• Si f no es negativa en todo el intervalo, el área de la región limitada por la gráfica de f,
b
el eje OX y las rectas x = a , x = b , viene dada por A = ∫ a
f (x ) dx .

• Si f no es positiva en todo el intervalo, el área de la región limitada por la gráfica de f, el


b
eje OX y las rectas x = a , x = b , viene dada por A = − ∫
a
f (x ) dx .

Por lo tanto, en el cálculo de áreas, lo primero a estudiar es el signo de la función en el


intervalo de integración.
Ejemplos

Ejemplo 9.5 Sea la función f (x ) = x 3 definida en el intervalo [0,1] . Para calcular el área

encerrada por f , el eje de abcisas y la recta x = 1 , basta ver que la función es positiva en todo
el intervalo y, como consecuencia, su área es
1
 x4
1  1

A = x dx =  
3
 4 
= .
0
  4
0

Ejemplo 9.6 Sea la función f (x ) = x 2 − 3 definida en todo R. 2

Para calcular el área de la región limitada por la gráfica de f y el 1

eje de abcisas, primeramente vemos cuál es esta región. Para


ello, necesitamos hallar los puntos de intersección de f con OX. Es -2 -1 1 2

fácil ver que f corta al eje en x = 3 y x = − 3 y que la región -1

pedida es la definida en el intervalo [− ]


3 , 3 . Como f es -2

negativa, su área es
-3

9-7
Integral definida Tema 9

3 3
3  x3   x3  3 3 
∫ (x )
− 3 dx = − − 3x  = −2  − 3x  = −2  −3 3 = 4 3 ,
2
A=−
− 3  3   3   3 
  − 3   0  

donde hemos utilizado que la función es simétrica respecto del eje OY.

π
Ejemplo 9.7 Para hallar el área encerrada por f (x ) = sen x , el eje de abcisas y la recta x =
4
 π
, en el intervalo − π, 4  , se estudia el signo de la función en dicho intervalo en cuestión. Es
 
 π
fácil ver que es negativa en [− π,0] y positiva en 0,  . Por lo tanto, el área es
 4
0 π4 2 6− 2
∫ ∫
0 π4
A=− sen x dx + sen x dx = (cos x ) −π
+ (− cos x ) 0 = 1+ 1− +1= .
−π 0 2 2

1
0.5

-5 -4 -3 -2 -1 1
-0.5
-1

9.6.1 Área de la región limitada por las gráficas de dos funciones

Sean f y g dos funciones integrables en [a , b ] , con f (x ) ≤ g (x ) para todo x del intervalo.


Entonces, el área de la región limitada por las gráficas de f y g es
b
A= ∫a
(g (x ) − f (x )) dx

Ejemplos

Ejemplo 9.8 Para hallar el área limitada por las gráficas de las
funciones f (x ) = x 2 − 5 y g (x ) = 3 − x 2 , primeramente hallamos
2
los puntos de intersección. Es fácil ver que éstos son x = −2 y
x = 2 . Por lo que la región limitada por las curvas es la
correspondiente al intervalo [−2,2] , donde se cumple además
que f (x ) ≤ g (x ) . En consecuencia, el área pedida es -2 -1 1 2
2 2 2 2
A= ∫ −2
(g (x ) − f (x )) dx = ∫
−2
(3 − x 2
− x 2 + 5 dx =) ∫ (8 − 2x )dx = 2∫ (8 − 2x )dx =
−2
2
0
2

-2
2
2  3   16  64
=2 ∫ (8 − 2 x ) dx =2
2  8x − 2x



= 2 16 − = ,
0
 3  0
 3  3
-4
en donde hemos utilizado que el integrando es una función
simétrica respecto del eje OY.

Ejemplo 9.9 Para hallar el área limitada por las gráficas de las funciones f (x ) = x 2 − 1 y
g (x ) = x 3 − x 2 − x + 1 , primeramente hallamos los puntos de intersección. Es fácil ver que

9-8
Integral definida Tema 9

éstos son x = −1 , x = 1 y x = 2 . Por lo tanto, la región a estudiar está compuesta por dos
zonas, una que se corresponde con el intervalo [−1,1] y otra con el
[1,2] . En el primer intervalo, f (x ) ≤ g (x ) , mientras que en el 4

segundo es f (x ) ≥ g (x ) ; así pues, el área pedida es


1 2
A= ∫ −1
(g (x ) − f (x )) dx + ∫ (f (x ) − g (x )) dx =
1 2
1 1
= ∫ −1
(g (x ) − f (x )) dx + ∫ (g (x ) − f (x )) dx =
2
1 1
∫ (x ) ∫ (x )
3
= − 2 x 2 − x + 2 dx + 3
− 2 x 2 − x + 2 dx =
−1 2 -2 -1 1 2
1 1
x 2x x4  3 2 x 4
2x x 3 2
= − − + 2x  + − − + 2x  =
 4 3 2   4 3 2 
  −1   2
-2
1 2 1 1 2 1  1 2 1  16 
= − − + 2 −  + − − 2 + − − + 2 −  4 − − 2 + 4 =
4 3 2 4 3 2  4 3 2  3 
1 16 17
= −2 − + = .
2 3 6 -4

Nota Si y = f (x ) es continua y biyectiva de [a , b ] en [c , d ] , el área


limitada por el eje OY, la gráfica de f y las rectas y = c , y = d , viene dada
por
d d
A= ∫ c
f −1 (y ) dy .

A efectos prácticos, el problema se reduce a hallar la función


inversa, esto es, despejar x y dejarla en función de y (lo cuál no siempre
es posible). f
c

Ejemplos
a b

Ejemplo 9.10 Para hallar el área limitada por la gráfica de f (x ) = x + 1 , el eje OY y las rectas
y = 2 , y = 4 , no existe ningún problema pues la gráfica de f es una recta y, por tanto,
continua y biyectiva en el intervalo que deseemos. Su inversa es f −1 (y ) = y − 1 , luego el área
4
 y2 4
pedida es A = (y − 1) dy = 
2  2


− y


= 8 − 4 − (2 − 2) = 4 .
2

Ejemplo 9.11 Para hallar el área limitada por la gráfica de f (x ) = sen x , el eje OY y las rectas
 π
y = 0 , y = 1 , no existe ningún problema pues la gráfica de f es continua y biyectiva de 0, 
 2
1
en [0,1] . Su inversa es f −1 (y ) = arcsen y , luego el área pedida es A= ∫ 0
arcsen y dy . Para

ello, calculamos una primitiva del integrando. En efecto,


 dx 
 u = arcsen x ; du = 
∫ arcsen x dx = 
 dv = dx ;
1− x 2  =

 v =x 

9-9
Integral definida Tema 9

x dx 1
= x arcsen x − ∫ 2
= x arcsen x +
2 ∫ (
− 2x 1 − x 2 )
−1 2
dx = x arcsen x + 1 − x 2 + C .
1− x
1 1
π
Por lo tanto, A = ∫ arcsen y dy =  x arcsen x − 1 − x 2  = arcsen 1 = .
0   0 2

9-10
Integral definida Tema 9

AMPLIACIÓN 1: Volúmenes de cuerpos de revolución

9.7.1 Revolución alrededor del eje de abcisas

Sea f una función integrable en [a , b ] . El volumen del cuerpo de revolución engendrado


al girar, alrededor del eje OX, la región del plano limitado por la gráfica de y = f (x ) , el eje OX y
las rectas x = a , x = b , es
b
VX = π ∫ a
(f (x ))2 dx .

Nota A diferencia del área, en la que se sumaban infinitos


rectángulos, aquí se suman infinitos cilindros de radio f (x ) y
altura dx . Como el volumen de un cilindro es igual al área
de la base por la altura del mismo, cada cilindro anterior
tendrá un volumen igual a π(f (x ))2 dx , de donde se sigue la
fórmula escrita anteriormente.

Ejemplos

Ejemplo 9.12 Para hallar el volumen que engendra al girar alrededor del eje de abcisas la
gráfica de y = x 2 , entre x = 0 y x = 1 , bastará aplicar
1
1  x5  π

= π x dx = π  
4
VX = .
0  5  5
  0

Ejemplo 9.13 Para hallar el volumen que engendra al girar


alrededor del eje de abcisas la región limitada por y = tg x ,
π π
y = 0, x = , x = , bastará aplicar
6 3
π3

π3
VX = π tg 2 (x ) dx = π (− x + tg x ) π6
=
π6

 π π
= π− + 3 + −
3   π 2 3 π 4 3 −π
= π− + = ( )
 3 6 3   6 3  6
   

9.7.1.1 Volúmenes de cuerpos de revolución engendrados al girar la


región limitada por dos curvas

Sean f y g dos funciones integrables en [a , b ] , con f (x ) ≤ g (x ) para todo x del intervalo.


Entonces, el volumen del cuerpo engendrado al girar alrededor del eje OX la región limitada por
las gráficas de f y g, entre a y b, es

∫ (g (x ) )
b
2
VX = π − f (x )2 dx
a
Nota Observa que el integrando es la diferencia de dos cuadrados y no el cuadrado de una
diferencia.

9-11
Integral definida Tema 9

Ejemplo 9.14 Para hallar el volumen que engendra al girar alrededor del eje OX el recinto
x2 x
limitado por g (x ) = 2 x − y f (x ) = , calculamos
2 2
primeramente los puntos de intersección. Es fácil ver que éstos
son x = 0 y x = 3 , y que en ese intervalo se tiene que
f (x ) ≤ g (x ) . Luego el volumen es
3 4 2 

∫ (g (x ) )
3
 4 x 2 + x − 2 x 3 − x  dx =
− f (x ) dx = π ∫
2 2
VX = π
0 0 4 4 

3
3 15 x 2 x4   x3 x5 x4 
=π ∫ 
0 4
+
4
− 2 x 3  dx = π  5


+ −
 4 20 2 



=
0

 135 243 81  27π


=π + − = .
 4 20 2 5

9.7.2 Revolución alrededor del eje de ordenadas

Sea f una función integrable en [a , b ] . El volumen del cuerpo de revolución engendrado


al girar, alrededor del eje OY, la región del plano limitada por la gráfica de y = f (x ) , el eje OX y
las rectas x = a , x = b , es
b
VY = 2π ∫ a
x f (x ) dx .

Nota A diferencia del volumen engendrado al girar alrededor del eje OX, aquí se suman las
áreas de infinitos tubos (caras laterales de cilindros) de radio de la base igual a x y altura f (x ) .
Como el área lateral de un tubo es el producto de la longitud de la circunferencia de la base
por la altura del mismo, cada tubo tendrá un área igual a 2π x f (x ) , de donde se sigue la
fórmula escrita anteriormente.

Ejemplo 9.15 El volumen del sólido engendrado al girar en torno al eje OY la región del plano
π π2
limitada por la gráfica de y = sen x , entre x = 0 y x =
2
, es VY = 2π
0
x sen x dx . Como ∫
una primitiva del integrando es F (x ) = − x cos x + sen x (se hace por partes), resulta que
π2

π2
VY = 2π x sen x dx = 2π (− x cos x + sen x ) 0
= 2π .
0

Nota Si lo que se pretende es hallar el volumen del sólido engendrado al girar, en torno al eje
OY , la región del plano limitada por la gráfica de f (x ) , el eje OY y las rectas y = c e y = d ,
siendo c = f (a ) , d = f (b ) y f biyectiva e integrable en [a , b ] , puede aplicarse una fórmula
análoga a la vista para el caso de OX, quedando
d 2

∫ (f (y ))
−1
VY = π dy .
c

Ejemplo 9.16 El volumen del sólido engendrado al girar, en torno al eje OY , la región del plano
limitada por la gráfica de f (x ) = x 2 , el eje OY y las rectas y = 1 e y = 4 , es
4
 x2 
∫ ( )
4 2 15 π
VY = π x dx = π   = .
1  2  2
  1

9-12
Integral definida Tema 9

Observa en el dibujo que el volumen pedido es la diferencia entre dos cilindros, menos
el volumen engendrado al girar, en torno al eje OY , la región del plano limitada por la gráfica
de f (x ) = x 2 , el eje OX y las rectas x = 1 e x = 2 . El cilindro grande tiene de radio de la base
2 y altura 4; mientras que el cilindro pequeño tiene de radio de la base 2 y altura 1. Para el
tercer volumen podemos aplicar la fórmula del ejemplo 9.20. En efecto,
2
2  x4   1  15 π
∫ x x dx = 15 π − 2π  
2
VY = 16 π − π − 2π
 4 
= 15 π − 2 π  4 −  = .
1
  1
 4 2

9-13
Integral definida Tema 9

AMPLIACIÓN 2: Lo que pudo hacer Arquímedes


2
Cálculo del el área limitada por la parábola y = x , la recta x = 1 y el semieje positivo
de las equis: Se divide el intervalo [0,1] en n partes iguales y, tomando como base el segmento
de cada enésima parte, se construye un rectángulo de altura igual a la imagen del punto de la
base con menor imagen. Como se aprecia en el dibujo, la suma de las áreas de todos estos
rectángulos es inferior al área pedida y, conforme n aumenta, esta suma se aproxima más al
área pedida.

1 1

0.8 0.8

0.6 0.6

0.4 0.4

0.2 0.2

0.2 0.4 0.6 0.8 1 0.2 0.4 0.6 0.8 1

Si, a continuación, los rectángulos se construyen con altura igual a la imagen del punto
de la base con mayor imagen, la suma de las áreas de todos estos rectángulos es superior al
área pedida y, conforme n aumenta, esta suma se aproxima más al área pedida.

1 1

0.8 0.8

0.6 0.6

0.4 0.4

0.2 0.2

0.2 0.4 0.6 0.8 1 0.2 0.4 0.6 0.8 1

1
Como la base de cada rectángulo mide , resulta que:
n
• la suma de las áreas de todos los rectángulos inferiores es

2 2 2
1 1  1 2  1  n − 1 12 + 2 2 + K + (n − 1)2
I = 0 +   +   +K+   = ,
n n  n n  n n  n3
2
i 
ya que la altura de cada rectángulo es   , con i = 0,1,K,n − 1 , respectivamente.
n
• la suma de las áreas de todos los rectángulos superiores es

2 2 2 2
1 1  1 2  1  n − 1 1 n  12 + 22 + K + (n − 1)2 + n 2
S =   +   +K+   +   =
n n  n n  n n  n n  n3

9-14
Integral definida Tema 9

2
i 
ya que la altura de cada rectángulo es   , con i = 1,1,K,n , respectivamente.
n
• y, además, la diferencia entre S e I se puede hacer tan pequeña como se quiera, pues
1
S − I = , que es menor cuanto mayor es n .
n
Pero Arquímedes, en su tratado Sobre Espirales, había demostrado que

n(n + 1)(2n + 1)
12 + 2 2 + K + n 2 = ,
6

por lo que sólo tenía que hallar el número A, que se corresponde con el área pedida, tal que

I=
(n − 1) n (2n − 1) < A < n (n + 1)(2n + 1) = S .
6n 3 6n 3

Y es aquí donde el método de Arquímedes (método de compensación) difiere del


actual. Se trata de hallar un número que sea el supremo de la primera sucesión y, a la vez, el
ínfimo de la segunda.
1
Evidentemente, este número es A = , pues
3

lim
(n − 1) n (2n − 1) = 1
= lim
n (n + 1)(2n + 1)
.
3
n →∞ 6n 3 n →∞ 6n 3

9-15
Integral definida Tema 9

AMPLIACIÓN 3:
Una demostración y algunos cálculos de integrales mediante el uso de la
definición

Demostración de una Proposición

Sean f una función real integrable en [a, b ] , m = inf {f (x ) : x ∈[a, b ]} y M = sup {f (x ) : x ∈[a, b ]} ,
b

entonces: m ⋅ (b − a ) ≤ f ≤ M ⋅ (b − a ) .
a

Demostración.- Por ser m el ínfimo, se tiene


b  n   n   n 
∫a n →∞
∑  n →∞ ∑
f (x )dx = lim  f (c i )(x i − x i −1 ) ≥ lim  m ⋅ (x i − x i −1 ) = lim  m ⋅
 n →∞ ∑ (x i − x i −1 ) =

 i =1   i =1   i =1 

 n 
n →∞

= m ⋅ lim  (x i − x i −1 ) = m ⋅ (b − a ) , con lo que está probada una desigualdad.

 i =1 

Análogamente, como M es el supremo:

b  n   n   n 
∫a
f (x )dx = lim 
n →∞
∑ f (c i )(x i − x i −1 ) ≤ lim 
 n →∞ ∑ M ⋅ (x i − x i −1 ) = lim  M ⋅
 n →∞ ∑ (x i − x i −1 ) =

 i =1   i =1   i =1 

 n 
n →∞

= M ⋅ lim  (x i − x i −1 ) = M ⋅ (b − a ) , con lo que está probada la proposición.

 i =1 

n
Aclaración ∑ (x
i =1
i − x i −1 ) = (x1 − x 0 ) + (x 2 − x1 ) + (x 3 − x 2 ) + K + (x n −1 − x n −2 ) + (x n − x n −1 ) =

= − x 0 + x n = −a + b = b − a .

Cálculo de integrales a partir de la definición. Ejemplos:

Ejemplo 9.17 La función f (x ) = x 2 es integrable en [0,2] , porque es continua. Para calcular la


2
∫0
2
x dx , dividimos el intervalo [0,2] en n partes iguales, por lo que cada intervalo tiene
2 2i
longitud , y tomamos como c i = , esto es, el punto del intervalo donde la función es mayor
n n
(obsérvese que la función es creciente en el intervalo). Así pues, por definición se tiene que
2  n  2i  2 2   n 8i 2  2 2 2
 = 8 lim 1 + 2 + 3 + K + n =
x 2 dx = lim   
∫ n →∞

 = lim 
n  n →∞ i =1 n 3  ∑ n3
 i =1  
0 n n →∞

n (n + 1)(2n + 1) 8
= 8 lim = .
n →∞ 6n 3 3

Ejemplo 9.18 La función f (x ) = x es integrable en [− 1,1] , porque es continua. Para calcular la


1
∫−1
x dx , dividimos el intervalo [− 1,1] en 2n partes iguales, por lo que cada intervalo tiene

1 i
longitud , y tomamos como c i = −1 + . Así pues, por definición se tiene que
n n

9-16
Integral definida Tema 9

1  2n i 1   n  i 1
n
 i  1 1 1
∫ −1
x dx = lim 
n →∞  ∑− 1+
  ∑
= lim  1 −  +
n n  n →∞ i =1  n  n i =n +1 ∑
 − 1+   = + = 1
n  n  2 2
 i =1
ya que
 n  i  1
n
1
n
i 1+ 2 + K + n n (n + 1) 1 1
 ∑
lim  1 −   = lim
 ∑ − lim ∑
2
= 1 − lim 2
= 1 − lim 2
= 1− =
 i =1  n  n  n → ∞ i = 1 n n → ∞ i =1 n 2 2
n →∞ n →∞ n n → ∞ 2n

 2n  i  1
2n
1
2n
i (n + 1) + (n + 2) + K + (n + n ) =
lim 
n →∞
∑  − 1 +   = − lim
 ∑+ lim ∑ 2
= −1 + lim
n2
 i =n +1 n  
n n n
n → ∞ n → ∞ n → ∞
i = n +1 i = n +1

n (n + 1 + n + n ) 3 1
= −1 + lim = −1 + = .
n →∞ 2n 2 2 2

Ejemplo 9.19 La función f (x ) = x 3 es integrable en [− 1,1] , porque es continua. Para calcular la


1
∫x [− 1,1]
3
dx , dividimos el intervalo en 2n partes iguales, por lo que cada intervalo tiene
−1
1 i
longitud , y tomamos como c i = −1 + . Así pues, por definición se tiene que
n n
1  2n

3
i  1   2n 3
i
2n 2
i
2n
i
2n
1 
∫−1
x 3 dx = lim   − 1 + 
n →∞  n∑ n
= lim 
 n →∞ n 4
−∑3
n 3 ∑
+ 3
n
∑2

n ∑

= 4−8+6−2 = 0 ya
 i =1   i =1 i =1 i =1 i =1 
 2n
i 3  13 + 2 3 + K + (2n )3 (2n )2 (2n + 1)2 = 4 ,
que lim 
n →∞
∑ = lim
n 4  n →∞ n4
lim
n →∞ 4n 4
 i =1

 2n
i 
2
12 + 2 2 + K + (2n )2 2n (2n + 1)(4n + 1) 8
lim 
n →∞
∑ 3 
= lim
n  n →∞ n 3
lim
n →∞ 6n 3
= ,
3
 i =1

 2n
i  1 + 2 + K + (2n ) (1 + 2n )2n = 2 , y lim  2n
1 2n
lim 
n →∞
∑ 2 
= lim
n  n →∞ n 2
lim
n →∞ 2n 2 n →∞
∑ n  = lim n →∞ n
= 2.
 i =1  i =1 

2
 n (n + 1) 
Aclaración En este ejemplo se ha aplicado la fórmula 13 + 2 3 + K + n 3 =   .
 2 

Ejemplo 9.20 (Integral de una función constante) La función f (x ) = k , ∀ x ∈R , es integrable


b
en cualquier intervalo [a , b ] , porque es continua. Para calcular la ∫ k dx , dividimos el intervalo
a
b−a
[a, b] en n partes iguales, por lo que cada intervalo tiene longitud , y no hay que
n
preocuparse de los c i , porque la imagen de todos ellos es siempre k. Así pues, por definición
se tiene que
b  n b−a  n 
 = lim  k (b − a ) 1  = k (b − a ) .
a
k dx = lim  k
n →∞ ∫ ∑
n  n →∞ n  ∑
 i =1 i =1

9-17
Integral definida Tema 9

Ejercicios resueltos

R.1. Calcula las siguientes integrales:

π π
dx 5 xdx 1
∫0 ∫ ∫1 ∫0 x
2
a) cos x dx b) 3
π c) d) ⋅ e 2 x dx
6 senx ⋅ cos x x −1
Soluciones:
π
∫ 0 cos x dx = [senx ]0 = senπ − sen0 = 0 .
π
a)

dx
π π
sen 2 x + cos 2 x dx π
senxdx( π
cos xdx ) [
π
]
b) ∫ 3
π
senx ⋅ cos x
6
= ∫ π3
6 senx ⋅ cos x
= ∫ π3
6 cos x
+ ∫ π3
6 senx
= − ln cos x + ln senx 3
π
6
=
π
1
[ ]
= ln tgx 3π = ln 3 − ln = 2 ln 3 = ln 3 .
6 3

5 xdx x = 5 → t = 2
c) ∫1 Hacemos el cambio x − 1 = t ⇒ x − 1 = t 2 con lo que dx = 2tdt y 
x −1 x = 1 → t = 0
5 2 (t 2
) 2 t3 
2

∫ (t )
xdx + 1 ⋅ 2tdt 28
∫ ∫
2
así que = =2 + 1 ⋅ dt = 2 + t  = .
1 x −1 0 t 0
 3  0 3

d) Se hace la integral indefinida aplicando la fórmula de integración por partes dos veces y se
1
 e 2x  e2 − 1
tiene:
1
∫0 x
2
⋅e 2x
dx =  (
2x 2 − 2 x + 1  = . )
 4  0 4

R.2. Aplicando el Teorema Fundamental del Cálculo Integral, calcula F ′(x ) en los
x x2 2 dt
siguientes casos: a) F (x ) = ∫ sen 2 t ⋅ dt b) F (x ) = ∫ e sent ⋅ dt c) F (x ) = ∫
3 1 x
1+ t 3

Solución: El Teorema Fundamental del Cálculo Integral dice que siendo f (t ) una función
x
continua en [a , x ] la función F (x ) = ∫ f (t ) ⋅ dt es derivable y además F ′(x ) = f (x ) .
a

x
a) Dada F (x ) = ∫ sen 2 t ⋅ dt , la aplicación del teorema nos dice directamente que F ′(x ) = sen 2 x
3

x2
b) La función F (x ) = ∫ e sent ⋅ dt se deriva considerando que al ser el límite superior x 2 y no x,
1

hay que aplicar la regla de la cadena. Así F ′(x ) = e senx ⋅ 2 x .

2 dt x dt
c) La variable está en el límite inferior de integración, como F (x ) = ∫ 3
= −∫ .
x
1+ t 2
1+ t 3
1
Aplicando el teorema tenemos que F ′(x ) = −
1+ x 3

ln x e3
R.3. Calcula la siguiente integral. S1 =
e x
dx . ∫
Solución: La integral es inmediata. Si se quiere se puede hacer el cambio ln x = t .
e3
e3 ln x  (ln x )2  32 1
S1 = ∫
e x
dx = 
 2  e
 = − =4.
2 2

9-16
Integral definida Tema 9

Áreas de recintos planos

R.4. Calcula el área del recinto plano formado por la gráfica de f (x ) = x 3 + x 2 y el eje OX.
Dibuja el recinto.

Solución: Los puntos de corte de la función con OX son P (0,0 ) y Q(− 1,0 ) , f ′(x ) = 3 x 2 + 2 x de
  2
creciente si x ∈  − ∞,−  U (0, ∞ )  2 4 
aquí se tiene que f (x )  3
con M  − ,  y m (0,0 )
 2 
decrecient e si x ∈  − ,0   3 27 
  3 

 1 2 
f ′′(x ) = 6 x + 2 con lo que el punto de inflexión es P .I . − , 
 3 27 
siendo cóncava a la derecha del P.I. y convexa a la izquierda. Se
dibuja el recinto y se observa que está por encima del eje OX y
0
 x 4 x3   1 1 1
por tanto S = ∫ −1
0
(x 3
)
+ x 2 dx =  +  = 0− −  =
 4 3  −1  4 3  12
unidades de área.

R.5. Calcula el área del recinto plano formado por la gráfica de f (x ) = x 3 − x , las rectas
x = 0 , x = 2 y el eje OX. Sin dibujar el recinto.

Solución: Precisamos encontrar los puntos de corte y estudiar las regiones. Los puntos de
corte son (-1,0); (0,0) y (1,0). La función toma valores positivos en x ∈ (− 1,0 ) U (1,+∞ ) y
negativos en x ∈ (0,1) y en esta zona está por debajo del eje OX mientras que de 1 a 2 está por
encima. Dividimos en dos integrales.
1 2
 x4 x2  x4 x2  5
1
0
(
S = − ∫ x − x dx + ∫
3
) 1
2
( 3
)
x − x dx = − +  + −  = u.a.
 4 2 
0  4 2 
1
2

cos x si x ≤ 0

R.6. Dada la función f (x ) = ax 2 + b si 0 < x < 2 . Calcula a y b para que sea continua.
 x + 3 si x ≥ 2

2
Calcula la ∫ − f (x ) ⋅ dx .¿Coincide con el área del recinto formado por la función y las
π
2

verticales en los límites de integración?.

Solución: Para que lim− f (x ) = lim+ f (x ) = f (0 ) → b = 1 y para que


x →0 x →0
cos x si x ≤ 0

lim− f (x ) = lim+ f (x ) = f (2) → a = 1 y la función es f (x ) =  x 2 + 1 si 0 < x < 2 y la integral
x →2 x →2
 x + 3 si x ≥ 2

2
π
2
0
∫ − f (x ) ⋅ dx = ∫ − π
2
2
(
cos x ⋅ dx + ∫ x 2 + 1 ⋅ dx =
0
)
2
 x3  17
= [senx ]0− π +  + x = . Si coincide con el área del recinto pues f (x ) es positiva en todo el
2
 3  0 3
intervalo de integración.

9-17
Integral definida Tema 9

R.7. Calcula el área encerrada por las gráficas de f (x ) = x 3 + x 2 y g (x ) = x 3 + 1 sin dibujar


el recinto.
y = x 3 + x 2 
Solución: Calculamos los puntos de corte. Resolvemos  ⇒ P (− 1,0 ) y Q(1,2)
y = x 3 + 1 
Entre los valores –1 y 1 debemos saber qué función está por encima, probamos con un valor
intermedio cualquiera, f (0 ) = 0, g (0 ) = 1 así que g está por encima de f en el recinto y por tanto.
1
 3
4
S=∫
1
−1
(g − f ) ⋅ dx = ∫ −11 (1 − x 2 )⋅ dx =  x − x  = u .a.
 3  3
−1

R.8. Calcula el área del recinto plano encerrado por las gráficas de f (x ) = 4 − x 2 y
g (x ) = 8 − 2 x 2 . Dibuja el recinto.

Solución: Las gráficas son dos parábolas con las ramas hacia abajo
de vértices respectivos (0,4) y (0,8). Calculamos sus puntos de corte
y = 4 − x 2 
y dibujamos el recinto. Resolvemos  ⇒ P (− 2,0 ) y Q(2,0 ) .
y = 8 − 2 x 2 
El recinto es simétrico por lo tanto.

2
 x3  32
S = 2∫
2
0
((8 − 2x ) − (4 − x ))⋅ dx = 2∫ (4 − x )
2 2
0
2 2
⋅ dx = 24 x −
3
 =
3
u.a.
  0

R.9. Dada la función y = 3 x − 1 . Se pide:


a) Calcula el área del recinto formado por la gráfica de la función la recta x = 1 y el
eje OX.
b) Calcula el área del recinto formado por la gráfica de la función la recta y = 2 el
eje OX y el eje OY.
c) Dibuja ambos recintos.

Solución: La función es media parábola horizontal y su gráfica es elemental.


1
a) S = ∫ 1 3 x − 1 ⋅ dx =  (3 x − 1)3  =
1 2 4 2
u.a.
3 9   1
3
9
2
2   3 
2
b) Integramos respecto al eje OY S = ∫  y + 1  ⋅ dy = 1  y + y  = 5 2 u .a.
0  3  3  3  0 9
 
Obsérvese que la suma de las dos áreas calculadas es 2 como corresponde a un rectángulo
de 1 unidad de base y 2 de altura. Los recintos respectivos son:

9-18
Integral definida Tema 9

R.10. Calcula el área del recinto formado por las gráficas de


f (x ) = x 2 y g (x ) = 2 − x 2 y h(x ) = 4 . Dibuja el recinto.

Solución: Puntos de corte :


y = x2 
f (x ) = x 2 con g (x ) = 2 − x 2 → (− 1,1) y (1,1) .
2
y = 2 − x 

y=4 
g (x ) = 2 − x 2 con h(x ) = 4 2 → x 2 = −2 → no hay .
y = 2−x 

y=4 
f (x ) = x 2 con h(x ) = 4  → (− 2,4 ) y (2,4 ) .
y = x2 
Las gráficas son sencillas dos parábolas y una recta horizontal. El recinto es simétrico.
1 2 1 2
S=2 ∫ 0
(h(x ) − g (x )) ⋅ dx + 2∫ (h(x ) − f (x )) ⋅ dx = 2∫
1 0
(2 + x )⋅ dx + 2∫ (4 − x )⋅ dx = 8u.a.
2
1
2

R.11. Dibuja el recinto formado por la gráfica de f (x ) = − x 2 + 4 x − 3 , su tangente en el


punto P (0,−3 ) y su eje de simetría. Calcula su área.
Solución: Derivamos y f ′(x ) = −2 x + 4 → f ′(0 ) = 4 la
ecuación de la tangente es y + 3 = 4 x → y = 4 x − 3 . El eje
de simetría es la recta vertical que pasa por el vértice y la
abcisa del vértice cumple f ′(x ) = 0 → x = 2 , el vértice es
v (2,1) . Dibujamos el recinto.
2
 x3  8
S= ∫0
2
((4x − 3) − (− x 2
)) 2
+ 4 x − 3 ⋅ dx = ∫ x 2 ⋅ dx =   = u.a.
0
  0 3
3

R.12. Dibuja el recinto delimitado por las curvas y = − x 2 + 2 x + 3 e y = x + 1 . Calcula su


área.
 x + 1 si x ≥ −1
Solución: y = x + 1 ⇒ y =  Calculamos
− x − 1 si x ≤ −1
y = − x 2 + 2 x − 3
el punto de corte  con x ≥ −1
y = x +1 

y = − x 2 + 2 x − 3
Al resolver  con x ≤ −1 no
y = −x − 1 
encontramos nuevos puntos de corte. Dibujamos el recinto.
2
∫ (− x )
2
El área S = + 2 x + 3 − x − 1 dx =
−1
2
 x3 x2  9
= − + + 2 x  = u.a.
 3 2  −1 2

R.13. Dada la función y = x ⋅ e x y las rectas x = 1 e y = 0 .


a) Dibuja la gráfica de la función para x ≥ 0 .
b) Señala el recinto plano comprendido entre las tres gráficas anteriores.
c) Calcula el área del recinto plano señalado.

9-19
Integral definida Tema 9

Solución: Analizamos y = x ⋅ e x , su derivada y ′ = (x + 1)e x nos


indica que la función es creciente en todo x ≥ 0 . La derivada
segunda y ′′ = (x + 2)e x nos dice que la función es cóncava en
todo x ≥ 0 . Los puntos de corte con x = 1 y con y = 0 son
respectivamente P (1, e ) y Q (0,0 ) , con esos datos el recinto es
obvio.
S=
1
∫0x ⋅e
x
[
dx = x ⋅ e x − e x ]
1
0 = 1 u.a.

R.14. Calcula el área del recinto plano limitado por las


rectas y = x , y = 2 x y la parábola y = x 2 .

Solución: Calculamos los puntos de corte, resolvemos


y=x 
 P1(0,0 ) ; P2 (1,1) Calculamos ahora el corte de
y = x2 
y = 2x 
 P1(0,0 ) ; P3 (2,4 ) El recinto es el de la derecha y el
y = x2 
área es

∫ 0 (2 x − x ) dx + ∫1 (2x − x ) dx , integrando
1 2 2
S=
1 2
 x2   x3  7
S =   + x 2 −  = u.a.
 2  0  3 
1
6

1 x
R.15. Dibuja el recinto limitado por y = x 2 , y = e y = . Calcula su área.
x 4
1 
y= 
Solución: Calculamos los puntos de corte de x  P (1,1)
y = x 2 
x 
y=  1 1 
Calculamos ahora el corte de 4  P (0,0 ) ; P  ,  ,
y = x 2   4 16 

1
 y=
x  1  1
P  2,  ; P  − 2,−  El recinto está
x
por último
y=   2  2
4
formado por una parábola, una recta y una hipérbola.

El área es:
1 2
1 x 2 1 x  x3 x2   x2  63
S = ∫ 1  x −  dx + ∫  −  dx S = 
2
−  + ln x −  = ln 2 − u.a.
4 4  4
1 x
 3 8  1  8 
1
384
4

R.16. Dibuja el recinto limitado por y = x 2 − 4 x , y = 2 x − 5 .


Calcula su área.

y = x 2 − 4 x 
Solución: Puntos de corte:  P1 (1,−3 ) ; P2 (5,5 )
y = 2 x − 5 
El recinto es el de la derecha y el área es:
S=∫
1
5
((2x − 5) − (x 2
)) 1
5
(
− 4 x dx = ∫ − x 2 + 6 x − 5 dx )

9-20
Integral definida Tema 9

5
 x3  32
S = − + 3x 2 − 5x  = u.a.
 3  1 3

R.17. Dibuja el recinto limitado por y 2 = 2 x , 2 x − y − 2 = 0 . Calcula su área.


y 2 = 2 x 
Solución: Resolviendo el sistema ⇒
2 x = y + 2
1 
P1  ,−1 ; P2 (2,2) y el recinto:
2 

1
S = 2∫ 2 2 x dx + ∫ 1
0
2

2
( 2x − (2x − 2)) dx
Integrando.
1
2
S =  (2 x )3  +  (2 x )3 − x 2 + 2 x  = u.a.
2 2 1 9
3 0 3  2 4
1

R.18. Halla la ecuación de la recta tangente a y = x 2 + 2 en


el punto de abcisa x = 1 . Calcula el área del recinto
limitado por y = x + 2 , la tangente anterior y el eje OY.

Solución: Derivando y ′ = 2 x luego m = 2 , y como f (1) = 3 la


tangente es y − 3 = 2(x − 1) o sea, y = 2 x + 1 representando
las tres funciones tenemos el recinto. Para calcular su área
observamos que el recinto es un triángulo de base 1 (en el eje
OY) y de altura 1 y por tanto su cálculo es elemental
1 1
S = 1⋅ 1 = u.a. y no tiene sentido integrar.
2 2

R.19. Calcula el área de la región del plano limitada por las gráficas de las funciones
y = − x 2 + 4 x − 4 e y = 2x − 7 .
y = − x 2 + 4 x − 4 
Solución: Resolviendo el sistema 
y = 2x − 7 
encontramos los puntos de corte P1 (− 1,−9 ) ; P2 (3,−1) . El
recinto es el de la derecha y el área:

∫ −1((− x ) ) ∫ −1(− x )
3 3
S= 2
+ 4 x − 4 − (2 x − 7 ) dx = 2
+ 2 x + 3 dx =
3
 x3  32
= − + x 2 + 3x  = u.a.
 3  −1 3

R.20. Halla el área del recinto limitado por las gráficas de las
funciones y = x 2 − 2 x e y = − x 2 + 4 x .

Solución: Resolviendo el sistema se encuentran los puntos de


corte (0,0) y (3,3) por otra parte la parábola y = x 2 − 2 x tiene
su vértice en v(1,-1) y las ramas hacia arriba y y = − x 2 + 4 x en
v’(2,4) y las ramas hacia abajo. El área:
S=∫
3
0
((− x 2
) ( ))
+ 4 x − x 2 − 2 x dx = ∫ 0 (− 2 x
3 2
)
+ 6 x dx =

9-21
Integral definida Tema 9

3
 2x 3 
= − + 3 x 2  = 9 u.a
 3  0

R.21. Calcula el área del recinto limitado por las curvas y = x 2 − 1 , y = 11 − x y el semieje
positivo de las equis. Dibuja el recinto.
y = x 2 − 1
Solución: Puntos de corte :  , son
y = 11 − x 
P1(−4,15 ) ; P2 (3,8 ) . El recinto está se debe
descomponer para calcular el área en dos recintos
independientes, la parábola y con la recta con OX.
3
(
S = ∫ x 2 − 1 dx + ∫
1
) 11
3
(11 − x ) dx =
3 11
 x3   x2  143
= − x  + 11x −  = u.a.
 3  1  2  3 3

R.22. Halla el área del recinto plano delimitado por las


curvas de ecuación y = x 2 − 2 e y = − x . Dibuja el recinto.
− x si x > 0
Solución: y = − x ⇒ y =  . Como las dos
 x si x < 0
gráficas son simétricas respecto al eje OY calculamos un
y = x 2 − 2
punto de corte  con x > 0 P (1,−1) y por simetría
y = −x 
el otro punto es Q(− 1,−1) . Dibujamos el recinto
1
1  x3 x2 
∫ (− x − x )
2 7
S=2 + 2 dx == 2 − − + 2 x  = u.a
0
 3 2  0 3

x2
R.23. Dada la parábola y = y la recta y = x .
4
a) Dibuja las gráficas de la parábola y de la recta.
b) Señala el recinto plano comprendido entre las
dos gráficas anteriores.
c) Calcula el área del recinto plano señalado.

Solución: La parábola tiene su vértice en (0,0) y las ramas


hacia arriba, así que basta con encontrar los puntos de
x 2 
corte, resolviendo y = 4  encontramos P1(0,0 ) ; P2 (4,4 )
y = x 
y el recinto es:
Y el área:
4
4 2  2 3
 x − x  dx =  x − x  = 8 u.a.
S= ∫ 0
 4   2 12  0 3

x 1
R.24. Representa el recinto plano limitado por las gráficas de y = 4 x , y = e y= .
4 x
Calcular su área.

9-22
Integral definida Tema 9

Solución: Buscamos los puntos de corte de cada pareja de funciones:


x  1 
y=  y=  1   1 
4  ⇒ P1 (0 ,0 ) ; x  ⇒ P2  ,2  ; P3  − ,−2  y
y = 4 x  y = 4 x  2   2 

x
y= 
4  ⇒ P  2, 1  ; P  − 2,− 1  . Las gráficas son dos
1 4 5
y=   2  2
x
rectas y una hipérbola, el recinto tiene dos partes iguales,
calculamos el área de una de ellas S1 y el área total
S = 2S1 . Se integra en dos partes.
1
 x 2 1 x
∫ ∫
2
S1 =  4 x −  dx +  −  dx =
 4 x 4
1
0 2
1
2
15 x 2  2  x2 
=  + ln x −  = 2 ln 2 u.a.
 8  0  8  1
2

por tanto S = 4 ln 2 .

R.25. Halla la ecuación de la recta tangente a la curva de


ecuación y = x 3 − 3 x en el punto de abcisa x = −1 .
Calcula el área del recinto limitado por la recta tangente
y la curva dada.

Solución: Derivando se encuentra que f ′(−1) = 0 y f (− 1) = 2


por tanto la tangente es la horizontal y = 2 . Resolvemos
y = x 3 − 3 x 
 de soluciones P1(−1,2 ) ; P2 (2,2) el recinto es.
y =2 
2
2  x 4 3x 2  27
∫ (2 − x )
3
S= + 3 x dx = 2 x − +  = u.a.
−1
 4 2  −1 4

R.26. Dadas las funciones y = − x 2 + 4 e y = x + 2 .


a) Dibuja ambas gráficas.
b) Señala el recinto plano comprendido entre las
gráficas anteriores.
c) Calcula el área del recinto plano señalado.

Solución: Las gráficas son elementales así que


calculamos los puntos de corte. Resolvemos
y = − x 2 + 4 
 con x > −2 ⇒ P (1,3 ) y también
y = x + 2 
y = − x 2 + 4
 con x < −2 ⇒ Q(− 2,0 ) . El recinto sólo
y = − x − 2 
está en la parte en que x > −2
1
1  x3 x2  9
∫ ( )
2
S= − x − x + 2 dx = − − + 2 x  = u.a..
−2  3 2  − 2 2

9-23
Integral definida Tema 9

R.27. Dada la curva y = x 2 − 4 x y la recta y = 3 x − 6 .


a) Dibuja la gráfica de ambas.
b) Señala el recinto plano comprendido entre ellas.
c) Calcula el área del recinto señalado.

Solución: Calculamos los puntos de corte. Resolvemos


y = x 2 − 4 x 
 ⇒ P (6,12 ), Q (1,−3 )
y = 3 x − 6 
6 6
∫ (3 x − 6 − x ) ∫ (− x )
2 2
S= + 4 x dx = + 7 x − 6 dx .. =
1 1
6
 x 3 7x 2  125
= − + − 6x  = u.a.
 3 2 1 6

Volúmenes de revolución

R.28. Calcula el volumen del sólido de revolución formado al girar el recinto encerrado
por la gráfica de f (x ) = 4 − x y los ejes de
coordenadas alrededor de OX.

Solución: La curva corta a los ejes en (4,0) y (0,2) . El sólido


es el dibujado a la derecha y el volumen es:
4
4  x2 
V = π∫ (4 − x ) ⋅ dx = π4 x −  =π(16 − 8 ) = 8π u.v .
0
 2 
0

R.29. Calcula el volumen del sólido de revolución


1
formado al girar la gráfica de f (x ) = entre las
(x + 1)2
verticales x = 0 y x = 3 alrededor de OX.

Solución: Se puede comprobar que la función en todo el


intervalo es decreciente y cóncava. El sólido que no es
necesario dibujar es el de la derecha.
El volumen
3
3 dx  −1  63 π
V = π∫ = π 3
= u.v .
0
(x + 1)4  3(x + 1)  0 192

R.30. Calcula el volumen del sólido de revolución formado al girar el recinto encerrado
por las gráficas y 2 = 2 x , x 2 + y 2 = 8 e y = 0 alrededor de
OX.

Solución: Se trata de una parábola horizontal con el vértice en


(0,0) y de una circunferencia de centro el origen y de radio 8 ,
la resolución del sistema nos da como puntos de corte P (2,2) y
Q (2,−2) . Sólo el recinto del primer cuadrante ya genera todo el
sólido al girar alrededor de OX. Hay que dividir los cálculos en
dos intervalos [0,2] con la parábola y 2, 8 con la [ ]
circunferencia, como se aprecia en el dibujo.

9-24
Integral definida Tema 9

8
 3 
(2x ) ⋅ dx + π∫ 2 8 (8 − x 2 )⋅ dx =π[x 2 ]0 +π8 x − x
2 2
V = π∫  =
0
 3 
2

=
π
3
( )
32 2 − 28 unidades de volumen.

R.31. Calcula el volumen del sólido de revolución formado al girar el recinto encerrado
por las gráficas y = x 2 − 1 e y = 3 a) alrededor de OX b) alrededor de OY.

Solución: El dibujo de las gráficas es elemental.


a) la figura es simétrica y se trata de dos funciones y = 3
por encima e y = x 2 − 1 por debajo, por lo tanto:
2
( ) 2
(
V = 2π∫ f 2 − g 2 ⋅ dx = 2π∫ 8 + 2 x 2 − x 4 ⋅ dx =
0 0
)
2
 2x 3 x 5  448 π
= 2π8 x + −  = u.v .
 3 5 
0
15
(Se ha dibujado solamente la parte de la derecha del sólido).

b) hay que trabajar con la función recíproca x = y + 1 e


integrar entre –1 y 3.
3
3 3 y2 
V = π ∫ x ⋅ dy =π∫ (y + 1) ⋅ dy = π
2
+ y  = 8π
−1 −1
 2  −1
unidades de volumen.

x2 y2
R.32. Calcula el volumen del elipsoide de revolución formado al girar + =1
a2 b2
alrededor de OX.

Solución: Es una elipse de semieje mayor a y semieje


menor b el elipsoide es el dibujado (en el que se ha
resaltado la mitad de la derecha).
b2
y 2 = b2 − 2 x 2
a
Integramos entre 0 y a y multiplicamos por 2.
a
 2 b2 2   3 
 ⋅ dx = 2πb 2  x − x  = 4π a ⋅ b 2
a
V = 2π ∫ b − x
0  a2  3a 2  0
   3

9-25
Integral definida Tema 9

Ejercicios propuestos

Cálculo de integrales definidas

P.1. Calcula:
1 dx 1 π 2 π 2
∫ ∫ (x − e ) ∫ ∫ senx ⋅ cos(2 x ) dx
x
a) b) cos x dx c) x sen 2 x dx d)
0 1+ x3 0 0 0

a + sen (πx ) , x ≤ −1

P.2. Determina a y b para que la función f (x ) =  ax + b , − 1 < x ≤ 0 sea continua y, después,
 x2 + 2 , x > 0

2
calcula ∫−2
f (x ) dx .

Cálculo de áreas

P.3. Halla el área limitada por la gráfica de la función f (x ) = x 3 − 6 x 2 + 8 x y el eje OX.

P.4. Calcula el área encerrada por las rectas x = 2 , x = 2 3 , la gráfica de la función


1
f (x ) = y el eje OX .
4 + x2

x2 + 3
P.5. Halla el área limitada por la gráfica de la función f (x ) = , el eje OX y las rectas
x 2 − 5x + 6
x =4 y x =6.

x2 y2
P.6. Halla el área limitada por la elipse de ecuación + = 1.
a2 b2

P.7. Calcula el área de la región del plano limitada por el eje de abcisas, la gráfica de la función
f (x ) = x ⋅ e x y las rectas x = 0 y x = 1 .

x2
P.8. Calcula el área de la figura plana limitada por las gráficas de f (x ) = − x +1 y
2
g (x ) = x + 1 .

1 x2
P.9. Dadas las funciones f (x ) = , g (x ) = , calcula el área de la región del plano
2 + x2 3
delimitada por ellas.

P.10. Calcula el área del recinto plano limitado por la gráfica de f (x ) = x 3 − x , y su recta
tangente en x = −1 .

P.11. Halla el área de la región limitada por las gráficas de las curvas f (x ) = 1 + x e x ,
ln x
g (x ) = y las rectas x = 1 , x = 2 .
x

P.12. Determina el área del recinto limitado por la parábola y 2 − 2 x = 0 y la recta que pasa por
los puntos (2,−2), 4,2 2 . ( )
P.13. Determina el área de la región acotada delimitada por la gráfica de la función
f (x ) = x 2 ln x , su tangente en el punto de abcisa e, y el eje OX.

9-26
Integral definida Tema 9

P.14. Dibuja el recinto limitado por las gráficas de las funciones f (x ) = 2 x y g (x ) = x 2 − 8 .


Calcula el área de dicho recinto.

P.15. Representa gráficamente y = 6 x − x 2 e y = 2 x . Determina el área limitada por ambas.

P.16. Dibuja el recinto limitado por las gráficas de las funciones f (x ) = x 2 − 2 x , g (x ) = − x 2 + 4 x


y h (x ) = 8 . Calcula el área de dicho recinto.

6
P.17. Calcula el área del recinto limitado por las gráficas de las funciones y = x − 1 , y = ,
x
x = 4 y el eje OX. Dibuja el recinto.

Cálculo de volúmenes de cuerpos de revolución

P.18. Halla el volumen del sólido encerrado por la gráfica de la función f (x ) = tg x , con dominio
 π
0, 4  , al girar alrededor del eje de abcisas.
 

P.19. Calcula el volumen del sólido generado por el triángulo de vértices A(0,0), B(1,1) y C(2,0),
al girar alrededor del eje de ordenadas.

P.20. Calcula el volumen limitado por el elipsoide de revolución generado por la elipse
2 x 2 + y 2 = 1 al girar alrededor del eje de ordenadas.

P.21. Calcula el volumen del cuerpo engendrado al girar alrededor del eje OX la superficie
comprendida entre las gráficas de las funciones f (x ) = x 2 − 5 x + 4 , g (x ) = x 2 + x − 20 y el eje
OY.

P.22. a) Calcula el volumen del sólido engendrado por la rotación alrededor del eje OX del
trapecio limitado por las rectas y = 3 x + 5 , x = 1, x = 4. b) ¿Cuál es el volumen si gira
alrededor del eje OY?

P.23. Calcula el volumen del sólido engendrado al girar alrededor del eje OX el recinto limitado
por la parábola y 2 = 2 px , la recta x = h y el eje OX.

P.24. Calcula el volumen del cuerpo engendrado al girar alrededor del eje OX, el recinto
limitado por las gráficas de las funciones f (x ) = 6 x − x 2 y g (x ) = x .

P.25. a) Halla el área del recinto del primer cuadrante limitado por las curvas y = sen x ,
y = cos x y el eje de ordenadas. b) Calcula el volumen engendrado al girar el anterior recinto
alrededor del eje de abcisas.

P.26. a) Halla el área limitada por la elipse x 2 + 9 y 2 = 9 . b) Halla el volumen engendrado por
esta elipse al girar alrededor del eje de abcisas.

Cálculo del volumen por secciones

P.27. Calcula el volumen de un cuerpo de la siguiente forma: su base es un círculo de radio 2 y


las secciones perpendiculares a un determinado diámetro son triángulos equiláteros.

9-27
Integral definida Tema 9

Soluciones

3 ln 2 + 3 π 2 − e (sen 1 + cos 1) −1
P.1. a) = 0´84 ; b) = −0´ 87 ;c) 0,44; d) . P.2. a = 1 ; b = 2 ;
9 2 3
2 55 2 2 4
∫−2
f (x )dx =
6 π ∫
+ = 9´8 . P.3 S = f (x )dx − f (x )dx = 4 − (− 4 ) = 8 u.a.
0 2 ∫
2 3 1 π
P.4. S =
2 ∫4+x 2
dx =
24
= 0´13 u.a.

6 x2 + 3
∫ dx =[12 ln(x − 3 ) − 7 ln(x − 2) + x ]4 ≈ ≈ 10'33 u.a.
6
P.5 S =
4 x 2 − 5x + 6
a
4b a 4b  a 2 x x a2 − x2 
∫  arcsen +  = π ⋅a⋅b
2 2
P.6. S = a − x dx =
a 0 a  2 a 2 
 0
1
P.7 S =
1

0
xe x dx = (x − 1)e x [ ]0
= 1 . P.8 S =
4

0
16
∫ [g (x ) − f (x )]dx = 3 u.a. .
1 x 1 2 2 2
P.9. S = 2 
0 2 + x2

− ∫ 
3 
dx = 2arctg
2
− ≈ 0'648 .
9
2   3 
 2 x + 2 −  x − x  dx = 27 .
P.10 S = ∫ −1

 3


 4
2
2 ln x   x (ln x )2 + x  = e 2 − (ln 2)2 + 1 ≈ 8'15 .
∫ dx = e (x − 1) −
x
P.11 S = 1 + xe − 
1  x   2  1 2

∫( ( 2 + 1)x + 2 )
2 4 14 + 10 2
P.12. S = 2 ∫0
2 x dx +
2
2x − 2 + 4 dx =
3
≈ 9'38 u.a. .
2e

∫ [x )]
e
P.13 S = ∫ 3
1
x 2 ln x ⋅ dx + 2e
2
(
ln x − 3ex − 2e 2 ⋅ dx ≈ 0'629 + 0'597 ≈ 1'227 .
3
4 4
∫ (2x − x ) ∫ (4x − x )dx = 3 u.a. 3 .
2 32 232
P.14 S = + 8 dx = 36 u.a. P.15 S =
−2 0

Recinto P.14 Recinto P.15

0 3 4

∫ [8 − (x )] ∫ [8 − (− x )] ∫ [8 − (x )]
2 2 2
P.16. − 2 x dx + + 4 x dx + − 2 x dx = 27 .
−2 0 3
3 4
6 4
P.17. ∫ (x − 1)dx + ∫ dx = 2 + 6 ln 
1 3
x 3

9-28
Integral definida Tema 9

Recinto P.16 Recinto P.17

1
4π − π 2
∫ [(2 − y ) ]
2
P.18. ≈ 0'6751u.v. 0,6751. P.19 π − y 2 dy = 2 π .
4
0
1
2 4

∫ (1 − 2x )dx = 3 . P.21. π ∫ (x ) − (x )


2π 2  − 5 x + 4 dx =768π .
2 2 2 2 2
P.20 2π + x − 20

0 0
4
 y −5
17
 2

∫ (3x + 5) dx =489 π ; b) π (4 − 1 )⋅ 8 + π ∫ y −  
2 2 2 2
P.22. a) π dy =120π + 81π = 201 π .
1 8
  3  
1 2 5

∫ ( 2px ) dx = π p h ∫( ) 625π
. P.24. π  6 x − x 2 − x 2 dx =
2 2
P.23. π .
  3
0 0
π π
4 4

∫ (cos ) π
∫ (cos x − senx )dx = − 1 +
2
P.25. a) 2 ; b) x − sen 2 x dx = .
2
0 0
3 3

∫ (9 − x )dx = 4 π . P.27.18,47.
4 2π
∫ 9 − x 2 dx = 3 π 2
P.26. a) ; b)
3 9
0 0

9-29
n
El conjunto R Tema 10

Tema 10

El conjunto Rn. Dependencia de vectores

10.1 Definición de Rn

Se define R n como el conjunto de n-tuplas ordenadas de números reales, es


{ }
decir, R n = (x1, x 2 ,K, x n ), x j ∈ R . A cada (x1, x 2 ,K, x n ) se le llama vector.

Ejemplos

Ejemplo 10.1 Los pares (3,1), (-1,1), (0,3) son elementos de R 2 .

Ejemplo 10.2 Las ternas (1,3,1), (2,1,1), (0,0,3) son elementos de R 3 .

Nota Dos elementos (x 1 , x 2 ,K , x n ) , (y 1 , y 2 ,K , y n ) , de R n son iguales si, y sólo si, x i = y i


para i = 1, 2,K , n .

10.2 Operaciones con elementos de Rn. Estructura

Definición. Dados dos elementos (x 1 , x 2 , K , x n ) y (y 1 , y 2 ,K , y n ) de R n se define la

operación suma como sigue (x1 , x 2 ,K , x n ) + (y 1 , y 2 ,K , y n ) = (x 1 + y 1 , x 2 + y 2 ,K , x n + y n ) .

Nota Es evidente que la suma de dos elementos de R n es otro elemento de R n cuyas


coordenadas son la suma de las coordenadas correspondientes de los sumandos.

Ejemplo 10.3 (1,3,2) + (4,1,2) = (5,4,4).

Definición. Para todo α de R y para todo (x 1 , x 2 ,K , x n ) de R n , se define el producto de


un número por un vector: α ⋅ (x1, x 2 ,K, x n ) = (α x1, α x 2 ,K, α x n ) .

Ejemplo 10.4 5 · (1,2,1) = (5,10,5) .

10-1
n
El conjunto R Tema 10

Nota Geométricamente R 2 se identifica con el conjunto de los vectores del plano con origen en
O. Análogamente, R 3 se identifica con el conjunto de los vectores del espacio con origen en
O. En ambos casos, la suma que se define entre dos vectores sigue la conocida ley del
paralelogramo.

El producto de un número real α por un vector es otro vector con la misma dirección,
mismo sentido si α > 0 , sentido contrario si α < 0 , y de módulo α veces el módulo del
primero.

n
Proposición (Estructura de R )
1. R n , con la operación suma, satisface las siguientes propiedades:
• La asociativa: a + (b + c ) = (a + b ) + c , ∀ a, b, c ∈ R n .
• La conmutativa: a + b = b + a , ∀ a, b ∈ R n .
• Existe un único elemento de R n , llamado elemento neutro, el (0,0,K ,0 ) , que
denotamos por 0, tal que a + 0 = a ∀ a ∈ R n .
• Para cada a = (x 1 , x 2 ,K , x n ) de R n existe un único elemento de R n , el
(− x1 ,− x 2 ,K ,− x n ) llamado elemento simétrico, que denotamos por −a , tal que
a + (−a ) = 0 .
2. La ley externa anteriormente definida verifica:
• α ⋅ (a + b ) = α ⋅ a + α ⋅ b
• (α + β) ⋅ a = α ⋅ a + β ⋅ a
• α ⋅ (β ⋅ a ) = (α ⋅ β) ⋅ a
• 1⋅ a = a , donde 1 es el elemento neutro de R para la operación producto habitual.

Por satisfacer las propiedades anteriores, se dice que el conjunto R n , con las
operaciones mencionadas tiene estructura de espacio vectorial sobre R y a sus elementos se
les llama vectores.

10.3 Combinación lineal

Definición Dado un subconjunto A = { v 1 , v 2 ,K , v m } de R n , llamamos combinación lineal


de elementos de A a todo vector v de R n que pueda ser escrito de la forma
v = α 1 v 1 + α 2 v 2 + K + α m v m , con α 1 , α 2 ,K α m ∈ R n .

Ejemplos

Ejemplo 10.5 Dada la familia A = {(3,1),(-1,1),(0,3)} de vectores de R 2 , dos combinaciones


lineales de ellos serían: -(3,1) y 4(-1,1)+2(0,3).
Ejemplo 10.6 Dada la familia A = {(1,3,1),(2,1,1),(0,0,3)} de vectores de R 3 , dos
combinaciones lineales de ellos serían: (1,3,1) y 2(1,3,1)-5(2,1,1)+(0,0,3).

10-2
n
El conjunto R Tema 10

Ejemplo 10.7 Dada la familia A = {(3,1),(-1,1),(0,3)} de vectores de R 2 , el vector (-2,5) es


combinación lineal de A porque (-2,5) = 2(-1,1)+(0,3).

Ejemplo 10.8 Dada la familia C = {(1,3,1),(2,1,1),(0,0,3) de vectores de R 3 , el vector (3,4,5)


es combinación lineal de C porque (3,4,5)=(1,3,1)+(2,1,1)+(0,0,3).

10.4 Familia linealmente dependiente

Definición Dado un subconjunto A = { v 1 , v 2 ,K , v m } de R n , se dice que A es linealmente


dependiente o familia ligada, si existe un elemento de A que es combinación lineal de los
restantes.

Ejemplos

Ejemplo 10.9 En R 2 , la familia A = {(1,2),(4,8)} es ligada, porque (4,8) = 4 (1,2).

Ejemplo 10.10 En R 2 , la familia A = {(20,30),(3,1),(1,2)} es ligada porque (20,30) = 2 (3,1) +


14 (1,2).

Nota Evidentemente, dada una familia A de vectores, si 0 ∈ A entonces A es linealmente


dependiente.

Proposición Una familia A = { v 1 , v 2 ,K , v m } de R n , es linealmente dependiente si, y sólo


si, existen α 1 , α 2 ,K , α m ∈ R no todos nulos, tales que 0 = α 1v 1 + α 2 v 2 + K + α m v m .

Ejemplo 10.11 Veamos, utilizando la proposición anterior, que la familia A = {(1,1,1), (1,0,1),
(0,1,0)} de R 3 es linealmente dependiente. En efecto, planteamos
α (1,1,1) + β (1,0,1) + γ (0,1,0 ) = (0,0,0 ) ,
que es equivalente a escribir
(α + β, α + γ, α + β) = (0,0,0 )
de donde resulta el sistema
α + β = 0

α + γ = 0
α + β = 0
que, resolviéndolo, da α = −β = − γ ; esto es, existen números reales no nulos que hacen cierta
la igualdad. Así que la familia es linealmente dependiente.

10.5 Familia linealmente independiente

Definición Dado un subconjunto A = { v 1 , v 2 ,K , v m } de R n , se dice que A es linealmente


independiente o familia libre, si no es linealmente dependiente; es decir, si ningún elemento
de A es combinación lineal de los restantes.

10-3
n
El conjunto R Tema 10

Ejemplo 10.12 Veamos que la familia A = {4,5), (2,3)} de R 2 es linealmente independiente.


En efecto, si no lo fuera, existiría un número α tal que (4,5) = α (2,3). Esto es,
(4,5) = (2α,3α ) . De donde, igualando coordenada a coordenada, resulta que 4 = 2α y 5 = 3α ,
lo cuál es imposible.

Proposición Una familia A = { v 1 , v 2 ,K , v m } de R n , es linealmente independiente si, y


sólo si, 0 = α 1v 1 + α 2 v 2 + K + α m v m implica que α 1 = 0, α 2 = 0,K , α m = 0 .

Ejemplo 10.13 Veamos, utilizando la proposición anterior, que la familia A = {1,1,1), (1,0,1),
(0,1,1)} de R 3 es linealmente independiente. Planteamos
α (1,1,1) + β (1,0,1) + γ (0,1,1) = (0,0,0 )
que es equivalente a escribir:
(α + β, α + γ , α + β + γ ) = (0,0,0)
de donde resulta el sistema
α+β = 0 

α+γ =0 
α + β + γ = 0 
que, resolviéndolo, da α = 0, β = 0, γ = 0 ; y, por lo tanto, es una familia linealmente
independiente.

Definición Dado un subconjunto A de R n , se llama rango de A al máximo número de


vectores de A que forman una familia linealmente independiente.

10.6 Propiedades

Proposición (Propiedades inmediatas)


• El conjunto {0} es linealmente dependiente.
• Si v ≠ 0 , el conjunto { v } es linealmente independiente.
• Dada una familia ligada, cualquier otra que la contenga es también ligada.
• Dada una familia libre, cualquier subconjunto de ella es también libre.

Proposición Si al añadir un vector a una familia libre, ésta se transforma en una ligada
entonces el vector añadido es combinación lineal de los restantes.

Proposición Si en una familia libre se sustituye un vector por una combinación lineal de él,
con coeficiente distinto de cero, y los restantes, la familia resultante es también libre.

10-4
n
El conjunto R Tema 10

Ejemplo 10.14 Sea la familia linealmente independiente A = { v 1 , v 2 , v 3 } veamos que la


familia B = { v 1 , v 2 , αv 1 + βv 2 + γv 3 } es también linealmente independiente. En efecto,
teniendo en cuenta la Proposición 10.3, planteamos a v 1 + b v 2 + c (αv 1 + βv 2 + γv 3 ) = 0 ,
que es equivalente a (a + c α ) v 1 + (b + c β)v 2 + c γ v 3 = 0 .
Recordemos que probar que B es linealmente independiente equivale a probar que
a = b = c = 0.
Ahora bien, como A es linealmente independiente, se tiene:
a + c α = 0

b + c β = 0
c γ = 0 
y, como γ ≠ 0 ,se deduce rápidamente que a = b = c = 0 , como se quería demostrar.

10.7 El método de la cascada


Hay un procedimiento que permite manipular los vectores de una familia sin que ésta
deje de ser libre o ligada, según lo fuera desde el principio.

Tal procedimiento es conocido por el nombre de reducción en cascada o triangulación


nula. Consiste en poner los vectores uno debajo de otro e intentar hacer cero los términos que
hay por debajo de la diagonal principal (la que va de arriba a la izquierda abajo a la derecha).

Ejemplo.

Dada la familia A = {(1,2,1), (3,4,0), (4,1,1)} de R 3 ,colocamos los vectores en columna,


uno encima del otro, como sigue:
1 2 1
3 4 0
4 1 1
Como el objetivo es hacer cero las coordenadas que están por debajo de la diagonal
principal, hay que conseguir algo como lo que sigue
1 2 1
0 ∗ ∗
0 0 ∗
Para ello, se fija la primera fila (1,2,1), se multiplica por -3 y el resultado se suma a la
segunda:
1 2 1
0 −2 −3
4 1 1
Se multiplica por -4 la primera fila y el resultado se suma a la tercera fila:
1 2 1
0 −2 −3
0 −7 −3
Como ya se tienen los ceros posibles en la primera columna, se fijan las dos primeras
filas.

Se multiplican por -7 y 2 la segunda y tercera fila, respectivamente, y se suman para


obtener la nueva tercera fila

10-5
n
El conjunto R Tema 10

1 2 1
0 −2 −3
0 0 15
Como ya se han hecho ceros todos los números que hay por debajo de la diagonal
principal es el momento de sacar conclusiones:

• La familia inicial y la {(1,2,1), (0,-2,-3), (0,0,15)} tienen el mismo tipo de dependencia.


• Si al realizar la triangulación nula, todas las filas son distintas del vector cero, la familia
es linealmente independiente. En caso contrario, esto es, si alguna fila es toda ceros, la
familia es linealmente dependiente. En nuestro caso podemos asegurar que la familia A
es linealmente independiente.
• Al número de vectores linealmente independientes, los que sean distintos de cero
después de introducir ceros en cascada, se llama rango del sistema de vectores.

10-6
n
El conjunto R Tema 10

Ejercicios propuestos
P.1. Comprueba si el vector (6,3) es combinación lineal A, siendo A = {(3,1),(-1,1),(0,3)} .

P.2. Comprueba si (4,6,5) es combinación lineal de C, siendo C = {(1,3,1),(2,1,1),(0,0,3)} .

P.3. Prueba que la familia A = {(1,2,1),(3,4,0),(-7,-8,2)} de R 3 es linealmente dependiente.

P.4. Estudia la dependencia en las familias siguientes. En caso de que exista, halla una
relación de dependencia, y el rango.

S1 = { (1,0,5,3), (3,1,-1,-1), (0,1,2,1), (7,1,1,0) }


S2 = { (1,0,3,1), (0,0,1,1), (1,3,0,1), (7,2,4,1), (1,1,1,1) }
S3 = { (5,2,-3,1), (4,1,-2,3), (1,1,-1,-2)}
S4 = { (4,2,0,1,0,5), (-2,0,1,2,3,0), (-2,-2,-1,0,0,0), (0,0,0,3,3,5) }
5
P.5. Calcula el rango del subconjunto de R formado por los vectores u1=(1,-1,1,-1,1),
u2=(1,1,0,0,3), u3=(3,1,1,-1,7), u4=(0,2,-1,1,2).
5
P.6. Sea S = { (2,3,0,1,1), (1,2,0,-3,2), (1,2,1,5,2) }. Encuentra un subconjunto de R que
contenga a S y sea libre.
4
P.7. Encuentra un subconjunto de R constituida en parte por los vectores A = { (2,-2,3,1),
(-1,4,-6,2), (1,14,-21,-7)} y que sea libre.

P.8. Determina t para que el vector x = ( 3,8, t ) se pueda expresar como combinación lineal
de la familia de vectores A = { u , v } , siendo u=(1,2,3) y v=(1,3,-1).

P.9. Determina b para que el conjunto de vectores A = { (b,−3,2), (2,3, b ), (4,6,−4 ) } tenga de
rango uno.

Soluciones

P.1. (6,3 ) = 3 ⋅ (3,1) + 3 ⋅ (− 1,1) − 1⋅ (0,3) . P.2. (4,6,5 ) = 8 ⋅ (1,3,1) + 6 ⋅ (2,1,1) + 11 ⋅ (0,0,3) .
5 5 5

P.3. (− 7,−8,2) = 2 ⋅ (1,2,1) − 3 ⋅ (3,4,0 ) .

P.4. S1 : Dependiente (1,0,5,3 ) = −2(3,1,−1,−1) + (0,1,2,1) + (7,1,1,0 ) , rango 3.


−1
S2 : Dependiente (1,1,1,1) = (1,0,3,1) + 2 (0,0,1,1) + 7 (1,3,0,1) + 1 (7,2,4,1) , rango 4.
27 3 27 9
S3 : Dependiente (5,2,−3,1) = (4,1,−2,3 ) + (1,1,−1,2) , rango 2.
S 4 : Dependiente (4,2,0,1,0,5 ) = − (− 2,0,1,2,3,0 ) − (− 2,−2,−1,0,0,0 ) + (0,0,03,3,5 ) , rango 3.

P.5, 2. P.6. B = S U {(0,0,01,1), (0,0,0,0,1)}.

P.7. B = {(2,−2,3,1), (− 1,4,−6,2), (1,14,−21,−7 ), (0,0,0,1)}. P.8. t = 1 . P.9. b = −2 .

10-7
Matrices Tema 11

Tema 11

Matrices. Operaciones con matrices

11.1 Definiciones
Sean I y J dos conjuntos finitos de índices; esto es, I = {1,2,3,K , m } y J = {1,2,3,K , n } .

Definición de matriz Llamamos matriz de números reales a toda aplicación a : I × J → ℜ ,


tal que a cada par (i , j ) le hace corresponder el elemento a(i , j ) de ℜ , que denotaremos
por a ij y al que se le llama elemento de la matriz.

Generalmente estos elementos se colocan en una tabla rectangular de la forma

 a11 . . . a1n 
 
 . .
 
 . .
 
 a m1 . . . a mn 

donde el elemento a ij ocupa la posición determinada por la fila i y la columna j. Toda matriz se
suele representar con una letra mayúscula y, abreviadamente, queda de la forma A = a ij ( )i , j∈I×J .

 2 −5 3 
Ejemplo 11.1  3 es una matriz de números reales, de dos filas y tres columnas en la
 −7 0 
 5 
3
que, por ejemplo, a11 = 2 , a 21 = y a13 = 3 .
5

Notas - Una matriz de orden m × n es un conjunto de m ⋅ n números reales ordenados en m filas


y n columnas. En el ejemplo anterior serían seis números reales ordenados en dos filas y tres
columnas.

- El conjunto de todas las matrices de orden m × n se denota por M m × n (ℜ ) .

Una matriz se llama matriz cuadrada si tiene el mismo número de filas que de columnas.
Se llama matriz columna si sólo tiene una columna. Se llama matriz fila si sólo tiene una
fila.

Nota El orden de una matriz cuadrada viene dado tan sólo por el número de filas (o de
columnas).El conjunto de todas las matrices cuadradas de orden n se denota por M n (ℜ ) .

11-1
Matrices Tema 11

 1 −2 3 
 
Ejemplo 11.2  0 1 77  es una matriz cuadrada de orden tres.
 43 − 5 2 
 

 − 6
 
 8 
 0  Es una matriz columna con 4 filas.
 
1 
 2

(1 2 55 − 8 9 ) Es una matriz fila con 5 columnas.

Dos matrices son iguales si, y sólo si, tienen el mismo orden y son iguales los elementos que
ocupan la misma posición.

Matriz transpuesta Dada la matriz A = a ij ( )


m×n
, se llama matriz transpuesta de A, y se denota
t
A' o A , a la matriz que se obtiene cambiando las filas por las columnas en la matriz A.

 1 4
 1 − 1 2  
Ejemplo 11.3 Sea A =   → A ′ =  − 1 1 
 4 1 0  2 0
 

Notas Observa que si A es de orden m × n , su transpuesta A' es de orden n × m y que (A ′)′ = A .

( )
Una matriz cuadrada A = a ij se llama simétrica cuando A ′ = A ; esto es, a ij = a ji ∀i , j . Y se
llama antisimétrica cuando A' = -A.

Ejemplos

2 4 5 
 
Ejemplo 11.4  4 1 3  es una matriz simétrica.
 
5 3 0 

 0 8 - 2
 
Ejemplo 11.5  - 8 0 5  es una matriz antisimétrica.
 
 2 -5 0 

11-2
Matrices Tema 11

Se llama diagonal principal de una matriz cuadrada a la formada por los elementos a ii .

2 4 5 
 
Ejemplo 11.6 (2,1,0) es la diagonal principal de la matriz  4 1 3  .
 
5 3 0 

Matriz diagonal Se dice que una matriz cuadrada es diagonal cuando a ij = 0 si i ≠ j .

3 0 0 
 
Ejemplo 11.7  0 2 0  es una matriz diagonal.
 
 0 0 1

Matrices triangulares Se dice que una matriz cuadrada A es triangular superior si todos los
elementos por debajo de la diagonal principal son nulos. Es decir, si a ij = 0 para i > j .
Análogamente se define matriz triangular inferior como la matriz cuyos términos por encima de
la diagonal principal son todos nulos.

1 0 2
 
Ejemplo 11.8  0 - 1 3  es una matriz triangular superior.
 
0 0 7 

11.2 Operaciones con matrices


Suma de matrices

( ) ( )
Definición Si A = a ij y B = bij son dos matrices del mismo orden m × n , se define la
( )
suma de A con B como la matriz C = A + B = c ij , de orden m × n , tal que
c ij = a ij + b ij ∀i , j .

Nota Observa que sólo se pueden sumar matrices que tengan el mismo orden y que, en tal caso,
la matriz suma es de dicho orden y cada uno de sus elementos es la suma de los elementos de los
sumandos que ocupan la misma posición.

11-3
Matrices Tema 11

Ejemplo 11.9
 1 2 − 1   2 − 3 5   3 − 1 4   0 − 4   3 − 6   3 − 10 
        
 3 −4 1 + 4 0 1  = 7 − 4 5  ;  3 5  +  7 32  =  10 37 
 3  2 6
     
1 7 − 6 3 −8

0   2 − 1 − 6   − 6 12   4 − 6   − 2 6 
 2   2 

Proposición (Propiedades algebraicas para la suma)

1) Conmutativa: A + B = B + A , ∀ A, B ∈ M m×n (R ) .
2) Asociativa: A + (B + C ) = (A + B ) + C , ∀ A, B ,C ∈ M m×n (R ) .
( )
3) La matriz 0 = a ij / a ij = 0 ∀i , j es el elemento neutro para la suma; esto es,
A + 0 = A , para toda matriz A ∈ M m×n (R ) .
4) Para toda matriz A = a ij( ) existe una matriz, a la que denotamos por –A y llamamos
matriz opuesta de A, tal que A + (− A ) = 0 . Dicha matriz es − A = (− a ij ) .
5) (A + B )′ = A ′ + B ′ , ∀ A, B ∈ M m×n (R )

Nota Por las propiedades anteriores, se dice que el conjunto M m×n (R ) tiene estructura de grupo
abeliano para la suma.

Ejemplos

0 0
 
Ejemplo 11.10 El elemento neutro para las matrices de orden 3 × 2 es 0 =  0 0  .
0 0
 
 − 3 2 − 6  3 − 2 6
Ejemplo 11.11 La matriz –A , opuesta de A =   , es − A =   .
 4 1 − 9 − 4 −1 9

Nota Restar dos matrices es sumar a la primera la opuesta de la segunda. Esto es, escribir A − B
es la manera más simple de escribir A + (−B ) .

Producto de un número real por una matriz

Definición Sean k ∈ R y A = aij ( )m×n se define el producto de k por A como la matriz


(
k ⋅ A = k ⋅ a ij )m×n , en la que cada elemento de A se ha multiplicado por k.

 − 1 4 3   − 5 20 15 
Ejemplo 11.12 5 ⋅   =   .
 3 2 − 2   15 10 − 10 

11-4
Matrices Tema 11

Proposición
∀ k , k 1 , k 2 ∈ R , ∀ A, B ∈ M m×n (R ) se verifica que:
1) k ⋅ (A + B ) = k ⋅ A + k ⋅ B .
2) (k1 + k 2 ) ⋅ A = k1 ⋅ A + k 2 ⋅ A .
3) k1 ⋅ (k 2 ⋅ A ) = (k1 ⋅ k 2 ) ⋅ A .
4) 1⋅ A = A .

Nota Por verificar las proposiciones de la suma y del producto de número real por matriz , se dice
que el conjunto M m×n (R ) es un R-espacio vectorial, de dimensión m × n , para las operaciones
suma y producto externo definidos anteriormente.

Producto de matrices

Dos matrices A = a ij ( )m×n y B = (bij )p×q se dice que son multiplicables si n = p ; es decir, si el
número de columnas de la primera es igual al número de filas de la segunda.

Dadas las matrices A = a ij ( )m×n , B = (bij )n×p , definimos el producto de A por B como la matriz
n
( )m×p tal que c ij = ∑ aik ⋅ bkj
, C = A ⋅ B = c ij ; es decir, el elemento de la fila i y columna j de C
k =1

es la suma de los productos ordenados de los elementos de la fila i de A por los de la


columna j de B.

Ejemplos

 2 1  0 1  2 ⋅ 0 + 1 ⋅ 2 2 ⋅1 + 1 ⋅1  2 3 
Ejemplo 11.13 Sean A =   , B =   → A ⋅ B =   =  
5 3   2 1  5 ⋅ 0 + 3 ⋅ 2 5 ⋅ 1 + 3 ⋅1   6 8 

 0 1
1 0 2   6 7 
Ejemplo 11.14 Sean las matrices A =   , B =  2 0  , entonces A ⋅ B =   y
2 1 0    2 2 
3 3 
2 1 0 
 
B ⋅ A = 2 0 4  .
 
9 3 6

Nota Observa que la existencia de A ⋅ B no implica que esté definido el producto B ⋅ A .

3 2   7 14 
  1 4  
Ejemplo 11.15 Sean A =  1 - 1  y B =   ; existe A ⋅ B =  − 1 3  , pero no B ⋅ A .
   2 1 2 8
2 0   

11-5
Matrices Tema 11

Proposición (Propiedades algebraicas para el producto)

1) Asociativa A ⋅ (B ⋅ C ) = (A ⋅ B ) ⋅ C
2) Distributiva respecto de la suma A ⋅ (B + C ) = A ⋅ B + A ⋅ C
3) (A ⋅ B )′ = B ′ ⋅ A′
4) En el caso de matrices cuadradas, existe elemento neutro llamado matriz unidad y que
se denota por I. Esto es, A ⋅ I = I ⋅ A = A , para todo A ∈ M n (R ) . La matriz unidad es
aquella cuyos elementos de la diagonal principal son 1 y el resto 0.

1 0 0 0
 
0 1 0 0
Ejemplo 11.16 La matriz unidad de orden 4 es I = 
0 0 1 0
 
0 0 0 1 

Notas - El producto de matrices no tiene la propiedad conmutativa. En general, A ⋅ B ≠ B ⋅ A .

- También puede suceder que A ⋅ B = 0 , a pesar de que A y B sean distintas de 0.

Ejemplos

 3 1  0 3   0 0 
Ejemplo 11.17   ⋅   =   .
 - 6 - 2  0 - 9  0 0 
0 3   3 1   − 18 − 6 
Si conmutamos los factores   ⋅   =   , el resultado es otro.
 0 - 9   - 6 - 2   54 18 
a b  x  e
Ejemplo 11.18 Para hallar la solución de la ecuación matricial   ⋅   =   , donde la
0 d   y   f 
x
incógnita es la matriz columna   , se realiza el producto indicado y, a continuación, se igualan
y 
cada elemento de la matriz del primer miembro con el correspondiente de la matriz del segundo
de - bf f
miembro, resulta así un sistema cuya solución es: x = , y = , que tiene sentido siempre
ad d
que a ⋅ d ≠ 0 , es decir, que a ≠ 0 y d ≠ 0 .

11.3 Matriz inversa

Definición Dada una matriz A, llamaremos matriz inversa de A a aquella matriz B tal que
A⋅B = B ⋅ A = I .

Notas - Observa que de la definición se deduce que para que pueda existir matriz inversa de una
matriz dada A, la primera condición es que A sea una matriz cuadrada. No obstante, tampoco toda
matriz cuadrada admite matriz inversa.

- Si existe matriz inversa de A, la denotaremos por A −1 .

11-6
Matrices Tema 11

Ejemplos

1 2 
Ejemplo 11.19 Veamos que la matriz A =   no admite inversa. En efecto, supongamos que
1 2 
admitiera inversa. En tal caso ésta tendría que ser cuadrada de orden dos y, por tanto, de la forma
a b  1 2   a b   1 0 
A −1 =   y, además, debería verificarse que A ⋅ A −1 = I . Ahora bien,   ⋅   =  
 c d  1 2   c d   0 1 
a + 2c = 1 
a + 2c = 0 
implica que  , lo cuál es absurdo.
b + 2d = 0
b + 2d = 1

1 1   2 - 1
Ejemplo 11.20 Sean las matrices A =   ; B =   , para comprobar que las matrices son
1 2   - 1 1
1 0
inversas la una de la otra, basta verificar que A ⋅ B = B ⋅ A =   , lo cuál es evidente en este
 0 1
caso.

Definición Llamaremos matrices regulares a aquellas que admiten inversa.

Proposición (Propiedades de la matriz inversa)

1) La matriz inversa, si existe, es única.


2) Si A y B admiten inversa, entonces (A ⋅ B )−1 = B −1 ⋅ A −1 .
3) Toda matriz triangular, tal que a ii ≠ 0 ∀i , es regular.
4) Toda matriz diagonal tal que a ii ≠ 0 ∀i , es regular.

Las matrices regulares tal que A ′ = A −1 se llaman matrices ortogonales.

11.4 Método de Gauss–Jordan para el cálculo de matrices inversas


El método de Gauss-Jordan para el cálculo de la matriz inversa de una matriz dada A,
requiere que, una vez escrita la matriz A, se coloque a su derecha la matriz unidad. Ambas, A e I,
aparecerán juntas y separadas por una raya.

 a11 K a1n 1 K 0 
 
 M O M M O M
 
 a n1 K a nn 0 K 1 

El objetivo es, mediante transformaciones con las filas, obtener la matriz unidad en el lugar
que ocupaba la matriz A. Una vez conseguido, puede afirmarse que la matriz que resulte en el lugar
de la unidad (la de la derecha) será la inversa de la dada.

11-7
Matrices Tema 11

 1 K 0 b11 K b1n 
 
M O M M O M 
 
 0 K 1 b n1 K bnn 

Si al realizar el proceso, en el lado izquierdo aparecen dos filas iguales o una fila se anula,
se dice que la matriz dada no tiene inversa.

El método consiste en realizar una doble triangulación nula sobre la matriz de la izquierda.

La primera triangulación es de arriba a abajo y se consigue que todos los elementos que
están por debajo de la diagonal principal sean nulos.

La segunda triangulación es de abajo a arriba, con lo que se consigue que sean nulos
todos los elementos que están por encima de la diagonal principal.

Según cómo se ha procedido, en el caso de que algún elemento de la diagonal principal no


sea uno, se divide toda la fila entre el número correspondiente.

Las transformaciones permitidas para las triangulaciones son consecuencia de considerar


a cada fila como un vector de R 2n y aplicarle las proposiciones vistas en el tema anterior. Se
pueden concretar en las siguientes:

1. Una fila se puede multiplicar o dividir por cualquier número real distinto de cero.
2. Una fila puede sustituirse por una combinación lineal de ella misma y otra u otras, siempre
que la fila que se sustituye no se multiplique por cero.
3. Dos filas pueden intercambiarse entre sí.

Ejemplos

 1 2 - 1
 
Ejemplo 11.21 Hallemos por el método de Gauss la inversa de la matriz A =  3 8 2  .
 
 4 9 - 1
Primera triangulación nula:

 1 2 − 1 1 0 0  1 2 − 1 1 0 0
   
 3 8 2 0 1 0  ⇒  0 2 5 − 3 1 0  f 2 − 3f1
 4 9 − 1 0 0 1  0 1 3 − 4 0 1  f − 4f
    3 1

se intercambian las filas f 3 y f 2

 1 2 − 1 1 0 0 1 2 − 1 1 0 0
   
 0 1 3 − 4 0 1 ⇒  0 1 3 − 4 0 1
0 2 5 − 3 1 0  0 0 1 − 5 − 1 2  − f + 2f
    3 2

Segunda triangulación nula:

 1 2 0 − 4 − 1 2  f1 + f 3  1 0 0 − 26 − 7 12  f1 − 2f 2
   
 0 1 0 11 3 − 5  f 2 − 3f 3 ⇒  0 1 0 11 3 − 5 ,
0 0 1 − 5 − 1 2  0 0 1 − 5 − 1 2 
   

11-8
Matrices Tema 11

 - 26 - 7 12 
 
la matriz inversa es A −1
=  11 3 - 5  .
 
 - 5 -1 2

1 2 1
 
Ejemplo 11.22 Hallemos la matriz inversa de A =  3 2 1 .
5 6 3 

 1 2 1 1 0 0 1 2 1 1 0 0 1 2 1 1 0 0
     
 3 2 1 0 1 0  ⇒  0 − 4 − 2 − 3 1 0  f 2 − 3f1 ⇒ 0 − 4 − 2 − 3 1 0
 5 6 3 0 0 1  0 − 4 − 2 − 5 0 1  f − 5f 0 0 0 − 2 − 1 1  f 3 − f 2
    3 1 

Al obtener una fila nula, puede afirmarse que no tiene matriz inversa.

11.5 Rango o característica de una matriz


Como ya se dijo, dada la matriz A ∈ M m×n , cada fila la puede ser considerada como un
vector de R n y cada columna como un vector de R m .
v1 K v n
w1 a11 K a1n
M M O M
wm a m1 K a mn

Proposición En toda matriz, el máximo número de vectores fila linealmente independientes


coincide con el máximo número de vectores columna linealmente independientes.

Definición de rango Se llama rango de una matriz A al máximo número de vectores fila (o
vectores columna) linealmente independientes.

Nota De la definición y de la proposición anterior, se sigue que el rango de una matriz coincide con
el rango de su traspuesta.

Para calcular el rango de una matriz numérica se puede utilizar el método de Gauss de
introducción de ceros en cascada manipulando filas o columnas a nuestra conveniencia. La matriz
resultante tiene el mismo rango que la original.

Ejemplos

 − 1 - 2 0
 
Ejemplo 11.23 Calculemos el rango de A =  - 1 3 4  .
 
 2 - 2 6
 − 1 - 2 0  − 1 - 2 0  − 1 - 2 0
     
 - 1 3 4 ⇔  0 5 4  f 2 − f1 ⇔  0 5 4 ,
     
 2 - 2 6  0 − 6 6  f 3 + 2f1  0 0 54  5f 3 + 6f 2

11-9
Matrices Tema 11

luego, como las tres filas son linealmente independientes, el rango de la matriz es 3 .

 t -1 0
 
Ejemplo 11.24 Estudiemos el rango de la matriz  - 1 t 0  , según los valores del parámetro t.
 
 2 - 2 t -1
Por comodidad, intercambiamos las dos primeras filas y hacemos cero los números que
hay por debajo de –1:
- 1 t 0  - 1 t 0
   
 t -1 0 ⇒  0 t 2 - 1 0  f 2 + t ⋅ f1 .
   
 
 2 - 2 t - 1  0 2t - 2 t - 1 f 3 + 2f1

Como todos los elemento de la segunda y tercera filas dependen de t , estudiamos si es


posible que se anulen a la vez:
t 2 - 1 = 0 , si t = ±1 ; 2t - 2 = 0 , si t = 1 y t − 1 = 0 , si t = 1 .

Luego, si t = 1 , la matriz tiene rango 1. Si t = −1 , es de rango 2. Y, si t ≠ ±1 , el rango es 3.

11-10
Matrices Tema 11

Ejercicios resueltos

Operaciones con matrices

R.1. Dadas las matrices


2 1 − 5 3 
 1 3 2 2 − 3  
    1 − 2 3 8  1 − 2 3
A= 2 1 1 , B =  0 1  , C =   y D =   . Calcula:
− 2    2 −4 −5 −3  2 1 4 
 2 3 1 1   
0 1 0 2 

a) A ′, − B y − C ′
b) B′ + D
c) 2 ⋅ B − 3 ⋅ D′

Soluciones:
− 2 − 1 − 2 0 
 1 2 − 2 − 2 3   
     −1 2 4 − 1
a) A ′ =  3 1 2  ; − B =  0 − 1 ; − C ′ = 
2 1 3   − 1 − 1 5 −3 5 0 
     
 − 3 − 8 3 − 2
 

 2 0 1  1 − 2 3   3 − 2 4 
b) B ′ + D =   +   =  
 − 3 1 1  2 1 4   − 1 2 5 

 4 − 6  3 6   1 − 12 
     
c) 2 ⋅ B − 3 ⋅ D′ =  0 2  −  − 6 3  =  6 −1 
 2 2   9 12   − 7 − 10 
     

2
 1  2 1  1 3   − 1 2   3 2   − 1 1
R.2. Calcula: a)  − ⋅   + 2⋅ 2 4   b) 2 ⋅   −   ⋅  
 2  4 2   1 
0    3 5   7 3   0 1
 

Soluciones:
2
 1  2 1  1 3 
2  1  3  2
0 1   0 − 1

a) − ⋅   + 2 ⋅  2 4  =   − 1 − 2  +  1 2   = 
  =  
 2  4 2


 1

0     
− 2 − 1  2 0  
 0 − 1  0 1 
   

 − 1 2   3 2   − 1 1  − 2 4   − 3 5   1 − 1
b) 2 ⋅   −   ⋅   =   −   =  
 3 5   7 3   0 1  6 10   7 10  13 0 

R.3. Calcula los siguientes productos de matrices:

3 1 2 5 2 1  2 1 2 4 
    2 1 3   
a)  2 1 3  ⋅  2 3 − 2  
b) A ⋅ B y B ⋅ A con A =  
 y B = 3 − 1 0 1 
 3 − 2 1  5 3 9   7 2 − 1 2 0 1 − 3
     

Soluciones:

11-11
Matrices Tema 11

 3 1 2   5 2 1   27 15 19 
     
a)  2 1 3  ⋅  2 3 − 2  =  27 16 27 
 3 − 2 1   5 3 9   16 3 16 
     

2 1 2 4 
2 1 3    13 1 7 0 
b) A ⋅ B =   ⋅  3 − 1 0 1  =   ; B ⋅ A no puede realizarse pues B
 7 2 − 1  2 0 1 − 3  18 5 13 33 
 
tiene 4 columnas y A tiene 2 filas .

 − 1 2 3 2
R.4. Dadas las matrices A =   y B =   . Se pide comprobar que:
 2 1  1 4
a) (A + B )′ = A ′ + B ′
b) (A ⋅ B )′ = B ′ ⋅ A ′
c) A ⋅ B ≠ B ⋅ A

Soluciones:

2 4  2 3  − 1 2 3 1  2 3
a) A + B =   → (A + B )′ =   ; A ′ =  ; B ′ =   → A ′ + B ′ =  
3 5  4 5  2 1 2 4  4 5

 − 1 2 3 2  − 1 6  − 1 7  3 1  − 1 2  − 1 7 
b) A ⋅ B =   ⋅   =   ; (A ⋅ B )′ =   ; B ′ ⋅ A ′ =   ⋅   =  
 2 1   1 4   7 8   6 8   2 4   2 1  6 8 

3 2   − 1 2  1 8 
c) B ⋅ A =   ⋅   =   ≠ A ⋅ B (que está calculado en el apartado b).
 1 4   2 1  7 6 

 1 1 1
 
R.5. Dada la matriz A =  0 1 1 . Calcula A 2 , A 3 , A 4 ,... hasta encontrar una forma de
 0 0 1
 
n
expresar A .

Solución:

1 1 1  1 1 1  1 2 3  1 1 1  1 2 3   1 3 6 
           
A2 = 0 1 1 ⋅  0 1 1 =  0 1 2  ; A 3 =  0 1 1 ⋅  0 1 2  =  0 1 3 
0 0 1  0 0 1  0 0 1   0 0 1  0 0 1   0 0 1 
      
1 1 1  1 3 6   1 4 10 
4      
A = 0 1 1 ⋅  0 1 3  =  0 1 4  ; para encontrar la forma de A n el único elemento
0 0 1  0 0 1   0 0 1 

1 n 
n  
conflictivo es el a13 , pues los demás quedan A =  0 1 n  , si se observa la secuencia de
0 0 1
 
operaciones que han dado lugar al término a13 en cada producto se puede ver que en
A 2 → a13 = 1 + 2 = 3 ; en A 3 → a13 = 1 + 2 + 3 = 6 ; en A 4 → a13 = 1 + 2 + 3 + 4 = 10

11-12
Matrices Tema 11

n ⋅ (n + 1)
en A n → a13 = 1 + 2 + 3 + ⋅ ⋅ ⋅ + n = . Se ha aplicado la fórmula de la suma de los n
2
 n ⋅ (n + 1) 
1 n 
n  2 
primeros términos de una progresión aritmética con a1 = 1 . Luego A =  0 1 n 
0 0 1 
 
 

 cos a − sena   cos b − senb 


R.6. Dadas las matrices A =   y B =   . Calcula A ⋅ B y B ⋅ A .
 sena cos a   senb cos b 
¿Quiere decir este resultado que el producto de matrices es conmutativo?.

Solución:

 cos a − sena   cos b − senb   cos a cos b − sena ⋅ senb − cos a ⋅ senb − sena ⋅ cos b 
A ⋅ B =   ⋅   =  =
 sena cos a   senb cos b   cos a ⋅ senb + sena ⋅ cos b cos a cos b − sena ⋅ senb 

 cos (a + b ) − sen (a + b )
=   .
 sen (a + b ) cos (a + b ) 

 cos b − senb   cos a − sena   cos (b + a ) − sen (b + a )


B ⋅ A =   ⋅   =   = A ⋅ B .
 senb cos b   sena cos a   sen (b + a ) cos (b + a ) 

El producto de matrices no es conmutativo, pues para que se pueda afirmar tal cosa
debería cumplirse que A ⋅ B = B ⋅ A ∀A y ∀B y no sólo para algunos casos.

1 2   2 1
R.7. Dadas las matrices A =   y B =   .
3 − 2 3 5
a) Comprueba que (A − B ) ⋅ (A + B ) ≠ A 2 − B 2 . ¿Por qué no se cumple la igualdad?.
b) Indica el desarrollo correcto de (A − B ) ⋅ (A + B )

Solución:

− 1 1  3 3  3 0 
a) A − B =   ; A + B =   → (A − B ) ⋅ (A + B ) =  
 0 − 7   6 3   − 42 − 21

 7 − 2 7 7  0 −9 
A 2 =   ; B 2 =   → A 2 − B 2 =   .
 − 3 10   21 28   − 24 − 18 

La igualdad no se cumple porque el producto de matrices no es conmutativo.

 0 − 9   8 11   5 2 
b) (A − B ) ⋅ (A + B ) = A 2 − B 2 + A ⋅ B − B ⋅ A =   +   −  =
 − 24 − 18   0 − 7   18 − 4 
 3 0 
=  
 − 42 − 21

Ecuaciones y sistemas matriciales

R.8. Resuelve los siguientes sistemas matriciales:

11-13
Matrices Tema 11

 − 2 3   3 1 17  
8 A − 5B =   3 A + 7B =   
 11 − 2  10 23 − 3  
a)  b) 
 0 1   5 − 12 1  
2 A − B =    5 A − 2B =   
 3 0    3 11 − 5  

Soluciones:
 − 2 3 
I) 8 A − 5B =  
 11 − 2  − 2 3   0 4  2 1
a)  ⇒ I − II ⋅ 4 → −B =   −   → B =   Sustituimos
 0 1   11 − 2  12 0   1 2
II) 2 A − B =   
 3 0  
1   0 1  2 1   1 1
en II) y despejamos A =   +  =  
2   3 0   1 2    2 1

 3 1 17  
I) 3 A + 7B =   
10 23 − 3   6 2 34   35 − 84 7 
b)  ⇒ 2 ⋅ I + 7 ⋅ II → 41 ⋅ A =   +   =
  20 46 − 6   21 77 − 35 
 5 − 12 1 
II) 5 A − 2B =   
 3 11 − 5  

 41 − 82 41  1 − 2 1 
=   → A =   ; sustituyendo A en II y despejando B se tiene:
 41 123 − 41 1 3 − 1

1  1 − 2 1   5 − 12 1    0 1 2 
B= 5 ⋅  −  =  
2  1 3 − 1  3 11 − 5    1 2 0 

 7 3 
3 X − 2Y =   
 16 4  
  
R.9. Resuelve el sistema de ecuaciones matriciales  .
 6 12  
X + 3Y =  
 − 2 27  
 

Solución:

Multiplicando por 3 la segunda ecuación y restando la primera se tiene:

 18 36   7 3   11 33   1 3
11 ⋅ Y =   −   =   → Y =   sustituyendo en la segunda
 − 6 81 16 4   − 22 77   − 2 7
 6 12   1 3 3 3
ecuación y despejando: X =   − 3 ⋅   =  
 − 2 27   − 2 7  4 6

2 0  2 3 
R.10. Calcula la matriz X tal que 3 ⋅ X − 2 ⋅ A = B con A =   y B =   .
 3 − 1  9 23 

Solución:

11-14
Matrices Tema 11

 2 3  4 0    2 1
Al despejar X se tiene: X =
1
(B + 2 ⋅ A ) = 1    + 2 ⋅    = 


3 3   9 23   6 − 2 5 7

Cálculo de la matriz inversa

R.11. Calcula por el método de Gauss la matriz inversa, si existe de:

2 1 2  0 2 0 
   
a) A =  1 0 − 1 b) B =  3 0 − 3 
3 1 2  0 1 2 
   

Soluciones:

a) Se indican a la derecha las transformaciones realizadas en cada línea.

2 1 2 1 0 0  1 1 1 1 0 0  1 f1 1 1 1 1 0 0 
   2 2 2  2 2
 1 0 − 1 0 1 0 ⇒  1 0 − 1 0 1 0 ⇒  0 − 1 − 2 − 1 1 0  f 2 − f1 ⇒
 3 1 2 0 0 1    2 2 
  3 1 2 0 0 1  0 − 1 − 1 − 3 0 1  f 3 − 3f1
   2 2 

1 1 1 1 0 0  1 1 1 1 0 0 
 2 2  2 2
⇒ 0 1 4 1 − 2 0  − 2f 2 ⇒  0 1 4 1 − 2 0 ⇒
   
0 − 1 − 1 − 3 0 1 0  1
0 1 − 1 − 1 1 f3 + 2 f2
 2 2   

1 1 0 3 1 − 1  f1 − f 3 1 0 0 −1 0 1  f1 − 21 f 2
 2 2  
⇒ 0 1 0 5 2 − 4  f 2 − 4f 3 ⇒  0 1 0 5 2 − 4
  0 0 1 − 1 − 1 1 
0 0 1 −1 −1 1   
 

− 1 0 1 
 
Por lo tanto A −1 =  5 2 − 4
− 1 − 1 1 

b)
0 2 0 1 0 0 1 2 −1 1 1 0 f + 1 f  1 2 − 1 1 1 0
   3  1 3 2  3 
 3 0 − 3 0 1 0  ⇒ 1 0 −1 0 1 0 1
f 2 ⇒  0 − 2 0 − 1 0 0  f 2 − f1 ⇒
 0 1 2 0 0 1  3  3
 
  0 1 2 0 0 1 0 1 2 0 0 1
   

1 2 − 1 1 1 0 1 2 −1 1 1 0 1 2 −1 1 1 0 
 3   3   3
⇒ 0 1 0 1  
0 0 − 2 f2 ⇒ 0 1 0
1 1 0 0  
⇒ 0 1 0 1 0 0  ⇒
 2   2   2 1
0 1 2 0 0 1 0 0 2 − 1 0 1 f3 − f 2 0 0 1 − 1 0 1  2 f3
   2   4 2

1 2 0 3 1 1 f + f 1 0 0 − 1 1 1  f − 2f − 3 4 6
 4 3 2 1 3  4 3 2 1 2
1  
⇒ 0 1 0 1 0 0  ⇒ 0 1 0 1 0 0  −1
B =  6 0 0
 2   2  12  
0 0 1 − 1 0 1  0 0 1 − 1 0 1  − 3 0 6
 4 2  4 2

11-15
Matrices Tema 11

 1 0 0
2 2
 
R.12. Resuelve la ecuación matricial A ⋅ X − B = A siendo: A =  0 2 0  ;
 0 0 1
 
−1 0 0
 
B =  0 −3 0  :
0 0 − 1

Solución:
 1 0 0
2
 
Llamamos C = A y calculamos su valor C =  0 4 0  , la ecuación queda:
 0 0 1
 
0 0 0
 
C ⋅ X − B = C → C ⋅ X = C + B =  0 1 0  y X = C −1 ⋅ (C + B ) . Calculamos C −1
0 0 0
 

 1 0 0 1 0 0  1 0 0 1 0 0 1 0 0
     
0 4 0 0 1 0 ⇒ 0 1 0 0 1 0  1 ⋅ f así que C −1 =  0 1 0 y
4 4 2 4
 0 0 1 0 0 1  0 0 1 0 0 1  0 0 1 
   

1 0 0 0 0 0 0 0 0
     
X = 0 1 0 ⋅ 0 1 0 = 0 1 0
4 4
0 0 1   0 0 0   0 0 0 

0 3 0
4 − 2 1   
R.13. Calcula la matriz X tal que X ⋅ B + A = C , siendo A =   , B =  2 0 1  y
5 1 − 3 0 − 3 2
 
 1 −3 5 
C =   .
 − 2 4 − 6

Solución:
− 3 −1 4 
X ⋅ B = C − A → X = (C − A ) ⋅ B −1 ; C − A =   Calculamos B −1
 − 7 3 − 3
0 3 0 1 0 0  2 0 1 0 1 0   1 0 1 0 1 0 1 f
2 1
    2 2
 2 0 1 0 1 0  cambiamos f1 y f2 ⇒  0 3 0 1 0 0  ⇒  0 3 0 1 0 0 ⇒
 0 − 3 2 0 0 1  0 − 3 2 0 0 1   0 − 3 2 0

0 1
     

1 0 1 0 1 0 1 0 1 0 1 0  1 0 0 − 1 1 − 1 f − 1 f
 2 2   2 2  4 2 4 1 2 3
⇒ 0 3 0 1 0 0 ⇒ 0 3 0 1 0 0  ⇒ 0 1 0 1 0 0  31 f 2
    3 
0 0 2 1 0 1  f3 − f 2 0 0 1 1 0 1  1 f3  0 0 1 1 0 1 
   2 2 2  2 2 

− 1 1 −1   − 3 6 − 3
 4 2 4
1  
luego B −1 =  1 
 12 
0 0 = 4 0 0  y
 3 
 1 0 1   6 0 6 
 2 2 

11-16
Matrices Tema 11

 − 3 6 − 3
1 − 3 −1 4    1  29 − 18 33   29 12 − 3 2 114 
X=   ⋅  4 0 0  =   =
 5 
12  − 7 3 − 3  
6 0 6  12  15 − 42 3   4 − 7 2 14 
 

Estudio del rango de una matriz

R.14. Calcula utilizando el método de Gauss el rango de las matrices:


 4 − 1 0 3  1 2 3 − 1
   
 0 1 3 1 2 4 6 3 
a) A =  b) B = 
2 1 1 3 0 1 2 − 2
   
 6 0 1 6 1 1 1 1 
   

Soluciones:

a)
4 − 1 0 3 4 − 1 0 3 4 − 1 0 3 4 − 1 0 3
       
0 1 3 1 0 1 3 1 0 1 3 1 0 1 3 1
2 1  ⇔  ⇔  ⇔
1 3 0 3 2 3 2f 3 − f1 0 3 2 3 0 0 −7 0  f 3 − 3f 2
       
6 0 1 6  0 3 2 3  2f 4 − 3f1 0 0 0 0  f 4 − f 3 0 0 0 0 
   
El rango es 3

1 2 3 − 1 1 2 3 − 1 1 2 3 − 1
     
2 4 6 3  0 0 0 5  f 2 − 2f1 0 0 0 5 
b)   ⇔  ⇔ El rango es 3
0 1 2 −2 0 1 2 −2 0 1 2 − 2
     
1 1 1 1  0 − 1 − 2 2  f − f 0 0 0 0  f 4 + f 3
   4 1 

R.15. Estudia según los valores del parámetro a el rango de las siguientes matrices:

2 − 5 a 1 1 1   a 1 1
     
a) A =  1 4 1  b) B =  a 1 a  c) C =  1 a 1 
3 − 1 2 1 a a2   1 1 a
     

Soluciones:

2 − 5 a 2 − 5 a  2 − 5 a 
     
a)  1 4 1  ⇔  0 13 2 − a  2f 2 − f1 ⇔  0 13 2 − a 
3 − 1 2  0 13 4 − 3a  2f − 3f  0 0 2a − 2  f − f
    3 1   3 2

Si a = 1 → r (A ) = 2; Si a ≠ 1 → r (A ) = 3

1 1 1  1 1 1   1 1 1
     
b)  a 1 a  ⇔  0 1 − a 0  f 2 − a ⋅ f1 Si a = 1 → B =  0 0 0  → r (B ) = 1
1 a a2   0 a − 1 a 2 − 1 f − f 0 0 0
    3 1  

 1 1 1
 
Si a = −1 → B =  0 2 0  → r (B ) = 2 ∀a ≠ 1 ∧ a ≠ −1 → r (B ) = 3
0 2 0
 

11-17
Matrices Tema 11

 a 1 1  1 1 a 1 1 a 
     
c) C =  1 a 1  intercambi amos las filas 1 y 3 ⇔  1 a 1  ⇔  0 a − 1 1 − a  f 2 − f1 ⇔
 1 1 a  a 1 1 0 1− a 1− a2  f − a ⋅ f
      3 1

1 1 a  Si a = 1 → r (C ) = 1
 
⇔  0 a − 1 1 − a  f 2 − f1 ⇔ Si a = −2 → r (C ) = 2
0 1− a 1− a2  f − a ⋅ f ∀a ≠ 1 ∧ a ≠ −2 → r (C ) = 3
  3 1

R.16. Calcula los valores de a y b para que el rango de la matriz A sea dos con

 2 − 2 1
 
2 − 1 a
A= .
4 − 2 b
 
 2 − b 1
 

Solución:

 2 − 2 1 2 − 2 1  2 − 2 1 
     
2 − 1 a 0 1 a − 1 2 1
f − f 0 1 a −1 
4 − 2 b ⇔ 0 2 b − 2  f 3 − 2f1
⇔
0 0 b − 2a  f − 2f
      3 2
 2 − b 1 0 2 − b 
0  f 4 − f1  (a − 1) ⋅ (b − 2) f 4 + (b − 2)f1
   0 0

Los valores de a y b que hacen que el rango sea 2 son las soluciones de

b − 2a = 0
(a − 1) ⋅ (b − 2) = 0
.La única solución del sistema es a = 1 y b = 2 .

R.17. Demuestra que toda matriz cuadrada se puede descomponer de modo único como suma
de una matriz simétrica y una antisimétrica.

Solución:

Supongamos que sea cierto; es decir, que A = S + H , siendo S una matriz simétrica y H una
antisimétrica. Entonces A′ = S ′ + H ′ = S − H , luego A + A′ = 2 ⋅ S y A − A′ = 2 ⋅ H . De donde,
A + A′ A − A′
resolviendo el sistema, se obtiene S = y H= . Como se ve, por la manera de ser
2 2
obtenida, ésta descomposición es única.

11-18
Matrices Tema 11

Ejercicios propuestos

P.1. Expresa mediante la tabla correspondiente cada una de las siguientes matrices: A = a ij , ( )
 2, si i ≥ j
de orden 3 × 4 , tal que: aij =  . ( )
B = bij , de orden 3 × 3 , tal que:
 - 1, si i < j
 4, si i + j > 3
b ij = 
 - 4, si i + j ≤ 3

P.2. Determina la matriz traspuesta de las siguientes:

-1 2 
   0 -1 6  -1 1 5 6 8
 3 - 1    
A=  ; B = - 2 - 4 2  ; C =  4 - 3 0 1 6 
 0 7    
   5 1 - 1  - 3 - 5 4 1 7 
 7 4
P.3. Resuelve el siguiente sistema matricial:
1 - 2 1  
2A - B =  
 
1 0 5  



2 2 2  
A+ B =  
 
5 3 4  
1 0 0
 1 0 1 1 3 0  
2
P.4. Calcula A′ ⋅ B − C Siendo: A =   ; B =   ; C =  0 2 1 
0 2 0   4 0 - 1  
0 2 2 
 1 2 1  1 0 1
   
P.5. Dadas las matrices A =  1 3 1  , B =  2 2 2  , hallar la matriz P que verifique
   
0 0 2  0 0 6 
P − B2 = A ⋅ B .
1 0 
P.6. Sabiendo que A =   , calcular: H = A + A 2 + L + A n .
1 1 
0 0 0   1 0 1 1 1 1 
     
P.7. Hallar A , B y C siendo: A =  1 0 1  ; B =  0 0 0  ; C = 1 1 1 
n n n
     
 0 0 1  1 0 1 1 1 1 
 cosa - sena 
P.8. Siendo A =   , calcula A p .
 sena cosa 

1 2
P.9. Sea la matriz A =   , determinar todas las matrices X de orden 2 tales que A ⋅ X = 0 .
3 a
Halla el valor de a para que el problema tenga solución distinta de la trivial.

1 2 - 3  1 7 - 13 
P.10. Sean B =   , C =   ¿Existe una matriz A tal que C = A ⋅ B ?
2 -1 4  5 0 5

11-19
Matrices Tema 11

 2 - 2 - 4  1 1 3
   
P.11. Si A =  - 1 3 4  , B =  5 2 6  . Calcula A 2 y B 3 . (A las matrices que cumplen
   
 1 - 2 - 3  - 2 -1 - 3 
lo que A 2 se les llama idempotentes, a las que verifican lo que B 3 se les llama nilpotentes.)

1 0 2 0 
P.12. Dadas las matrices A =   y R =   , hallar la matriz S = A + A ⋅ R + L + A ⋅ R n −1 .
0 2  0 3 

P.13. Resolver la ecuación matricial A ⋅ X ⋅ B = C , siendo:

1 0 1 1   1 1
A =   , B =   , C =  
0 1 1 2  0 0 

P.14. Resolver la ecuación matricial M ⋅ X + N = P , siendo:

 -1 0  1 2 4 3 
M =   , N =   , P =  
 0 - 1 3 4   2 1
 0 0 1
 
P.15. Demuestra que A =  0 - 1 0  es inversa de sí misma.
 
1 0 0

 -1 0 0  0 1 6 
   
P.16. Demuestra que las matrices: A =  2 0 0  y B =  0 2 1  son linealmente
   
 3 0 0 0 5 2 
independientes.

P.17. Determina el rango de cada una de las siguientes matrices:


- 4 1 1 2   -1 2
3 4 4 0   -1 2 1 0     
     3 1 2 1  1 5
A1 =  1 3 2 - 2  , A2 =  6 0 1 0  , A3 =   , A4 =  
     -1 2 3 3   1 6
2 1 2 2   0 3 1 0    
 4 1 5 0  2 1

 4 1 2
-1 5 3 1 0 4 6 8 10   
     0 1 2
 2 1 0 1 0 1 2 3 3  
A5 =   , A6 =   , A7 =  0 0 0
 1 1 1 0 1 3 4 5 7  
     1 0 0
 2 1 0 1 0 2 2 2 4  
 -1 2 0

1 2 3 t
 
P.18. Calcula el rango de la matriz  2 4 6 8  para los distintos valores de t:
 
 3 6 9 12 

 0 1 2  1 0 2 0
   
P.19. Halla una matriz X tal que:  - 1 1 3  ⋅ X =  - 1 3 1 0 
   
 4 -1 - 5  - 5 -1 4 0

P.20. Hallar la matriz X que verifica A ⋅ X ⋅ B + C = D , siendo:

11-20
Matrices Tema 11

1 0 0
 1 0    1 2 3 3 1 0 
A =   B =  0 1 0  C =   D =  
 -1 0     -1 2 - 3  0 1 0 
0 -1 1

P.21. Encontrar una matriz X que verifique la ecuación: A ⋅ X + B = C siendo:

1 0 0  1 0 0 3 0 0 
     
A = 1 2 0  B =  0 1 0  C =  2 5 2  .
     
1 2 4   0 0 1 0 1 3 

P.22. Calcula por el método de Gauss la inversa de las matrices:

2 0 1 3 4 1
   
a)  1 2 1  b)  2 1 4 
   
0 0 1 4 5 2 

Soluciones:

 2 − 1 − 1 − 1  − 4 − 4 4  0 −2 5 
    − 1 3 0 7  
P.1. A =  2 2 − 1 − 1 , B =  − 4 4 4  . P.2.   ; − 1 − 4 1  ;
2 2 2 − 1  4 4 4   2 − 1 7 4 6 2 − 1
  
 − 1 4 − 3
 
 1 − 3 − 5 0 3 0   6 4 18 
5  1 2 1  1 0 1    
0 4  . P.3. B =  ; A =   . P.4.  8 − 6 − 6  . P.5. 13 10 31  .
   3 2 1  2 1 3  1 − 5 − 6  0 0 48 
6 1 1     
 
8 6 7 

 n 0 0 0 0  1 0 1 1 1 1
    n −1   n −1  
P.6. n (n + 1) n
. P.7. A =  0 0 n
1 ; B = 2 n
⋅ 0 0 0 ; C = 3 ⋅ 1 1 1
 n
0 0
 2   1   1 0 1
 
1 1 1
 

 cos (pa ) − sen (pa )  x = −2 x 21


P.8.   P.9.Si a = 6 → ∀x 21 , x 22 ∈ R con  11 ;si a ≠ 6 → X = (0 ) .
 sen (pa ) cos (pa )   x 12 = −2 x 22
 3 − 1 2n − 1 0   1 0
P.10.   . P.11. A 2 = A, B 3 = 0 . P.12.  . P.13.   .
1 2  0 3 − 1
n
   0 0
 − 3 − 1
P.14.   . P.15. Basta comprobar que A ⋅ A = I . P.16. Trivial.
 1 3 
P.17. A1 : 2 , A2 : 3 , A3 : 3 , A4 : 2 A5 : 3 , A6 : 2 , A7 : 3 P.18. Si t = 4 → r = 1, t ≠4→r =2
 1 1
   0 − 
 − 10 8 3 0  2 0 0  2 2
 
P.19.  25 − 22 − 2 0  . P.20. No existe. P.21.  0  1 1 1
2 1 . P.22. a)  − − 
 1   4 2 4
 − 12 11 2 0  −
3
0  0
 − 0 1 
 2 4   
 
b) No admite inversa.

11-21
Determinantes Tema 12

Tema 12

Determinantes.Matriz inversa. Rango de una matriz.

12.1 Conceptos previos. Permutaciones de n elementos

Definición Se llama permutación de los n elementos del conjunto A = {a1 , a 2 , a 3 ,L , a n } a


cualquier forma de ordenar los n elementos de A.

Ejemplo 12.1 Si A = {1, 2 } , una permutación es {1, 2 } y la otra es { 2,1 }

Notas - A partir de ahora, consideraremos tan sólo las permutaciones de los n primeros números
naturales 1, 2, 3,K , n . Y al conjunto de todas las permutaciones posibles de los n primeros números
naturales lo denotaremos por S n .

Ejemplos

Ejemplo 12.2 Con el conjunto formado por los números 1 , 2 y 3, hay 6 permutaciones posibles:

1 2 3, 1 3 2, 2 1 3 , 2 3 1, 3 1 2 y 3 2 1.

Proposición 12.1 El número total de permutaciones de n elementos es n! = 1⋅ 2 ⋅ 3 ⋅ L ⋅ n .

Ejemplo 12.3 Con los números 1, 2, 3 y 4 se pueden hacer 4! = 24 permutaciones distintas.

Definición Se llama permutación principal a aquella en la que los números están en el orden
natural.

Ejemplo 12.4 La permutación σ = 1 2 3 4 5 es la permutación principal de los cinco primeros


números naturales.

12-1
Determinantes Tema 12

Definición Dos elementos se dice que están en permanencia si se encuentran en la misma


posición relativa que en la permutación principal. En caso contrario se dice que están en
inversión.

Ejemplo 12.5 En la permutación 3 2 1 4, los números 3 y 1, 3 y 2, 2 y 1 están en inversión; 1 y 4


están en permanencia.
En la permutación 3 1 5 2 4, los números 3 y 1, 3 y 2, 5 y 2, 5 y 4, están en inversión; 3 y 5,
3 y 4, 1 y 5, 1 y 2, 1 y 4, 2 y 4, están en permanencia.

Una permutación se dice de clase par si el número de inversiones es par y en caso contrario
se dice de clase impar.

Ejemplo 12.6 La permutación 3 2 1 4, como se ha visto en el ejemplo anterior, tiene 3 inversiones


por lo que es de clase impar.

La permutación 3 1 5 2 4, como se ha visto en el ejemplo anterior, tiene 4 inversiones por lo


que es de clase par.

Nota De las n! permutaciones de los n primeros números naturales la mitad son de clase par y la
otra mitad de clase impar.

La demostración es muy simple, pues si una vez formadas todas las permutaciones
cambiamos entre si dos elementos fijos dados en cada permutación se siguen teniendo las mismas
permutaciones y las que eran de clase par se convierten en clase impar y viceversa.

Definición Llamamos signatura de una permutación σ , y la denotamos por s (σ ) , al número


s (σ ) = (− 1) p , donde p es el número de inversiones de la permutación.

 1, si σ es par
Nota Evidentemente, s (σ ) =  .
 − 1, si σ es impar

12.2 Determinante de una matriz cuadrada. Definición

 a11 a12 L a1n 


 
 a21 a 22 L a 2n 
Sea A =  una matriz cuadrada de orden n.
M M M M 
 
a 
 n1 a n 2 L ann 
Llamamos determinante de una matriz cuadrada A, y lo representamos por A , al número
A = ∑ s(σ )a σ ( ) ⋅ a σ ( ) ⋅L ⋅ a σ ( ) ,
σ ∈ Sn
1 1 2 2 n n

donde σ (1), σ (2), ... , σ (n ) es una permutación.

12-2
Determinantes Tema 12

Es decir: es el número que resulta al sumar todos los productos posibles de n términos ,
tales que no haya dos de la misma fila, ni dos de la misma columna, con signo más o menos según
la paridad de la permutación que resulta al considerar los segundos subíndices sea par o impar.

Nota Observa que según la definición de un determinante:

- Hay n! sumandos.
- Cada sumando se compone de n factores tales que no hay dos de la misma fila ni dos de
la misma columna.
- El signo de cada sumando es positivo si la permutación σ de los subíndices de las
columnas es de clase par y negativo si es de clase impar.

Calcular el valor del determinante de una matriz cuadrada a partir de la definición resulta
complicado, no obstante la definición es el punto de partida en las demostraciones de las
propiedades de los determinantes.

A continuación vamos a plantear reglas que faciliten calcular el valor de determinantes de orden
dos y tres; posteriormente se verá como calcular determinantes de orden mayor que tres.

12.2.1 Determinantes de orden dos

a11 a12
Al aplicar la definición al cálculo de un determinante de orden dos; es decir, ,
a21 a22
recordemos que con los números 1 y 2 sólo hay dos posibles permutaciones, la 1 2 y 2 1; la
a a12
primera es de clase par y la segunda de clase impar, por lo que 11 = a11 ⋅ a 22 − a12 ⋅ a 21 .
a21 a22

1 −3 1 −3
Ejemplo 12.7 El valor de es = 1⋅ 5 − (− 3 ) ⋅ (− 2) = −1 .
−2 5 −2 5

12.2.2 Determinantes de orden tres. Regla de Sarrus.

a11 a12 a13


En el caso de un determinante de orden 3 tal como a21 a22 a23 recordemos que
a31 a32 a33
hay seis permutaciones posibles con los números 1, 2 y 3. Tres de clase par: 123, 231 y 312, y
tres de clase impar: 321, 213 y 132. Por lo tanto, según la definición de determinante se tiene

a11 a12 a13


a21 a 22 a23 = a11a 22 a33 + a13 a21a32 + a12 a 23 a31 − a13 a22 a31 − a12 a 21a33 − a11a23 a32
a31 a32 a33

No obstante, para mayor comodidad, existe una regla geométrica para recordar este
desarrollo, llamada Regla de Sarrus, según la cual:

Los términos de signo positivo provienen de la diagonal principal y de los dos triángulos de
base paralela a dicha diagonal.

Los términos de signo negativo provienen de la diagonal secundaria y de los dos triángulos
de base paralela a dicha diagonal, tal y como se ve en los siguientes esquemas.

12-3
Determinantes Tema 12

2 −2 5
Ejemplo 12.8 Para calcular el valor de − 3 4 0 nos fijamos en los esquemas anteriores y
6 −1 1
2 −2 5
− 3 4 0 = 2 ⋅ 4 ⋅ 1 + (− 3 ) ⋅ (− 1) ⋅ 5 + (− 2) ⋅ 0 ⋅ 6 − 5 ⋅ 4 ⋅ 6 − 2 ⋅ 0 ⋅ (− 1) − (− 2) ⋅ (− 3 ) ⋅ 1 =
6 −1 1
= 23 − 126 = − 103
1 2 3
4 5 6 = (1⋅ 5 ⋅ 9 + 4 ⋅ 8 ⋅ 3 + 2 ⋅ 6 ⋅ 7) − (3 ⋅ 5 ⋅ 7 + 1⋅ 6 ⋅ 8 + 2 ⋅ 4 ⋅ 9) =
7 8 9
= (45 + 96 + 84 ) − (105 + 48 + 72 ) = 0 .

12.3 Propiedades de los determinantes


Se enuncian sin demostrar las propiedades fundamentales de los determinantes que, entre
otras cosas, van a servir para facilitar el cálculo de determinantes de orden superior a tres.

Observa que de no utilizar estas propiedades resulta excesivamente laborioso el cálculo de


un determinante, ya para el de orden 4 requiere hacer 24 sumas de 4 factores cada sumando y el
de orden 5 tiene 120 sumandos de 5 factores cada uno y, así, sucesivamente.

1. El determinante de una matriz A es igual al de su traspuesta.

2 −3 6
Ejemplo 12.9 Calculemos el valor de − 2 4 − 1 correspondiente a la matriz traspuesta de la
5 0 1
primera del ejemplo 12.9.
2 −3 6
−2 4 − 1 = 2 ⋅ 4 ⋅ 1 + (− 3 ) ⋅ (− 1) ⋅ 5 + (− 2) ⋅ 0 ⋅ 6 − 5 ⋅ 4 ⋅ 6 − 2 ⋅ 0 ⋅ (− 1) − (− 2) ⋅ (− 3 ) ⋅ 1 = −103
5 0 1
Observa que tenemos exactamente los mismos sumandos en el desarrollo que en el ejemplo
anterior.

12-4
Determinantes Tema 12

Nota Como consecuencia de esta propiedad, todas las que siguen son válidas indistintamente
para filas o para columnas. Se hablará entonces, de líneas aplicando dichas propiedades a filas o a
columnas según interese.

2. El determinante de una matriz triangular es igual al producto de los elementos de la


diagonal principal.

2 1 0 −3
0 4 2 4
Ejemplo 12.10 = 2 ⋅ 4 ⋅ (− 3 ) ⋅ 5 = −120 , puesto que los demás sumandos tendrán
0 0 −3 −7
0 0 0 5
siempre un factor nulo.

3. Si todos los elementos de una línea son 0, el determinante es 0.

4. Al multiplicar una línea, fila o columna, por un número el determinante queda multiplicado por
ese número.

2 3
Ejemplo 12.11 Calculemos 4 ⋅ .
−1 4
2 3
Por un lado, se tiene que 4 ⋅ = 4 ⋅ (2 ⋅ 4 − 3 ⋅ (− 1)) = 44 y aplicando la propiedad
−1 4
8 3
resulta = 8 ⋅ 4 − 3 ⋅ (− 4 ) = 44 , que es lo mismo. Observa que al multiplicar la primera
−4 4
columna por 4, cada sumando se ha multiplicado también por 4.

Nota Como consecuencia: el producto de un número por un determinante es igual al determinante


que resulta al multiplicar una sola línea, y sólo una, por ese número.

Ejemplo 12.12 Se comprueba, entre otras posibilidades, que


3 2 4 9 2 4 3 2 4
3⋅ 1 2 1 = 3 2 1 = 3 6 3 = −93
5 3 −1 15 3 − 1 5 3 −1

5. Si se permutan entre sí dos filas o columnas de un determinante éste cambia de signo.

6. Si una línea de un determinante está formada por términos que son suma de varios
sumandos, el determinante es igual a la suma de los determinantes que se obtienen al sustituir
dicha línea por los primeros sumandos, los segundos, etc. y dejar el resto de las líneas iguales.

12-5
Determinantes Tema 12

3 2 4
Ejemplo 12.13 Es fácil comprobar por un lado que 1 2 1 = −31 y por otra parte se puede
5 3 −1
considerar la siguiente descomposición:
3 2 4 3 1+ 1 4 3 1 4 3 1 4
1 2 1 = 1 −2+4 1 = 1 −2 1 + 1 4 1 = 54 − 85 = −31 , que también da lo
5 3 −1 5 2 +1 −1 5 2 −1 5 1 −1
mismo.

Nota La propiedad anterior se puede entender de la siguiente forma:

a11 a12 a13 a11 b12 a13 a11 a12 + b12 a13
a21 a 22 a 23 + a21 b22 a 23 = a 21 a 22 + b22 a 23 como regla para sumar determinantes.
a31 a32 a33 a31 b32 a33 a31 a32 + b32 a33

7. Un determinante con dos líneas iguales es 0.

8. Si un determinante tiene dos líneas proporcionales, el determinante es nulo.

9. Un determinante no varía si se le suma a una línea una combinación lineal de las


restantes.

Ejemplo 12.14 Veamos con un ejercicio teórico las propiedades 6, 7, 8 y 9.

a11 a12 a13


Sea A = a 21 a 22 a 23 , sumemos a la primera fila una combinación lineal de las otras y veamos
a31 a32 a33
que el determinante no varía:

a11 a12 a13 a11 + αa 21 + βa31 a12 + αa 22 + βa32 a13 + αa23 + βa3
A = a 21 a 22 a 23 = a 21 a22 a 23 =
a31 a32 a33 a31 a32 a33

aplicando la propiedad 6, resulta

a11 a12 a13 αa 21 αa 22 αa 23 βa31 βa32 β a3


= a21 a 22 a 23 + a21 a 22 a 23 + a 21 a22 a 23 = A + 0 + 0 = A ,
a31 a32 a33 a31 a32 a33 a31 a32 a33

pues los sumandos 2º y 3º tienen dos líneas proporcionales.

12-6
Determinantes Tema 12

Idea clave Esta propiedad sugiere que para resolver un determinante se puede triangular la matriz
(hacer 0 los elementos por debajo de la diagonal principal) utilizando combinaciones lineales, sin
que altere el valor del determinante y, a continuación, aplicar la propiedad 2.

Nota Hay que poner especial atención en el número de la combinación lineal que multiplica a la
línea sustituida por aquella, porque el determinante queda multiplicado por dicho número.

1 2 3
Ejemplo 12.15 a) Calculemos 3 − 1 0
2 1 1

1 2 3 1 2 3   1 2 3  
     8
3 − 1 0 = 0 − 7 − 9 f 2 − 3f1  = 0 − 7 − 9   = 1 ⋅ (− 7 ) ⋅  −  = 8
 7
2 1 1 0 − 3 − 5 f 3 − 2f1  0 0 −8 f − 3 f 
 3 7 2
7

−1 1 1 1
1 −1 1 1
b) Calculemos
1 1 −1 1
1 1 1 −1

−1 1 1 1 −1 1 1 1   −1 1 1 1
 
1 −1 1 1 0 0 2 2 f 2 + f1  0 2 0 2
=  =− [cambio f 2 y f 3 ] =
1 1 −1 1 0 2 0 2 f 3 + f1  0 0 2 2
1 1 1 −1 0 2 2 0 f 4 + f1  0 2 2 0

−1 1 1 1 −1 1 1 1
0 2 0 2 0 2 0 2
=− =− = −16 .
0 0 2 2 0 0 2 2
0 0 2 − 2 f4 − f2 0 0 0 − 4 f4 − f3

10. Un determinante es 0 si una fila es combinación lineal de las restantes.

11. El determinante de la matriz producto de dos matrices cuadradas es el producto de los


determinantes.

3 2 5 2
Ejemplo 12.16 Dadas A =   y B =   comprobamos que A ⋅ B = A ⋅ B .
 1 7   8 1
 31 8 
A ⋅ B =   ; A = 21 − 2 = 19 ; B = 5 − 16 = −11 ; A ⋅ B = 279 − 488 = −209 = 19 ⋅ (− 11) , con lo
 61 9 
que queda comprobada la propiedad.

12-7
Determinantes Tema 12

12.4 Matriz adjunta


Definición Dada una matriz cualquiera llamaremos menor al determinante de la matriz
cuadrada que resulta de suprimir un determinado número de filas y de columnas.

3 2 1 4 2 1 4
  3 1
Ejemplo 12.17 En  3 1 8 9  , = 1 es un menor de orden dos, 1 8 9 es un menor
2 1 1 0 2 1
  1 1 0
3 2 4
de orden tres, su valor es –37, 3 1 9 es otro menor de orden tres.
2 1 0

Dada una matriz cuadrada llamaremos menor complementario de un elemento al menor que
resulta de suprimir su fila y su columna.

3 1 2
  3 1
Ejemplo 12.18 En  0 − 1 5  , el menor complementario de 5 es , su valor es 6.
0 2 1 0 2
 

Llamamos adjunto de un elemento aij y lo representaremos Aij , al menor complementario si


i + j es par, y al menor complementario cambiado de signo si i + j es impar.

3 1 2
  3 2
Ejemplo 12.19 En  0 − 1 5  , el adjunto de a 32 = 2 es A32 = − = −15 .
0 2 1 0 5
 

( )
Llamamos matriz adjunta de una matriz cuadrada aij a la matriz (Aij ) , cuyos elementos son
los adjuntos Aij de la primera.

Así, para una matriz de orden 3, (Aij ) sería:

 a 22 a 23 a 21 a 23 a 21 a 22 
 − 
 a 32 a 33 a 31 a 33 a 31 a 32 
 a12 a13 a11 a13 a11 a12 
(Aij ) = −
 a 32 a 33 a 31 a 33

a 31 a 32


 a12 a13 a11 a13 a11 a12 
 − 
 a 22 a 23 a 21 a 23 a 21 a 22 
 

12-8
Determinantes Tema 12

12.5 Desarrollo de un determinante por los adjuntos de una línea

Teorema El determinante de una matriz cuadrada es igual a la suma de los productos de


los elementos de una línea (fila o columna) por sus correspondientes adjuntos.

Veamos la comprobación para una matriz de orden tres desarrollando por la primera
columna:
a11 a12 a13
a 22 a 23 a12 a13 a12 a13
a 21 a 22 a 23 = a11 ⋅ − a 21 ⋅ + a 31 ⋅ =
a 32 a 33 a 32 a 33 a 22 a 23
a 31 a 32 a 33
= a11a 22 a 33 − a11a 23 a 32 − a 21a12 a 33 + a 21a13 a 32 + a 31a12 a 23 − a 31a13 a 22 = A

Nota Este teorema da un método para hallar el valor de los determinantes de orden superior a 3.
Previamente, utilizando las propiedades de los determinantes enunciadas anteriormente, se
introducen ceros en una línea para simplificar operaciones.

Ejemplos

2 1 0 3
2 4 2 4
Ejemplo 12.20 Para el cálculo de empezamos haciendo algunos ceros:
3 −1 2 5
2 −3 3 4
2 1 0 3 2 1 0 3
2 4 2 4 0 3 2 1 f 2 − f1
= =
3 −1 2 5 3 −1 2 5
2 −3 3 4 0 −4 3 1 f 4 − f1

y luego desarrollamos por adjuntos a partir de la primera columna:

3 2 1 1 0 3
= 2⋅ −1 2 5 + 3⋅ 3 2 1 = 6 .
−4 3 1 −4 3 1

Ejemplo 12.21 Desarrollo por adjunto de la primera fila se tiene:

a b c d
x 0 0 −1 0 0 −1 x 0 −1 x 0
−1 x 0 0
= a ⋅ −1 x 0 − b ⋅ 0 x 0 + c ⋅ 0 −1 0 − d ⋅ 0 −1 x =
0 −1 x 0
0 −1 x 0 −1 x 0 0 x 0 0 −1
0 0 −1 x
= ax 3 + bx 2 + cx + d

12.6 Matriz inversa de una matriz cuadrada


En el tema anterior se ha visto que dada una matriz A, la matriz inversa A −1 es la matriz
que verifica A ⋅ A −1 = A −1 ⋅ A = I

12-9
Determinantes Tema 12

1
Se puede probar que A −1 = Aij ′ ; es decir, la matriz traspuesta de la adjunta ( o la
( )
A
adjunta de la traspuesta, que es lo mismo) dividida por el determinante de A.

Por tanto, para que una matriz tenga inversa es necesario y suficiente que su determinante
sea distinto de cero, es decir, que la matriz sea regular.

 1 2 3
 
Ejemplo 12.22 Dada la matriz A =  0 2 1  calculemos A −1 .
 0 − 1 1
 
1 0 0 
 
Primero calculamos su determinante: A = 3 . A continuación la matriz traspuesta A ′ =  2 2 − 1
3 1 1 
 
y en esta matriz se calculan los adjuntos:

2 −1 2 −1 2 2
A11 = =3 A12 = − = −5 A13 = = −4
1 1 3 1 3 1
0 0 1 0 1 0
A21 = − =0 A22 = =1 A23 = − = −1
1 1 3 1 3 1
0 0 1 0 1 0
A31 = =0 A32 = − =1 A33 = =2
2 −1 2 −1 2 2

 −5 4
1 − 
3 − 5 − 4  3 3
1  1 1
Entonces: A −1 =  0 1 − 1 =  0 − 
3  3 3
0 1 2   1 2 
0 
 3 3 

12.7 Rango de una matriz. Método de orlar menores


Se vio en el tema anterior el método de Gauss para calcular el rango de una matriz.
Veremos ahora otro modo de calcular ese rango utilizando el concepto de determinante.
Enunciamos el siguiente teorema:

El rango de una matriz coincide con el orden del mayor menor no nulo de dicha matriz.

La búsqueda del menor de mayor orden, distinto de cero, se hace por un método llamado
orlar menores.

Dado un menor de orden h, llamamos menor orlado al determinante de orden h + 1 que resulta
al añadir una fila y una columna cualesquiera de la matriz a dicho menor.

12-10
Determinantes Tema 12

 2 −6 0 − 5
3
 
 − 15 1 4 11 
3 1 3
Ejemplo 12.23 En la matriz   un menor de orden dos es , algunos
6 3 2
9 −9 3 2
 
 5 4 0 6 
1

2 −6 3 1 3 11
menores orlados del anterior son: − 15 1 3 (con f1 y c1 ); 3 2 − 9 (con f 4 y c 5 )
6 3 2 4 1 6

Para el cálculo del rango de una matriz mediante el teorema anterior conviene tener en
cuenta la siguiente propiedad:

Proposición Sea Ai un menor no nulo de orden h de una matriz A. Si todos los menores
obtenidos orlando Ai con una fila m y con el resto de las columnas de A son nulos, entonces la
fila m es combinación lineal de las h filas de A que corresponden a Ai ( lo mismo para las
columnas).

Para hallar el rango de una matriz numérica seguiremos el siguiente proceso:

• Se parte de un menor de orden h no nulo.


• Se orla con una línea fija, por ejemplo una fila, y el resto de las columnas.
o Si alguno de esos menores es distinto de cero el rango de la matriz es, al menos,
h + 1 y con ese menor de orden h + 1 repetiríamos el proceso de orlar.
o Si al orlar, todos los menores fueran cero, la fila elegida es combinación lineal de
las h incluidas en el menor y dicha fila no cuenta para el rango de la matriz (la
suprimimos).
• Se continúa el proceso, si es preciso, con otras filas. Si todos los menores son nulos el
rango es h.

Ejemplos

1 −1 2 0 7 
 
Ejemplo 12.24 Calculemos el rango de  3 − 1 0 3 − 2  .
4 − 2 2 3 5 
 

Como sólo tiene tres filas, el rango es, como mucho, 3.


−1 2
Tomamos un menor no nulo de orden 2, por ejemplo .
−1 0
A continuación, lo orlamos con la fila 3 y con cada una de las columnas no elegidas en el
menor:

1 −1 2 −1 2 0
orlando con f3 y c1 se tiene 3 − 1 0 = 0 , orlando con f3 y c 4 se tiene − 1 0 3 = 0 y
4 −2 2 −2 2 3
−1 2 7
orlando con f3 y c 5 resulta − 1 0 − 2 = 0 .
−2 2 5

12-11
Determinantes Tema 12

Por lo tanto, la fila tercera es combinación lineal de las dos primeras y en consecuencia el
rango es 2.

2 3 − 1
 
4 3 2 
Ejemplo 12.25 Calculamos el rango de  0 3 − 4 .
 
1 1 0 
 
0 0 1 

El rango es como mucho 3, pues sólo tiene tres columnas.

2 3
Tomamos un menor no nulo de orden 2, por ejemplo = −6 ≠ 0 y orlamos con la
4 3
tercera columna y el resto de las filas (tercera, cuarta y quinta):

2 3 −1 2 3 −1
orlando con f3 y c 3 se tiene 4 3 2 = 0 ; orlando con f 4 y c 3 resulta 4 3 2 = 1 ≠ 0 . Por
0 3 −4 1 1 0
lo tanto, ya no es preciso seguir y se puede concluir que el rango de la matriz es 3.

Nota Para el caso de matrices numéricas, parece más rápido hallar el rango a base de triangular
o, bien, triangular y estar atentos a menores no nulos que puedan dar el rango directamente.

Para hallar el rango de una matriz con parámetros lo conveniente suele ser empezar
anulando los menores más grandes que se puedan formar y, a continuación, hacer el estudio
distinguiendo entre los valores que lo anulan y los que no lo anulan.

Ejemplos

k 3 2 
 
Ejemplo 12.26 Calculemos para los distintos valores de k el rango de  3 2 k  .
 
2 k 3 
Empezamos haciendo cero el menor de mayor orden que, en este caso, coincide con el
determinante de la matriz.
k 3 2
3 2 k = 18k - 35 - k 3 = 0 ⇔ k = −5 .
2 k 3

De aquí podemos concluir:


5 3
a) para k = −5 , el rango es 2, ya que el menor es distinto de cero;
3 2
b) para k ≠ −5 , el rango es 3, ya que el determinante de la matriz es distinto de cero.

Ejemplo 12.27 Calculemos para los distintos valores de x e y el rango de la matriz:

12-12
Determinantes Tema 12

 -1 - 2 3 
 
 0 x 0 
 
 2 4 -6  .
 
 1 y -3 
 
 0 0 1

En este caso, es fácil ver que la tercera fila es combinación lineal de la primera, luego el
rango de la matriz dada coincide con el rango de la matriz

 −1 − 2 3 
 
 0 x 0 
 .
1 y −3 
 
 0 0 1 

Entre los posibles menores de mayor orden, que es tres, tomamos, por ejemplo, el
−1 − 2 3
0 x 0 que se anula sólo para x = 0 . Luego, para x ≠ 0 el rango es 3, independiente mente
0 0 1
del valor de y. Veamos qué pasa para x = 0 . La matriz e estudiar es
 −1 − 2 3 
 
 1 y −3 ,
 0 0 1 

−1 − 2 3
cuyo determinante vale 1 y − 3 = 2 − y = 0 ⇔ y = 2 . Así que la matriz que resulta para
0 0 1

 −1 − 2 3 
 
x = 0 e y = 2 es  1 2 − 3  , cuyo rango es 2.
 0 0 1 

−1 − 2 3
Ahora bien, como el menor 1 y − 3 es independiente de x, para y ≠ 2 el rango es
0 0 1
tres.

Resumiendo:

x y rango
R − {0 } R 3
R R − {2 } 3
0 2 2

12-13
Determinantes Tema 12

Ejercicios resueltos

Determinantes de orden 2 y 3

R.1. Calcula el valor de los siguientes determinantes utilizando la Regla de Sarrus:

1 3 −2 5 −2 0
a) 4 2 1 b) 1 1 2
5 −3 7 3 −3 1

Soluciones:

1 3 −2
a) 4 2 1 = (14 + 24 + 15 ) − (− 20 − 3 + 84 ) = −8
5 −3 7

5 −2 0
b) 1 1 2 = (5 + 0 − 12 ) − (0 − 2 − 30 ) = 25
3 −3 1
x 2x 2
x 3
R.2. Resuelve las ecuaciones: a) =0 b) 2 1 3 =0
2 x
1 2 4
Soluciones:

x 3
a) = 0 . Desarrollamos y queda x 2 − 6 = 0 ⇔ x = ± 6 .
2 x

x 2x 2
1
b) 2 1 3 = 0 ⇔ 4 x + 8 + 6 x − 2 − 6 x − 16 x = 0 ⇔ x = .
2
1 2 4

Propiedades de los determinantes

x y  x y
R.3. Dada la matriz A =   y sabiendo que = 3 , utiliza las propiedades de los
 z t  z t
determinantes para calcular razonadamente:
x+y y x y 2y 2x
a) b) 3 3 c) d) 3 ⋅ A .
z+t t 2z 2t 2t 2z

Soluciones:

x+y y x y y y
a) = + = 3 + 0 = 3 [propiedades 6 y 7].
z+t t z t t t

x y x y
1
b) 3 3 = ⋅2⋅ = 2 [propiedad 4] .
2z 2t 3 z t

12-15
Determinantes Tema 12

2y 2x y x
c) = 2⋅2⋅ = 2 ⋅ 2 ⋅ (− 3 ) = −12 [propiedades 4 y 5].
2t 2z t z

3 x 3y
d) 3 ⋅ A = = 27 [ propiedad 4] (recuérdese el producto de número por matriz).
3z 3t

 1 3 2 3 − 1 2
   
R.4. Dadas las matrices A =  5 2 1  y B =  2 1 0  comprueba que:
 − 1 2 4 3 1 3
   
a) A + B ≠ A + B b) A ⋅ B = A ⋅ B c) α ⋅ B = α 3 ⋅ B

Soluciones:
 4 2 4 4 2 4
 
a) Calculamos A + B =  7 3 1 y A + B = 7 3 1 = 38 , por otra parte se tiene que
2 3 7 2 3 7
 
1 3 2 3 −1 2
A = 5 2 1 = −33 y B = 2 1 0 = 13 , la desigualdad es evidente.
−1 2 4 3 1 3

 15 4 8
 
b) A ⋅ B = A ⋅ B Calculamos A ⋅ B =  22 − 2 13  y
 13 7 10 
 
15 4 8
A ⋅ B = 22 − 2 13 = −429 = −33 ⋅ 13 = A ⋅ B
13 7 10

 3α − 1α 2α  3α − 1α 2α 3 −1 2
 
c) (α ⋅ B ) =  2α 1α 0  y 2α 1α 0 = α ⋅ 2 1 0 = α3 ⋅ B
3

 3α 1α 3α  3α 1α 3α 3 1 3
 

R.5. Sabiendo que los números 102, 119 y 255 son múltiplos de 17, demuestra aplicando las
2 0 1
propiedades de los determinantes que 9 1 1 es también múltiplo de 17.
5 5 2
Solución:

Si en la primera columna hacemos la transformación c1 + 10 ⋅ c 2 + 100 ⋅ c 3 el valor del


2 0 1 102 0 1 6 0 1
determinante no varía 9 1 1 = 119 1 1 = 17 ⋅ 7 1 1 y por tanto múltiplo de 17.
5 5 2 255 5 2 15 5 2

Determinantes de orden superior

12-16
Determinantes Tema 12

3 1 −1 2 2 1 3 5
2 3 5 2 −2 2 4 2
R.6. Calcula introduciendo ceros el valor de: a) b)
3 0 1 −7 −1 3 5 7
−2 2 3 4 3 −9 3 2

Soluciones:

a) Se van a introducir ceros en la 2ª columna por tanto hay que manejar filas.

3 1 −1 2 3 1 −1 2  
 
2 3 5 2 −7 0 8 − 4 f 2 − 3f1 
=   desarrollamos por la segunda columna
3 0 1 −7 3 0 1 −7 
−2 2 3 4 −8 0 5 0 f 4 − 2f1 

3 1 −1 2
−7 8 −4
−7 0 8 −4
=− 3 1 − 7 = 60 − 448 + 32 + 245 = −111
3 0 1 −7
−8 5 0
−8 0 5 0

b) Introducimos ceros en la segunda fila, por tanto manejaremos columnas.

2 1 3 5 7 7  cambios 
2 3
 
− 2 2 4 2 − 2 0 0 0  c 2 ⇒ c 2 + c1 
=   desarrollamos por la segunda fila
− 1 3 5 7 − 1 2 3 6 c 3 ⇒ c 3 + 2c1 
3 −9 3 2 3 − 6 9 5  c 4 ⇒ c 4 + c1 

2 3 7 7
0  cambios 
3 7 7 3 7
−2 0 0 0  
= 2 ⋅ 2 3 6 = 2 ⋅ 2 3 3 c 3 ⇒ c 3 − c 2  = 2 ⋅ (− 36 − 126 − 81 + 56 ) = −374
−1 2 3 6
−6 9 5 − 6 9 − 4  

3 −6 9 5

x a b x
a x x b
R.7. Calcula el valor de:
b x x a
x b a x
Solución:

Hacemos c1 ⇒ c1 − c 4 , c 2 ⇒ c 2 − c 3 y sacamos factor común a − b

x a b x 0 a−b b x 0 1 b x
a x x b a−b 0 x b 1 0 x b
= = (a − b )2 ⋅ =
b x x a b−a 0 x a −1 0 x a
x b a x 0 b −a a x 0 −1 a x

sumamos a la fila tercera la fila segunda e intercambiamos la primera y la segunda

12-17
Determinantes Tema 12

0 1 b x 1 0 x b
1 0 x b 0 1 b x
= (a − b )2 ⋅ = −(a − b )2 ⋅ =
0 0 2x a + b 0 0 2x a + b
0 −1 a x 0 −1 a x

sumamos a la fila cuarta la fila segunda

1 0 x b

= −(a − b )2 ⋅
0 1
0 0 2x
b
a+b
x
= −(a − b )2 ⋅
2x
a+b
a+b
2x
[
= −(a − b )2 ⋅ 4 x 2 − (a + b )2 . ]
0 0 a+b 2x

R.8. Demuestra que el determinante de Vandermonde de orden tres vale:

1 1 1
a b c = (c − b ) ⋅ (c − a ) ⋅ (b − a )
a2 b2 c2
Solución:

A la 3ª fila le restamos la 2ª multiplicada por a y a la 2ª la 1ª multiplicada por a

1 1 1 1 1 1
a b c =0 b−a c −a desarrollamos por la 1ª columna y sacamos factor común
2 2 2 2 2
a b c 0 b − b ⋅a c − c ⋅a

1 1 1
b−a c −a 1 1
= 0 b−a c −a = = (b − a ) ⋅ (c − a ) ⋅ = (b − a ) ⋅ (c − a ) ⋅ (c − b )
2 2 b ⋅ (b − a ) c ⋅ (c − a ) b c
0 b − b⋅a c −c ⋅a

Cálculo de la matriz inversa. Ecuaciones matriciales

R.9. Halla la matriz inversa de las siguientes matrices utilizando determinantes:


3 −1 2   3 −2 4
 1 2    
a) A =   b) B =  2 − 2 1  c) C =  5 1 11
 − 2 4  3 1 − 2 − 3 7 1 
  
Soluciones:
1 2  1 − 2
a) Calculamos A = = 8 , la matriz traspuesta es A t =   y los adjuntos son
−2 4 2 4 
1 1
1  4 − 2  2 − 
4.
A11 = 4 ; A12 = −2 A21 = 2 A22 = 1 . Por tanto, A −1
=  =
8  2 1   1 1 
 
4 8 
3 −1 2 3 2 3 
t  
b) B = 2 − 2 1 = 18 ; B =  − 1 − 2 1  . Calculamos los adjuntos de esta matriz:
3 1 −2 2 1 − 2 

−2 1 −1 1 −1 − 2
B11 = =3 B12 = − =0 B13 = =3
1 −2 2 −2 2 1

12-18
Determinantes Tema 12

2 3 3 3 3 2
B21 = − =7 B22 = = −12 B23 = − =1
1 −2 2 −2 2 1

2 3 3 3 3 2
B31 = =8 B32 = − = −6 B33 = = −4
−2 1 −1 1 −1 − 2

 1 1 
3 0 3   0 
   6 6 
1    7 2 1 
Por lo tanto B −1 =  7 − 12 1  =  −
18    18 3 18 

  4 1 2
 8 − 6 − 4   9 − − 
 3 9

3 −2 4
c) Como C = 5 1 11 = 0 , la matriz C no tiene inversa.
−3 7 1

 x 2 − 1
 
R.10. Estudia para qué valores de x la matriz A =  1 x 3  no tiene inversa. Después, calcula la
2 3 2 
 
matriz inversa de A para x = 3 .

Solución:
x 2 −1
 x = 1
La matriz no tiene inversa si A = 0 ; A = 1 x 3 = 2x 2 − 7x + 5 = 0 ⇔  5 .
x=
2 3 2 
 2

 3 2 − 1  3 1 2
   
Calculamos la matriz inversa de A =  1 3 3  . A = 2 , A′ =  2 3 3  y los adjuntos son:
2 3 2   − 1 3 2
   

3 3 2 3 2 3
A11 = = −3 A12 = − = −7 A13 = =9
3 2 −1 2 −1 3

1 2 3 2 3 1
A21 = − =4 A22 = =8 A23 = − = −10
3 2 −1 2 −1 3

1 2 3 2 3 1
A31 = = −3 A32 = − = −5 A33 = =7
3 3 2 3 2 3

−3 −7 9 
− 3 − 7 9   
   2 2 2 
1
con lo que A −1 = ⋅ 4 8 − 10  =  2 4 − 5 .
2   
− 3 − 5  −3 −5 7 
 7   2 
 2 2 

R.11. Resuelve la ecuación matricial X ⋅ A = B con:

12-19
Determinantes Tema 12

 0 1 − 1  7 1 − 19 
   
A =  1 0 2  ; B =  − 3 7 11 
−1 1 3   1 7 −9 
   

Solución:

Despejando, X = B ⋅ A −1 . Necesitamos hallar A −1 .


 − 2 4 − 2
−1 1 
Como A = −6 , si se halla la matriz adjunta de la traspuesta se llega a que A =  5 1 1 ,
6 
 −1 1 1 
 19 5 16 
 − 
 7 1 − 19   − 2 4 − 2   3 3 3 
1   
Haciendo X = B ⋅ A −1 se tiene X =  − 3 7 11  ⋅  5 1 1  =  3 1 4 .
6  
  −1 1 1  
 1 7 − 9    23 1 −2 
 
 3 3 3 

Rango de una matriz por medio de determinantes

R.12. Calcula, estudiando sus menores, el rango de las siguientes matrices:

3 2 1 0
1 0 −1 1 3   
  2 5 1 4
a)  4 1 − 1 7 18  b) 
 2 1 1 5 12  3 −1 2 5
   
4 1 4 2 

Soluciones:

a) Como sólo tiene tres filas, el rango es menor o igual que 3. Elegimos un menor de orden 2
1 0
distinto de cero; por ejemplo, el = 1 ≠ 0 y lo orlamos con la fila 3 y las diferentes columnas
4 1
hasta encontrar un menor distinto de cero, si es que lo hay.

1 0 −1 1 0 1
Con f3 y c 3 → 4 1 − 1 = 1 − 4 + 2 + 1 = 0 ; Con f3 y c 4 → 4 1 7 = 5 + 4 − 2 − 7 = 0
2 1 1 2 1 5

1 0 3
Con f3 y c 5 → 4 1 18 = 12 + 12 − 6 − 18 = 0 .
2 1 12
Como no hay ningún menor de orden tres distinto de cero, entonces el rango de la matriz es 2.

b) El rango es menor o igual que 4. Elegimos un menor de orden 2 distinto de cero; por ejemplo, el
3 2
= 11 ≠ 0 y lo orlamos con la fila 3 y las diferentes columnas hasta encontrar un menor
2 5
distinto de cero, si es que lo hay.
3 2 1
2 5 1 = 30 − 2 + 6 − 15 + 3 − 8 ≠ 0
3 −1 2
Al ser distinto de cero, ya podemos decir que el rango es mayor o igual que 3. Orlamos
este menor con la única fila y la única columna que quedan, resulta

12-20
Determinantes Tema 12

3 2 1 0 3 2 1 0
3 2 1 3 2 1
2 5 1 4 −6 3 −7 0 f2 − 2f 4
= = −5 ⋅ − 6 3 − 7 + 2 ⋅ − 6 3 − 7 ≠ 0 .
3 −1 2 5 3 −1 2 5
4 1 4 3 −1 2
4 1 4 2 4 1 4 2

Por tanto el rango de la matriz es 4.

Rango de matrices dependientes de parámetros

k 3 2
 
R.13. Estudia para los distintos valores de k el rango de.  3 2 k 
2 k 3
 
Solución:
k 3 2
Calculamos 3 2 k = 0 ⇔ −k 3 + 18k − 35 = 0 ⇔ k = −5 .
2 k 3
Por lo tanto, si k ≠ −5 el rango es 3. Y si k = −5 , como hay menores de orden dos distintos
de cero, el rango es 2.

1 t t 2 
 
R.14. Estudia el rango de A = 1 1 1  según los valores del parámetro t ∈ R
 
1 −1 1 
 

Solución:
1 t t2
Calculamos. 1 1 1 = 2 − 2 ⋅ t 2 = 0 ⇔ t = ±1 .
1 −1 1
1 1
Por tanto si t ≠ ±1 el rango es tres. Y si t = ±1 , el rango es 2 , pues el menor es
−1 1
distinto de cero para todo t de R.

a + 1 1 −a a 
 
R.15. Estudia el rango de A =  1 a + 1 0 2a  según los valores del parámetro a ∈ R .
 a 1 1 0 

Razona si para algún valor de a existe A −1 .

Solución:

a +1 1 −a
El rango de A es menor o igual que 3. Calculamos: 1 a + 1 0 = a ⋅ (a + 1)2 , luego si a ≠ 0 , o
a 1 1
bien a ≠ −1 el rango de A es tres. Veamos qué pasa si a = 0 y, después, si a = −1 .

 1 1 0 0
 
Si a = 0 , la matriz es A =  1 1 0 0  , donde todos los menores de orden 3 son nulos y
0 1 1 0
 

12-21
Determinantes Tema 12

1 1
encontramos un menor ≠ 0 y por tanto el rango de A es 2.
0 1

 0 1 1 − 1
 
Si a = −1 , queda A =  1 0 0 − 2  , donde suprimiendo la columna segunda, queda
−1 1 1 0 
 
0 1 −1
1 0 − 2 = 1 ≠ 0 . Por lo que el rango de A también es 3.
−1 1 0

Es absurdo pretender que una matriz que no es cuadrada tenga inversa.

12-22
Determinantes Tema 12

Ejercicios propuestos
Determinantes de orden 2 y 3

2 5 −3
P.1. Calcula el valor del determinante: − 2 1 4
2 0 1

P.2. Resuelve las siguientes ecuaciones:


x 3 2 x x − 2 2x 3 1 x
x +1 4
a) =0 b) x + 1 1 0 = 0 c) 5 2 1 =0 d) 2 x − 1 = 17
2 x −1
3 0 −2 −2 1 3 −1 0 x +1

Propiedades de los determinantes

3 x
P.3. A partir del valor de A = = 5 , calcula utilizando las propiedades de los determinantes el
2 y
6 x 3−x x
valor de: a) b) c) A 2 d) 2 ⋅ A
4 y 2−y y

 1 0 3  3 1 7
   
P.4. Dadas las matrices A =  2 − 1 2  y B =  − 1 0 1  comprueba que:
 1 1 1  2 1 3
   
a) A − B ≠ A − B b) A t = A c) α ⋅ A = α 3 A d) A ⋅ I = A

Determinantes de orden superior

P.5. Calcula introduciendo ceros, si es preciso, el valor de:

1 2 2 0 1− α 0 2 0
2 5 3 1 2 0 2−α 0
a) b)
3 8 4 2 3 1 −2 5
4 14 1 4 3 0 0 2−α

a a a a 1+ a 1 1 1
a b b b 1 1+ b 1 1
P.6. Calcula: a) b)
a b c c 1 1 1+ c 1
a b c d 1 1 1 1+ d

−x 1 0 0 0
0 −x 1 0 0
P.7. Demuestra que: 0 0 −x 1 0 = − x 5 + ax 4 + bx 3 + cx 2 + dx + e
0 0 0 −x 1
e d c b a−x
−1 0 0 −1 1
1 −1 0 0 0
P.8. Calcula 0 1 1 0 −1
a 0 0 −d 0
0 b −c 0 0

12-23
Determinantes Tema 12

P.9. demuestra que el determinante de Vandermonde de orden 4

1 1 1 1
a b c d
= (d − c ) ⋅ (d − b ) ⋅ (d − a ) ⋅ (c − b ) ⋅ (c − a ) ⋅ (b − a ) ⋅
a2 b2 c2 d2
a3 b3 c3 d3

Cálculo de la matriz inversa. Ecuaciones matriciales

 1 0 − 1
 
P.10. Dada la matriz A =  0 x 3  averigua para qué valores del parámetro x la matriz A no
4 1 − x
 
tiene inversa. Calcula A −1 cuando x = 2 .

P.11. Calcula las matrices inversas por medio de determinantes de:

 3 1 − 2 12 2 0   1 0 − 1
2 5       
a)   b)  5 2 1  c)  3 1 2  d)  2 0 − 1
3 − 8  4 2 − 3  2 − 1 1 − 6 −1 0 
     

0 3 0
4 − 2 1     1 −3 5 
P.12. Resuelve X ⋅ B + A = C con: A =   , B =  2 0 1  y C =  
5 1 − 3 0 − 3 2  − 2 4 − 6
 

P.13. Resuelve la ecuación matricial A ⋅ X = B siendo:

 1 0 − 2  1 −3 2 
   
A = 0 1 3  y B =  5 7 − 2
 1 0 − 4 −1 7 0 
  

Rango de una matriz por medio de determinantes

P.14. Calcula el rango de:


− 3 1 2 7 
 1 2 1 5  
   2 8 1 − 3
a)  3 5 2 13  b) 
− 2 3 5 4  2 1 4 2 
   
 3 1 − 2 2 

Rango de matrices dependientes de parámetros

P.15. Determina en función del parámetro α el rango de cada una de las siguientes matrices:
α 3 − 1 α α + 3 1   1 −1 α + 3 1 + α 1 0 
       
a)  0 α + 1 2  b)  2 4 − 1 c)  1 α 0  d)  1 1 1
4 0 1  1 −6 0  − 2 1 0   0 1 2α 
   

 2α + 2 3 α  1 1 1  3 − 2 1 
     
e)  4α − 1 α + 1 2α − 1  f)  α 1 α − 1 g)  α 1 α + 2
 5α − 4 α + 1 3α − 4  1 α 1  0 α + 3 4 
   

12-24
Determinantes Tema 12

Soluciones:

34 9
P.1. 58 P.2. a) x = ±3 , b) x = 0 , c) x = −, d) x = 2 y x = − P.3. a) 10, b) 5, c) 25, d) 20
6 4
P.4. a) A − B = −5 , A = 6 y B = −5 luego A − B ≠ A − B , b) trivial,
α 0 3α
(
c) α ⋅ A = 2α − α 2α = α 3 A d) trivial. P.5. a) 0, b) (α − 2) ⋅ α 2 − 3α − 2 )
α α α
P.6. a) − a ⋅ (a − b ) ⋅ (b − c ) ⋅ (c − d ) , b) bcd + acd + abd + abc + abcd P. 7. Se hace
c 4 ⇒ c 4 + x ⋅ c 5 y se desarrolla por f 4 y se repite el proceso hasta llegar a orden 2. P.8.
−d ⋅ b + a ⋅ c P.9. Haciendo (en este orden) f 4 ⇒ f 4 − a ⋅ f3 , f3 ⇒ f3 − a ⋅ f2 , f2 ⇒ f2 − a ⋅ f1 ,
desarrollando por c1 y sacando factor común se llega al determinante de Vandermonde de
− 7 −1 2 
−1 
orden 3. P.10. x = 1, x = 3 A =  12 2 − 3 
− 8 −1 2 
 
 8 1 − 5  3 −2 4   −1 1 0
1 8 5  1  1    
P.11. a)   b)  − 19 1 13  c)  1 12 − 24  d)  6 − 6 − 1
31  3 − 2  9  38 
 − 2 2 − 1  − 5 16 6  − 2
 1 0 
 1 1 1
− − 
 4 2 4  4 0 − 2
1 1  29 − 18 33  1 
P.12. B −1 =  0 0 , X =   . P.13. A −1 =  − 3 2 3 ,
 3  12  15 − 42 3  2
 1 1   1 0 − 1 
 0 
 2 2 
 3 − 13 4 
  19
X =  2 22 − 5  . P.14. a) 2, b) 4 P.15. a) r = 3 ∀α ; b) si α = − →r = 2 , si
1 − 5  7
 1 
19 1 1
α≠− → r = 3 ; c) si α = −3 ∨ α = − → r = 2 , si α ≠ −3 ∧ α ≠ − → r = 3 ; d) si
7 2 2
1 1
α = 1∨ α = − → r = 2 , si α ≠ 1∧ α ≠ − → r = 3 ;
2 2
e) si α = 3 ∨ α = 2 ∨ α = 1 → r = 2 , si α ≠ 3 ∧ α ≠ 2 ∧ α ≠ 1 → r = 3 ; f) si α = 1 → r = 2 , si
α ≠ 1 → r = 3 g) r = 3 ∀α

12-25
Sistemas de ecuaciones lineales Tema 13

Tema 13

Sistemas de ecuaciones lineales

13.1 Definiciones

Se llama ecuación lineal respecto a las n incógnitas x1 , x 2 , x 3 ,K x n a toda igualdad que

puede ser escrita de la forma: a1x1 + a2 x 2 + a3 x 3 + L + a n x n = b , donde a1 ,L a n , b son

elementos de R. A los números a1 ,L a n se les llama coeficientes de las incógnitas

x1 ,L x n , respectivamente, y al número b se le llama término independiente o segundo

miembro de la ecuación.

Ejemplo 13.1 La ecuación 3 x + 2y = 4 es una ecuación lineal respecto de las incógnitas x, y .

Se llama sistema de m ecuaciones lineales con n incógnitas x1 , x 2 , x 3 ,K x n a un conjunto


de m ecuaciones lineales con esas n incógnitas que deben ser verificadas
simultáneamente. Lo escribiremos así:
a11 x1 + a12 x 2 + a13 x 3 + L + a1n x n = c1 
a21 x1 + a22 x 2 + a23 x 3 + L + a 2n x n = c 2 
.
LLLLLLLLLLLL 
a m1x1 + am 2 x 2 + a m3 x 3 + L + a mn x n = c m 

4x − y + z = 1 
Ejemplo 13.2 El sistema  es un sistema de 2 ecuaciones lineales con 3
x + 2 y + 3 z = 2
incógnitas.

13-1
Sistemas de ecuaciones lineales Tema 13

Se llama solución del sistema a todo vector s = (s1 , s 2 , s 3 ,K s n ) de R n que satisface las
m ecuaciones. Esto es, que al sustituir x 1 por s1 , x 2 por s 2 y, así sucesivamente, en
todas las ecuaciones del sistema, se hacen ciertas cada una de ellas. Por tanto, se verifica:

a11s1 + a12 s 2 + a13 s 3 + L + a1n s n = c1 


a21s1 + a 22 s 2 + a 23 s 3 + L + a 2n s n = c 2 

LLLLun
Nota Resolver LL LLLL
sistema esLhallar
L la solución
 o soluciones del mismo, en el caso de que
a m1s1 + a m 2 s 2 + a m3 s 3 + L + a mn s n = c m 
tenga.

4x − y + z = 1 
Ejemplo 13.4 El vector ( 1,2,−1 ) es solución del sistema  , porque para
x + 2 y + 3 z = 2
x = 1, y = 2, z = −1 se satisfacen las dos ecuaciones del sistema.

Todo sistema de m ecuaciones lineales con m incógnitas tal como

a11 x1 + a12 x 2 + a13 x 3 + L + a1n x n = c1 


a21 x1 + a22 x 2 + a23 x 3 + L + a 2n x n = c 2 

LLLLLLLLLLLL 
a m1x1 + am 2 x 2 + a m3 x 3 + L + a mn x n = c m 

puede ser escrito matricialmente de la forma

 a11 a12 L a1n   x1   c 1 


     
 a 21 a 22 L a2 n   x2   c2 
L ⋅ = o, abreviadamente, A ⋅ X = C ,
L L L   K   K 
     
a a L a mn   x n   c m 
 m1 m2

 a11 a12 L a1n   x1 


   
a L a2 n 
 es la matriz de los coeficientes, X =  2  la matriz incógnita y
a22 x
donde A =  21
L L L L M
   
a L ann   
 n1 an 2  xn 

 c1 
 
c 
C =  2  la matriz de los términos independientes.
M
 
c 
 n
 a11 a12 L a1 n c1 
 
a a 22 L a2 n c2 
A la matriz A =  21

, formada al añadir a la matriz A la matriz
L L L L L
 
a L amn c m 
 m1 am 2

columna de los términos independientes, la llamaremos matriz ampliada.

13-2
Sistemas de ecuaciones lineales Tema 13

4x − y + z = 1   4 −1 1  4 −1 1 1
Ejemplo 13.5 En el sistema  , A =   y A ∗ =   son la
x + 2 y + 3 z = 2  1 2 3   1 2 3 2 
matriz de los coeficientes y la matriz ampliada, respectivamente.

13.2 Clasificación de los sistemas de ecuaciones lineales


- Atendiendo a su solución, los sistemas pueden ser:

1) Compatible: cuando tiene solución. Se llama Compatible Determinado si tiene


solución única. Y se llama Compatible Indeterminado si tiene infinitas soluciones.

2) Incompatible: cuando no tiene solución.

- Atendiendo a los términos independientes, los sistemas pueden ser:

1) Homogéneo: si todos los términos independientes son cero.

2) No homogéneo: si existe algún término independiente distinto de cero.


Nota Observa que un sistema homogéneo es siempre compatible ya que (0, 0, 0, 0,L 0 ) es
siempre una solución del sistema. A ésta se le llama solución trivial.

De lo anterior, resulta el siguiente esquema de clasificación:

Determinados ( Solución trivial)

Homogéneos Compatibles

Indeterminados (Infinitas sol.)


Sistemas
Determinados ( Una solución)

Compatibles

No homogéneos Indeterminados ( Infinitas sol.)

Incompatibles ( Sin solución)

Ejemplos

2 x + 3 y = −5 
Ejemplo 13.6 El sistema  es compatible determinado porque tiene una única
7 x + 2y = 8 
solución que es ( 2, − 3 ) .

2 x + 3y − z = 2 
Ejemplo 13.7 Es sencillo comprobar que el sistema  es compatible
4 x + 6 y − 2z = 4 
indeterminado, pues toda solución de la primera ecuación lo es también de la segunda y la
primera tiene infinitas soluciones.

13-3
Sistemas de ecuaciones lineales Tema 13

3x + y = 2 

Ejemplo 13.8 Es sencillo comprobar que el sistema 6 x + 2 y = −2 es incompatible, ya que
x + 2 y = 5 
ninguna solución de la primera ecuación puede satisfacer la segunda.

13.3 Sistemas equivalentes


Para poder resolver un sistema conviene dar la siguiente definición:

Dos sistemas se dicen equivalentes si toda solución del primero lo es del segundo y
viceversa.

El proceso para resolver un sistema es transformarlo en otro equivalente más sencillo. A


continuación se dan algunos tipos de transformaciones que, realizadas en un sistema, dan
lugar a otro equivalente.

Algunas transformaciones que dan lugar a otro sistema equivalente son:

1. Cambiar de orden las ecuaciones del sistema.

2. Sumar a los dos miembros de una ecuación un mismo número.

3. Multiplicar los dos miembros de una ecuación por un mismo número distinto de cero.

4. Sustituir una ecuación por una combinación lineal de dicha ecuación (con coeficiente
distinto de cero) y de las restantes.

5. Suprimir una ecuación que es combinación lineal de otras.

Nota Observa la similitud que existe entre la transformación 4. y las transformaciones que
pueden hacerse mediante el método de Gauss con las filas de una matriz sin que varíe su
rango. De hecho, las transformaciones que mantienen el rango de las matrices asociadas a un
sistema conducen a matrices que corresponden a un sistema equivalente al de partida.

13.4 Sistemas tipo Cramer. Regla de Cramer.

Definición Un sistema se dice que es del tipo Cramer (o sistema de Cramer) si tiene igual
número de ecuaciones que de incógnitas y, además, el determinante de la matriz de
coeficientes es distinto de cero.

Teorema Todo sistema tipo Cramer tiene solución y es única.

Demostración.- Dado un sistema de n ecuaciones con n incógnitas podemos considerar su

13-4
Sistemas de ecuaciones lineales Tema 13

 a11 a12 L a1n   x1   c1 


     
a a 22 L a2 n   x 2   c 2 
forma matricial  21 ⋅ = que, como vimos, equivale a A ⋅ X = C .
L L L L   M   M 
     
a L a nn   x n   c n 
 n1 an 2

Como el sistema es tipo Cramer, A ≠ 0 y, por tanto, existe A −1 con lo que despejando en la
 x1   A11 A21 L An1   c1 
     
−1  x 2  1  A12 A22 L An 2   c 2 
ecuación matricial se tiene que X = A ⋅ C ; es decir   = ⋅ ⋅ ,
M A  L L L L   M 
    
x   A1 n A2 n L Ann   c n 
 n 
lo que implica que el sistema tiene solución y que es única.

Regla de Cramer. Si, en la demostración anterior, se despeja la incógnita x i se tiene


A1 i ⋅ c1 + A2 i ⋅ c 2 + L + An i ⋅ c n
xi =
A
y es fácil ver que
a11 a12 L a1i −1 c1 a1i +1 L a1 n
a 21 a 21 L a 2i −1 c2 a 2i +1 L a2 n
A1 i ⋅ c1 + A2 i ⋅ c 2 + L + An i ⋅ c n =
M M M M M M
a n1 a n 2 L a n i −1 c n a n i +1 L a nn

sin más que desarrollar el determinante por la columna i.

De aquí, podemos afirmar que:

La solución de cada incógnita de un sistema de Cramer es un cociente, en el que el


numerador es el determinante formado al sustituir en la matriz de los coeficientes la columna
correspondiente a los coeficientes de la incógnita por la columna de términos independientes y
el denominador es el determinante de la matriz de coeficientes.

Ejemplos

4 x + 5 y + 3 z = -4

Ejemplo 13.9 Resolvamos el sistema: 4 x + y + 4z = 0 
4 x + 3 y + 3 z = -5
4 5 3
Como │A│= 4 1 4 = 8 , y hay 3 ecuaciones y 3 incógnitas, se trata de un sistema
4 3 3
Cramer cuya solución es:

-4 5 3 4 -4 3 4 5 -4
0 1 4 4 0 4 4 1 0
-5 3 3 - 49 4 -5 3 1 4 3 -5
x= = ; y= = ; z= =6.
8 8 8 2 8

13-5
Sistemas de ecuaciones lineales Tema 13

2 x + y + 4z − t = 1 

Ejemplo 13.10 Resolvamos, usando la Regla de Cramer, el sistema: 3 x + y − z + t = 2
x + y − 3z + 2t = 5
No tenemos el mismo número de ecuaciones que de incógnitas y en principio parece
que no es posible resolverlo por la Regla de Cramer, sin embargo si pasamos los términos con
t al segundo miembro y los consideramos como parámetro queda un sistema de Cramer con
incógnitas x, y, z:

2 x + y + 4z = 1 + t  2 1 4

3 x + y − z = 2 − t  , ya que 3 1 − 1 = 12 ≠ 0 , cuyas soluciones son
x + y − 3z = 5 − 2t  1 1 3

1+ t 1 4 2 1+ t 4 2 1 1+ t
2 − t 1 −1 3 2−t −1 3 1 2−t
5 − 2t 1 − 3 t − 13 1 5 − 2t −3 58 − 10t 1 1 5 − 2t 5t − 5
x= = , y= = , z= = .
12 12 12 12 12 12

13.5 Teorema de Rouché – Fröbenius

Dado un sistema de m ecuaciones con n incógnitas, siendo A la matriz de los coeficientes


y A ∗ la matriz ampliada, la condición necesaria y suficiente para que el sistema tenga
( )
solución es que rango (A ) = rango A ∗ .
Además, si llamamos r a dicho rango, se tiene:
a) Si r = n , el sistema tiene solución única (compatible determinado).
b) Si r < n , el sistema tiene infinitas soluciones (compatible indeterminado).

Demostración.-

Condición necesaria: [⇒]


a11 x1 + a12 x 2 + a13 x 3 + L + a1n x n = c1 
a21 x1 + a22 x 2 + a23 x 3 + L + a 2n x n = c 2 
Matricialmente, el sistema  puede ser escrito del
LLLLLLLLLLLL 
a m1x1 + am 2 x 2 + a m 3 x 3 + L + a mn x n = c m 
modo siguiente:
 a11   a12   a1n   c1 
       
 a21   a   a   c2 
  x1 +  22  x 2 + ... +  2n  xn =  .
 ...   ...   ...   ... 
       
       
 am1   am2   amn   cm 

13-6
Sistemas de ecuaciones lineales Tema 13

 c1 
 
 c2 
Afirmar que el sistema tiene solución equivale a decir que el vector   puede ponerse
 ... 
 
 
 cm 
 a11   a12   a1n 
     
 a21   a22   a2n 
como combinación lineal de los vectores columna  ,   , ... ,   .
 ...   ...   ... 
     
     
 am1   am2   amn 

Por lo tanto, podemos eliminar la columna de términos independientes en el cálculo del


rango de la matriz ampliada y, en consecuencia, rango (A ) = rango A ∗ . ( )
Condición suficiente: [⇐]

( )
Supongamos que rango (A ) = rango A ∗ = r y probemos que el sistema tiene solución.

Sin restar generalidad, podemos suponer que el menor de orden r no nulo que
determina el rango incluye las r primeras ecuaciones y las r primeras incógnitas. Esto quiere
decir que las m − r filas restantes de la matriz ampliada son combinación lineal de las otras,
luego las podemos eliminar, quedando el sistema equivalente

a11 x1 + a12 x 2 + a13 x 3 + L + a1r x r + a1r +1 x r +1 + L + a1n x n = c1 



a21 x1 + a22 x 2 + a23 x 3 + L + a 2r x r + a 2r +1x r +1 + L + a 2n x n = c 2 

LLLLLLLLLLLL 
a r 1x1 + a r 2 x 2 + a r 3 x 3 + L + ar r x r + a r r +1x r +1 + L + a r n x n = c r 

y pasando al segundo miembro los sumandos correspondientes a las n − r últimas incógnitas

a11 x1 + a12 x 2 + a13 x 3 + L + a1r x r = −a1r +1x r +1 − L − a1n x n + c1 



a21 x1 + a22 x 2 + a23 x 3 + L + a 2r x r = −a 2r +1x r +1 − L − a2n x n + c 2 

LLLLLLLLLLLL 
a r 1x1 + a r 2 x 2 + a r 3 x 3 + L + a r r x r = −a r r +1x r +1 − L − ar n x n + c r 

que es un sistema tipo Cramer y, por lo tanto, tiene solución, presentándose dos casos:

1. Si r = n , el segundo miembro se reduce a los términos independientes ci y el sistema


tiene solución única.

2. Si r < n , las r primeras incógnitas quedan en función de las n − r restantes que se


pueden considerar como parámetros y dando a éstos valores arbitrarios se obtienen
infinitas soluciones.

Nota importante . En los sistemas homogéneos los rangos de A y de A ∗ siempre coinciden


sabemos que siempre existe solución. En tal caso, si r = n , la única solución es la trivial y, si
r < n , tiene infinitas soluciones.

13.6 Método de Gauss para la resolución de sistemas lineales

13-7
Sistemas de ecuaciones lineales Tema 13

También llamado de triangulación o de introducción de ceros en cascada. Gauss divide su


algoritmo en dos partes:

1. Triangulación nula: Mediante transformaciones del tipo 4. se introducen ceros del mismo
modo que con el método de Gauss para el estudio del rango de una matriz. Así, si el sistema
tiene solución, se llega a un sistema que presenta alguna de las dos siguientes formas:

a) Igual número de incógnitas que de ecuaciones:


a11 x1 + a12 x 2 + a13 x 3 + L + a1n x n = c1 
a 22 x 2 + a 23 x 3 + L + a2n x n = c 2 
a33 x 3 + L + a3 n x n = c 3  , con todos los aii ≠ 0 .
LLLLLLLLLLLLLLL

a nn x n = c n 

b) Mayor número de incógnitas que de ecuaciones:


a11 x1 + a12 x 2 + a13 x 3 + L + a1n x n = c1 
a 22 x 2 + a 23 x 3 + L + a 2n x n = c 2 
a 33 x 3 + L + a 3 n x n = c 3  , con todos los aii ≠ 0 .
LLLLLLLLLLLLLL

a jj x j + L + a jn x n = c j 

2. Sustitución inversa: Una vez triangulado el sistema, se despeja la incógnita correspondiente


al último coeficiente a jj distinto de cero y se sustituye (su valor o su expresión, según suceda
a) o b) en la ecuación anterior. Después se despeja la incógnita correspondiente al coeficiente
a j −1 j −1 y se vuelve a sustituir ésta en la ecuación anterior y, así, sucesivamente.

Lo acostumbrado es trabajar con la matriz ampliada del mismo modo a como se hace
para estudiar su rango y, conseguida la triangulación, se vuelve a recomponer el sistema.

Ejemplos

Ejemplo 13.9 Veamos el método de Gauss aplicado a la resolución del siguiente sistema:

x + y − z = −2  x + y − z = −2  x + y − z = −2 
  
3 x − y + 2z = 4  ⇔ − 4 y + 5z = 10  (e 2 − 3 ⋅ e1 ) ⇔ − 4 y + 5z = 10  ,
− x + 2y − z = 12 
3 y − 2z = 10  (e3 + e1 ) 
7z = 70 (4e3 + 3 ⋅ e2 )

de la última ecuación se obtiene z = 10 y da comienzo la sustitución inversa. Sustituyendo z en


la segunda ecuación: −4 y + 50 = 10 ⇔ y = 10 y, por último, sustituyendo z e y en la primera,
x = −2 . Luego la solución es la terna ( − 2,10,10 ) .

Otra forma: Veamos cómo se resuelve utilizando la matriz ampliada:

 1 1 -1   x   - 2 
    
el sistema se escribiría:  3 - 1 2   y  =  4 
     
 - 1 2 - 1   z   12 

transformamos la matriz ampliada:

13-8
Sistemas de ecuaciones lineales Tema 13

 1 1 -1 - 2   1 1 -1 - 2   1 1 -1 - 2 
     
 3 - 1 2 4  →  0 - 4 5 10  (f2 − 3 ⋅ f1 ) →  0 - 4 5 10 
     
 - 1 2 - 1 12   0 3 - 2 10  (f3 + f1 )  0 0 7 70  (4e3 + 3 ⋅ e2 )

x + y − z = −2 

y de aquí volvemos a reconstruir el sistema − 4 y + 5z = 10  , que se resuelve repitiendo el
7z = 70 
proceso de la sustitución inversa ya efectuado anteriormente.

Ejemplo 13.10 Resolvamos por el método de Gauss el siguiente sistema de ecuaciones:

x - 2y + 2t = -5
2 y + 4z - 5t = 7 

x + y - 4z + 2t = -8
3 y - 4z = -3

- x - y + 4z - 2t = 8 
Eliminamos la quinta ecuación porque es igual a la tercera multiplicada por (-1). Se trabaja
ahora con la matriz ampliada:

 1 -2 0 2 -5   1 -2 2 -5 
0
   
 0 2 4 -5 7   0 2 4 -5 7 
 ⇔ 
 1 1 -4 2 -8   0 3 - 4 0 - 3  f3 − f1
   
 0 3 -4 0 -3   0 3 -4 0 -3 

como la fila cuarta es igual a la tercera, se suprime:

 1 -2 0 2 -5   1 -2 0 2 -5 
   
 0 2 4 -5 7  ⇔ 0 2 4 -5 7  .
   
 0 3 -4 0 -3   0 0 - 20 15 - 27 2
 3f − 3f 2

Como ya no se puede triangular más, se reconstruye el sistema:

x - 2y + 2t = −5 

2y + 4z − 5t = 7 ,

− 20 z + 15t = −27 
 15t + 27 
se despeja z  z =  de la tercera ecuación y se sustituye en la segunda ecuación, de
 20 
 5t + 4 
donde se despeja la y y =  . Por último, se sustituyen la z y la y en la primera
 5 
−17
ecuación, de donde se obtiene que x = .
5
27 + 15t 4 + 5t -17
La solución es z = ; y= ; x= , con t cualquier número real.
20 5 5
Así pues, se trata de un sistema compatible e indeterminado con infinitas soluciones que se
obtienen dando valores al parámetro t.

13-9
Sistemas de ecuaciones lineales Tema 13

4 x + 5 y + 3z = - 4
4 x + y + 4z = 0 

Ejemplo 13.11 Resolvamos el sistema: 4 x + 3 y + 3z = - 5 
4 x - 3 y + 5z = 4 

- 2y + z = 5 
Hacemos ceros en la matriz A ∗ y separamos con una línea la matriz A:
4 5 3 − 4  4 5 3 − 4
   
4 1 4 0   0 − 4 1 4  f2 − f1
4 3 3 − 5  ⇔  0 − 2 0 − 1  f3 − f1 , como f 4 = 2f 2 suprimimos f 4 y se tiene:
   
4 −3 5 4   0 − 8 2 8  f4 − f1
   
0 −2 1 5  0 − 2 1 5 

 4 5 3 − 4 4 5 3 − 4
     4 5 3 − 4
0 − 4 1 4  0 − 4 1 4   
 0 − 2 0 − 1 ⇔ 0 0  eliminamos f 4 ⇔ 0 − 4 1 4 
1 6 − 2f3 + f 2 0 0 1 6 
     
0 − 2 1 5   0 0 − 1 − 6  − 2f + f
    4 2

( )
Luego, el rango (A) = rango A ∗ = 3 = nº incógnitas : sistema compatible y determinado. Lo
resolveremos por Gauss. Reconstruimos el sistema:

4 x + 5 y + 3z = - 4

− 4 y + z = 4 . Ya que z=6, procediendo por sustitución inversa, resulta:
z = 6
1 49
y = , x=− .
2 8

x+ y - z+t = 4 

Ejemplo 13.12 Resolvamos el sistema: 2 x - y + 3z + 2t = - 1 .
- 4 x + 5 y - 11z - 4t = 11
Se transforma la matriz ampliada para hallar los rangos de A y A ∗

 1 1 −1 1 4 1 1 −1 1 4  1 1 −1 1 4 
     
 2 −1 3 2 − 1 ⇔  0 − 3 5 0 − 9  f 2 − 2f1 ⇔  0 − 3 5 0 − 9 
 − 4 5 − 11 − 4 11   0 9 − 15 0 27  f + 4f 0 0 0 0 0  f3 + 3f 2
    3 1 

( )
por lo que rango (A) = rango A ∗ = 2 < 4 = nº incógnitas . Se trata de un sistema compatible
indeterminado. Dejando x e y en el primer miembro, y recomponiendo el sistema a partir de la
última matriz, resulta:

 x + y = 4+ z - t 9 + 5z -3 + 2z + 3t
 , de donde y = , x= .
 - 3 y = -9 - 5 z 3 -3

 2x - y = 1

Ejemplo 13.13 Resolvamos el sistema:  x + 3 y = - 2 .
 5 x - 4y = 7

13-10
Sistemas de ecuaciones lineales Tema 13

2 -1 1
2 −1
Como ≠0, rango (A ) = 2 . Por otra parte, como 1 3 - 2 = 24 ≠ 0 , el
1 3
5 -4 7
( )
rango A ∗ = 3 y, por lo tanto, el sistema es incompatible.

13.7 Discusión y resolución de sistemas con parámetros


En ocasiones se plantean sistemas en los que algunos de los coeficientes de las
incógnitas o los términos independientes son desconocidos y se quieren analizar las posibles
soluciones del sistema en función de esos valores desconocidos, a los que se llaman
parámetros.
Existen dos métodos de análisis y de resolución de estos sistemas que se
corresponden con las dos formas de estudiar el rango de sus matrices asociadas:

Primer método: método de Gauss

Se utiliza la definición de rango como máximo número de líneas (filas o columnas)


linealmente independientes. Generalmente, se dan los siguientes pasos:

• Se triangula la matriz ampliada.


• Para los valores de los parámetros que hacen cero alguno de los términos de la
ampliada se estudian los rangos de A y A ∗ .
• Se resuelve el sistema por sustitución inversa, para aquellos valores de los parámetros
que lo hacen compatible (determinado o indeterminado).

Segundo método: método de Rouché

Se utiliza la definición de rango a partir de los determinantes (mayor menor no nulo). La


organización del estudio es similar a la anterior:

• Se estudia el rango de A y A ∗ para cada uno de los valores de los parámetros.


• Se comparan los rangos y se concluye la compatibilidad o incompatibilidad del sistema.
• Se resuelve el sistema, generalmente, utilizando la Regla de Cramer.

Ejemplos

αx + y + z = 1 

Ejemplo 13.14 Discute según los valores del parámetro α el sistema: x + αy + z = α  y
x + y + αz = α2 
resuélvelo para algún valor de α que lo haga compatible.

a) Método de Gauss.

Manejamos A y A ∗ conjuntamente, conviene que el parámetro quede lo más abajo y lo


más a la derecha posible, por tanto pasamos la primera ecuación al tercer lugar e introducimos
ceros en cascada.

13-11
Sistemas de ecuaciones lineales Tema 13

 1 1 α α2  1 1 α α 2  1 1 α α2 
    
 1 α 1 α  ⇔ 0 α − 1 1− α α − α  f 2 − f1 ⇔  0 α − 1
2
1− α α−α 2
 f 2 − f1
   3   
α 1 1 1 0 1− α 1− α2 1 − α  f3 − αf1 0 0 2 − α − α2 − α − α + α + 1 f3 − αf1
3 2
     

Por tanto:
Para α ≠ 1 , α ≠ −2 hay ceros en ( )
cascada y rango (A) = rango A ∗ = 3 = nº incógnitas :
Sistema compatible determinado.
1 1 1 1
 
Para α=1, las matrices son:  0 ( )
0 0 0  luego rango (A) = 1 = rango A ∗ : Sistema
0 0 0 0 

compatible indeterminado, con soluciones dependientes de 2 parámetros.
1 1 − 2 4 
 
Para α = −2 , las matrices son ∗
( )
A A =  0 − 3 3 − 6  , se observa que el
0 0 0 3 

( )
rango (A ) = 2 y el rango A ∗ = 3 y el sistema es incompatible.

b) Método de Rouché.

Se forma el mayor determinante posible con la matriz de coeficientes y se hallan los


valores de α que lo anulan:

α 1 1
1 α 1 = α 3 − 3α + 2 , resolviendo α 3 − 3α + 2 = 0 por Ruffini da α = 1 (doble) y α = −2 .
1 1 α
Por tanto:
( )
Para α ≠ 1 , α ≠ −2 rango (A) = rango A ∗ = 3 = nº incógnitas : Sistema compatible
determinado.
( )
Para α=1, rango (A) = 1 = rango A ∗ : Sistema compatible indeterminado, con soluciones
x = 1− s − t

dependientes de 2 parámetros: y = s .
z = t

Para α = −2 , vemos en A un menor de orden dos distinto de cero por lo que el rango de A es 2.
− 2 1 1 1  -2 1 1
 
Además, A ∗ =  1 − 2 1 − 2  , tiene un menor de orden tres 1 - 2 - 2 ≠ 0 . Por
 1 1 − 2 4 
  1 1 4
( )
tanto rango A ∗ = 3 , y el sistema es incompatible.

αx + 2 y = 1 
x + 2y = 3
Ejemplo 13.15 Discute y resuelve el sistema  , según los valores de α y β,
- x + 3 y = 2
2 x + βy = 0
resolviéndolo cuando sea posible.
Como hay un menor de orden dos distinto de cero, el rango de A es 2
independientemente de los valores de α y β. Formamos los mayores determinantes posibles
con la matriz ampliada y hallamos los valores de α y β que los anulan:

13-12
Sistemas de ecuaciones lineales Tema 13

α 2 1 1 2 3
1 2 3 = - 5α - 5 ; α = -1; - 1 3 2 = - 5β - 10 ; β = -2 .
-1 3 2 2 β 0
( )
Para α ≠ −1 ó β ≠ −2 , rango (A) = 2 < rango A ∗ = 3 : Incompatible.
( )
Para α = −1 y β = −2 se tiene rango (A) = 2 = rango A ∗ = nº incógnitas : sistema
compatible determinado. Eliminamos las dos últimas ecuaciones y queda el sistema:
− x + 2y = 1
 , cuya solución es x = 1, y = 1 .
x + 2y = 3 

x - αy + z = 4 

Ejemplo 13.16 Discutamos, según los valores de α y β, el sistema: 2 x - αy + 2z = 6  .
βx + y + z = 4
Calculamos el mayor determinante posible, que coincide con
A = α(1 − β ) = 0 → α = 0 o β = 1 .
( )
Caso I: si α = 0 ⇒ rango (A) = 2 y = rango A ∗ = 3 Sistema incompatible.
 1 -α 1  1 -α 
   
Caso II: si β = 1 , entonces: A =  2 - α 2  → rang (A) = rang  2 - α 
   
 1 1 1   1 1 
1 -α 1 -α
Estudiaremos sus menores de orden dos: = α ; pero = 1+ α , luego para
2 -α 1 1
todo α hay un menor de orden 2 distinto de cero, ya que no se anulan a la vez. Es decir,
rango (A ) = 2 ,  ∀α ∈ R .
Veamos cuál es el rango de A ∗ :
 1 -α 1 4   1 -α 4  1 -α 4
   
Como rang  2 - α 2 6  = rang  2 - α 6  , 2 - α 6 = 0 → α = −1 , distinguiremos:
   
 1 1 1 4   1 1 4  1 1 4
( )
Caso IIa.: si β = 1 y α = −1 s ⇒ rango (A) = rango A ∗ = 2 < nº incógnitas . Sistema compatible e
indeterminado.
( )
Caso IIb.: si β = 1 y α ≠ −1 ⇒ rango (A) = 2 y = rango A ∗ = 3 . Sistema incompatible.
Caso III: si α ≠ 0 y β ≠ 1 ⇒ rango (A) = rango A ( ) = 3 = nº incógnitas .

Sistema compatible
determinado.

x+y +z=0 

Ejemplo 13.17 Resuelve según los valores de α: αx + 3 y + z = 0
4 x + 5 y − z = 0 

( )
Como en un sistema homogéneo rango (A ) = rango A ∗ , estudiaremos A , que vemos
que se anula sólo para α = 2 . Entonces:
( )
Para α ≠ 2 , rango (A ) = rango A∗ = 3 = nº incógnitas : solución trivial, x = y = z = 0 .
Para α = 2, ( )
rango (A ) = rango A∗ = 2 < nº incógnitas : sistema compatible
indeterminado con soluciones dependiendo de un parámetro.

13-13
Sistemas de ecuaciones lineales Tema 13

Eliminamos la última ecuación y pasamos la z al otro miembro, quedando:


x+y =z 
 , que es fácil de resolver. Obteniendo: x = 4z , y = −3z que se puede expresar
2 x + 3y = −z 
 x = 4t

también de la forma: y = −3t con t ∈ R .
 z=t

13.8 Eliminación de parámetros


Definición Un sistema de ecuaciones paramétricas es aquel en el que las incógnitas están
escritas en función de unos determinados parámetros, de manera que al dar valores a los
parámetros se obtienen las distintas soluciones del sistema.

Ejemplos

 x = 2 - 3α + β

Ejemplo 13.18 El sistema  y = 3 + 3β + γ es un sistema de ecuaciones paramétricas. Para
 z = - 2 + 6β + 2γ

cada valor de los parámetros α , β , γ obtenemos los valores de x, y, z correspondientes.

Llamamos eliminación de parámetros al proceso de obtener un sistema de ecuaciones


lineales o cartesianas que tenga las mismas soluciones que un sistema paramétrico dado.

Ejemplo 13.19 Eliminar los parámetros del sistema del ejemplo anterior es trata de obtener
unas ecuaciones que ligan x, y, z en las que no aparezcan los parámetros.

Nota De alguna manera, podría considerarse que se trata de recorrer el camino contrario al
que hemos seguido hasta ahora y a partir de las soluciones de un sistema compatible
indeterminado llegar éste o a uno equivalente.

Generalmente, los pasos a dar son:

• Dejar en un miembro sólo los parámetros.


• Considerar el sistema donde los términos independientes son tanto las incógnitas
como las constantes, mientras que los parámetros hacen el papel de incógnitas.
• Calcular el rango de la matriz de los coeficientes de los parámetros.
• Obligar a que el rango de la matriz ampliada coincida con el rango anterior.

El número de ecuaciones que ligan las incógnitas iniciales viene dado por

nº ecuaciones = nº incógnitas − rango matriz coeficient es

Nota Ante la pregunta: ¿qué se puede decir cuando el número de incógnitas es igual al rango
de la matriz de los coeficientes?, ya que en tal caso no resulta ninguna ecuación, la respuesta
es: todo vector de R n , con n = nº incógnitas , es solución del sistema.

Ejemplos

13-14
Sistemas de ecuaciones lineales Tema 13

 x = 2 - 3α + β

Ejemplo 13.20 Eliminemos los parámetros α, β, γ del sistema  y = 3 + 3β + γ .
 z = - 2 + 6β + 2γ

Primero, se dejan solos los parámetros a los que consideramos incógnitas del sistema:

- 3α + β = x − 2   −3 1 0   −3 1 0 x −2 
    
3β + γ = y − 3  , de donde A =  0 3 1  y A ∗ =  0 3 1 y − 3  .
6β + 2γ = z + 2  0 6 2  0 6 2 z+2 
   

-3 1 0
−3 1
Se calcula el rango de A, como ≠0 y 0 3 1 = 0 , se tiene que
0 3
0 6 2
rang(A) = 2 .
−3 1 x −2
Se obliga a que rang A ( ) = 2 que es equivalente a que

0 3 y − 3 = 0 , de donde
0 6 z+2
se obtiene la ecuación 2y − z − 8 = 0 .
x = 1− t

Ejemplo 13.21 Eliminemos el parámetro t del sistema y = −2 + 3t .
 z = 1 − 2t

Primero, se dejan solos los parámetros a los que consideramos incógnitas del sistema:
 − t = x −1  −1   −1 x −1 
   ∗  
 3t = y + 2 , de donde A =  3  y A =  3 y +2.
− 2t = z − 1  −2  − 2 z −1 
    
Es evidente que rang(A) = 1 , ya que sólo tiene una columna.
( )
Se obliga a que rang A ∗ = 1 :
x -1 -1 x -1 -1
como los menores de orden dos han de ser cero, =0; = 0 , se obtienen
y+2 3 z -1 - 2
3x + y − 1 = 0 
las ecuaciones cartesianas  .
− 2 x + z + 1 = 0
 x = 2 - 3α + β

Ejemplo 13.22 Eliminemos los parámetros α, β, γ del sistema  y = 3 + 3β + γ .
 z = -2 - α +γ

- 3α + β = x - 2 

Se dejan solos los parámetros 3β + γ = y - 3  .
- α + γ = z + 2
Se estudia el rango de A: como A = −9 − 1 = −10 → rang(A) = 3 .
− 3 1 0 x − 2
 
Se obliga a que rang A( )∗ ∗
= 3 , siendo A =  0 3 1 y − 3  . Pero, observamos que
 −1 0 1 z + 2
 
( )
rang A ∗ = 3 ∀x , y , z ∈ R . Luego, en este ejemplo, no hay ecuaciones cartesianas. Lo cual
significa que el sistema paramétrico anterior genera todo R 3 .

13-15
Sistemas de ecuaciones lineales Tema 13

Ejercicios resueltos

Resolución de sistemas

R.1. Resuelve, por el método de Gauss:

x + y + z + t = 4
x + 2 y − 3 z + t = 2
 2 x − y + 3z − t = 0 
a) 2 x + y − z + 2t = 1 b) 
3 x − 2y + z + 2t = 3 
x − 2y + z + t = 2 
− x + y − z + t = 2 

Soluciones:

a) Obtenemos las matrices asociadas al sistema e introducimos ceros en cascada.

 1 2 − 3 1 2 1 2 − 3 1 2  1 2 − 3 1 2 
     
( ∗
)
A A =  2 1 − 1 2 1  ⇒  0 − 3 − 5 0 − 3  f 2 − 2f1 ⇒  0 − 3 − 5 0 − 3  ⇒
 1 − 2 1 1 2 0 − 4 4 0 0  f − f 0 1 − 1 0 0  1
    3 1   − f3
4
1 2 − 3 1 2  x + 2 y − 3 z + t = 2
   3
⇒ 0 − 3 − 5 0 − 3 que corresponde al sistema − 3 y − 5z = −3  de donde z = −
0 0 2
 2 0 − 3  3f3 + f 2 2z = −3 

3
sustituyendo en la segunda ecuación y despejando y = − sustituyendo en la primera y pasando t
2
1 − 2t  1 − 2t 3 3  
al 2º miembro queda x = . La solución es:  ,− ,− , t  ∀t ∈ R  .
2  2 2 2  

 1 1 1 1 4  1 1 1 1 4 
   
 2 − 1 3 −1 0   0 − 3 1 − 3 − 8  f 2 − 2f1
b) Las matrices asociadas son  ⇒ dividimos
3 −2 1 2 3   0 − 5 − 2 − 1 − 9  f3 − 3f1
   
−1 1 −1 1 2   0 2 0 2 6  f 4 + f1

1 1 1 1 4   1 1 1 1 4
   
0 1 0 1 3   0 1 0 1 3  f 2 − 2f1
por 2 la 4ª fila y la pasamos a la 2ª  ⇒ ya
0 − 3 1 − 3 − 8   0 0 1 0 1  f 3 − 3f1
   
0 − 5 − 2 − 1 − 9 0 0 − 2 4 6 f + f
    4 1
x + y + z + t = 4
y + t = 3 
tenemos ceros en cascada. El sistema asociado es:  y la solución por sustitución
z = 1
− 2z + 4t = 6 
inversa es: (x , y , z, t ) = (0,1,1,2)
x + 2y + z = 0  x + 2 y − 3z + t = 0
 
R.2. Resuelve utilizando la regla de Cramer: a) 2 x + y + 3z = 9  b) 2 x + y − z + t = 1 
3 x + 5 y + z = −4 3 x + y + 4 z − t = 2
Soluciones:
1 2 1
a) A = 2 1 3 = 7 ≠ 0 luego es un sistema de Cramer. Su solución es:
3 5 1

13-16
Sistemas de ecuaciones lineales Tema
13

0 2 1 1 0 1 1 2 0
9 1 3 2 9 3 2 1 9
−4 5 1 3 −4 1 3 5 −4
x= = 1, y = = −2 , z = =3
7 7 7
b) La matriz de los coeficientes no es cuadrada, si pasamos t al segundo miembro queda
x + 2 y − 3 z = −t  1 2 −3

2 x + y − z + = 1 − t  con 2 1 − 1 = −14 ≠ 0 podemos resolverlo por la regla de Cramer:
3 x + y + 4z = 2 + t  3 1 4
−t 2 −3 1 −t −3 1 2 −t
1− t 1 −1 2 1− t −1 2 1 1− t
2+t 1 4 9 − 7t 3 2+t 4 7t − 3 3 1 2+t 7t + 1
x= = , y= = , z= = .
− 14 14 − 14 14 − 14 14

x - y + z=1
R.3. Añade una ecuación al sistema  de modo que el sistema obtenido sea
2 x - y - z = 2
compatible indeterminado. Razona la respuesta.

Solución:

1 -1
Como ≠ 0 el sistema inicial ya es Compatible Indeterminado. Por eso, tan sólo resta añadir
2 -1
una combinación lineal de las dos ecuaciones dadas; por ejemplo, la suma de ambas:
x - y + z=1

2 x - y - z = 2 . Pasar la solución a forma paramétrica es eliminar las incógnitas x e y, dejándolas
3 x − 2y = 3 
x - y = 1− z 
en función de z. Utilizaremos la regla de Cramer. El sistema es  , por lo que, como
2x - y = 2 + z
1 -1 1- z - 1 1 1- z
=1 x = = 1+ 2z análogamente y = = 2z quedando las ecuaciones
2 -1 2+ z -1 2 2+ z
paramétricas:

 x = 1+ 2α

 y = 2α .
 z=α

Discusión y resolución de sistemas con un parámetro

3 x + 3 y − z = 0

R.4. Dado el sistema homogéneo: − 4 x − 2y + mz = 0 Calcula m para que tenga solución
3 x + 4 y + 6z = 0
distinta de la trivial y resuélvelo.

Solución:

Para que tenga solución distinta de la trivial, el determinante de la matriz de los coeficientes debe
ser cero. Veamos para qué valores de m se cumple esto.
3 3 -1
46 46
- 4 - 2 m = 46 - 3m = 0 → m = . Por tanto, tendrá solución distinta de la trivial si m = .
3 6
3 4 6

13-17
Sistemas de ecuaciones lineales Tema 13

3 3 3x + 3y - z = 0 
Como ≠ 0 resolveremos el  . Restando las dos ecuaciones (2ª y 1ª) queda:
3 4 3x + 4y + 6z = 0 
y + 7z = 0
22z
y = −7z . Sustituyendo en la primera queda: 3 x − 21z − z = 0 → x = . El conjunto solución
3
será:
 22z  
 , − 7z, z  con z ∈ R  .
 3  

x + y + az = 1 

R.5. Discute en según los valores del parámetro a el sistema: ax + y + z = 3 y resuélvelo
x + 2 y + az = 5
para a = 2 .

Solución:

Lo discutimos y resolveremos por los dos métodos.

1. Método de Gauss.

 1 1 a 1 1 1 a 1
   
Las matrices asociadas son:   
a 1 1 3 ⇔ 0 1− a 1− a 2 3 − a  f 2 − af1 Debemos
   
 1 2 a 5 0 1 0 4  f3 − f1
 
detenernos en los valores de a que nos den ceros que afecten al rang (A ) y ver qué pasa con
( )
rang A∗ .

( )
- Si a ≠ {− 1,1} → rang (A ) = 3 = rang A∗ = nº incógnitas ⇒ Sistema compatible y determinado.

 1 1 1 1
  rang (A ) = 2
- Si a = 1 las matrices son  0 0 0 2  y ⇒ Sistema incompatible.
  rang A∗ = 3( )
0 1 0 4

 1 1 − 1 1  1 1 − 1 1
    rang (A ) = 2
- Si a = −1 las matrices quedan  0 2 0 4  ⇔  0 2 0 4  ⇒ Sistema
    rang A∗ = 3 ( )
0 1 0 4  0 0 0 4  2f3 − f 2
incompatible.

x + y + 2z = 1 

Lo resolvemos para a = 2 el sistema inicial es equivalente a − y − 3z = 1  que por sustitución
y = 4
1 5
inversa comenzando por el valor de y nos da x = , y = 4, z = − .
3 3

2. Método de Rouché.

1 1 a
 a =1
Estudiamos rang(A ) , a 1 1 = 0 → a 2 − 1 = 0 →  por tanto al haber sólo 3 ecuaciones:
a = −1
1 2 a

13-18
Sistemas de ecuaciones lineales Tema
13

( )
- Si a ≠ {− 1,1} → rang (A ) = 3 = rang A∗ = nº incógnitas ⇒ Sistema compatible y determinado.
1 1 1
( )
- Si a = 1 rang (A ) = 2 y rang A = 3 pues encontramos

1 1 3 = −2 ≠ 0 ⇒ Sistema
1 2 5
incompatible.
1 1 1
( )
- Si a = −1 rang (A ) = 2 y rang A = 3 pues encontramos

− 1 1 3 = 4 ≠ 0 ⇒ Sistema
1 2 5
incompatible
x + y + 2z = 1 

Lo resolvemos para a = 2 el sistema con el que tenemos que trabajar es el inicial 2 x + y + z = 3
x + 2 y + 2z = 5
Lo resolvemos con la regla de Cramer. A = 3 y se tiene:

1 1 2 1 1 2 1 1 1
3 1 1 2 3 1 2 1 3
5 2 2 1 1 5 2 1 2 5 5
x= = , y= = 4, z = =− .
3 3 3 3 3

x − y = 5
y + z = a 
R.6. Discute según los valores del parámetro a el sistema:  . Resuélvelo cuando
x − 2z = 3 
2 x − 3z = a 
sea compatible.

Solución:

Lo discutimos y resolveremos por los dos métodos.

1. Método de Gauss.

1 −1 0 5 1 −1 0 5 
   
0 1 1 a 0 1 1 a 
Obtenemos las matrices asociadas A A ∗ ( ) =
1 0 −2 3
⇔
0 1 −2 − 2  f 3 − f1

   
2 0 − 3 a   0 2 − 3 a − 10  f − 2f
   4 1

1 −1 0 5  1 −1 0 5 
   
0 1 1 a  0 1 1 a 
⇔ ⇔ a la vista de las matrices:
0 0 − 3 − 2 − a  f3 − f 2 0 0 −3 −2−a 
   
 0 0 − 5 − a − 10  f − 2f 0 0 0 2a − 20  3f 4 − 5f3
  4 2 

rang (A ) = 3
- Si a ≠ 10  ⇒ Sistema incompatible.
( )
rang A∗ = 4

( )
- Si a = 10 → rang (A ) = 3 = rang A∗ = nº incógnitas ⇒ Sistema compatible y determinado.

x−y=5   x = 11
 
El sistema inicial es equivalente a y + z = 10  es inmediato ver que y = 6
− 3z = −12 z = 4

13-19
Sistemas de ecuaciones lineales Tema 13

2. Método de Rouché.

1 −1 0 
  1 −1 0
0 1 1 
Estudiamos rang (A ) : A =  y 0 1 1 = 3 ≠ 0 por tanto rang (A) = 3 ∀a ∈ R
1 0 − 2
  1 0 −2
2 0 − 3
 
1 −1 0 5
0 1 1 a
( )
La matriz ampliada es cuadrada de orden 4 y rang A ∗ = 3 si
1 0 −2 3
= 0 calculamos el

2 0 −3 a
determinante introduciendo ceros en la segunda columna.

1 −1 0 5 1 −1 0 5
1 1 a+5
0 1 1 a 1 0 1 a + 5 f 2 + f1
= = 1 −2 3 = −2a + 20 = 0 → a = 10 y en conclusión:
1 0 −2 3 1 0 −2 3
2 −3 a
2 0 −3 a 2 0 −3 a

rang (A) = 3
- Si a ≠ 10  ⇒ Sistema incompatible.
( )
rang A ∗ = 4

( )
- Si a = 10 → rang (A ) = 3 = rang A ∗ = nº incógnitas ⇒ Sistema compatible y determinado.

La resolución del sistema puede hacerse por la regla de Cramer o por cualquier otro método. Se
x−y=5 

puede elegir el sistema y + z = 10  con las tres primeras ecuaciones pues A = −3 ≠ 0 y se llega,
x − 2z = 3 
por supuesto, a la misma solución.

ax + y + z + t = 1 

R.7. Discute en según los valores del parámetro a el sistema: x + a y + z + t = 2
x + y + z + at = a 

Solución:

Lo discutimos por el método de Gauss. Intercambiamos la 1ª y la 3ª ecuaciones .

 1 1 1 a a 1 1 1 a a 
   
Obtenemos: ( ∗
)
A A =  1 a 1 1 2 ⇔ 0 a − 1 0 1 − a 2 − a  f 2 − f1 ya se pueden
 a 1 1 1 1  0 1 − a 1 − a 1 − a 2 1 − a 2  f − af
    3 1
analizar los rangos. Se tiene:

rang (A ) = 1
- Si a = 1  ⇒ Sistema incompatible.
( )
rang A ∗ = 2

( )
- Si a ≠ 1 rang (A ) = 3 = rang A∗ ⇒ Sistema compatible indeterminado con soluciones
dependientes de un parámetro. Si se pasa t al segundo miembro puede comprobarse que las
soluciones son:
2−a a 2 (t − 1) + a (t − 1) − 2t + 3
x = t − 1, y = + t, z = .
a −1 1− a

13-20
Sistemas de ecuaciones lineales Tema
13


x + 2y + 3z = 2

R.8. Discute en según los valores del parámetro m el sistema: .
2 x + 5 y + 4 z = −1
2 
x + 3 y + m z = 3m 
Resuélvelo cuando sea compatible y determinado.

Solución:

Utilizamos el método de Gauss.

Obtenemos (A A∗ )
1 2 3 2  1 2 3 2  1 2 3 2 
     
2 5 4 − 1 ⇔ 0 1 −2 − 5  f2 − 2f1 ⇔  0 1 −2 −5  así que:
 1 3 m2 
3m   2   2 
  0 1 m − 3 3m − 2  f3 − f1  0 0 m − 1 3m + 3  f3 − f2

rang (A ) = 2
- Si m = 1  ⇒ Sistema incompatible.
( )
rang A ∗ = 3

( )
- Si m = −1 rang (A ) = 2 = rang A ∗ ⇒ Sistema compatible indeterminado con un grado de
libertad.

( )
- Si m ≠ {− 1,1} → rang (A ) = 3 = rang A ∗ = nº incógnitas ⇒ Sistema compatible y determinado.

x + 2y + 3z = 2 

Resolvemos y − 2z = −5 donde hemos simplificado dividiendo la 3ª ecuación por m + 1 y por

( )
m2 −1 z = 3 
 21 6 3 
sustitución inversa, es fácil comprobar que: (x , y , z ) = 12 − ,−5 + , .
 m −1 m − 1 m − 1

ax + y + z = 1 

R.9. Discute en según los valores del parámetro a el sistema: x + ay + z = 1  . Resuélvelo
x + y + az = −2
cuando sea compatible y determinado.
-
Solución:

Lo discutimos por el método de Rouché.

Estudiamos rang (A ) .
a 1 1
a = 1 (doble)
1 a 1 = a 3 − 3a + 2 = 0 → 
1 1 a a = -2

En consecuencia:

( )
- Si a ≠ {− 2,1} → rang (A) = 3 = rang A ∗ = nº incógnitas ⇒ Sistema compatible y determinado.

13-21
Sistemas de ecuaciones lineales Tema 13

− 2 1 1 1
 
- Si a = −2 las matrices son  1 − 2 1 1 la columna de los términos independientes
 
 1 1 − 2 − 2 
coincide con la 3ª luego no aumenta el rang A ∗ así que: ( )
( )
rang (A) = 2 = rang A < 3 = nº incógnitas ⇒ Sistema compatible e indeterminado.

1 1 1 1
  rang (A ) = 1 1 1
- Si a = 1 las matrices son 1 1 1 1  , pues se encuentra ≠ 0 así que el
  ( )
rang A ∗ = 2 1 −2
1 1 1 − 2 
sistema es incompatible.

Lo resolvemos para a ≠ {− 2,1} utilizando la regla de Cramer.


1 1 1 a 1 1 a 1 1
1 a 1 1 1 1 1 a 1
−2 1 a a2 + a − 2 1 1 −2 a 1 1 1 −2 −2
x= = = , y= = , z= =
2
(a − 1) (a + 2) (a − 1) (a + 2) 2 a −1 2
(a − 1) (a + 2) a − 1 2
(a − 1) (a + 2) a − 1

2x + y - z = 1 

R.10. Discute, según los valores de t ∈ R el sistema: tx - y + 2z = 3  y resuélvelo, si es
3 x - 2tz = t - 1
posible, para t = −1 .

Solución:

Empleamos el método de Rouché.

Consideremos las matrices asociadas al sistema:

2 1 −1 1 
 
(A )
A∗ =  t − 1 2 3 .
 3 0 − 2t t − 1
 

1 -1
Estudiamos el rango de A y A ∗ : Como ≠ 0, el rg (A) ≥ 2, ∀t. Veamos cuándo puede
-1 2
ser tres:

2 1 -1
- 4 ± 16 - 24
t -1 2 = 2 t 2 + 4t + 3 = 0 → t = ∉ ℜ por lo que rang (A ) = 3 ∀t ∈ R . Por lo que el
4
3 0 - 2t

sistema es siempre Compatible y determinado. Aplicando la regla de Cramer, obtendremos la


solución:

1 1 -1 2 1 -1 2 1 1
3 -1 2 -1 3 2 -1 -1 3
0 0 2 3 -2 2 3 0 -2
x= = - 10, , y = = 25, z = =6.
1 1 1

13-22
Sistemas de ecuaciones lineales Tema
13

x+ y + z=1 

R.11. Discute según los valores de k y resuelve cuando sea indeterminado: x + y + kz = k 

x + y + kz = k 3 

Solución:

Empleamos el método de Rouché .


1 1 1 1
 
Obtenemos la matriz de los coeficientes y la matriz ampliada: 1 1 k k  . Estudiamos los
 
1 1 k 
 k3 
1 1 1
menores de orden 3. 1 1 k = 0, ∀ k .
1 1 k

1 1 1
1 k k = k 4 - k 3 - k 2 + k = 0 ↔ k = 0, k = 1, k = -1 . Por tanto:
1 k k3

( )
- Si k = 0 → rang (A ) = rang A ∗ = 2 ⇒ S. Compatible indeterminado.
 x=α
y + z = 1- x  
el sistema sería:  , con solución: y = −α ∀α ∈ R .
y = -x   z =1

- ( )
Si k = 1 → rang (A ) = rang A ∗ = 1 ⇒ S. Compatible e indeterminado. El sistema sería:
 x=α

x + y + z = 1 , con solución:  y = β ∀α, β ∈ R .
z = 1 − α − β

-

( )
- Si k = −1 → rang (A) = rang A ∗ = 2 ⇒ S. Compatible e indeterminado.
 x=α
y + z = 1- x  
el sistema sería:  , con solución: y = −α ∀α ∈ R .
y - z = -1 - x   z =1

( )
- Si k ≠ {1,0,−1} → rang (A ) = 2 < rang A ∗ = 3 ⇒ S. Incompatible.

Discusión y resolución de sistemas con dos parámetros

mx + ny + z = 1

R.12. Discute el sistema para los distintos valores de m y n: x + mny + z = 1
x + ny + mz = 1
Solución:
m n 1 1
 
Consideremos la matriz de los coeficientes y la matriz ampliada:  1 mn 1 1 .
 1 n m 1
 

13-23
Sistemas de ecuaciones lineales Tema 13

m n 1
( )
Calcularemos 1 mn 1 = 0 → n ⋅ m 3 − 3m + 2 = 0 → n = 0, m = 1, m = −2 .
1 n m

 m 0 1 1
 
I) Para n = 0 , queda:  1 0 1 1 y por tanto:
 
 1 0 m 1

( )
- Si m = 1, rang (A) = rang A ∗ = 1 ⇒ S. Compatible e indeterminado.

m 1 1
m 1
- Si m ≠ 1 , calculamos 1 1 1 = (m - 1 )2 ≠ 0. Y = m - 1 ≠ 0 Por tanto, rang(A) = 2 y
1 1
1 m 1
rang A ( ) = 3 ⇒ S. Incompatible.

II) Para n ≠ 0 :
1 n 1 1
 
 
( )
- Si m = 1, queda: 1 n 1 1 , rang(A) = rang A ∗ = 1 ⇒ S. C. indeterminado.
1 n 1 1

- 2 n 1 1
 
- Si m = −2 , queda:  1 - 2n
 
( )
1 1 , donde, rang(A) = 2 pero rang A ∗ = 3 ya que
 1 n - 2 1

-2 1 1
1 - 2 1 ≠ 0 ⇒ S. Incompatible.
1 1 1

( )
III) Para n ≠ 0 y m ≠ {1,−2} ⇒ rang (A) = rang A∗ = 3 ⇒ S. Compatible Determinado.

R.13. Calcular a y b para que el siguiente sistema sea compatible y determinado:


ax + 2y = 1
x + 2y = 3 

- x + 3 y = 2
2 x + by = 0 

Solución:

( )
Será Compatible y determinado cuando rang (A ) = rang A ∗ = 2 .

 a 2 1
 
∗  1 2 3 1 2
Consideremos las matrices A y A :   . Como ≠ 0 ⇒ rang (A ) = 2 ∀a, b ∈ R .
- 1 3 2 -1 3
 
 2 b 0

13-24
Sistemas de ecuaciones lineales Tema
13

a 2 1 1 2 3
Entonces, para que rang A ( ) = 2 , obligaremos a que:

1 2 3 = - 1 3 2 = 0 , de donde se
-1 3 2 2 b 0
obtiene que : a = −1 y b = −2 . Es decir, el sistema será Compatible Determinado si, y sólo si
a = −1 y b = −2 .

Eliminación de parámetros

 x = 2 + 3 β1 + β 2

y = 3 - 2 β1 + β2
R.14. Elimina los parámetros β1 y β 2 en el siguiente sistema: 
z = β1 + β2
 t = 1- β + β
 1 2
Solución:

Se dejan los parámetros que queremos eliminar en un miembro, quedando un sistema de


 3 1 x − 2
 
 − 2 1 y − 3
ecuaciones de incógnitas β1 y β 2 cuyas matrices asociadas son:  . Como el
1 1 z 
 
 −1 1 t −1 
 
3 1
menor ( )
≠ 0 el rang (A) = 2 , por lo que tenemos que obligar a que el rang A∗ = 2 . Para ello
-2 1
haremos cero los dos menores posibles de orden tres. Y éstos serán los que nos determinarán las
dos ecuaciones cartesianas.
3 1 x-2
- 2 1 y - 3 = 0 → 3 x + 2 y - 5z - 12 = 0
1 1 z
3 1 x-2
3 x + 2y - 5z - 12 = 0 
- 2 1 y - 3 = 0 → x + 4 y - 5t - 9 = 0 quedando el sistema: .
x + 4 y - 5t - 9 = 0
-1 1 t -1

 x = 2 + 3 β1 - β 2 + 2 β3

R.15. Elimina los parámetros β1 , β 2 y β 3 en el siguiente sistema:  y = 3 + β1 + 5 β2 + 6 β3
 z= 2- β +β
 1 2
Solución:
 3 −1 2 x − 2
 
(∗
)
Consideramos las matrices del sistema: A A =  1 5 6 y − 3  . Estudiaremos el rango de
−1 1 0 z − 2
 
3 -1
la matriz de los coeficientes, como ≠ 0 , el rango es como mínimo 2. Veamos si es tres,
1 5
3 -1 2
calculamos: 1 5 6 = 0 , luego el rango de A es 2, y debemos obligar que el rango de A ∗ sea
-1 1 0
también 2. Para esto, debemos hacer cero el menor de orden tres que queda, obteniendo así la
única ecuación cartesiana:

13-25
Sistemas de ecuaciones lineales Tema 13

3 -1 x - 2
1 5 y - 3 = 0 → 6 x - 2 y + 16 z - 38 = 0 .
-1 1 z-2

 3 β1 + β2 + β 3 = 1

 β1 - β2 + 2 β3 = 2
R.16. Elimina los parámetros β1 y β 2 en el siguiente sistema: 
 β1 + β2 - β3 = 1
3 β + 2 β + β = 3
 1 2 3

Solución:

Dejaremos en un mismo miembro sólo los parámetros β1 y β 2 . Las matrices del sistema serán:

 3 1 1− β3 
 
 1 − 1 2 − 2β 3  3 1
 1 1 1 + β  . Como ≠ 0 el rango de A es 2.
 3  1 -1
3 2 3 − β 
 3 

Así que debemos obligar que el rango de A ∗ sea dos. Para ello, basta hacer cero los dos menores
de orden tres:
3 1 1 - β3

1 - 1 2 - 2 β3 = 0 → - 6 - 2 β3 = 0 → β3 = -3

1 1 1+ β3
3 1 1 - β3
13
1 - 1 2 - 2 β3 = 0 → - 13 + 5 β3 = 0 → β3 =
5
3 2 3 − β3

Lo cual es absurdo, porque β 3 no puede tener dos valores distintos al mismo tiempo.
Luego, es imposible eliminar los parámetros que nos pedían.

13-26
Sistemas de ecuaciones lineales Tema
13

Ejercicios propuestos

Resolución de sistemas.

Resuelve, por el método de Gauss, los siguientes sistemas:

3 x − 4 y + z − 4t = −4  3 x − 3 y + 4z = −3  x + y − 2z + t = −1
  
P.1. − x − y + 2z − 3t = −4  P.2. 2 x − y + z = −4  P.3. 2 x − y + 3z + 6t = 2 
7x − 7y − 5t = −4 3 x + y + 2z = −4  3 x + y − z + 6t = 0 
x +y +z+t =0 
3 x + 5y = 1  x + y + z = 6
  x − y + z − t = 12 
P.4. 2 x − y = 2 P.5. 2 x − 3 y + 5z = 5  P.6. 
x + y − z + t = −8
2 x + 25 y = 0 3 x − 2 y + 6z = 8 
x − y − z + t = −6
x + y − z = 6
 2 x − y + z − t = 4
P.7. 2 x − 3 y + z = 3  P.8. 
3 x − 2y + z − 2t = 6 
6 x − y − z = 7 

− 5 x − 3y + z = 0 

P.9. Resuelve utilizando la regla de Cramer: 2 x + 3 y − 4z = 4 
3x − 4z = −3
Resuelve:

− 2 x − 3 y − 3z + 3t = 1  − x + 4 y + 4z = 3 
− 2 x + 4z − 10t = 4  6 x − y = −4
P.10.  P.11. 
3 x + 3 y + z + 2t = −3 − 5 x − 3 y − 4z = 1 
x + 3 y + 5z − 8t = 1  y + z = 0 

Discusión y resolución de sistemas.

Discute los siguientes sistemas y resuélvelos cuando sean compatibles:

3 x − 4y = −1 3 x − 2y + z = 2 
2 x + 2y = 4  x + y − 5z = −12 
P.12.  P.13. 
2 x + 2y = 11 6 y − 7z = −9 
− 9 x − 2y = 11 − 2 x + y − 6z = −18 

Discusión y resolución de sistemas con un parámetro.

x + y =1 

P.14. Dado el sistema de ecuaciones lineales ty + z = 0  determina t, de modo que: a)
x + (1 + t )y + tz = t + 1
el sistema tenga solución única; b) el sistema tenga infinitas soluciones; c) el sistema no tenga
solución.

Calcula a para que sea compatible el sistema y resuélvelo:

x+y +z=2 
2 x − 3 y + z = 0
x + 2y − 3z = 8  
P.15.  P.16. x − a y − 3z = 0
ax − y − z = 1 
5 x + 2 y − z = 0
x − y + z = −2

13-27
Sistemas de ecuaciones lineales Tema 13

2x + y + z + w = 0
ax − 2y + z + 4w = 0
P.17. Discute y resuelve el sistema para los distintos valores de a: 
3 x − y − z + 4w = 0
− 2 x + 4 y − z + 9w = 0

x − 3 y + 5z = 2 

P.18. Discute el sistema para los distintos valores de m: 2 x − 4 y + 2z = 1  .
5 x − 11 y + 9z = m 

P.19. Halla el valor de k para que el siguiente sistema tenga sólo la solución trivial:
x − 5 y = 0

4 x − 6 y + 2z = 0 
5 x − 4 y + kz = 0

P.20. Halla el valor de m para que el sistema homogéneo siguiente tenga solución distinta de la
x + y + mz = 0

trivial: 3 x + 2 y + 4mz = 0
2 x + y + 3z = 0

Discute y resuelve los siguientes sistemas en función del parámetro m:

3 x + y + 2z = 1 − m  2 x − 5 y + 3 z = 0
 
P.21. (1 + m ) x + 2 y + z = m  P.22. x − y + z = 0
mx − y + z = 1 − m  3 x + my + z = 0

x + 2y − 5z + mt = 0
(m − 1) x + (m − 1) y = m  
P.23.  P.24. 2 x − 3 y + 2z + 3t = 0
mx + (m − 1) y = m − 1
mx − 7 y + z − 6t = 0

x + 2y − z = 8 

P.25. Dado el sistema 2 x − 3 y + z = −1 a) Halla el valor de k que hace el sistema compatible;
3 x − y + kz = 5 
b) Halla el valor de k para el cual el sistema es compatible y además z = −1 ; c) Para el valor de k
hallado en b), resuelve el sistema.

2 x + y − 4z = p 
2y − z = p 
P.26. Calcula el valor de p que hace compatible el sistema: 
y +z=6 
3 x − 2z = 11

6 x + 18 y − mz = 0 

P.27. Estudia según los valores del parámetro m el sistema: 7 x − 2y − 4z = 0  y resuélvelo
4 x + 10 y − 6z = 0 
cuando tenga solución distinta de la trivial.

13-28
Sistemas de ecuaciones lineales Tema
13

5 x − 11y + 9z = k 

P.28. Discute y resuelve en caso de compatibilidad el siguiente sistema: x − 3 y + 5z = 2 
2 x − 4 y + 2z = 1 

P.29. Estudia, en función del parámetro a, la compatibilidad del siguiente sistema, y resuélvelo en
2 x + 4 y + a z = 2

uno de los casos en que sea compatible: x + y + z = 0
2 x + y + 2z = 2

x + y + az = a 2 

P.30. Discute el siguiente sistema según los valores del parámetro a: x + a y + z = a 
ax + y + z = 1 

P.31. Sea el sistema


2
( )
m − 1 x + (m + 1) y = m + 1
2
 . a) Discútelo, según los posibles valores de m.
(m + 1) x + (m − 1) y = m + 1
b) Resuélvelo en caso de que sea compatible determinado.

P.32. Discute y resuelve, en el caso de ser compatible, el sistema:


ax − y + z = 1 
(a + 3) x + (a + 1) z = 1 + a 
2y + az = 0 

P.33. Discute el sistema según los valores del parámetro t. Resuélvelo cuando sea posible.
2 x + 3 y + tz = 0 

ty − z = 0 
tx + ty + tz = 0 

Sistema con dos parámetros.

P.34. Estudiad, en función de los parámetros a y b, la compatibilidad del siguiente sistema:


3 x + 2y − z = a 

2 x − y + bz = −1
5 x + 8 y − 9z = 3 

Eliminación de parámetros.

 x = 12 − β1 + β 2 + β3

P.35. Elimina los parámetros β1 y β 2 en el siguiente sistema:  y = 3 + β1 + β3
 z= 2- β +β
 1 2

 β1 + β 2 + β3 + β 4 + β5 = 3

P.36. Elimina β1 y β 2 si es posible, en:  β1 + 2β 2 − β3 + β 4 − β5 = 2
 2β − β + β − 2 β + β = 5
 1 2 3 4 5
Soluciones:

 12 + 7z − 8t 16 + 7z − 13t    5 1 3
P.1.  , , z, t ∀z, t ∈ R  . P.2. − , ,  .
 7 7    2 2 2
P.3. {(− 2t ,1 − t ,1 − t ,t )∀t ∈ R} . P.4. Incompatible. P. 5. Incompatible. P.6. (2,−1,4,−5 ) .

13-29
Sistemas de ecuaciones lineales Tema 13

 21 41   25 122 1 
7.  − 4,− ,−  . P.8. {(2 − z,−z − t , z,t )∀z, t ∈ R} . P.9. − , ,− 
 2 2  21 63 7 
 3 − 7z + 13t  
P.10.  − 2 + 2z − 5t , , z, t ∀z , t ∈ R  . P.11. (− 3,−1,4,14 ) . P.12. Incompatible.
 3  
P.13. (1,2,3 ) . P.14. si t ∈ R − {0,1} Comp. Det., si t = 0 Comp. Indet., si t = 1 Incomp. P.15.
a = 2 (1,2,−1) . P.16. Compatible ∀a ∈ R , si a ≠ −8 sol. Trivial, si a = −8 Comp. indet.
 z 7z  
 , , z ∀z ∈ R  . P.17. si a ≠ 11 sol. Trivial, si a = 11 Comp. indet. {(x ,2 x ,−3 x ,− x )∀x ∈ R} .
 19 19  
P.18. si m ≠ −4 Incompatible, si m = −4 Comp. Indet. P.19. k ≠ 3 P.20. m = 1 P.21. si m = 1
Incompatible, si m ≠ 1 comp.. Det. P.22. m ≠ 3 sol. Trivial, si a = 3 Comp. indet.
 − 2z z  
 , , z ∀z ∈ R  P.23. si m = 1 Incomp. Si m ≠ 1 Comp. Det. P.24. Comp. Indet. ∀m ∈ R
 3 3  

x=
11z(m + 15 )
, y=
(
z 2m 2 + 13m + 75 ), t=
11z(m − 7 )
. P.25. a) k ≠ 0 b) k = 2 c)
2 2
3m − 8m + 63 3m − 8m + 63 3m 2 − 8m + 63

(3,2,−1) P.26. p = 6 . P.27. si m = 10 Comp. indet.  2z , z , z ∀z ∈ R  . P.28. si k = 4 Comp.


 3 3  
 5 3  
indet.  − + 7z,− + 4z, z ∀z ∈ R  si k ≠ −4 Incompatible. P.29. si a = 2 Incomp. Si a ≠ 2
 2 2  
Comp. Det.

P.30. si a ∈ R − {1−
, 2} Comp. Det., si a = 1 Comp. Indet., si a = −2 Incomp. P.31. si m ∈ R − {0,−1}
m2 + m + 2 m2 − m − 2
Comp. Det., si m = −1 Comp. Indet., si m = 0 Incomp. x = , y =− . P.32. si
4m 4m
1 
a ∈ R − {1,2,−1} Comp. Indet., si a = −1  ,0,0  ., si a = 1 ó a = −2 Incomp. P.33. si t = 0 ó t = 1
 2 
1
Comp. Indet., si t ∈ R − {0,1} sol. trivial (0,0,0 ) P.34. si b ≠ 3 Comp. Det., si b = 3 y a = Comp.
3
1
Indet., si b = 3 y a ≠ Incompatible. P.35. todo R 3 P.36. 4 − 7 β3 − 4 β 4 − 7 β5 = 0 .
3

13-30
Espacio afín y euclídeo Tema 14

Tema 14

Vectores en el espacio

14.1 Introducción
Con este tema iniciamos el primero de los tres que se dedican a la Geometría. A lo largo
de ellos se podrá apreciar la estrecha unión que existe entre los conceptos algebraicos vistos y las
nuevas nociones geométricas. De hecho, todos los temas de álgebra y geometría que aparecen en
el libro pertenecen a una porción de las Matemáticas conocida por el nombre de Álgebra Lineal.
Se empieza con el concepto de espacio afín R 3 , cuyos elementos esenciales son puntos,
rectas y planos. Después se introduce un producto escalar que permite introducir los conceptos de
distancia y ángulo. Finalmente, se dan los productos vectorial y mixto con el fin de abordar
problemas de cálculo de áreas y volúmenes.
La idea de que los alumnos puedan usar cuanto antes estas herramientas justifica que
hayamos antepuesto este tema al de las nociones de plano y recta.

14.2 El espacio afín R3

En el tema 10 se vio que el conjunto R 3 era un R-espacio vectorial, ahora se verá que puede
ser considerado a la vez como un espacio afín cuyo espacio vectorial asociado es él mismo.

De manera informal puede decirse que un espacio afín es un conjunto tal que cada par de
elementos determina un vector del espacio vectorial asociado.

Definición Sea V un R-espacio vectorial. Se llama espacio afín asociado al espacio vectorial V
a la terna (Ω,→,V ) donde Ω es un conjunto no vacío y → una aplicación
Ω × Ω→V
(A, B ) a AB
llamada aplicación vector, que satisface los siguientes axiomas:
1. Para todo A de Ω y para todo v de V existe un único B de Ω tal que v = AB .
2. Regla del paralelogramo (ley de Chasles): si A, B y C son de Ω , entonces
AB + BC = AC .

Nota A los elementos del espacio afín se les llama puntos para distinguirlos de los elementos del
espacio vectorial a los que, como se dijo en el tema 10, se les llama vectores.

14-1
Espacio afín y euclídeo Tema 14

Nota Si un vector v de V es de la forma v = AB , se dice que A es el origen del vector y B su


extremo.

Ejemplo 14.1 Si consideramos la aplicación que a cada par de elementos A = (a1 ,a 2 ,a3 ) y
3
B = (b1 ,b2 ,b3 ) de R le hace corresponder el vector AB = (b1 − a1 ,b2 − a 2 ,b3 − a3 ) , es fácil
3
comprobar que satisface la definición 14.1 y que, por lo tanto, R es un espacio afín.

3
Por ser R un espacio afín satisface las siguientes propiedades:
3
Para todo A, B, C, D de R se verifica:
1. AB = 0⇔ A = B .
2. AB = −BA .
3. Si AB = CD , entonces AC = BD .

Dimensión Se llama dimensión del espacio afín a la dimensión del espacio vectorial
asociado.

3
En el caso de R , su dimensión es tres.

3
Nota A partir de aquí, los elementos de R pueden considerarse tanto como puntos de un espacio
afín o como vectores de un espacio vectorial. Para distinguir el uso que hagamos de ellos
escribiremos letras mayúsculas para referirnos a los puntos, y letras minúsculas o dos letras
mayúsculas bajo una flecha cada vez que los tratemos como vectores.

14.3 Sistema de referencia


3
Como es interesante representar geométricamente los elementos de R se introduce el
concepto de sistema de referencia que dará paso a la utilización de coordenadas cartesianas para
3
todo punto de R .

3
Definición Se llama sistema de referencia (cartesiano) de R a todo par {O;Β} donde O es un
3 3
punto de R llamado origen del sistema de referencia y Β una base de R .

3
En el caso del espacio afín R , el sistema de referencia que tomaremos está formado por
3
el punto O = (0,0,0 ) y la base canónica de R , Β = {e1 = (1,0,0 ),e2 = (0,1,0 ),e3 = (0,0,1)} . En física, y
r r r
en ocasiones también aquí, se denotan los vectores e1 , e 2 , e 3 por i , j , k , respectivamente.
z

e3
e2
y
e1

14-2
Espacio afín y euclídeo Tema 14

3
Coordenadas. Definición Se llaman coordenadas cartesianas de un punto P de R respecto
de un sistema de referencia a las componentes del vector OP respecto de la base del sistema.

3
En el caso de tomar la base canónica, las coordenadas cartesianas de un punto P de R
tal que OP = x1e1 + x 2 e2 + x 3 e3 son P ≡ (x1 ,x 2 ,x 3 ) . Esto es, las componentes de P como vector
3
del espacio vectorial R coinciden con sus coordenadas cartesianas como punto del espacio afín
3
R .
OP
z

e3
e2
y
e1

Al vector OP se le llama vector de posición del punto P.

A partir de las coordenadas cartesianas de dos puntos A y B, pueden obtenerse las


coordenadas del vector AB en la base de referencia del modo siguiente.

Si A ≡ (a1 ,a 2 ,a3 ) y B ≡ (b1 ,b2 ,b3 ) ,

como AB = AO + OB , entonces AB = −OA + OB = (b1 − a1 )e1 + (b2 − a 2 )e2 + (b3 − a3 )e3 , por
lo que las componentes del vector AB son (b1 − a1 ,b2 − a2 ,b3 − a3 ) .

14.4 Producto escalar


El producto escalar es uno de los instrumentos idóneos para estudiar las propiedades
métricas en el espacio ya que permite introducir las nociones de distancia y ángulo.

Intuitivamente hemos introducido el sentido de un vector como el que establece el orden


señalado por su origen y su extremo; con el producto escalar se introduce un segundo elemento: el
módulo.

En el tema siguiente se verá el tercer elemento: la dirección.

3
Definición Dados dos vectores v = (x1 ,y 1 ,z1 ) y w = (x 2 ,y 2 ,z 2 ) de R se define el producto
escalar de esos dos vectores, y se denota v ⋅ w , al número x1 x2 + y1 y2 + z1 z2 . Escribiremos:
v ⋅ w = x1 x 2 + y 1 y 2 + z1 z 2 .

14-3
Espacio afín y euclídeo Tema 14

3
Proposición Para todo v ,w , u de R y para todo α , β de R se verifica que:
• v ⋅ v ≥ 0 y v ⋅ v = 0⇔v = 0
• v ⋅w = w ⋅v
• (αv + βw ) ⋅ u = α(v ⋅ u ) + β(w ⋅ u )

3
Espacio vectorial euclídeo. Al espacio vectorial R con el producto escalar así definido, se
3
le llama espacio vectorial euclídeo. Y teniendo en cuenta que R es también un espacio afín,
3
nos referiremos a R como espacio afín euclídeo.

La palabra euclídeo viene de Euclides y al designar de esta manera al espacio estamos


aceptando el modo en que éste lo concibió.

Aquí, por tanto, se verifica su polémico axioma V o axioma de las paralelas que viene a
decir que por un punto exterior a una recta pasa una, y sólo una, recta paralela a ésta.
r 3 r
Módulo de un vector. Dado un vector a = (a1 , a2 , a3 ) de R se llama módulo del vector a (o
r r r
también norma o longitud del vector) al número a = a ⋅ a = a12 + a2 2 + a3 2 .

Un vector se llama unitario si tiene módulo 1.

r r
Ejemplo 14.2 Si a = (1 ,1 , 2) , entonces a = 12 + 12 + 2 2 = 6 .

r r 1 r
Observa que dado un vector v , el vector w = r v tiene módulo unidad y, además, tiene la
v
r
misma dirección y sentido que v .

r r 3
Propiedades. Sean v y w de R , entonces se verifican las siguientes propiedades:
• v ≥0 y v =0 ⇔v =0.
• α ⋅v ≤ α w , para todo α de R.
• Desigualdad de Schwarz: v ⋅ w ≤ v w .
• Desigualdad triangular: v + w ≤ v + w .
2 2 2
• Teorema de Pitágoras: v + w = v + w ⇔ v ⋅w = 0
14.3 Distancia y ángulo

14-4
Espacio afín y euclídeo Tema 14

Distancia entre dos puntos. Definición La distancia entre dos puntos P ≡ (a1 , b1 , c1 ) y
3
Q ≡ (a 2 , b2 , c 2 ) de R es el módulo del vector PQ .

Entonces, se tiene que d (P ,Q ) = PQ = PQ ⋅ PQ = (a1 − a 2 )2 + (b1 − b2 )2 + (c1 − c 2 )2 .

Ejemplo 14.3 Dados los puntos P (1,3,5 ) y Q(2,1,7 ) la distancia entre ambos

d (P ,Q ) = PQ == (1 − 2)2 + (3 − 1)2 + (5 − 7)2 =3.

Nota La desigualdad de Schwarz se puede escribir de una manera equivalente, como sigue:
v ⋅w
v ⋅ w ≤ v w ⇔ − v w ≤ v ⋅ w ≤ v w ⇔ −1 ≤ ≤1 .
v w
Esto permite dar la siguiente definición:

3
Ángulo de vectores. Definición Dados dos vectores v y w de R el ángulo, ∠( v , w ) que
v ⋅w
forman esos vectores es cos (∠( v ,w )) =
v w
.

Nota Si v = (x1 , y 1 , z1 ) y w = (x 2 , y 2 , z 2 ) , es fácil deducir la


fórmula anterior. Basta considerar el triángulo cuyos lados son
los dos vectores y el segmento que une los extremos de éstos.
En dicho triángulo conocemos los lados v , w y la distancia
entre los extremos que no es otra que
2 2 2
(x 1 − x 2 ) + (y 1 − y 2 ) + (z1 − z 2 ) . Y ya que el ángulo
buscado es el opuesto a este último lado, aplicando el teorema
del coseno sale inmediatamente la fórmula anterior.

Interpretación geométrica del producto escalar:

De la anterior definición se deduce que u ⋅ v = u v cos (∠(u , v )) .

Ejemplo 14.4 El ángulo formado por los vectores u = (3,2,−1) y v = (4,−3,5 ) es:
12 - 6 - 5 7
cos (∠( u ,v )) = = ⇔ ∠( u ,v ) = 87 o 50' 2' ' .
14 ⋅ 50 70

Definición Dos vectores v y w se dicen ortogonales (perpendiculares) si su producto escalar es


cero. En tal caso, se escribe v ⊥ w .

14-5
Espacio afín y euclídeo Tema 14

Definición Se llama base ortogonal a aquella cuyos vectores son ortogonales dos a dos. Se
dirá base ortonormal a toda base ortogonal formada por vectores unitarios.

3
Ejemplo 14.5 Es sencillo probar que la base canónica de R es una base ortonormal.

Nota Es fácil probar que cualquier familia de vectores ortogonales es linealmente independiente.

14.5 Producto vectorial


Vamos a introducir una operación entre dos vectores cuyo interés reside en que el
resultado va a ser un vector que es ortogonal a los dos dados.
r r 3
Definición Dados dos vectores a = (a1 ,a2 ,a3 ) y b = (b1 ,b2 ,b3 ) de R , se define su producto
r r  a2 a3 a3 a1 a1 a2 
vectorial como el vector a ∧ b =  , ,  . Se suele escribir como
 b2 b3 b3 b1 b1 b2 
r r r
i j k
r r
a ∧ b = a1 a 2 a3 .
b1 b2 b3

r r
Ejemplo 14.6 Dados dos vectores a = (1,3,7 ) y b = (5,−2,−1) su producto vectorial es el vector
r r  3 7 7 1 1 3 
a∧b =  , ,  = (11, 36, − 17 )
 − 2 −1 −1 5 5 − 2 
 

Por tratarse de un determinante es fácil comprobar las siguientes propiedades:

r r r 3
Propiedades del producto vectorial Sean u , v y w dos vectores de R , entonces se verifica:
r r r r
• u ∧ v = −v ∧ u (propiedad anticonmutativa).
r r r r r r r
• u ∧ (v + w ) = u ∧ v + u ∧ w (distributiva).
• (αvr ) ∧ wr = vr ∧ (αwr ) = α(vr ∧ wr ) , para todo α ∈R .
r r
• u ∧u = 0 .
r r r r
• u ∧ v = 0 ⇔ {u ,v }es linealmente dependiente.

14-6
Espacio afín y euclídeo Tema 14

Proposición (definición geométrica de producto vectorial)


r r r r
1. El vector a ∧ b es ortogonal a los vectores a y b .

a1 a 2 a3
r r r r
2. Si a = (a1 ,a 2 ,a3 ) , b = (b1 ,b2 ,b3 ) y , a ∧ b = (c1 ,c 2 ,c 3 ) entonces b1 b 2 b3 > 0 .

c1 c 2 c3
r r r r r v
3. a ∧ b = a ⋅ b ⋅ sen ∠ a, b ( ( ))

Demostración.-

a1 a2 a3
r r r a2 a3 a3 a1 a1 a2
( )
1. a ⋅ a ∧ b = a1 + a2 + a3 = a1 a2 a3 = 0 .
b2 b 3 b3 b1 b1 b2
b1 b2 b3

r r r r
2. Por 1. sabemos que a ∧ b es ortogonal a los vectores a y b , luego el sistema {a , b , a ∧ b } es
a1 a 2 a3

linealmente independiente y, por lo tanto, b1 b2 b3 > 0 .

c1 c 2 c3

r r2 r2 r2 r r
3. Probamos previamente la igualdad a ∧ b = a ⋅ b − a ⋅ b ( ) 2

Si desarrollamos el primer miembro y ordenamos los sumandos se tiene


2 2 2
r r 2  a2 a3   a3 a1   a a2 
a∧b =   +  + 1  que operando nos lleva a la expresión:
b b3   b3 b1  b b 
 2  1 2 

r r2 2 2 2 2 2 2 2 2 2 2 2 2 2 2
a ∧ b = a 2 ⋅ b3 + a3 ⋅ b 2 + a3 ⋅ b1 + a3 ⋅ b1 + a1 ⋅ b3 + a1 ⋅ b 2 + a 2 ⋅ b1 −

− 2(a 2 a3 b 2 b3 + a1a3 b1b3 + a1a 2 b1b 2 ) .

Calculando ahora el valor del segundo miembro se tiene

r2 r2 r r
a ⋅ b − a⋅b ( ) = (a
2
1
2 2 2
)( 2
+ a 2 + a3 ⋅ b1 + b2 + b3
2 2
) − (a b
1 1 + a 2 b 2 + a3 b3 )
2
y operando en esta
expresión es fácil comprobar que coincide con la de desarrollar el primer miembro.

Basándonos en esta igualdad probaremos el punto 3. Se tiene:

r r2 r2 r2 r r r2 r2 r2 r2 r v r2 r2 r v
a ∧b = a ⋅ b − a⋅b ( ) 2
( ( ))
= a ⋅ b − a ⋅ b ⋅ cos 2 ∠ a, b = a ⋅ b ⋅ 1 − cos 2 ∠ a, b ( ( ( ))) =

14-7
Espacio afín y euclídeo Tema 14

r2 r2 r v r r r r r v
( ( )) y en consecuencia
= a ⋅ b ⋅ sen 2 ∠ a, b ( ( ))
a ∧ b = a ⋅ b ⋅ sen ∠ a, b

Esta última proposición puede utilizarse, si se prefiere, como definición de producto


vectorial y a partir de ella se debería demostrar como teorema la definición que hemos utilizado.

Para poder dar la interpretación geométrica del producto vectorial conviene recordar que
dado un sistema linealmente independiente de vectores, éste tiene la misma orientación que la
base del sistema de referencia cuando el determinante formado por sus componentes es positivo.
En tal caso, se dice que el sistema dado tiene sentido dextrógiro (regla del sacacorchos).

Como consecuencia de la anterior proposición surge la Interpretación geométrica del


r r
producto vectorial: El producto vectorial de dos vectores a y b es otro vector de módulo
r r r r r v
( ( ))
a ∧ b = a ⋅ b ⋅ sen ∠ a, b , dirección perpendicular al plano que contiene los vectores y cuyo
sentido viene dado por la regla del sacacorchos (o del pulgar de la mano derecha).

Proposición 14.6 (Aplicación del producto vectorial al cálculo de áreas)


r r r r
1. u ∧ v es el área del paralelogramo determinado por u y v .

AB∧ AC
2. El área del triángulo de vértices A , B y C viene dada por .
2

Ejemplo 14.7 Para hallar el área del triángulo de vértices A(1,1,0), B(0,1,0) y C(2,2,3), hallamos
primeramente AB = ( − 1 , 0 , 0 ) y AC = (1 , 1 , 3 ) , después, aplicamos la fórmula.
(− 1,0 , 0 ) ∧ (1,1, 3 ) 10
Área = = unidades cuadradas.
2 2

14-8
Espacio afín y euclídeo Tema 14

Nota Para calcular el área de un polígono, basta descomponerlo en suma de triángulos y aplicar
en cada uno de ellos el resultado 2. de la proposición anterior.

Ejemplo 14.8 Para hallar el área del cuadrilátero de vértices A(2,1,0), B(0,2,0), C(-3,0,0) y
∧ ∧
D(0,-1,0), descomponemos el cuadrilátero en los triángulos ABC y ACD . El área del cuadrilátero
será la suma de las áreas de los dos triángulos.
Como AB ∧ AC = ( 0 , 0 , 7 ) y AC ∧ AD = ( 0 , 0 , 8 ) , el área del cuadrilátero será

AB∧ AC AD ∧ AC 7 + 8 15
+ = = unidades cuadradas.
2 2 2 2

14.6 Producto mixto


r r r 3
Definición Dados dos vectores a = (a1 ,a2 ,a3 ) , b = (b1 ,b2 ,b3 ) y c = (c1 ,c 2 ,c 3 ) de R , se define
rrr r r r
[ ] (
el producto mixto de esos tres vectores como a,b,c = a ⋅ b ∧ c . )

a1 a2 a3
rrr
[
Observa que es un número que también se puede escribir como a,b,c = b1 b2 b3 . ]
c1 c 2 c 3

Proposición 14.7 (Aplicación del producto mixto al cálculo de volúmenes)


3
El producto mixto de tres vectores de R es igual, salvo el signo, al volumen del
paralelepípedo construido sobre ellos.

Demostración.-

Teniendo en cuenta la figura siguiente y que el volumen del paralelepípedo es el área de la


base por su altura, se tiene
r r
área de la base = b ∧ c , en virtud del apartado 1. de la proposición 14.6,
r r r r r r
altura = h = a ⋅ cos α , pues h es la proyección de a sobre w , siendo w = b ∧ c y
rr
( )
α = ∠ b ,c .
r r r r r rrr
r r
(
Consecuentemente el volumen V = b ∧ c ⋅ a ⋅ cos(∠(a,w )) = a ⋅ b ∧ c = a,b,c . ) [ ]

14-9
Espacio afín y euclídeo Tema 14

Consecuencia: volumen de una pirámide de base triangular

Como el volumen de una pirámide es igual a un tercio del área de la base por la altura, si
consideramos la pirámide de la figura razonando de modo análogo a la demostración de la
proposición anterior, se tiene:
r r rrr
1 b ∧c r r r r r [ ]
a,b,c
volumen = ⋅
3 2
1
6
( )
⋅ a ⋅ cos(∠(a,w )) = ⋅ a ⋅ b ∧ c =
6

El volumen es siempre positivo de ahí que consideremos el valor absoluto en la fórmula


anterior.

Ejemplo 14.9 Para calcular el volumen de la pirámide de vértices A(-5,-2,1), B(-1,4,-1), C(4,1,6) y
D(0,0,-4), consideramos los vectores DA = ( − 5,− 2,5 ) , DB = ( − 1,4,3 ) y DC = (4,1,10 ) . A
continuación, aplicamos la fórmula anterior y resulta
-5 -2 5
1 314
Volumen = ⋅ - 1 4 3 = unidades cúbicas.
6 6
4 1 10

14-10
Espacio afín y euclídeo Tema 14

Ejercicios resueltos

R.1. Consideramos el sistema de referencia S ′ = {O ′, A} donde O ′ ≡ (3,2,1) y


r r r r r r
A = {u1 = (2,1,0 ),u 2 = (0,−1,2),u 3 = (0,0,3 )}referidos al sistema canónico. S = {O,e1, e 2 , e 3 }
a) Halla las coordenadas del punto P = (5,7,3 ) respecto del nuevo sistema S′
b) Halla las coordenadas en el sistema canónico del punto Q que, en el sistema S′
tiene de coordenadas Q = (3,5,2 ) .

Solución:
→ → →
a) Partimos de la igualdad vectorial OP = OO′+ O ′P y llamando (x ′, y ′, z′) a las coordenadas del

vector O ′P en el sistema S′ , se tiene (5,7,3 ) = (3,2,1) + x ′(2,1,0 ) + y ′(0,−1,2) + z′(0,0,3 ) en la que todo
está referido en la base canónica. Operamos e igualamos coordenadas con lo que se llega a
2x′ = 2 → x ′ = 1

x ′ − y ′ = 5 → y ′ = −4 .
10
2 y ′ + 3z ′ = 2 → z ′ =
3
→ → →
b) Al igual que en el apartado anterior planteamos OQ = OO′+ O′Q y si llamamos (x, y , z ) a las

coordenadas del vector OQ en el sistema S , se tiene:

(x, y, z ) = (3,2,1) + 3(2,1,0) + 5(0,−1,2) + 2(0,0,3) → (x, y, z ) = (9,0,17)


r r r r
R.2. Dados los vectores a (3,2,−5 ) y b (− 2,1,6 ) referidos a una base ortonormal, se pide: a , b
r r
( )
y ángulo α a, b .

Solución:

r r r r
a = 9 + 4 + 25 = 38 , b = 4 + 1 + 36 = 41 y a ⋅ b = −6 + 2 − 30 = −34 .
r r
r r
( )
a ⋅b
A partir de la definición de producto escalar se tiene que α a, b = arccos r r = arccos
− 34
38 41
y
a⋅b
α = 149'47º .
r r r r r r r r
R.3. Se considera la base {u1, u 2 , u3 } en la que u1 = u 2 = 2 , u3 = 1 , ∠(u1, u 2 ) = 60º ,
r r r r r r
∠(u 2 , u3 ) = ∠(u1, u3 ) = 90º y se consideran los vectores a (2,3,5 ) y b (1,1,−1) referidos a la base
r r r r
anterior, se pide: a , b y ángulo α a, b . ( )
Solución:
r r r r r r r r r r r r r r r r r
a = a ⋅ a , y a ⋅ a = 4u1 ⋅ u1 + 9u 2 ⋅ u 2 + 25u3 ⋅ u3 + 2 ⋅ 6 ⋅ u1 ⋅ u 2 ya que u1 ⋅ u3 = u 2 ⋅ u3 = 0
r r r 2 r r r 2 r r r 2 r r r r
como u1 ⋅ u1 = u1 = 4 , u 2 ⋅ u 2 = u 2 = 4 , u3 ⋅ u3 = u3 = 1 y u1 ⋅ u 2 = u1 ⋅ u 2 cos 60 = 2 , se tiene
r r r
que a ⋅ a = 16 + 36 + 25 + 24 = 101 y a = 101

14-11
Espacio afín y euclídeo Tema 14

r r r r r r r r r r r r r
Del mismo modo se tiene que b = b ⋅ b y b ⋅ b = 1⋅ u1 ⋅ u1 + 1⋅ u 2 ⋅ u 2 + 1⋅ u3 ⋅ u3 + 2 ⋅ 1⋅ u1 ⋅ u 2 = 13 y
r r r r r r r r r r r
b = 13 y a ⋅ b = 2 ⋅ u1 ⋅ u1 + 3 ⋅ u 2 ⋅ u 2 − 5 ⋅ u3 ⋅ u3 + (2 + 3 ) ⋅ u1 ⋅ u 2 = 25
r r
r r
( ) a ⋅b
y al igual que en el ejercicio anterior α a, b = arccos r r = arccos
25
101 13
= 46'37º
a⋅b

r r
R.4. Dados los vectores a (2,3,−7 ) y b (3,4,0 ) referidos a una base ortonormal, se pide:

r r
a) Calcula las coordenadas de un vector unitario u con la misma dirección y sentido que a
r r
b) Calcula las coordenadas de un vector v con la misma dirección que b , sentido contrario
y de módulo 7.

Solución:

r 1 r r
a) como a = 4 + 9 + 49 = 62 el vector ⋅ a tendrá la misma dirección y sentido que a y será
62
r  2 3 −7 
unitario, por lo tanto u =  , ,  .
 62 62 62 
r
b) Buscamos primero un vector unitario con la misma dirección y sentido que b actuando como en
r r 3 4 
el apartado anterior, b = 9 + 16 + 0 = 5 luego u =  , ,0  y multiplicando el vector obtenido por
5 5 
r r  21 28 
−7 se llega al vector v pedido que será v =  − ,− ,0  .
 5 5 

R.5. Dados los puntos A(1,2,0 ) , B(5,−2,3 ) y C (4,7,8 ) se pide:

a) Calcula las coordenadas del punto D de manera que ABCD sea un paralelogramo.
b) Calcula el área de dicho paralelogramo.

Solución:
→ →
a) El punto D debe cumplir que AB = DC , (hay que tener
cuidado en elegir correctamente el sentido).

Los vectores son AB = (4,−4,3 ) y si llamamos (x, y , z ) a las

coordenadas del punto D, DC = (4 − x,7 − y,8 − z ) igualando
4−x = 4 

se tiene: 7 − y = −4  → D = (0,11,5 ) .

8 − z = 3 
→ → → →
b) El área del paralelogramo S = AB ∧ DC , como AB ∧ DC = (− 47,−23,32) , se tiene que

→ →
S = AB ∧ DC = 472 + 232 + 322 = 3 418 .

R.6. Dados los puntos A(0,1,3 ) , B(4,1,0 ) , C (2,1,4 ) y D (1,2,1) se pide:

14-12
Espacio afín y euclídeo Tema 14

a) Comprueba que no son coplanarios.


b) Calcula el volumen del tetraedro (pirámide de base triangular) de vértices ABCD.

Solución:

→ → → 
a) Si el producto mixto de los vectores  AB, AC, AD  ≠ 0 los vectores no son coplanarios.
 
4 0 −3
→ → →  → → → 
AB(4,0,−3 ) , AC (2,0,1) , AD (1,1,−2 ) y  AB, AC, AD  = 2 0 1 = −10 ≠ 0 .
 
1 1 −2
1 → → →  − 10 5
b) El volumen del tetraedro es V =  AB, AC, AD  = = u.v.
6   6 3

r r r
R.7. Calcula el valor de α para que los vectores u (0,1, α ) , v (3,2,4 ) y w (0,3,−5 ) sean
coplanarios. Calcula el volumen del paralelepípedo que tiene sus aristas coincidentes con
r r r
los vectores u , v y w para α = 1 .

Solución:
0 1 α
r r r 5
Debe cumplirse que [u,v ,w ] = 0 , o sea 3 2 4 =0⇒α =− .
3
0 3 −5
0 1 1
r r r
El volumen será V = [u,v ,w ] = 3 2 4 = 24 = 24u.v.

0 3 −5

R.8. Calcula el valor de α para que el volumen del paralelepípedo que tiene de vértices
A(− 2,1,4 ) , B(0,1,1) , C (0,0,2 ) y D (3,5, α ) sea 19u.v..

Solución:
→ → →
Los vectores son : AB(2,0,−3 ) , AC (2,−1,−2) , AD (5,4, α − 4 ) y
2 0 −3
1 → → →  1
V=  AB, AC, AD  = 2 − 1 − 2 = 19 → − 2α − 15 = 19 → α = 2 .
6   6
5 4 α −4

14-13
Espacio afín y euclídeo Tema 14

Ejercicios propuestos

P.1. Si del sistema canónico de referencia se pasa al sistema ℜ = {O′,Β} , donde O ′ ≡ (1,2,1) y
r r r
Β = {u1 = (1,1,0 ),u 2 = (0,2,0 ),u 3 = (0,1,2)}, se pide: a) halla las coordenadas del punto P ≡ (3,5,3 )
respecto del nuevo sistema ℜ ; b) halla las coordenadas en el sistema original del punto Q que, en
el sistema ℜ , tiene de coordenadas Q ≡ (1,− 1,1) .

r r r r
P.2. Sabiendo que u = v = 2 , que u ⋅ v = 0,1 , calcula el ángulo que forman los vectores
r r r r r r
a = u + v y b = u − 2v .
r r r r r r r r
P.3. Sea B = {e1,e2 ,e3 } una base de R , tal que e1 = e2 = e3 = 2 y cos ∠ ei , e j
3
( ( )) = 0 si i ≠ j .
r r r r r r r r r r
Calcula a 2 − a ⋅ b , para a = 3e1 − 6e2 + e3 y b = e2 + 2e3 .

r r r
P.4. Si el espacio euclídeo tridimensional está referido a una base B = {e1,e2 ,e3 } formada por
vectores unitarios que forman entre sí ángulos de 60°, calcula el coseno del ángulo que forman los
r r r r r r r
vectores a = e1 +e2 y b = e1 − e 2 + e3 .

r r
P.5. Halla dos vectores ortonormales a los vectores u = (2,− 2,3 ) y v = (3,− 2,3 ) .

P.6. Halla el área del triángulo de vértices A ≡ (1,1,1) , B ≡ (0,3,5 ) , C ≡ ( 4,0,2 ) .

P.7. Dados los puntos A ≡ (1,2,0 ) y B ≡ (0,1,2 ) , determina el lugar geométrico del tercer vértice
para que el triángulo resultante tenga área constante igual a 3.

Soluciones

P.1. a) (3,5,3 ) = (1,2,1) + x (1,1,0 ) + y (0,2,0 ) + z (0,1,2) → P (x, y , z ) = P (2,0,1) , b) Q(2,2,3 ) .


r r r r r
v
( )
P.2. a = 8'2 , b = 19'6 , a ⋅ b = −4'1 , ∠ a, b = 108'8º .
r r r r r r r r r r r r r r r r r r
( )
P.3. a 2 − a ⋅ b = a ⋅ a − b = (3e1 − 6e2 + e3 ) ⋅ (3e1 − 7e2 − e3 ) = 200 . P.4. a ⋅ a = 3 , b ⋅ b = 2 , a ⋅ b = 1 ,
6 r r r  3 13 2 13  r  
cos α = . P.5. u ∧ v = (0,3,2) , u1 =  0, ,  y u 2 =  0,− 3 13 ,− 2 13  .
6  13 13   13 13 
 
→ → → → 230 → →
P.6. AB ∧ AC = (6,13,−5) , S = AB ∧ AC = . P.7. AB ∧ AC = (− z − 2y + 4,2 x + z − 2, x − y + 1)
2

(− z − 2y + 4)2 + (2x + z − 2)2 + (x − y + 1)2 = 6 o bien


5 x 2 + 5 y 2 + 2z 2 − 2 xy + 4 xz + 4 yz − 6 x − 18y − 12z − 15 = 0

14-14
Ecuaciones de recta y plano Tema 15

Tema 15

La recta y el plano en el espacio afín

15.1 Introducción
Se verá que un plano viene dado por una ecuación lineal con tres incógnitas, mientras
que la recta quedará perfectamente expresada mediante un sistema de dos ecuaciones con
tres incógnitas que sea compatible indeterminado, esto es, la intersección de dos planos. Así,
cualquier problema de posiciones relativas se podrá reducir a un problema de estudio de
rangos de un sistema. Por otra parte, no olvidemos que el sistema de referencia elegido tiene
base ortonormal.

15.2 Ecuaciones de la recta


r
Definición dados un punto P ≡ (x 0 , y 0 , z0 ) y un vector v = (a, b , c ) no nulo, al que
llamaremos vector director, se llama recta que pasa por el punto P y tiene vector dirección
r r
v , al conjunto de puntos X del espacio afín tales que OX = OP + t ⋅ v . Se escribirá:
r
r ≡P+ v .

r
A la ecuación OX = OP + t ⋅ v se le llama
ecuación vectorial de la recta r y en un sistema de
referencia se escribe:
(x , y , z ) = (x 0 , y 0 , z0 ) + t ⋅ (a, b,c ) .

r
Obsérvese que OX = OP + t ⋅ v ⇔
r → → →
⇔ OX − OP = t ⋅ v y que, como PX = OX − OP ,
→ r
entonces la ecuación de la recta se podría también escribir como PX = t ⋅ v , lo que significa
→ r r
que el vector PX es combinación lineal de v y, por lo tanto, paralelo a v .

r
Nota Como vector director de una recta se puede elegir cualquier vector paralelo a v .

15-1
Ecuaciones de recta y plano Tema 15

Ejemplo 15.1 La ecuación vectorial de la recta r que pasa por P (3,5,−1) y tiene vector
r
dirección v (5,−3,0 ) es (x , y , z ) = (3,5,−1) + t ⋅ (5,−3,0 ) .
Para comprobar si un punto pertenece a la recta basta ver si para algún valor de t se
satisface la ecuación. Por ejemplo, el (1,1,3 ) pertenece a la recta si existe algún valor de t que
haga cierta la ecuación vectorial (1,1,3 ) = (3,5,−1) + t ⋅ (5,−3,0 ) . Igualando las primeras
2
coordenadas se tiene que 1 = 3 + 5t ⇒ t = − , pero para ese valor de t se tiene que no se
5
31
cumple la ecuación para las segundas coordenadas, pues − 3t + 5 = ≠ 1.
5
Para obtener algún punto de la recta basta elegir un valor para t y sustituirlo en la
ecuación vectorial. Por ejemplo, para t = 2 se tiene (x , y , z ) = (3,5,−1) + 2 ⋅ (5,−3,0 ) = (13,−1,−1)
luego Q(13,−1,−1) ∈ r .

a) Ecuaciones paramétricas de la recta:

Si operamos en la ecuación vectorial de la recta e igualamos las coordenadas se llega


 x = x0 + a ⋅ t

a y = y 0 + b ⋅ t , que son tres ecuaciones escalares que equivalen a la ecuación vectorial de la
z = z + c ⋅t
 0
recta llamadas ecuaciones paramétricas de la recta que pasa por el punto P ≡ (x 0 , y 0 , z0 ) y
r
tiene vector dirección v = (a, b , c ) , en las que t ∈ R es el parámetro.

Ejemplo 15.2 Las ecuaciones paramétricas de la recta r determinada por el punto P (3,5,−1) y
 x = 3 + 5t
r 
el vector v (5,−3,0 ) , son: r ≡ y = 5 − 3t .
 z = −1

Nota El obtener puntos de la recta y el comprobar si un punto pertenece o no a la recta se


hace ahora con más comodidad que con la ecuación vectorial.

b) Ecuación en forma continua y ecuaciones implícitas (o cartesianas) de la recta:

Si eliminamos el parámetro t del sistema de ecuaciones paramétricas anterior, se


obtienen dos ecuaciones cartesianas.
a ⋅ t = x − x 0

En efecto, si consideramos t como incógnita se tiene b ⋅ t = y − y 0 , donde (x , y , z ) es
c ⋅t = z − z
 0
un punto de la recta si el sistema es compatible, es decir, si el rango de la matriz de los
a  a x − x0 
  ∗  
coeficientes A =  b  y de A =  b y − y 0  coinciden.
c  c z − z 
   0 
r r
Como el rango de A es 1, ya que v ≠ 0 , el rango de A ∗ debe ser 1, y, en tal caso, las dos
columnas de la matriz A ∗ han de ser proporcionales, por lo que se llega a las ecuaciones

15-2
Ecuaciones de recta y plano Tema 15

x − x 0 y − y 0 z − z0
= = llamadas ecuaciones en forma continua de la recta que pasa por el
a b c
r
punto P ≡ (x 0 , y 0 , z0 ) y tiene vector dirección v = (a, b , c ) .

r r
Nota importante. Aunque v (a, b, c ) ≠ 0 puede suceder que alguna o algunas (no todas) de sus
coordenadas sí sean cero, en ese caso el cociente o los cocientes que aparecen deben
considerarse en términos de proporcionalidad ya que de otro modo carecen de sentido.

Ejemplo 15.3 Las ecuaciones en forma continua de la recta r determinada por P (3,5,−1) y el
r x − 3 y − 5 z +1
vector v (5,−3,0 ) , son: r ≡ = = en las que una ecuación que implique el último
5 −3 0
miembro hay que entenderla como que z + 1 = 0 y no como cociente.

De que el rango de A ∗ sea 1 también se deduce que

a x − x0 a x − x0
=0 y =0.
b y − y0 c z − z0

 Ax + By + Cz + D = 0
Desarrollando estos determinantes se obtienen dos ecuaciones  que
 A′x + B ′y + C ′z + D ′ = 0
equivalen a la ecuación vectorial de la recta, por lo que se las conoce como ecuaciones
implícitas o cartesianas de la recta que pasa por el punto P ≡ (x 0 , y 0 , z0 ) y tiene vector
r
dirección v = (a, b , c ) .

c) Ejemplos del modo de pasar de unas ecuaciones a otras

Como en el planteamiento de un problema la ecuación de una recta puede venir


expresada de distintas formas, conviene ser capaces de llegar a cualquiera de las ellas a partir
de una cualquiera de las otras.

Ejemplo 15.4 (De las ecuaciones paramétricas a la ecuación en forma continua) Si sabemos
interpretar los datos, pasar de las ecuaciones paramétricas a continuas es inmediato, pues los
coeficientes de t son las coordenadas del vector dirección y los números independientes son
las coordenadas de un punto de la recta.
 x = −1 + 2t

Así, dada la recta r ≡  y = 3 − 2t , es inmediato que pasa por P ≡ ( − 1, 3 , − 5 ) y tiene
z = −5 + 7t

r
de vector dirección v = ( 2 , − 2 , 7 ) , por lo que las ecuaciones en forma continua son
x +1 y − 3 z + 5
r ≡ = = .
2 −2 7
Otra forma: despeja t en cada ecuación e iguala los resultados.
x +1 y − 3 z + 5
Así se llega a = = .
2 −2 7

Nota Del mismo modo, interpretando los datos, es inmediato pasar de las ecuaciones en
forma continua a las ecuaciones paramétricas.

15-3
Ecuaciones de recta y plano Tema 15

Ejemplo 15.5 (De las ecuaciones paramétricas a las ecuaciones implícitas) Para pasar de las
ecuaciones paramétricas a las implícitas basta eliminar el parámetro t.
Así, en la recta anterior, como rango de A = rango de A ∗ = 1, entonces
2 x +1 2 x +1
= 0 ⇔ 2 x + 2y − 4 = 0 ⇔ x + y − 2 = 0 , y = 0 ⇔ 7 x − 2z − 3 = 0 .
−2 y −3 7 z+5
 x +y −2 =0
Luego la recta en ecuaciones implícitas es  .
7 x − 2z − 3 = 0
Otra forma: es pasar a la forma continua tal como se ha visto en el ejemplo anterior. Después,
x +1 y − 3
 =
se seleccionan dos de las ecuaciones, como por ejemplo  2 − 2 , y operando se llega
x +1 z + 5
 =
 2 7
− 2 x − 2 = 2 y − 6  x + y − 2 = 0
a  ⇒ .
 7 x + 7 = 2z + 10 7 x − 2z − 3 = 0

Ejemplo 15.6 (De las ecuaciones implícitas a las ecuaciones paramétricas) Dada la recta
 2x − 3y − z + 1 = 0
r ≡ , para obtener las ecuaciones paramétricas se pasa una de las
4 x + y − 2z − 3 = 0
incógnitas al segundo miembro y se considera como parámetro, con la única condición de que
en el sistema resultante el rango de la matriz de los coeficientes sea 2. Por ejemplo, aquí no
2 −1
se puede elegir y como parámetro ya que = 0 . Sin embrago, sí se puede elegir z. Por
4 −2
2 x − 3 y = z − 1
tanto, pasando z al segundo miembro queda:  que, resolviendo por la regla de
4 x + y = 2z + 3
z −1 − 3 2 z −1
2z + 3 1 4 1 4 2z + 3 5
Cramer, resulta: x = = + z ,y = = .
2 −3 7 2 2 −3 7
4 1 4 1
 4 1
x = 7 + 2 α
 5
Luego, las ecuaciones paramétricas serán:  y = con α ∈ R
 7
 z=α


Ejemplo 15.7 (De las ecuaciones implícitas a las continuas) Una manera de hacerlo es pasar
de las implícitas a las paramétricas y de éstas, de modo análogo al ejemplo 15.4, pasar a las
continuas.
4 5
x− y−
En efecto, eliminando α quedarían: r ≡ 7 = 7 = z , que en una versión mas cómoda
1 0 1
2
4 5
x− y−
resulta r ≡ 7 = 7 =z.
1 0 2

15-4
Ecuaciones de recta y plano Tema 15

Algunas consideraciones de interés:


- Conviene que quede claro que una vez se tiene un vector dirección de una recta
siempre se puede sustituir éste por uno linealmente dependiente, esto es, de coordenadas
 1 
proporcionales. En el ejemplo anterior hemos sustituido el  , 0 ,1  por el ( 1, 0 , 2 ) .
 2 

- Más adelante, en el ejemplo 15.19 , se verá cómo pasar directamente de las


ecuaciones implícitas a cualquiera de las otras ecuaciones. La idea es encontrar un vector
dirección y un punto de la recta.

- Hasta ahora hemos visto que dados un punto y un vector director queda determinada
una recta, pero también queda determinada si nos dan dos puntos P y Q. En efecto, en tal
r →
caso, basta tomar como vector dirección el v = PQ y como punto uno cualquiera de los dados,
de manera que r ≡ P + PQ y, también, r ≡ Q + PQ .

Ejemplo 15.8 Hallemos las ecuaciones vectorial, paramétricas, continua e implícitas de la


recta que pasa por los puntos P(0,2,1) y Q(1,2,3).
r →
Tomando v = PQ , la ecuación vectorial es: (x , y , z ) = (0,2,1) + α(1,0,2) . Las ecuaciones
 x= α
 x y − 2 z −1
paramétricas son:  y = 2 . En forma continua: = = . De cuyas dos igualdades
 z = 1+ 2α 1 0 2

podemos sacar las ecuaciones implícitas, del modo siguiente: del primer y segundo miembro
sacamos: y − 2 = 0 ; mientras que del primer y tercer miembro obtenemos: 2 x = z − 1 ,
 y −2 =0
quedando:  .
 2 x − z + 1= 0

15.3 Ecuaciones del plano


r r
Definición Dados un punto P (x 0 , y 0 , z0 ) y dos vectores v (v 1 ,v 2 ,v 3 ) y w (w 1 ,w 2 ,w 3 )
linealmente independientes entre si, llamados vectores directores, se llama plano que pasa
r r
por el punto P con vectores dirección v y w al conjunto de puntos X del espacio afín tales
r r r r
que OX = OP + α ⋅ v + β ⋅ w , con α ,β ∈ R . Se escribirá: π ≡ P + v ,w .

r r → r r
Nota Obsérvese que OX = OP + α ⋅ v + β ⋅ w equivale a PX = α ⋅ v + β ⋅ w ,es decir que si X es

un punto del plano entonces PX es combinación lineal
r r
de los vectores v y w .
r r
A la ecuación OX = OP + α ⋅ v + β ⋅ w se le llama
ecuación vectorial del plano que pasa por el punto P con
r r
vectores dirección v y w . En función de las
coordenadas en un sistema de referencia afín, la
ecuación puede ser escrita como

(x , y , z ) = (x 0 , y 0 , z 0 ) + α ⋅ (v 1 ,v 2 ,v 3 ) + β ⋅ (w 1 , w 2 , w 3 ) .

15-5
Ecuaciones de recta y plano Tema 15

Ejemplo 15.9 La ecuación vectorial del plano que pasa por el punto P (1,0,2) y tiene como
r r
vectores dirección los vectores v (2,1,0 ) y w (0,1,1) , es: π ≡ (x , y , z ) = (1,0,2 ) + α ⋅ (2,1,0 ) + β ⋅ (0,1,1) .

a) Ecuaciones paramétricas

Si operamos e igualamos coordenada a coordenada se obtiene


 x = x0 + α ⋅ v1 + β ⋅ w1

y = y 0 + α ⋅ v 2 + β ⋅ w 2 ,
z = z + α ⋅v + β ⋅w
 0 3 3
que son las llamadas ecuaciones paramétricas del plano que pasa por el punto P con vectores
r r
dirección v y w .

Ejemplo 15.10 Las ecuaciones paramétricas del plano que pasa por el punto P (1,0,2) y tiene
r r
como vectores dirección los vectores v (2,1,0 ) y w (0,1,1) , son
 x = 1+ 2α

 y = α+β .
 z=2 +β

Eligiendo valores cualesquiera para α y β pueden hallarse tantos puntos del plano
como se deseen. Por otro lado, un punto X de coordenadas (x , y , z ) pertenece al plano si es
posible encontrar dos valores α ,β ∈ R que satisfagan las tres ecuaciones.

b) Ecuación del plano en forma de determinante

Eliminando los parámetros α y β se llega a una sola ecuación de incógnitas x, y, z. En


efecto, si en las ecuaciones paramétricas consideramos α y β como incógnitas se tiene el
sistema
α ⋅ v1 + β ⋅ w1 = x − x0 

α ⋅v2 + β ⋅w2 = y − y0  ,
α ⋅ v 3 + β ⋅ w 3 = z − z0 
r r
( )
que será compatible si rang(A ) = rang A ∗ . Ahora bien, como rang(A) = 2 ya que v y w son
( )
linealmente independientes, necesariamente rang A ∗ = 2 y, por lo tanto, debe suceder que
v1 w1 x − x0
v2 w2 y − y 0 = 0 , que es la ecuación del plano en forma de determinante.
v3 w3 z − z0

Ejemplo 15.11 La ecuación en forma de determinante del plano que pasa por el punto P (1,0,2)
r r
y tiene como vectores dirección los vectores v (2,1,0 ) y w (0,1,1) es

x -1 y z-2
2 1 0 =0.
0 1 1

15-6
Ecuaciones de recta y plano Tema 15

c) Ecuación general del plano

Desarrollando y operando en el determinante anterior se llega a una ecuación del tipo


A ⋅ x + B ⋅ y + C ⋅ z + D = 0 , llamada ecuación general del plano.

Ejemplo 15.12 Desarrollando el determinante del ejemplo anterior se obtiene su ecuación


general o cartesiana: π ≡ x − 2y + 2z − 5 = 0 .

A igual que en el caso de la recta conviene ser capaces de llegar a cualquiera de las
formas de ecuaciones de un plano partiendo de otra u otras, lo que veremos en los siguientes
ejemplos.

Ejemplo 15.13 Dada la ecuación general de un plano, por ejemplo π ≡ 2 x + 3 y − 5z − 2 = 0 ,


para obtener las ecuaciones paramétricas del mismo basta hacer x = α , y = β y escribir z en

 x=α

función de α y β, quedando:  y = β
 2 2 3
 z= − + α+ β
 5 5 5

d) Ecuación del plano determinado por tres puntos.

En las anteriores ecuaciones de un plano se ha visto que éste queda perfectamente


determinado mediante un punto y dos vectores directores. Pero, también queda perfectamente
determinado por 3 puntos no alineados del plano.
r →
En efecto, sean P (x 0 , y 0 , z0 ) , Q(q1 , q 2 , q 3 ) y R (r1 , r2 , r3 ) , bastará tomar v = PQ y
r → → → →
w = PR . Entonces, X (x , y , z ) será del plano si: PX = α PQ + β PR , es decir, si
x − x0 y − y0 z − z0

q1 − x 0 q 2 − y 0 q 3 − z0 = 0 .

r 1 − x0 r 2 − y0 r 3 − z0

Con esta expresión podemos responder si 4 puntos X, P, Q, y R dados están todos en


el mismo plano.
Ejemplos

Ejemplo 15.14. Para hallar la ecuación del plano que pasa por los puntos A(1,0,2), B(2,-1,3) y
C(4,1,0), primeramente hay que comprobar que estos puntos no están alineados, para ello
basta comprobar que el determinante de orden tres formado por ellos es distinto de cero. Como
1 0 2
2 − 1 3 = 4 + 8 − 3 ≠ 0 , no están alineados y, por tanto, definen un plano. El plano quedará
4 1 0

15-7
Ecuaciones de recta y plano Tema 15

 A x −1 y z − 2
→
determinado por  AB y su ecuación será π ≡ 1 −1 1 = 0 que, en forma general, se
→ 3 1 −2
 AC
escribe como π ≡ x + 5 y + 4z − 9 = 0 .

Ejemplo 15.15 Para comprobar que los puntos P(1,-2,0), Q(0,-1,2) y R(3,0,4) no son
→ → → →
coplanarios con el punto T(-1,2,4), bastará verificar que, PT ≠ α PQ + β PR ; PT (− 2,4,4 ) ,
−2 4 4
→ →
PQ (− 1,1,2) y PR (2,2,4 ) y como − 1 1 2 = 16 ≠ 0 , T no está en el mismo plano que P, Q y
2 2 4
R.

e) Ecuación segmentaria del plano.

Partiendo de la ecuación general o implícita de un plano A ⋅ x + B ⋅ y + C ⋅ z + D = 0


pasamos D al segundo miembro A ⋅ x + B ⋅ y + C ⋅ z = −D y si dividimos por – D ambos
A B C x y z
miembros se llega a ⋅x + ⋅y + ⋅ z = 1⇔ + + = 1 y llamando
−D −D −D D D D
− − −
A B C
D D D x y z
a = − ,b = − ,c = − nos queda + + = 1 llamada ecuación segmentaria del plano
A B C a b c
en la que a, b y c son los segmentos que determina el plano con los ejes X, Y y Z
respectivamente, es decir corta a los ejes en los puntos (a ,0,0 ) , (0, b,0 ) y (0,0, c ) .

f) Ecuación normal del plano

Dados un punto P (x 0 , y 0 , z0 ) y un vector


r
n (A, B ,C ) perpendicular al plano π , recordando el
concepto de producto escalar, es fácil ver que un
r →
punto X ∈ π si, y sólo si, n ⋅ PX = 0 , condición
que puesta en función de las coordenadas nos
lleva a
A ⋅ (x − x 0 ) + B ⋅ (y − y 0 ) + C ⋅ (z − z0 ) = 0
conocida como ecuación normal del plano que
pasa por el punto P y es perpendicular al vector
r
n (A, B ,C ) .

v
Nota Al vector n se le llama vector característico del plano y como, operando en la ecuación
normal del plano, se tiene A ⋅ x + B ⋅ y + C ⋅ z + D = 0 que es la ecuación implícita del plano,
deducimos que siempre que se nos dé la ecuación del plano en forma implícita un vector
perpendicular a él es el que tiene por coordenadas los coeficientes de x, y, z.

Ejemplos

Ejemplo 15.16 Un vector perpendicular al plano de ecuación x + 2y − 2z + 7 = 0 es el que


tiene por coordenadas ( 1, 2 , − 2 ) .

15-8
Ecuaciones de recta y plano Tema 15

Ejemplo 15.17 La ecuación del plano que pasa por P (1,2,3 ) y tiene como vector perpendicular
r
n (2,1,5 ) , es 2(x − 1) + (y − 2 ) + 5(z − 3 ) = 0 ⇒ π ≡ 2 x + y + 5z − 19 = 0 .

Otra forma: la ecuación del plano será π ≡ 2 x + y + 5z + D = 0 . Para determinar D basta


sustituir el punto dado en dicha ecuación: 2 ⋅ 1 + 2 + 5 ⋅ 3 + D = 0 ⇔ D = −19 . Por lo tanto, el
plano buscado tiene de ecuación π ≡ 2 x + y + 5z − 19 = 0 .

Ejemplo 15.18 Para determinar un vector perpendicular al plano


1 2 x
π de ecuación 2 0 y − 7 = 0 , basta interpretar que sus
3 1 z +1
r r
vectores directores son v (1,2,3 ) y w (2,0,1) y que su producto
r r v
vectorial n = v ∧ w es perpendicular al plano π
r r r
i j k
r 2 3 3 1 1 2
n = 1 2 3 =  , ,  = (2,5,−4 ) .
 0 1 1 2 2 0 
2 0 1

g) La recta como intersección de planos

 Ax + By + Cz + D = 0
Las ecuaciones implícitas de una recta r ≡  pueden ser
 A′x + B ′y + C ′z + D ′ = 0
interpretadas como intersección de los planos π y π ′ de ecuaciones
Ax + By + Cz + D = 0 y A ′x + B ′y + C ′z + D ′ = 0 ,
respectivamente.
En tal caso, como hemos visto que los
vectores de coordenadas (A, B ,C ) y (A ′, B ′, C ′)
son perpendiculares a π y π ′ , resulta que un
vector director de r es el producto vectorial de
ambos. Lo que nos da un nuevo procedimiento
para pasar de las ecuaciones implícitas de una
recta a las paramétricas o continuas o
simplemente obtener un vector director de dicha
recta.

 Ax + By + Cz + D = 0
Nota Al decir que r es una recta ya queda claro que el sistema:  es
 A ′x + B ′y + C ′z + D ′ = 0
compatible. Pero no todo sistema de dos ecuaciones con tres incógnitas representa una recta.

Ejemplo 15.19 Para obtener las ecuaciones vectorial, paramétricas y continuas de una recta
2 x + y + z − 3 = 0
dada en forma implícita, tal como r :  , sólo se necesita saber sacar de éstas
 x + 2y − z + 4 = 0
un punto y un vector dirección.

El vector dirección será el producto vectorial de los vectores característicos de los


planos y para obtener el punto basta dar un valor a una de las incógnitas siempre que el
determinante de la matriz de los coeficientes del sistema que resulta sea distinto de cero.

15-9
Ecuaciones de recta y plano Tema 15

r r r
i j k
r
Así pues, el vector director de r será v = (2,1,1) ∧ (1,2,−1) = 2 1 1 = (− 3,3,3 ) .
1 2 −1
2 1
Como ≠ 0 , esto es, el menor formado por los coeficientes de x e y es distinto de
1 2
 2x + y = 3 10
cero, elegimos z = 0 y resolvemos el sistema  , cuya solución es x = ,
 x + 2y = −4 3
11  10 11 
y = − . Luego un punto de la recta es el punto es  ,− ,0  .
3  3 3 
En consecuencia,
(x , y , z ) =  10 ,− 11 ,0  + α(− 3,3,3) , es la ecuación vectorial y
 3 3 
10 11
x− y+
3 = 3 = z , son sus ecuaciones en forma continua.
−3 3 3

15.4 Posición relativa de dos rectas

Consideraremos dos posibilidades:


a) que una recta está dada en forma continua, vectorial o paramétrica (conocemos un
punto y un vector dirección) ,
b) que las dos rectas vienen dadas por sus ecuaciones implícitas.

P (x , y , z )  Q (x , y , z )
Caso a) Sean las rectas r1 :  r 1 1 1 y r 2 :  r 2 2 2 ,es decir, de sus ecuaciones
v (v 1 ,v 2 ,v 3 ) w (w 1 ,w 2 ,w 3 )

podemos conocer un punto y un vector director. Formaremos el vector PQ y consideramos las
 v1 w1   v1 w1 x1 − x 2 
   
matrices A =  v 2 w 2  y B =  v 2 w2 y1 − y 2  .
v  v z1 − z 2 
 3 w3   3 w3
Recordando que el rango de una matriz es el rango de los vectores columna que la
forman (el máximo números de vectores linealmente independientes), pueden darse los
siguientes casos:
r r r r →  r r →
1. rang (A ) = rang (B ) = 1 ⇔ rang (v ,w ) = rang  v ,w , PQ  = 1 ⇒ Los vectores v , w y PQ tienen
 
la misma dirección (son paralelos), luego las rectas son coincidentes.
r r r r →  r r
2. rang (A ) = 1, rang (B ) = 2 ⇔ rang (v ,w ) = 1, rang  v ,w , PQ  = 2 ⇒ Los vectores v y w tienen
 

la misma dirección y el vector PQ tiene dirección distinta a ellos, luego las rectas son
paralelas.
r r r r →  r r
3. rang (A ) = rang (B ) = 2 ⇔ rang (v ,w ) = rang  v ,w , PQ  = 2 ⇒ Los vectores v y w tienen
 

distinta dirección y el vector PQ es combinación lineal de ellos (esto es, está en el plano que
generan los otros dos vectores), luego las rectas se cortan en un punto.

15-10
Ecuaciones de recta y plano Tema 15

r r r r →  r r
4. rang (A ) = 2 , rang (B ) = 3 ⇔ rang (v ,w ) = 2, rang  v ,w , PQ  = 3 ⇒ Los tres vectores v , w y
 

PQ tienen distinta dirección (esto es, no existe un plano que contenga a los tres), luego las
rectas que se cruzan.

Rectas coincidentes. Rectas paralelas.

Rectas que se cortan Rectas que se cruzan.

x + 2 y −1
Ejemplo 15.20 Estudiemos la posición relativa de las rectas r: = =z y
−3 2
 x+ y - z=0
s: .
 x − y − 5z − 8 = 0
Primeramente obtendremos un vector dirección y un punto de s (ver ejemplo 15.19).
Haciendo el producto vectorial de los vectores característicos se obtiene el vector dirección
(3,−2,1) . Eliminando el parámetro z (pasándolo al segundo miembro y despejando x e y en
función de z), se obtiene el punto Q(4,−4,0 ) . De la recta r es fácil ver que ( − 3 , 2 ,1 ) es un

vector dirección y que P ( − 2 ,1, 0 ) es un punto. Entonces, PQ = (6,−5,0 ) . Por tanto, como
rang {(− 3,2,1), (3,−2,1)} = 2 y rang {(− 3,2,1), (3,−2,1), (6,−5,0 )} = 3 , las rectas se cruzan.

Caso b) Supongamos que las dos rectas vengan dadas por sus ecuaciones reducidas o
implícitas. Sean
 Ax + By + Cz + D = 0 (1)  A′′x + B ′′y + C ′′z + D ′′ = 0
r: y s: . (2)
 A′x + B ′y + C ′z + D ′ = 0  A′′′x + B ′′′y + C ′′′z + D ′′′ = 0
Ambas rectas definen un sistema de cuatro ecuaciones con tres incógnitas:
 Ax + By + Cz + D = 0
 A' x + B' y + C' z + D' = 0
(3) 
 .
 A′′x + B ′′y + C ′′z + D ′′ = 0
 A′′′x + B ′′′y + C ′′′z + D ′′′ = 0

15-11
Ecuaciones de recta y plano Tema 15

Observemos previamente que tanto en el sistema (1), como en el (2), por separado, el
rango de la matriz de los coeficientes coincide con el de la ampliada y vale dos. Entonces, en el
sistema (3) pueden darse los siguientes casos:

Casos Rango de la matriz Rango de la Tipo de sistema Interpretación


de los coeficientes matriz ampliada geométrica
1º 2 2 Compatible Rectas
Indeterminado coincidentes
2º 2 3 Incompatible Rectas
paralelas
3º 3 3 Compatible Se cortan en un
determinado punto
4º 3 4 Incompatible Se cruzan

Nota Si bien la interpretación geométrica de los casos 1º y 3º es evidente, quizás merezca


aclarar la de los casos 2º y 4º.

Si entiendes el caso 2º, la interpretación del 4º sale por reducción de posibilidades en


el espacio.
Para ello, considera cada recta como intersección de dos planos, cada uno de ellos con
su correspondiente vector característico, así por ejemplo en la primera recta los vectores
r r
característicos de cada plano son n = ( A , B ,C ) y n ′ = ( A′ , B ′ ,C ′ ) , respectivamente.
r r
Análogamente, los de la otra son n ′′ = ( A′′ , B ′′ ,C ′′ ) y n ′′′ = ( A′′′ , B ′′′ ,C ′′′ ) .
Decir que el rango de la matriz de los coeficientes es 2 (donde cada fila es uno de
estos cuatro vectores), es afirmar que los últimos dos vectores dependen linealmente de los
r r r r r r
dos primeros y, por tanto, podemos escribir: n ′′ = α n + β n ′ y n ′′′ = µn + σn ′ . Ahora
bien, como la dirección de la recta viene dada por el producto vectorial de ambos, y como
r r r r r r r r
n ′′ ∧ n ′′′ = ( αn + βn ′ ) ∧ (µn + σn ′ ) = ( α ⋅ σ − β ⋅ µ )(n ∧ n ′ ) ,
se concluye que las dos rectas son paralelas.

Ejemplo 15.21 Para hallar la posición relativa de las rectas


 2x − 3y − 8 = 0  2x + y + 5 = 0
r : s:
 4 x − 3 z + 5 = 0  4x + z + 8 = 0
se estudian los rangos de la matriz ampliada y de los coeficientes por el método de Gauss, por
ejemplo. Trabajando con la matriz ampliada se tiene
2 − 3 0 8  2 − 3 0 8  2 − 3 0 8 
     
 4 0 − 3 − 5  0 6 − 3 − 21  0 6 − 3 − 21
2 1 ⇔ ⇔ .
0 − 5 0 4 0 − 13  0 0 12 6 
     
4 0 1 − 8  0 0 4 − 3  0 0 0 15 
  
( )
Como rang (A ) = 3 y rang A ∗ = 4 ⇒ las rectas se cruzan.

15.5 Posición relativa de dos planos en el espacio

Consideraremos también dos posibilidades:


a) que de cada plano se conozca un punto y dos vectores directores ,

15-12
Ecuaciones de recta y plano Tema 15

b) que los dos planos vengan dados por sus ecuaciones implícitas.

Caso a) Los planos vienen dados mediante sus ecuaciones en forma paramétrica o de
determinante; es decir, conocemos dos vectores directores y un punto de cada uno. Sean
r r r r r r r r
π ≡ P + v ,w y π' ≡ Q + v ′ ,w ′ , y recordemos que rang {v ,w } = rang {v ′ ,w ′ } = 2 , entonces
pueden darse los siguientes casos:
r r r r r r r r → 
1. rang (v ,w ,v ′,w ′) = rang  v ,w ,v ′,w ′, PQ  = 2 ⇒ son planos coincidentes.
 
r r r r r r r r → 
2. rang (v ,w ,v ′,w ′) = 2 y rang  v ,w ,v ′,w ′, PQ  = 3 ⇒ son planos paralelos.
 
r r r r
3. rang (v ,w ,v ′,w ′) = 3 ⇒ se cortan en una recta.

No tiene sentido considerar que el rango sea cuatro ya que en R 3 el máximo rango es tres.

P (0,1,0 )  Q(3,2,1)
r r
Ejemplo 15.22 Para hallar la posición relativa de los planos: π :  v (1,2,1) y π′ : v ′(0,2,2) ,
wr (1,1,0 )  wr ′(1,0,1)
 
calcularemos el rang {(1,2,1), (1,1,0 ), (0,2,2), (1,0,1)} por el método de Gauss:
 1 1 0 1 1 1 0 1  1 1 0 1 
     
 2 1 2 0  ⇔  0 − 1 2 − 2  ⇔  0 − 1 2 − 2
 1 0 2 1 0 1 2 0  0 0 0 2 
     

Como es 3, los planos se cortan en una recta.

Caso b) Los planos vienen dados por sus ecuaciones implícitas.


Sean π : Ax + By + Cz + D = 0 y π′ : A ′x + B ′y + C ′z + D ′ = 0 , y consideremos el sistema
de 3 incógnitas que definen las 2 ecuaciones, entonces:

Casos Rango de la matriz Rango de la Tipo de sistema Interpretación


de los coeficientes matriz ampliada geométrica
1º 1 1 Compatible Planos
Indeterminado coincidentes
2º 1 2 Incompatible Planos paralelos
3º 2 2 Compatible Se cortan en
indeterminado una recta

15-13
Ecuaciones de recta y plano Tema 15

Ejemplo 15.23 Para estudiar la posición relativa de los planos: π : 2 x − y + 3 z = 4 y


π′ : x + y + z = 3 , se considera la matriz de los coeficientes y la matriz ampliada:
2 −1 3 4 2 − 1 3 4
  ⇒   ⇒ r = r ′ = 2 . Luego los planos se cortan en una recta.
 1 1 1 3   0 3 − 1 2  2f2 − f1

15.6 Posición relativa de una recta y un plano

Distinguiremos tres casos:


a) de cada uno se conocen un punto y su dirección,
b) ambos viene dados en forma implícita,
c) la recta viene dada en forma paramétrica y el plano en forma implícita.

r r r r r
Caso a) Sean la recta r ≡ P + v y el plano π ≡ Q + w , u . Como rang {w , u } = 2 , pueden
darse los siguientes casos:
r r r r r r → 
1. rang (v ,w ,u ) = rang  v ,w ,u , PQ  = 2 ⇒ recta contenida en el plano
 
r r r r r r → 
2. rang (v ,w , u ) = 2 y rang  v ,w , u , PQ  = 3 ⇒ recta paralela al plano, pero no contenida en él.
 
r r r
3. rang (v ,w ,u ) = 3 ⇒ la recta corta al plano en un punto.

P (− 2,−5,1)
Ejemplo 15.24 Para estudiar la posición relativa de la recta r :  r y el plano
v (5,9,1)
Q(0,0,2)
r →
π : w (7,0,−4 ) , hallamos el vector PQ = (2,5,1) y, a continuación, estudiamos los rangos de
uv (0,7,3 )

r r r → 
{vr ,wr ,ur } y  v ,w ,u , PQ  :
 

15-14
Ecuaciones de recta y plano Tema 15

5 9 1  5 9 1  5 9 1 
     
7 0 − 4  0 − 63 − 27  5f 2 − 7f1  0 − 63 − 27 
0 ⇔ ⇔ .
7 3  0 7 3  0 0 0  9f3 + f 2
     
2 5 1  0 7 3  5f 4 − 2f1 0 0 0  9f 4 + f 2
  

De donde rang {(5,9,1), (7,0,−4 )(0,7,3 )} = 2 y rang {(5,9,1), (7,0,−4 )(0,7,3 ), (2,5,1)} = 2 ; luego, es una
recta contenida en el plano.

Caso b) Consideramos la recta y el plano dados por sus ecuaciones implícitas


 A′x + B ′y + C ′z + D ′ = 0
π : Ax + By + Cz + D = 0 y s :  . Juntas, forman un sistema de 3
 A′′x + B ′′y + C ′′z + D ′′ = 0
ecuaciones con 3 incógnitas.
Observemos que, por ser s una recta, el rango de la matriz de sus coeficientes y el
rango de su matriz ampliada es dos.

Casos Rango de la matriz Rango de la Tipo de sistema Interpretación


de los coeficientes matriz ampliada geométrica
1º 2 2 Compatible Recta contenida
Indeterminado en el plano
2º 2 3 Incompatible Recta y plano
paralelos
3º 3 3 Compatible Se cortan en un
determinado punto

 2x + y − z = 1
Ejemplo 15.25 Para estudiar la posición relativa de la recta r :  y el plano
 x − 2 y + z = −1
π : 6 x + 3 y − 3z = 1 , se considera la matriz de los coeficientes y la ampliada:
2 1 −1 1  2 1 −1 1 
   
 1 − 2 1 − 1 ⇔  0 − 5 3 − 3  2f 2 − f1 .
6 3 − 3 1  0 0 0 − 2  f 3 − 3f1
  
Como el rango de la matriz de los coeficientes es 2 y el rango de la matriz ampliada es 3, la
recta r es paralela al plano π .

 x = x0 + a ⋅ t

Caso c) La recta r viene dada en forma paramétrica y = y 0 + b ⋅ t y el plano por su ecuación
z = z + c ⋅t
 0

implícita π : Ax + By + Cz + D = 0 . Se resuelve el sistema por sustitución de x, y, z en la


ecuación del plano quedando una sola ecuación con el parámetro t como incógnita, entonces:

1. si se llega a 0 ⋅ t = a con a ≠ 0 , no hay solución. La recta y el plano son paralelos


2. si se llega a 0 ⋅ t = 0 todo valor de t es solución. La recta pertenece al plano
3. si se llega a a ⋅ t = b con a ≠ 0 , hay una sola solución. La recta corta al plano en un punto.

15-15
Ecuaciones de recta y plano Tema 15

Es el caso más cómodo para estudiar la posición relativa o encontrar, en su caso, el


punto de corte, como se ve en el siguiente ejemplo:

 x = 2 + 3α

Ejemplo 15.26 Sean r :  y = 1 − 3α y π : 2 x + y − 3z + 1 = 0 . Estudiamos su posición relativa
 z=α

sustituyendo x, y, z de r en π :
2(2 + 3α ) + (1 + 3α ) − 3α + 1 = 0 → 0α = 6 ⇒ r y π no tienen ningún punto en común,
luego son paralelos.

15.7 Posición relativa de tres planos en el espacio

Sean π1 : Ax + By + Cz + D = 0 , π2 : A′x + B′y + C ′z + D′ = 0 y π3 : A′′x + B′′y + C ′′z + D′′ = 0 , las


ecuaciones de tres planos. Juntas, forman un sistema de 3 ecuaciones con 3 incógnitas.

Sean r al rango de la matriz de los coeficientes y r' el rango de la matriz ampliada,


entonces pueden darse los siguientes casos:

1. r = r ′ = 1 ⇒ S. C. Indeterminado: planos coincidentes.

2. r = 1, r ′ = 2 ⇒ Incompatible: Observando la matriz ampliada puede ser que:


a) No haya dos filas proporcionales: los tres planos son paralelos.
b) Hay una fila proporcional a otra: dos planos son coincidentes y el tercero paralelo.

3. r = r ′ = 2 ⇒ S. C. Indeterminado: Observando la matriz ampliada puede ser que:


a) No haya dos filas proporcionales: tres planos distintos con una recta en común.
b) Hay una fila proporcional a otra: dos planos son coincidentes y el tercero se corta con
ellos en una recta.

4. r = 2, r ′ = 3 ⇒ Incompatible: Observando la matriz de los coeficientes puede ser que:


a) No haya dos filas proporcionales: los tres planos se cortan dos a dos en tres rectas
paralelas entre sí.
b) Hay una fila proporcional a otra: dos planos son paralelos y el tercero corta a los dos en
dos rectas paralelas.

5. r = r ′ = 3 ⇒ S. C. Determinado: los tres planos se cortan en un punto.

15-16
Ecuaciones de recta y plano Tema 15

Nota Para entender la interpretación basta considerar los vectores característicos y la


resolución de sistemas.

Ejemplo 15.27 Estudiemos la posición relativa de los tres planos siguientes:


π1 : x + 3 y − 4z = 7
π2 : 3 x − 2y + 3z = −5
π3 : 4 x + y − z = −3
( )
Estudiando los rangos de las matrices obtenemos: rang (A ) = 2 , rang A∗ = 3 ; por lo tanto, los
tres planos no tienen ningún elemento en común y se cortan dos a dos determinando tres
rectas paralelas.

15.8 Haz de planos

a) Haz de planos paralelos:


Si en la expresión de un plano cualquiera π : Ax + By + Cz + D = 0 se fijan los
coeficientes A, B y C dejando variar D, la nueva expresión: Ax + By + Cz + D = 0 representa un
conjunto de planos, llamado haz de planos paralelos.

Ejemplo 15.28 La ecuación general del haz de planos paralelos al plano π : 3 x − y + z = 3 es


3x − y + z = D .

Nota Para seleccionar el plano del haz que pasa por un punto dado, bastará sustituir sus
coordenadas en el haz y extraer de esta condición un valor concreto para D que corresponde al
plano particular pedido.

Ejemplo 15.29 Para hallar el plano del haz del ejemplo anterior que pasa por el punto (1,2,1) ,
basta calcular D y como ha de pasar por el punto (1,2,1) , sustituyendo se tiene que
3 − 2 + 1 = D → D = 2 . Por tanto, el plano del haz será: 3 x − y + z = 2 .

b) Haz de planos que pasan por una recta

Llamamos haz de planos de arista una recta r dada, al conjunto de los planos que
contienen a dicha recta r.
Supongamos que una recta viene dada en su forma implícita:
 Ax + By + Cz + D = 0
r ≡ .
 A′x + B ′y + C ′z + D ′ = 0

15-17
Ecuaciones de recta y plano Tema 15

Hemos visto que para que un plano π ≡ ax + by + cz + d = 0 contenga a la recta r debe


 A B C   A B C D 
   
suceder que rang  A′ B ′ C ′  = rang  A ′ B ′ C ′ D ′  = 2 y, por tanto, (a , b , c , d ) debe ser
 a b c   a b c d 
   
combinación lineal de los dos vectores que quedan en la matriz ampliada. De aquí se sigue que
la ecuación del haz de planos de arista la recta r es:

α(Ax + By + Cz + D ) + β(A′x + B ′y + C ′z + D ′) = 0 , con α y β no simultáneamente nulos.

Nota Para α = 0 y β = 0 se obtienen respectivamente los planos A′x + B′y + C ′z + D′ = 0 y


Ax + By + Cz + D = 0 que evidentemente contienen a r.
β
, la ecuación (Ax + By + Cz + D ) + µ(A′x + B′y + C ′z + D′) = 0 ,
Si α ≠ 0 y llamamos µ =
α
cuando µ varía en R, representa a todos los planos del haz de arista r salvo el plano
A′x + B′y + C ′z + D′ = 0 . Teniendo en cuenta esta consideración, resulta más sencillo trabajar
con (Ax + By + Cz + D ) + µ(A′x + B′y + C ′z + D′) = 0 que con la otra ecuación.

Ejemplos

 x + y + 2z = 4
Ejemplo 15.30 La ecuación del haz de planos que contiene a la recta r :  es
2 x − y + z = 3
α (x + y + 2z − 4 ) + β(2 x − y + z − 3 ) = 0 .

Ejemplo 15.31 La ecuación del plano del haz del ejemplo anterior que pasa por el punto
P (1,1,0 ) debe verificar: α(1+ 1+ 0 − 4 ) + β(2 − 1 + 0 − 3 ) = 0 → α = −β . En particular, tomando
α = 1 → β = −1 , se obtiene el plano pedido: − x + 2y + z = 1 . (Se obtendría el mismo plano para
cualquier otro valor de α ).

Ejemplo 15.32 Si se pregunta si el plano x + y + z + 2 = 0 pertenece al haz determinado por la


 x + 2y − z − 1 = 0
recta r :  , una forma de hacerlo es estudiar la posición relativa; esto es, ver
 x − 3 y + 4z + 2 = 0
1 2 − 1 1  1 2 −1
 
si el plano contiene a la recta. Se considera 1 − 3 4 2  y, como 1 − 3 4 ≠ 0,
1 1 1 2  1 1 1

( )
rang (A ) = rang A ∗ = 3 , luego el plano y la recta se cortan en un punto y el plano no pertenece
a ese haz.

c) Haz de planos que pasan por un mismo punto (radiación de planos)

Teniendo en cuenta la ecuación normal de un plano, se deduce que la ecuación del haz
de planos que pasan por un mismo punto P (x 0 , y 0 , z0 ) es
A ⋅ (x − x 0 ) + B ⋅ (y − y 0 ) + C ⋅ (z − z0 ) = 0 ,
donde variando A, B y C se van obteniendo los distintos planos.

15-18
Ecuaciones de recta y plano Tema 15

Ejemplo 15.33 Halla el plano que contiene al punto de corte de los planos
π1 ≡ 2 x − 2y − 2z − 4 = 0 , π 2 ≡ x + y − z = 0 , π 3 ≡ x + 2 y + 3z + 1 = 0 , y a la recta de ecuación
 x = 1+ α

r :  y = 2 − α , Debemos calcular el punto de corte de los planos que es la solución del sistema
z = 3 − 5α

4 −2 −2
0 1 −1
2 x − 2 y − 2z − 4 = 0 
 −1 2 3
x + y − z = 0 , utilizamos la Regla de Cramer, con lo que x= = 1,
2 −2 −2
x + 2y + 3z + 1 = 0
1 1 −1
1 2 3
2 4 −2 2 −2 4
1 0 −1 1 1 0
1 −1 3 1 2 −1
y= = −1 y z = = 0 El punto es P (1,−1,0 ) y la radiación de planos que
2 −2 −2 2 −2 −2
1 1 −1 1 1 −1
1 2 3 1 2 3
r
contiene a P es A ⋅ (x − 1) + B ⋅ (y + 1) + C ⋅ z = 0 . La recta tiene como vector director v (1,−1,−5 ) y
si se recuerda la ecuación normal del plano, se llega al plano de ecuación
1⋅ (x − 1) − 1⋅ (y + 1) − 5z = 0 ⇒ x − y − 5z − 2 = 0 .

15-19
Ecuaciones de recta y plano Tema 15

Ejercicios resueltos
Ejercicios básicos de ecuaciones de rectas y planos

R.1. Expresa mediante sus ecuaciones paramétricas, continuas e implícitas la recta r


r
que pasa por P (− 1,5,6 ) y es paralela al vector v ( 4, −8,12 ) .

Solución:
r
El vector w (1, −2, 3 ) es también vector director de r por ser paralelo al anterior, por tanto:
 x = −1 + α

 y = 5 − 2α son sus ecuaciones paramétricas.
 z = 6 + 3α

x +1 y − 5 z − 6 x +1 y −5
En forma continua = = . Operamos = ⇒ 2x + y − 3 = 0 ;
1 −2 3 1 −2
x +1 z − 6 2 x + y − 3 = 0
= ⇒ 3 x − z + 9 = 0 y las ecuaciones implícitas son  .
1 3 3x − z + 9 = 0

R.2. Obtén las ecuaciones paramétricas de la recta cuyas ecuaciones implícitas son
 y −2= 0
 .Indica un punto y un vector director de la recta.
2 x − z + 1 = 0

Solución:
 y =2
Podemos elegir z como parámetro y despejar x e y con lo que se tiene  las
2 x = z − 1
 1 1
x = − 2 + 2 α
  1  r 1 
ecuaciones paramétricas son:  y = 2 . Un punto es P  − ,2,0  , un vector v  , 0,1 .
z = α  2   2 


x -1 y z −3
R.3. Calcula las ecuaciones de la recta r que está contenida en π : 1 2 1 =0 y
2 1 −1
en π ′ : x − 3 y + 3 = 0 .

Solución:

Sólo se trata de buscar la ecuación general de π, desarrollando el determinante se


− x + y − z + 4 = 0
llega a − x + y − z + 4 = 0 con lo que la recta en forma implícita es r :  .
 x − 3y + 3 = 0

R.4. Ecuación general del plano que contiene a P (1,1,−2) y tiene como vectores directores
r r
v (2,1,3 ) y w (− 3,1,0 ) .

Solución:
Obtenemos primero la ecuación en forma de determinante y después la general.

x -1 y −1 z + 2
2 1 3 = 0 ⇒ −3 x − 9 y + 5z + 22 = 0 .
−3 1 0

15-20
Ecuaciones de recta y plano Tema 15

r
R.5. Ecuación del plano determinado por los puntos A (1,1, 0 ) , B ( 2, 2, 3 ) y v (1, 2, 0 ) .

Solución:
uuur
El vector AB (1,1, 3 ) es director del plano y por tanto su ecuación en forma de determinante es
x -1 y −1 z
1 1 3 = 0.
1 2 0

 x = 5 + 3t − 2s

R.6. Sea π el plano de ecuaciones  y = −2 + 4t − 5s . a) Determina dos puntos del plano
z = 6 − t + s

π.
b) Calcula la ecuación de dos rectas secantes contenidas en π . c) Expresa π mediante
su ecuación general.

Solución:

a) Para t = 0,s = 0 → P ( 5, −2, 6 ) . Para t = 1,s = 0 → Q ( 8, 2, 5 ) .


 x = 5 + 3t  x = 5 − 2s
 
b) Las ecuaciones de dos rectas secantes contenidas en π :  y = −2 + 4t y  y = −2 − 5s
z = 6 − t z = 6 + s
 

 x − 5 = 3t − 2s

c) Dejamos solos los parámetros t y s. Luego los eliminamos.  y + 2 = 4t − 5s Las matrices del
 z − 6 = −t + s

 3 −2 x − 5 
 
sistema son  4 −5 y + 2  Obligaremos que el determinante de la matriz ampliada sea cero:
 −1 1 z − 6 
 
3 −2 x − 5
4 −5 y + 2 = 0 ⇒ x + y + 7 z − 45 = 0 .
−1 1 z−6
 x = 3t

R.7. Demuestra que la recta de ecuaciones paramétricas  y = 5t está contenida en el
z = 2 + t

plano x − y + 2z − 4 = 0 .

Solución:

Cualquier punto de la recta es de la forma ( 3t, 5t, 2 + t ) . Si la recta está contenida en el plano,
cualquier punto de ella debe satisfacer la ecuación del plano. Veamos, pues, que t se verifica la
ecuación del plano: x − y + 2z − 4 = 0 3t − 5t + 2 ( 2 + t ) − 4 = 0 → 0 ⋅ t = 0 que se cumple
∀t ∈ R .

15-21
Ecuaciones de recta y plano Tema 15

Posiciones relativas de rectas y planos

 2 x + 3 y − 5 z = −7

R.8. Estudia la posición relativa de los planos  3 x + 2y + 3z = 1
7 x + 8 y − 7 z = −13

Solución:

Estudiamos el sistema formado por las ecuaciones de los tres planos. Las matrices asociadas
 2 3 −5 −7  2 3 −5 −7  2 3 −5 −7 
     
son:  3 2 3 1  ↔  0 −5 21 23  2f2 − 3f1 ↔  0 −5 21 23  como
 7 8 −7 −13   0 −5 21 23  2f3 − 7f1 0 0 0 
0  f3 − f2
   
rang ( A ) = rang ( A∗ ) = 2 los planos tienen una recta en común.

x = 4 − t
x y −1 z − 3 
R.9. Estudia la posición relativa de las rectas r : = = y s : y = 7 .
2 3 −1 z = 1 + 2t

Solución:

 P ( 0,1, 3 )  Q ( 4, 7,1) r r
La recta r viene determinada por  r y s por  r como el rango de {v ,w } es 2,
v ( 2, 3, −1) w ( −1, 0, 2 )
uuur
las rectas tienen distinta dirección. El vector PQ = ( 4, 6, −2 ) , estudiamos el rango de
2 3 −1
r r uuur
{ }
v ,w ,PQ , −1 0 2 = 0 el rango es 2 por lo que las rectas se cortan.
4 6 −2
x + 2 y −1 z
R.10. Estudia la posición relativa de la recta r : = = y el plano
1 4 3
π : x + 5 y − 3z = 15

Solución:
 x = −2 + t

Las ecuaciones paramétricas de la recta r son r :  y = 1 + 4t Calculamos la intersección de la
z = 3t

recta y el plano −2 + t + 5 (1 + 4t ) − 3 ( 3t ) = 15 → t = 1 por tanto la recta y el plano se cortan en
un punto.

R.11. Estudia la posición relativa de los siguientes planos: π1 : ax + y + z = 1 ;


π2 : x + ay + z = 1 ; π3 : x + y + az = 1 según los valores del parámetro a ∈ R .

Solución:

Discutimos el sistema formado por las ecuaciones de los tres planos. Las matrices asociadas
a 1 1 1
 
son:  1 a 1 1 . Calculamos el determinante de la matriz de los coeficientes, para ver qué
1 1 a 1

15-22
Ecuaciones de recta y plano Tema 15

a 1 1
valores de a lo anulan. 1 a 1 = 0 ⇔ a 3 − 3a + 2 = 0 ⇔ a = 1 y a = −2. y analizando para
1 1 a
estos valores los rangos de las dos matrices concluimos que:

1. si a = 1 → rang ( A ) = rang ( A∗ ) = 1 ⇒ planos coincidentes.

2. si a = −2 → rang ( A ) = 2, rang ( A∗ ) = 3 y como los tres sistemas formados por parejas de


ecuaciones son compatibles π1 y π2 se cortan en una recta y π3 es paralelo a ella, pero no la
contiene.

3. si a ∈ R − {1, −2} → rang ( A ) = rang ( A∗ ) = 3 ⇒ Los tres planos se cortan en un punto.

R.12. Determina para qué valores del parámetro real p se cortan las rectas de
x = 1− a
 x + 2y − z = 5 
ecuaciones: r :  ; s : y = 2+a .
3 x + y + z = p z
 = 3a
Solución:

Pasando las ecuaciones de la recta s a forma implícita, obtenemos el siguiente sistema:

 x + 2y - z = 5 1 2 −1 5 
 3x + y + z = p  
 3 1 1 p
 de matrices asociadas . Como el rang ( A ) = 3 , se cortarán
 3 x + z = 3 3 0 1 3
 
 3 y - z = 6 0 3 −1 6 
cuando rang ( A∗ ) = 3 , por lo que debemos hallar el valor de p que hace cero el determinante
1 2 −1 5
3 1 1 p
de la matriz ampliada. = 0 ⇒ p = 5 , luego para p = 5 las rectas se cortan.
3 0 1 3
0 3 −1 6

Otras determinaciones de rectas y planos

R.13. Ecuación del plano determinado por los puntos A ( 2,1, 0 ) , B(5,2,1) y C (3,2,−1) .
Obtén un vector característico o perpendicular al plano.

Solución:
uuur uuur
Los vectores AB ( 3,1,1) y AC (1,1,−1) son paralelos al plano y como punto podemos elegir
x − 2 y −1 z
uno cualquiera, elegimos A, la ecuación es 3 1 1 = 0 ⇒ −2 x + 4 y + 2z = 0 y el vector
1 1 −1
r
v (− 2,4,2) es el asociado del plano.

R.14. Halla la ecuación del plano que pasa por el punto P (1,1,2) y es paralelo al plano
π : x − y + 3z − 4 = 0 .

15-23
Ecuaciones de recta y plano Tema 15

Solución:

Se puede plantear de dos modos muy parecidos.

Modo I

El plano tiene el mismo vector característico por lo que su ecuación es de la forma


x − y + 3 z = D como debe pasar por P sustituimos sus coordenadas y se tiene
1 − 1 + 6 = D ⇒ D = 6 y la ecuación x − y + 3z = 6

Modo II

Directamente imponemos la condición de que P pertenezca al plano la ecuación es.


1 ⋅ ( x − 1) − 1 ⋅ ( y − 1) + 3 ⋅ ( z − 2 ) = 0 ⇒ x − y + 3z − 6 = 0 .

R.15. Calcula las ecuaciones de la recta r que pasa por A (1,1, −1) y es paralela a la recta
 x +y −z =1
s: .
2 x + y + z = 2

Solución:

Buscamos un vector director de s que será también director de r. Como s viene dada como
r r
intersección de dos planos de vectores característicos v (1,1, −1) y w (2,1,1)
r r r
i j k  x = 1 + 2t
r r 
v ∧ w = 1 1 −1 = ( 2, −3, −1) es el vector director buscado, la recta sería  y = 1 − 3t .
2 1 1  z = −1 − t

R.16. Calcula las ecuaciones de la recta r que pasa por P (3,1,−5 ) y es perpendicular al
plano π : x − 2y − 3z + 1 = 0 .

Solución:
r
El vector v (1, −2, −3 ) es ortogonal a π y por lo tanto es un vector director de r, así que las
x = 3 + t

ecuaciones de r son  y = 1 − 2t .
 z = −5 − 3t

R.17. Calcula las ecuaciones de la recta que pasa por P (3,3,2) y es paralela a los planos
π : x + y + z = 0 y π′ : 2x − y − z = 2

Solución:
r r
Los vectores característicos de los planos son v (1,1,1) y w ( 2, −1, −1) y su producto vectorial
x = 3
r r 
v ∧ w = ( 0, 3, −3 ) es un vector director de la recta. Sus ecuaciones son y = 3 + 3α .
z = 2 − 3α

R.18. Halla la ecuación del haz de planos que tiene como arista la recta que pasa por
A (1, 2,1) y B ( 2, −3, −2 ) . Calcula la ecuación del plano perteneciente a ese haz que:
a) Pasa por el punto P (1,1,1) .

15-24
Ecuaciones de recta y plano Tema 15

 x − 2y + z = 4
b) Es paralelo a la recta de ecuaciones  .
2 x + 4 y + z + 1 = 0

Solución:

Calculamos las ecuaciones implícitas de la recta que pasa por A y B. Un vector director es
uuur x −1 y − 2 z −1  5x + y − 7 = 0
BA = ( −1, 5, 3 ) sus ecuaciones en forma continua = = ⇒ .
−1 5 3 3 y − 5z − 1 = 0
La ecuación del haz es:

α ( 5 x + y − 7 ) + β ( 3 y − 5z − 1) = 0 ⇔ ( 5α ) x + ( α + 3β ) y + ( −α − 5β ) z + ( −7α − β ) = 0 .

a) Sustituimos las coordenadas de P y elegimos un valor de α y de β que satisfaga la


igualdad ( 5α ) + ( α + 3β ) + ( −α − 5β ) + ( −7α − β ) = 0 → −2α − 3β = 0 si α = 3 → β = −2
con estos valores el plano es el de ecuación 15 x − 3 y + 7 z − 19 = 0 .

b) Un vector director de la recta dada es (1, −2,1) ∧ ( 2, 4,1) = ( −6,1, 8 ) y haciendo, por
ejemplo y = 2 encontramos un punto de la recta (1, 2,1) .

Si la recta y el plano han de ser paralelos el producto escalar del vector característico del
plano y el director de la recta es 0.

Elegimos unos valores de α y de β que cumplan la condición.

−6 ( 5α ) + 1( α + 3β ) + 8 ( −α − 5β ) = 0 → α + β = 0 si α = 1 → β = −1 con esos valores el plano


que se obtiene es −5 x − 2y + 4z − 6 = 0 .

El plano obtenido podría contener a la recta dada, así que para comprobar que son
realmente paralelos debemos sustituir las coordenadas (1, 2,1) en la ecuación del plano
−5 ⋅ 1 − 2 ⋅ 2 + 4 ⋅ 1 − 6 ≠ 0 luego la recta y el plano son paralelos.

R.19. halla la ecuación del plano que pasa por el punto P (3,2,−1) y contiene a la recta
 x+y −z =3
r : .
2 x − y − 2z = 0

Solución:

El ejercicio podría resolverse como el apartado a) del ejercicio anterior buscando la


ecuación del haz de planos de arista r. Lo resolvemos de otro modo.
x = t

Pasamos la recta a forma paramétrica r :  y = 2 de donde se lee que uno de los
 z = −1 + t

r
puntos de la recta es Q ( 0, 2, −1) y un vector director es v (1, 0,1) .

r
El plano queda determinado por el punto P (3,2,−1) y los vectores v (1, 0,1) y
x - 3 y − 2 z +1
uuur
QP ( 3, 0, 0 ) , por lo que la ecuación del plano es 1 0 1 = 0 ⇒ y −2 = 0.
3 0 0

15-25
Ecuaciones de recta y plano Tema 15

R.20. Halla la ecuación del plano que es perpendicular al plano 2 x − y + 3z − 1 = 0 ,


x+1 y z −8
paralelo a la recta = = , y que pasa por P (1,1,1) .
2 3 −1
Solución:

Por el enunciado, podemos considerar como vectores dirección del plano pedido: el
característico del plano dado ( 2, −1, 3 ) y el de la recta dada ( 2, 3, −1) . La ecuación del plano
pedido vendrá dada por:
x −1 y −1 z −1
2 −1 3 = 0 ⇒ −x + y + z − 1 = 0 .
2 3 −1

R.21. Calcula la ecuación del plano que pasa por A (1,1, 3 ) y es perpendicular a los planos
π1 : x + 2y − 3z = 0 y π2 : − x + y + z − 8 = 0 .

Solución:

Modo I

Se puede poner directamente la ecuación del plano pedido en forma de determinante ya que
los vectores asociados a π 1 (1,2,−3) y a π 2 (− 1,1,1) son paralelos al plano pedido. Su
x -1 y −1 z − 3
ecuación es 1 2 −3 = 0 ⇒ 5 x + 2y + 3z − 16 = 0 .
−1 1 1
Modo II
r
El vector w = (1, 2, −3 ) ∧ ( −1,1,1) = ( 5, 2, 3 ) es ortogonal al plano y su ecuación
5 ( x − 1) + 2 ( y − 1) + 3 ( z − 3 ) = 0 ⇒ 5 x + 2y + 3z − 16 = 0 .

R.22. Halla la ecuación de la recta que pasa por el punto P (1,1, 2 ) y corta a las rectas
x −1 y x z −1
r : = = 1 − z y s : = y == .
3 2 2 2
Solución:

1. Vemos la posición relativa de r y s. Como


rang {( 3, 2, −1) , ( 2,1, 2 )} = 2 las rectas tienen distinta
dirección. El vector que va del punto P1 (1, 0,1) de r al
uuuur
punto P2 ( 0, 0,1) de s es P1P 2 ( −1, 0, 0 ) y como
rang {( 3, 2, −1) , ( 2,1, 2 ) , ( −1, 0, 0 )} = 3 , las rectas se cruzan.
x -1 y −1 z − 2
2. Plano que pasa por P y contiene a r: 3 2 −1 = 0 ⇒ x − y + z − 2 = 0 .
0 1 1

x -1 y −1 z − 2
3. Plano que pasa por P y contiene a s: 2 1 2 = 0 ⇒ x − z +1= 0
1 1 1

15-26
Ecuaciones de recta y plano Tema 15

Por lo tanto, la recta pedida es la formada por las dos anteriores ecuaciones; esto es, la
x − y + z − 2 = 0
intersección de los dos planos obtenidos.  que son sus ecuaciones implícitas.
x − z + 1 = 0

R.23. Halla unas ecuaciones paramétricas de la recta t que es paralela a los planos
p : x + 2y + 5z = 8 y q : 2 x − y = 5 y pasa por la intersección de las rectas
 x − y + 2z = 1 x + y + z = 3
r : , s: .
y + z = 1 2 x + y − z = 5

Solución:

1. Hallamos el punto H intersección de las rectas, resolviendo el sistema por el método Gauss:

 1 −1 2 1  1 −1 2 1  1 −1 2 1  1 −1 2 1
       
0 1 1 1
↔
0 1 1 1
↔
0 1 1 1
↔
0 1 1 1
de donde,
1 1 1 3  0 2 −1 2   0 0 −3 0  0 0 −3 0
       
 2 1 −1 5   0 3 −5 3   0 0 −8 0  0 0 0 0 
x − y + 2z = 1 

y + z = 1  ⇒ z = 0, y = 1, x = 2 ⇒ H ( 2,1, 0 ) .
−3z = 0 

2. Construimos un plano paralelo a p, que pase por (2,1,0): será x + 2y + 5 z = D ; sustituimos


las coordenadas del punto H queda D = 4 . Luego, se trata del plano x + 2 y + 5z = 4 .

3. Construimos un plano paralelo a q que pase por (2,1,0): será 2 x − y = C ; sustituimos las
coordenadas del punto H, obteniendo C = 3 . Luego el plano es 2 x − y = 3 .
 x + 2y + 5z = 4
4. La intersección de estos dos planos es la recta pedida  y en forma
2 x − y = 3
x=β

paramétrica  y = −3 + 2β .
 z = 2−β

R.24. Halla las ecuaciones de una recta que pasa por P (1,1,1) , es coplanaria a la recta
y z
r : x −1= = y es paralela al plano π : − x + 2y + z = 0 .
2 3

Solución:

1. Hallamos la ecuación del plano π′ paralelo a π y que pasa por P . La ecuación de un plano
cualquiera paralelo a π es de la forma − x + 2y + z = D , si obligamos a que pase por P se tiene
−1 + 2 + 1 = D ⇒ D = 2 y la ecuación de π′ es − x + 2y + z = 2 .

2. Calculamos el punto Q intersección de π′ y r , las ecuaciones paramétricas de r son

15-27
Ecuaciones de recta y plano Tema 15

x = 1 + t
 1
 y = 2t sustituyendo en la ecuación de π′ se tiene − 1 − t + 4t + 3t − 2 = 0 ⇒ t = y
 z = 3t 2

3 3
sustituyendo en las ecuaciones de r obtenemos el punto Q  ,1,  .
2 2
 1
x = 1 + 2 t
uuur  1 1 
3. La recta pedida pasa por P y Q, como PQ  , 0,  las ecuaciones son  y = 1 .
2 2  1
z = 1 + t
 2

R.25. Calcula las ecuaciones de la recta r que pasa por el punto P(2,−1,0) y corta
x = 1 + t

perpendicularmente a s :  y = −2 + 3t .
z = 2

Solución:

1. Hallamos la ecuación del plano π que pasa por P y es


perpendicular a s. Su ecuación sería
1 ⋅ ( x − 2 ) + 3 ⋅ ( y + 1) + 0 ⋅ z = 0 ⇒ x + 3 y + 1 = 0 .

2. Calculamos el punto H de corte de π con s


2 7 4 
1 + t + 3 ⋅ (− 2 + 3t ) + 1 = 0 ⇒ t = y H  ,− ,2  .
5 5 5 

uuur  3 1  uuur
3. La recta pedida pasa por P y H , PH  − , , 2  , el vector 2PH = ( −3,1,10 ) es paralelo al
 5 5 
x − 2 y +1 z
anterior y por tanto sus ecuaciones son: = = .
−3 1 10

15-28
Ecuaciones de recta y plano Tema 15

Ejercicios propuestos

P.1. ¿Están alineados los puntos A(− 1,8,7 ) , B(4,1,5 ) y C (− 7,6,5 ) ?.

P.2. Halla t para que los puntos P (3,2,0 ) , Q(0,−1,0 ) , R (3, t ,1) y S (2,1,−2) sean coplanarios.

P.3. Halla las ecuaciones paramétricas de los planos XZ, XY, YZ.

P.4. Halla la ecuación del plano que pasa por los puntos (2,−1,0 ), (3,0,2) y (2,−3,4 ) .

P.5. Halla las ecuaciones paramétricas del plano que pasa por el punto (2,3,−1) y es paralelo
al plano 2 x − 3 y + z = 3 .
x −1 y − 2 z +1
P.6. Sea la recta r de ecuación = = . Determina la ecuación de un plano que
5 −3 7
contenga a r.

P.7. Halla la ecuación del plano que pasa por el punto (1,1,2) y es paralelo a las rectas
 3x + y = 0  2 x − 2y = 0
r : y r′:  .
 4x + z = 0  y −z =3

P.8. Escribe la ecuación del plano que pasa por el origen y contiene a la recta r de ecuaciones:
x + y + z = 1
r : .
x − y = 2
x −1 y −1
P.9. Halla la ecuación del plano que pasa por la recta = =z y es paralelo a la recta
2 3
que pasa por (2,0,0 ) y (0,1,0 ) .

P.10. Halla la ecuación del plano que pasa por los puntos A(5,0,1) y B(4,1,0 ) y es paralelo a la
 x − 3 y − 11 = 0
recta: r :  .
4 y − z + 4 = 0

x + y − z + 3 = 0 3 x − y + 4z = 0
P.11. Comprueba si las rectas r :  y r′:  se cortan.
 2 x − 3 y + z − 8 = 0 8 x + 5 y − 8z − 1 = 0
Encuentra, en caso afirmativo, el punto de corte.

y −1 z + 2 x+5 y −3 z
P.12. Se consideran las rectas r : x − 1= = y r′ : = = , demuestra que
−1 4 4 −2 3
se cortan en un punto y encuentra la ecuación del plano que determinan.

P.13. Estudia la posición relativa de los planos: π : 2 x + 3 y − 5z + 7 = 0 y π ′ : 3 x + 2y + 3z + 1 = 0


y π ′′ : 7 x + 8 y − 7z + 13 = 0 .

P.14. Sean los planos π : mx + y + z − 1 = 0, π ′ : x + my + z − 1 = 0, π ′′ : x + y + z − 1 = 0 . Estudia


su posición relativa dependiendo de los valores del parámetro m.

15-29
Ecuaciones de recta y plano Tema 15

P.15. Sean π : 3 x − y + 4z = 0, π ′ : 5 x − 4 y + 3z + 2 = 0 . Se pide: a) ecuación del plano paralelo a


π y que pasa por (− 1,2,3 ) ; b) ecuación de la recta r determinada por π y π ′ ; c) ecuación del
plano perteneciente al haz de arista r y que pasa por (− 7,3,2) .

P.16. Determina t ∈ R para que las rectas r y s siguientes se corten en un único punto. Calcula
x + y + z = 2 tx − y − z = 1
el punto de corte. r :  s: .
 x + 2 y − 3z = 8  x − y + z = −2

 x = 1 + 2α
  x+ y + z=1
P.17. Calcula t para que las rectas r : y = 2 + α s:  estén situadas en un
z = α  x − 2y + 2z = t

plano.

P.18. Calcula t para que las rectas r y s se corten en un punto. Encuentra ese punto.
x −1 y + 4 z+1  x + 2y + z = t
r: = = ; s:
2 −3 5  2 x − y − z = −2

x − y + z − 1 = 0  x − y −3= 0
P.19. Estudia la posición relativa de las rectas: r :  s: .
 x + 2y + z + 2 = 0  2y + z = 0

 x = z −1  x − 4 = 5z
P.20. Dadas las rectas r :  y s:
 y = 2 − 3z  y = 4z − 3
a) Indica si se cortan, son paralelas o se cruzan. b) Halla las ecuaciones de la recta que pasa
por el origen y corta a las dos.

P.21. Determina las ecuaciones de la recta que pasa por el punto (0,1,−1) y es secante a las
x 1 x −1
rectas r : = y = z − , r′: = y −2=z .
3 3 2

P.22. Halla la ecuación del plano que pasa por el punto P (− 1,2,1) , es perpendicular al
x − y = 0
plano x − y − 2z = 0 y paralelo a la recta: r :  .
z = 2

P.23. Obtén la ecuación de un plano que contenga a la recta r y sea perpendicular al plano π ,
 x=t −s
x −1 y + 4 z+1 
siendo: r: = = , π: y=t .
2 −3 5  z= s

15-30
Ecuaciones de recta y plano Tema 15

Soluciones

x = t x = t x = 0
  
P.1. No. 2. t = 2 . P. 3. XZ : y = 0 ∀s, t ∈ R XY :  y = s ∀s, t ∈ R , YZ :  y = t ∀s, t ∈ R
z = s z = 0 z = s
  
x = α

P.4. 4 x − 2y − z − 10 = 0 . P.5. y = β . P.6. Hay infinitos planos con ecuación
z = −6 − 2α + 3β

vectorial (x, y, z ) = (1,2,−1) + t (5,−3,7) + s (a, b, c ), con (a, b, c )∈ R 3 . P.7. x − 5 y + 4z − 4 = 0 . P.8.
x + 3 y + 2z = 0
P.9. x + 2 y − 8z − 3 = 0 . P.10. 5 x + y − 4z − 21 = 0 . P.11. Se cruzan. P.12.
{ }
rang {(1,−1,4 ), (4,−2,3 )} = rang (1,−1,4 ), (4,−2,3 ), PQ (6,−2,−2) = 2 , x + 5 y + 2z − 2 = 0 . P.13. Se
cortan dos a dos sin paralelismo. P.14. Si m = 1 planos coincidentes; si m ≠ 1 se cortan en un
3 x − y + 4z = 0
punto. P.15. a) 3 x − y + 4z − 7 = 0 ; b)  ; c) 37 x + 25 y + 108 z − 32 = 0 . P.16.
5 x − 4 y + 3z + 2 = 0
25  57 47 
t = 2 , (1,2,−1) . P.17. t = −4 P.18. t = − ,  − 8, ,−
2  6
 . P.19. rang(A) = 3; rang A∗ = 4 , por
2 
( )
2 x + y + z = 0 − y + z + 2 = 0
tanto se cruzan. P.20.. a) Se cruzan; b)  . P.21.  .
3 x + 4 y − 31z = 0 − 7 x + 12y + 9z − 3 = 0
P.22. x − y + z + 2 = 0 . P.23. 2 x + 3 y + z + 11 = 0 .

15-31
Proyecciones, distancias y ángulos Tema 16

Tema 16

Proyecciones, distancias y ángulos

16.1 Proyecciones. Puntos simétricos


a) Punto medio de un segmento. Punto simétrico respecto de un punto.

Dados los puntos P (x1 , y 1 , z1 ) y Q(x 2 , y 2 , z2 ) , si M (x 0 , y 0 , z0 ) es el punto medio del


→ →
segmento que une P con Q, entonces PQ = 2 PM y, en consecuencia, es inmediato que
x + x2 y + y2 z + z2
x0 = 1 , y0 = 1 , z0 = 1 . (16.1)
2 2 2

Ejemplos

Ejemplo 16.1 Para hallar el punto medio M del segmento AB ,con A(1,2,3 ) y B(5,2,1) , basta
aplicar las fórmulas de (16.1) y se llega a M (3,2,2)

El punto P ′(x ′, y ′, z ′) ,simétrico de P (x1 , y 1 , z1 ) respecto de Q(x 2 , y 2 , z2 ) es el que


→ →
cumple la relación PP ′ = 2 PQ , por lo que hallar sus coordenadas es inmediato.

Ejemplo 16.2 Sean los puntos A(1,2,3 ) y B(5,2,1) , para hallar el punto A' ≡ ( x , y , z ) simétrico
de A respecto de B, basta tener en cuenta que
→ →
AA′ = 2 AB → (x ′ − 1, y ′ − 2, z ′ − 3 ) = 2(4,0,−2) → A′(9,2,−1) .
Otra forma: También se podría haber resuelto considerando que B es el punto medio del
1+ x 2+y 3+z
segmento AA’, por lo que 5 = , 2= ,1 = y, despejando, se obtiene
2 2 2
A' ≡ ( 9 , 2 , − 1 ) .

b) Proyección de un punto sobre un plano. Punto simétrico respecto de un plano.

Definición Dado un punto P (x 0 , y 0 , z0 ) exterior al plano π : Ax + By + Cz + D = 0 , se llama


proyección ortogonal de P sobre π al punto H (x1 , y 1 , z1 ) , que es la base de la perpendicular
trazada desde P al plano.

16-1
Proyecciones, distancias y ángulos Tema 16

El punto simétrico al punto P respecto del plano π es el punto P ′(x ′, y ′, z ′) tal que H es el
→ →
punto medio del segmento PP’ ; esto es, PP ′ = 2 PH .

Ejemplo 16.3 Para hallar la proyección, H, de P (1,2,3 ) sobre el plano π : 3 x + y + z − 1 = 0 ,


primero se halla la recta que pasa por P y es perpendicular al plano; después se calcula la
intersección de ésta con el plano dado.
El vector (3,1,1) perpendicular al plano es un vector director de la recta, por tanto las
 x = 1 + 3α

ecuaciones de la recta son:  y = 2 + α . Sustituyendo en la ecuación del plano, se tiene que H
z =3+α

7  10 15 26 
3(1 + 3α ) + (2 + α ) + (3 + α ) − 1 = 0 → α = − , y por tanto, H ≡  − , , .
11  11 11 11 
Para hallar el punto P’ simétrico de P respecto de ese mismo plano, basta tener en
cuenta que H es el punto medio de PP’ , luego
10 x ′ + 1 15 y ′ + 2 26 z ′ + 3  31 8 19 
− = , = , = , de donde P ′ ≡  − , , .
11 2 11 2 11 2  11 11 11 

c) Proyección de un punto sobre una recta. Punto simétrico respecto a una recta.

Definición Dado un punto P (x 0 , y 0 , z0 ) exterior a una recta r , se llama proyección


ortogonal de P sobre r al punto H (x1 , y 1 , z1 ) intersección de r con el plano
π : Ax + By + Cz + D = 0 perpendicular a r que contiene a P .

El punto simétrico de P respecto a la recta anterior es el punto P ′(x ′, y ′, z ′) tal que


→ →
PP ′ = 2 PH .

Ejemplos

x −1 y
Ejemplo 16.4 Para calcular la proyección de P (1,2,−2) sobre la recta r : = =z:
3 2
1º Se halla la ecuación del plano π perpendicular a r y que pasa por P:
r
como el vector u (3,2,1) , que es director de la recta, es perpendicular al plano, la ecuación de
éste es 3(x − 1) + 2(y − 2) + (z + 2 ) = 0 → 3 x + 2y + z − 5 = 0 .
 10 2 1 
2º Se halla la intersección de la recta r y el plano π que es fácil ver que es H ≡  , ,  .
 7 7 7

x −1 y
Ejemplo 16.5 Para hallar el punto P’ simétrico de P (1,2,−2) respecto de r : = = z se
3 2
calcula el punto H como en el ejemplo anterior y, a continuación, se repite el proceso del
 13 10 16 
ejemplo 16.3. La solución es P ′ ≡  ,− ,  .
 7 7 7 

16-2
Proyecciones, distancias y ángulos Tema 16

16.2 Distancia de un punto a un plano. Distancia entre planos paralelos.


Distancia de una recta a un plano paralelo.
a) Distancia de un punto a un plano

Dados un plano π : Ax + By + Cz + D = 0 y un punto P (x 0 , y 0 , z0 ) exterior al plano, se define


la distancia del punto al plano como la distancia del punto P a su proyección ortogonal
sobre el plano π .

Demostraremos que:

Ax 0 + By 0 + Cz0 + D
d (P , π) = . (16.2)
2 2 2
A + B +C

En efecto, sea H (x1 , y 1 , z1 ) la proyección de


P sobre el plano π . La distancia del punto al plano

es el módulo del vector HP , que es paralelo al
r
vector v (A, B ,C ) perpendicular al plano.
→ r → r
Por la definición de producto escalar se tiene que HP ⋅ v = HP ⋅ v ⋅ (± 1) , pues el ángulo

r →
que forman v y HP mide 0º o 180º, dependiendo de que tengan o no el mismo sentido.

→ r → r
En cualquiera de los dos casos puede afirmarse que HP ⋅ v = HP ⋅ v y, en función

de las coordenadas, puede escribirse

(x 0 − x 1 )A + (y 0 − y 1 )B + (z 0 − z1 )C = d (P , π) ⋅ A 2 + B 2 + C 2 ,

de donde, operando y despejando, se llega a

(Ax 0 + By 0 + Cz0 ) − (Ax1 + By 1 + Cz1 )


d (P , π ) =
A2 + B 2 + C 2
y, como H (x1 , y 1 , z1 ) es un punto de π debe verificar su ecuación − (Ax1 + By 1 + Cz1 ) = D , por
lo que la fórmula de la distancia de un punto a un plano queda
Ax 0 + By 0 + Cz0 + D
d (P , π) = .
A2 + B 2 + C 2

Nota La distancia de un punto P a un plano también se puede calcular hallando las


coordenadas de su proyección, H, sobre el plano y calculando después la distancia entre esos
dos puntos, pero es claro que este método bastante más laborioso.

b) Distancia entre planos paralelos

La distancia entre dos planos paralelos es la distancia de un punto cualquiera de uno


de ellos al otro.

Si dos planos π y π′ son paralelos, sus ecuaciones generales respectivas siempre


podrán ponerse de la forma π : Ax + By + Cz + D = 0 y π′ : Ax + By + Cz + D ′ = 0 .

16-3
Proyecciones, distancias y ángulos Tema 16

Si un punto P (x 0 , y 0 , z0 ) ∈ π , entonces Ax 0 + By 0 + Cz0 = −D y, aplicando la fórmula


(16.2) de la distancia de P a π′ se obtiene

Ax 0 + By 0 + Cz0 + D ′ D′ − D
d (P , π′) = = (16.3)
2 2 2
A + B +C A + B2 + C2
2

como fórmula para la distancia entre dos planos paralelos cuyas ecuaciones generales son de
la forma anterior.

Ejemplo 16.6 La distancia entre los planos paralelos π : 4 x + 7 y − 2z + 1 = 0 y


1+ 3 4 4 69
π ′ : 4 x + 7 y − 2z − 3 = 0 será d (π,π ′) == = = .
42 + 7 2 + 22 69 69

c) Distancia de una recta a un plano paralelo

La distancia entre una recta y un plano paralelos es igual a la distancia desde cualquier
punto de la recta al plano.

 x − 2 y − 2z = 1
Ejemplo 16.7 Para hallar la distancia entre la recta r :  y el plano
 x + 5y − z = 0
π : 14 x + 7 y − 23 z = 0 , primero se comprueba que la recta y el plano son paralelos.

Para ello resolvemos el sistema formado por las tres ecuaciones aplicando el método
de Gauss las matrices del sistema son

 1 −2 −2 1  1 −2 −2 1   1 −2 −2 1 
     
 1 5  
−1 0 ⇔ 0 7  
1 −1 ⇔ 0 7 1 −1 
     
     
 14 7 − 23 0   0 35 5 − 14   0 0 0 −9 

Como el rango de A es 2 y el rango de A ∗ es 3, la recta y el plano son paralelos.

Entonces la distancia de cualquier punto de la recta al plano es la pedida.

Se obtiene un punto de la recta haciendo, por ejemplo y = 0 , con lo que el punto


14 + 23 9
P (− 1,0,−1) es un punto de la recta y en definitiva d (P , π ) = = .
2 2 2 774
14 + 7 + 23

La distancia entre una recta y un plano que no sea paralelo es cero.

16.3 Distancia de un punto a una recta. Distancia entre rectas paralelas


a) Distancia de un punto a una recta

Se define la distancia de un punto P a una recta r como la distancia entre P y su


proyección ortogonal P’ sobre r.

16-4
Proyecciones, distancias y ángulos Tema 16

Esta distancia se puede hallar de dos modos:

Método I:

Calculando el punto P’ y hallando la distancia de P a P’.


Q(x , y , z )
Dados el punto P (x 0 , y 0 , z0 ) y la recta r :  r 1 1 1 se siguen los siguientes pasos:
v (v 1 , v 2 , v 3 )

1. Se halla la ecuación del plano perpendicular a r desde P: el vector director de


r
la recta v (v 1 , v 2 , v 3 ) será perpendicular al plano y por lo tanto su ecuación será
v 1 ⋅ (x − x 0 ) + v 2 ⋅ (y − y 0 ) + v 3 ⋅ (z − z 0 ) = 0 .

2. Se halla el punto P’ intersección de r y el plano anterior resolviendo el sistema.

3. Se calcula la distancia de P a P’.

Método II:
r (16.4)
v ∧ QP
A partir de la fórmula d (P , r ) = r ,
v
r
donde Q es un punto cualquiera de la recta y v el vector
dirección de la misma.

Recordando la interpretación del módulo del producto


r
vectorial se sabe , por un lado, que el área del paralelogramo con lados paralelos a QP y v es
r
A = v ∧ QP .
r
Por otra parte, por geometría elemental, A = v ⋅ d (P , r ) . Igualando ambas expresiones
r
v ∧ QP
y despejando resulta: d (P , r ) = r .
v

b) Distancia entre rectas paralelas

La Distancia entre dos rectas paralelas será la de un punto cualquiera de una de ellas a
la otra.

x −1 z−2  x − 2y − 3 = 0
Ejemplo 16.8 Para calcular la distancia entre r : =y = y s: , se
2 3  3y − z + 1 = 0
estudia la posición relativa de las rectas.
Como el punto (1,0,2) de r no pertenece a s y los vectores directores de r y s son
(2,1,3 ) y (1,−2,0 ) ∧ (0,3 − 1) = (2,1,3 ) , respectivamente, son paralelas. Por lo tanto, se puede
aplicar la fórmula (16.4) con P ≡ (1,0,2) . El punto de s se calcula haciendo, por ejemplo y = 1 ,
r
con lo que se obtiene el punto Q ≡ (5,1,4 ) . Como PQ = (4,1,2) y v = (2,1,3 ) , entonces:

(2,1,3) ∧ (4,1,2) (− 1)2 + 8 2 + (− 2)2 69


d = = = .
(2,1,3) 14 14

16-5
Proyecciones, distancias y ángulos Tema 16

16.4 Distancia entre dos rectas que se cruzan

La distancia entre dos rectas se define como la menor de las distancias entre dos puntos
cualesquiera de las mismas.
Se demuestra que, dadas dos rectas r y s que se cruzan, la mínima distancia entre
ambas es la distancia desde un punto cualquiera de r al plano que, conteniendo a s, es paralelo
ar.
Veamos dos métodos diferentes para calcular esta distancia.
r r
Sean r ≡ P + < u > y s ≡ Q + < v > .

Método I

Consiste en:

1. Se calcula la ecuación del plano que contiene a s


r r
y es paralelo a r, dicho plano es π ≡ Q + < u , v > , del
que interesa su ecuación general.

2. Se calcula la distancia de P al plano anterior.

Método II

Consiste en aplicar la fórmula


[ QP , ur,vr ]
d (r , s ) = r r , (16.5)
u ∧v
que se deduce de la interpretación geométrica del
producto mixto.
→ r
En efecto, si se consideran los vectores QP , u y
r →
v (daría igual PQ ) , se sabe que el valor absoluto del
producto mixto de esos tres vectores coincide con el volumen del paralelepípedo de aristas
→ r r r r
QP , u y v , es decir: V = [ QP , u , v ] .

Por otro lado, dicho volumen es también el área de la base por la altura. Como el área de la
r r r r
base es A = u ∧ v y la altura es la distancia buscada, también V = u ∧ v ⋅ d (r , s ) .

[ QP , ur,vr ]
Igualando y despejando se tiene d (r , s ) = r r .
u ∧v

x −1 y + 2
Ejemplo 16.9 Para calcular la mínima distancia entre las rectas r : = =z y
2 −3
y −1 z
s : x +2 = = , primero hay que estudiar la posición relativa de ambas. Es fácil ver que
2 3
las rectas se cruzan. Calcularemos la distancia por los dos métodos enunciados.

Método I

1. El plano π que contiene a r y es paralelo a s es


x −1 y + 2 z
π≡ 2 −3 1 = 0 ⇔ π ≡ 11x + 5 y − 7z − 1 = 0 .
1 2 3

16-6
Proyecciones, distancias y ángulos Tema 16

2. La distancia de Q (− 2,1,0 ) ∈ s al plano π es


− 22 + 5 − 1 18
d (r , s ) = d (Q, π ) = = .
2
11 + 5 + (− 7 )
2 2 195
Método II
→ r r
Consideramos los vectores QP ( 3, −3, 0 ) , u (1, 2, 3 ) y v ( 2, −3,1) y hallamos el módulo

3 1 2
r r
de su producto mixto: [ QP , u , v ] = − 3 2 − 3 = 6 − 18 + 27 + 3 = 18 ,
0 3 1

r r r r
por otro lado, u ∧ v = (11, 5, −7 ) ⇒ u ∧ v = 195 .

 →r r 
 QP,u,v 
  18
Aplicando (16.5) : d ( r ,s ) = r r = .
u ∧v 195

16.5 Ángulo de dos rectas

Se entiende como ángulo de dos rectas al menor de los ángulos que forman.

También se habla de ángulo de dos rectas


cuando éstas se cruzan y, en este caso, es igual al
ángulo que forma una de ellas con la proyección de
la otra sobre el plano que contiene a la primera y es
paralelo a la segunda.

El cálculo del ángulo de dos rectas se halla


a partir del ángulo que forman sus respectivos
vectores directores.

Se toma en valor absoluto para que sea el


ángulo mínimo, menor de 90º. Así pues, dadas
r r
r ≡ P+ < u > y s ≡ Q + < v > , el ángulo que
forman es
r r
u ⋅v (16.6)
cos α = r r .
u ⋅v

x −1
Ejemplo 16.10 Para halla el ángulo que forman las rectas = y −3 = z y
r:
2
x+2 y +5 r
s: = ,z = 0 , se consideran sus vectores directores respectivos, que son u ( 2,1,1) y
−3 4
r r
r u ⋅v −6 + 4 2
v ( −3, 4, 0 ) , y se aplica la fórmula (16.6): cos α = r r = = ⇒ α = 80º 40′ .
u ⋅v 6 ⋅ 25 5 ⋅ 6

16-7
Proyecciones, distancias y ángulos Tema 16

16.6 Ángulo de dos planos


Es el menor de los dos ángulos diedros que determinan. Coincide con el ángulo
que forman sus vectores perpendiculares o el suplementario de éste.

Por tanto, dados los planos π : Ax + By + Cz + D = 0


y π : Ax + By + Cz + D ′ = 0 el ángulo que forman vendrá

dado por

( A,B,C ) ⋅ ( A′,B ′,C ′ ) (16.7)


cos α = .
A 2 + B 2 + C 2 ⋅ A′2 + B ′2 + C ′2

Ejemplo 16.11 Para hallar el ángulo que forman los planos π : 2x + y + z + 2 = 0 y


( 2,1,1) ⋅ ( 3,3,0 ) 3
π′ : 3 x + 3 y + 1 = 0 , se aplica la fórmula (16.7): cos α = = ⇒ α = 30º .
6 ⋅ 18 2

16.7 Ángulo de recta y plano

Es el ángulo que forma la recta con su proyección ortogonal sobre el plano.

Se halla a partir del ángulo que forman el vector director


de la recta y el perpendicular al plano.

El ángulo buscado es el complementario del formado por


los vectores anteriores.
P ( x ,y ,z )
Sean π : Ax + By + Cz + D = 0 y r :  r 0 0 0 entonces
 v (v1 ,v 2 ,v 3 )

( A,B,C ) ⋅ (v1 ,v 2 ,v 3 )
senα = cos β = .
2 2 2 (16.8)
A2 + B 2 + C 2 ⋅ v1 + v 2 + v 3

Ejemplo 16.12 Para hallar el ángulo que forman el plano π : 2 x + y + z + 2 = 0 y la recta


x − y + 1 = 0
r: , se obtiene primero un vector director de la recta y después se aplica la
z = 1
fórmula (16.8).

16-8
Proyecciones, distancias y ángulos Tema 16

Un vector director de la recta es (1, −1, 0 ) ∧ ( 0, 0,1) = ( −1, −1, 0 ) .


( 2,1,1) ⋅ ( −1, −1,0 ) 3 3
Luego, senα = cos β = = ⇒ α = arcsen = 60º .
6⋅ 2 2 2

16.8 Ecuaciones de la perpendicular común a dos rectas que se cruzan


Se trata de hallar la recta que, cortando a dos rectas que se cruzan, forma con ambas
un ángulo de 90º.

Veamos dos procedimientos para llegar a las ecuaciones de esta recta.


r r
Sean r ≡ P + < u > y s ≡ Q + < v > .

Método I

1. Se halla la ecuación del plano π que contiene


a una de ellas (por ejemplo r) y es paralelo a
la otra (en nuestro caso s). Dicho plano es
r r
π ≡ P + < u,v > .

2. Se hallan las ecuaciones de los planos


π1 y π2 perpendiculares a π y que contienen
a r y a s , respectivamente. Dichos planos son
r r r r r r
π1 ≡ P + < u , u ∧ v > y π 2 ≡ Q + < v , u ∧ v > .

3. La intersección de π1 y π2 es la recta buscada.

Método II

1. Se halla la ecuación del plano π que contiene a una de ellas (por ejemplo a r) y es
r r
paralelo a la otra. Dicho plano viene determinado por π ≡ P + < u , v > .

2. Se halla la ecuación del plano π1 perpendicular a π y que contiene a una de ellas


r r r
(por ejemplo r ) : π1 ≡ P + < u , u ∧ v > .

3. Se halla el punto H intersección de s con π1 . Este punto pertenece a la recta


buscada.
r r
4. La recta buscada es p ≡ H + < u ∧ v > .

Ejemplo 16.13 Hallemos la ecuación de la perpendicular común a las rectas


x −1 y + 2 y −1 z
r: = = z y s : x +2 = = del ejemplo 16.10, que se sabe que se cruzan.
2 −3 2 3

Usaremos los dos métodos.

Método I

1. La ecuación del plano π que contiene a r y es paralelo a s es:

16-9
Proyecciones, distancias y ángulos Tema 16

x −1 y + 2 z
2 −3 1 = 0 → 11x + 5 y − 7z − 1 = 0 ,
1 2 3

su vector característico (ortogonal a π ) es v a (11,5,−7 ) .

2. La ecuación del plano π1 perpendicular a π y que contiene a r es:


x −1 y + 2 z
2 −3 1 = 0 → 16 x + 25 y + 43z + 34 = 0 .
11 5 −7

3. La ecuación del plano π2 perpendicular a π y que contiene a s es:

x + 2 y −1 z
1 2 3 = 0 → 29 x − 40 y + 17z + 98 = 0 .

11 5 −7

16 x + 25 y + 43 z + 34 = 0
4. La perpendicular común es la recta p : 
29 x − 40 y + 17 z + 98 = 0

Método II

Los pasos 1 y 2 son los mismo que los del método anterior.

3. Calculamos el punto H intersección de s con π1 :

 x = −2 + t

y = 1 + 2t 27  417 141 81 
 ⇒t=- ⇒ H − , ,− .
z = 3t 195  195 195 195 

16 x + 25y + 43z + 34 = 0

 417 141 81 
4. La recta es p ≡  − , ,− + < ( 11, 5 , − 7 ) > . Sus ecuaciones paramétricas son:
 195 195 195 
 417
 x = − 195 + 11t
 141
p : y = + 5t .
 195
z = − 81 − 7t
 195

Puede comprobarse que evidentemente se trata de la misma recta aunque


determinada con diferentes ecuaciones.

16-10
Proyecciones, distancias y ángulos Tema 16

Ejercicios resueltos

Proyecciones y simetrías

z−3
R.1. Dada la recta r ≡ x + 1 = y − 2 = y el punto P (1, 2,1) . Halla las coordenadas del
4
punto P’ simétrico de P respecto a r.

Solución:

Seguiremos el siguiente proceso:

1º Calculamos la ecuación del plano que pasa por P y


es perpendicular a r. Será

( x − 1) + ( y − 2 ) + 4 ( z − 1) = 0 → x + y + 4z − 7 = 0 .
 x = −1 + t

2º Hallamos el punto Q intersección de r :  y = 2 + t con el plano obtenido.
z = 3 + 4t

1  4 5 5
( −1 + t ) + ( 2 + t ) + 4 ( 3 + 4t ) − 7 = 0 → 18t + 6 = 0 → t =− → Q− , ,  .
3  3 3 3
3º Teniendo en cuenta que Q es el punto medio de P y P’ nos queda:
4 x +1 11 5 y + 2 4 5 z +1 7
− = →x=− , = →y = , = →z= así que el punto P’ simétrico
3 2 3 3 2 3 3 2 3
 11 4 7 
de P respecto de r es P ′  − , ,  .
 3 3 3

x z
R.2. Halla la proyección de la recta r ≡ = y − 1 = sobre el plano π : x + 2y − z + 4 = 0 .
2 3
Solución:

Primero vemos la posición relativa de la


recta y el plano pasando r a forma paramétrica y
 x = 2t

resolviendo el sistema. r : y = 1 + t ,
z = 3t

2t + 2 (1 + t ) − 3t + 4 = 0 → t = −6 con lo que el
punto de corte es P1 ( −12, −5, −18 ) éste es ya un
punto de la recta proyección de r, tomaremos
otro punto de r y calcularemos su proyección ortogonal sobre π . Elegimos, por ejemplo,
t = 0 → Q ( 0,1, 0 ) .
Calculamos las ecuaciones de la recta que pasa por Q y es perpendicular a π , será
 x = α

 y = 1 + 2α
 z = −α

Calculamos la intersección Q1 de la recta con π , α + 2 + 4α + α + 4 = 0 → α = −1 así


que Q1 ( −1, −1,1) .

16-11
Proyecciones, distancias y ángulos Tema 16

uuuuur
La recta buscada es la que pasa por P1 y Q1, el vector P1Q1 (11, 4,19 ) y sus ecuaciones
x +1 y +1 z −1
son = = .
11 4 19

R.3. Determina las ecuaciones de la recta simétrica de r : x − 1 = y − 2 = z − 3 respecto


del punto P ( 3, 2,1) .
Solución:

Seguimos el siguiente proceso:

1. Calculamos la ecuación del plano que pasa por P


y es perpendicular a r, viene determinado por
P ( 3, 2,1)
 r π: x+y +z−6 =0.
 v (1,1,1)

2. hallamos la intersección R de r y π 1 + α + 2 + α + 3 + α − 6 = 0 → α = 0 → R (1, 2, 3 ) .

3. calculamos el punto R’ simétrico de R respecto a P considerando que P es el punto


medio entre R y R’ , de donde R ′ ( 5, 2, −1) .

4. La recta pedida es la paralela a r por R’, sus ecuaciones: r ′ : x − 5 = y − 2 = z + 1 .

x −1 y −1 z − 2
R.4. Halla la ecuación de la proyección ortogonal r’ de la recta r : = =
2 1 2
sobre el plano α : x − 3 y + 2z + 12 = 0 .

Solución:

El punto P’ de corte de la recta y el plano es uno de los puntos de r’, para ello conviene
 x = 1 + 2t

expresar r en forma paramétrica r :  y = 1 + t , sustituyendo en la ecuación del plano:
z = 2 + 2t

14  31 17 34 
1 + 2t − 3 (1 + t ) + 2 ( 2 + 2t ) + 12 = 0 → t = → P′ , ,  .
3  3 3 13 
Calculamos ahora la proyección del punto A (1,1, 2 ) sobre el plano.
x = 1 + µ

- Ecuaciones de la recta s que pasa por A y es perpendicular a α , s :  y = 1 − 3µ
z = 2 + 2µ

- Punto A’ de corte de s y α , 1 + µ − 3 (1 − 3µ ) + 2 ( 2 + 2µ ) + 12 = 0 → µ = −1 → A ′ ( 0, 4, 0 )

 x = 31λ
uuuuur 
- Ecuaciones de la recta que pasa por P’ y A’ , 3 A ′P ′ = ( 31, 5, 34 ) , luego: r ′ :  y = 4 + 5λ .
z = 34λ

Distancias

R.5. Dados los puntos A (1, 2, 5 ) y B ( 3, 2, −1) y los planos π : 2 x − 7 y + z − 2 = 0 y


π ′ : 2 x − 7 y + z + 13 = 0 . Calcula:

16-12
Proyecciones, distancias y ángulos Tema 16

a) Distancia entre los puntos.


b) Distancia del punto A al plano π .
c) Distancia entre los dos planos.

Solución:

2 2 2
a) La distancia entre A y B es: d ( A,B ) = ( 3 − 1) + ( 2 − 2 ) + ( −1 − 5 ) = 40 .

2 ⋅1− 7 ⋅ 2 + 5 − 2 9 6
b) Aplicamos la fórmula y se tiene: d ( A, π ) = = = .
4 + 49 + 1 54 2

−2 − 13 15 5 6
c) Teniendo en cuenta que son planos paralelos: d ( π, π ′ ) = = = .
4 + 49 + 1 54 6

x = 1 + t

R.6. Calcula la distancia del punto B ( 3, 2, −1) a la recta r :  y = 2t
z = −3 + t

Solución:

Calcularemos la distancia de dos modos diferentes:

Método I

Consiste en buscar la proyección ortogonal H de B sobre r y calcular la distancia entre


B y H.

Se siguen los siguientes pasos:

1. Calculamos la ecuación del plano π perpendicular a r desde B.


1( x − 3 ) + 2 ( y − 2 ) + 1( z + 1) = 0 → x + 2y + z − 6 = 0 .

2. Calculamos el punto H de corte de r y π resolviendo el sistema formado por sus


4 7 8 5
ecuaciones. 1(1 + t ) + 2 ( 2t ) + 1( −3 + t ) = 0 → t = → H  , , −  .
3 3 3 3

3. Aplicamos la fórmula para calcular la distancia de B a H

2 2 2
 7  8  5 12
d ( B,h ) =  3 −  +  2 −  +  −1 +  = .
 3   3   3  3

Método II

Aplicamos la fórmula a partir del producto vectorial, siendo P (1, 0, −3 ) un punto y


uuur r
r PB ⋅ v uuur uuur r
v (1, 2,1) un vector director de r d ( B,r ) = r como PB = ( 2, 2, 2 ) y PB ∧ v = ( −2, 0, 2 ) se
v
4+0+4 8 12
tiene que d ( B,r ) = = = .
1+ 4 + 1 6 3

16-13
Proyecciones, distancias y ángulos Tema 16

x + y − z = 4 x = 2
R.7. Calcula la distancia entre las rectas r :  y s: .
 x + 2y = 7  y = −5
Solución:

Primeramente estudiamos la posición relativa de r y s. Buscamos sus ecuaciones


paramétricas con el fin de obtener un punto y un vector director de cada recta.
1 −1
En las ecuaciones de r podemos elegir x como parámetro ya que ≠ 0 . Se tiene:
2 0

x = t
 x = 2
 7 1 Pr (1, 3, 0 ) ( para t = 1)  Ps ( 2, −5, 0 )
r : y = − t →  r y s :  y = −5 → r
 2 2 
v r ( 2, − 1, 1) z = s v s ( 0, 0,1)
 1 1 
 z = − + t
2 2
1 −8 0
uuuuur
El vector Pr Ps = (1, −8, 0 ) y como 2 −1 1 = 15 ≠ 0 las rectas se cruzan.
0 0 1

A partir de aquí calcularemos la distancia de dos formas:

Método I

Se siguen los siguientes pasos:

1. Calculamos la ecuación del plano π que contiene a r y es paralelo a s. Su ecuación es:


x −1 y − 3 z
2 −1 1 = 0 ⇒ x + 2y − 7 = 0 .
0 0 1

2. La distancia de cualquier punto de s a π es la distancia mínima entre ambas rectas.


2 + 2 ( −5 ) − 7 15
d ( Ps , π ) = = =3 5.
1+ 4 5

Método II

Aplicando la fórmula se tiene:

uuuuur r r 1 −8 0
Pr Ps ,v s ,v r  uuuuur r r
  r r
d ( r ,s ) = r r como Pr Ps ,v s ,v r  = 0 0 1 = −15 y v s ∧ v r = (1, 2, 0 ) entonces:
vs ∧ v r
2 −1 1

uuuuur r r
Pr Ps ,v s ,v r 
  15 15
d ( r ,s ) = r r = = =3 5.
vs ∧ vr 1+ 4 + 0 5

Ángulos

R.8. Calcula el ángulo diedro formado por los planos de ecuaciones π1 : x − 2 y + z − 1 = 0


x = 1 + α + β

y π2 :  y = 2 − α − 3β .
z = β

16-14
Proyecciones, distancias y ángulos Tema 16

Solución:

En primer lugar debemos buscar la ecuación general de π2 .

1 1 x −1
Eliminando los parámetros llegamos a −1 −3 y − 2 = 0 ⇒ x + y + 2z − 3 = 0
0 1 z

r r
El ángulo pedido es el de sus vectores característicos v1 (1, −2,1) y v 2 (1,1, 2 ) luego
r r
v1 ⋅ v 2 1− 2 + 2 1
α ( π1 , π2 ) = arccos r r = arccos = arccos = 80º 24 ′ 21′′ .
v v2 6 6 6

2 x − y + z = −4
R.9. Calcula el ángulo formado por las rectas r :  y
y + z = 1
z−2
s : −x − 2 = 1 − y = .
2

Solución:
r
Calculamos un vector director de r , v r = ( 2, −1,1) ∧ ( 0,1,1) = ( −2, −2, 2 ) .

Obtenemos también un vector director de s (¡cuidado! que no viene dada en forma


r
continua) v s = ( −1, −1, 2 )

Calculamos el ángulo formado por estos dos vectores


r r
vr ⋅ vs 2+2+4 8
α ( r ,s ) = arccos r r = arccos = arccos = 19º 28 ′ 16 ′′ ¨.
vr vs 4 + 4 + 4 1+ 1+ 4 72

z−2
R.10. Calcula el ángulo formado por la recta s : − x − 2 = 1 − y = y el plano
2
π1 : x − 2 y + z − 1 = 0 de los ejercicios anteriores.

Solución:

El ángulo es el complementario del ángulo agudo formado por las direcciones del
r
vector director de la recta y el asociado del plano que son respectivamente v s = ( −1, −1, 2 ) y
r r
r v s ⋅ v1 −1 + 2 + 2 1
v1 (1, −2,1) por tanto α ( s, π1 ) = arc sen r r = arc sen = arc sen = 30º .
v s v1 6 6 2

Otros ejercicios

 x − 4y + 9 = 0
R.11. Considera la recta dada por r :  .
3 y − z − 9 = 0
a) Determina el plano que pasa por el punto P (1, 4, 0 ) y contiene a r.
b) ¿Para cualquier valor de λ , el plano x − 4 y + 9 + λ ( 3 y − z − 9 ) = 0 contiene a r?.

16-15
Proyecciones, distancias y ángulos Tema 16

c) Determina los valores de λ para que el plano diste 3 unidades del origen de
coordenadas.

Solución:

a) La ecuación del plano pedido puede obtenerse de dos modos diferentes:

Método I

Buscamos las ecuaciones de r en forma paramétrica, haciendo y = t se tiene:


 x = −9 + 4t

r : y = t , un punto de r es Q ( −9, 0, −9 ) que pertenece al plano , y un vector director de r
z = −9 + 3t

r uuur uuur r
es v ( 4,1, 3 ) que junto con el vector QP (10, 4, 9 ) pertenecen al plano. QP ∧ v = ( 3, 6, −6 ) es
perpendicular al plano, así su ecuación es 3 ( x − 1) + 6 ( y − 4 ) − 6z = 0 → x + 2 y − 2z − 9 = 0 .

Método II

Escribimos la ecuación del haz de planos de arista r α ( x − 4 y + 9 ) + β ( 3 y − z − 9 ) = 0


y obligamos a que P pertenezca al haz sustituyendo las coordenadas se llega a
−6α + 3β = 0 → si α = 1 → β = 2 y sustituyendo estos valores en la ecuación del haz se
obtenemos x + 2y − 2z − 9 = 0 .

b) Si sustituimos x, y y z de las ecuaciones paramétricas de la recta en la expresión


dada se llega a −9 + 4t + −4t + 9 + λ ( 3t + 9 − 9t − 9 ) = 0 → 0 = 0 , por tanto la recta está
contenida en el plano para todo valor de λ .

c) La ecuación de todos esos planos es de la forma x + ( 3λ − 4 ) y − λz + 9 − 9λ = 0 ,


aplicando la fórmula de distancia de un punto a un plano se tiene:
9 − 9λ
d = = 3 → 9λ 2 − 18λ + 9 = 10λ 2 − 24λ + 17 → λ 2 − 6λ + 8 = 0 cuyas
2
1 + ( 3λ − 4 ) + λ 2

soluciones son λ = 4 y λ = 2 .

x = 1 + λ x = µ
 
R.12. Dadas las rectas r :  y = λ y s :  y = 2 + 2µ . Halla la ecuación de una recta que
z = −λ z = 0
 
sea perpendicular simultáneamente a r y s.

Solución:

Primero estudiamos la posición relativa de las rectas, éstas vienen determinadas


 P (1, 0, 0 ) P ( 0, 2, 0 ) uuuuur
respectivamente por r :  r r y s :  rs . El vector Pr Ps = ( −1, 2, 0 ) .
v r (1,1, −1)  v s (1, 2, 0 )

−1 2 0
uuuuur r r
 
Hacemos el producto mixto de los tres vectores Pr Ps ,v r ,v s  = 1 1 −1 = −4 ≠ 0 los
1 2 0
tres vectores no son coplanarios y por tanto las rectas se cruzan.

16-16
Proyecciones, distancias y ángulos Tema 16

Para hallar las ecuaciones de la perpendicular común en este caso seguiremos los
siguientes pasos:

1. Calculamos la ecuación del plano π que contiene a r y es paralelo a s. Su vector


r r r
i j k
r r
característico será v r ∧ v s 1 1 −1 = ( 2, −1,1) y su ecuación 2 x − y + z − 2 = 0 .
1 2 0

2. Calculamos la ecuación del plano π1 que contiene a r y es perpendicular a π .

x −1 y z
Su ecuación es 1 1 −1 = 0 → y + z = 0 .
2 −1 1

3. Calculamos la ecuación del plano π2 que contiene a s y es perpendicular a π .

x y −2 z
Su ecuación es 1 2 0 = 0 → 2 x − y − 5z + 2 = 0 .
2 −1 1

y + z = 0
4. La recta pedida en forma implícita es la de ecuaciones  .
2 x − y − 5z + 2 = 0

x − y + z = 1
R.13. Dados los puntos A ( −2, −4, 3 ) y B ( 2, 6, 5 ) , y la recta r ≡  , averiguar
2 x + y − 3z = 2
si existe alguna recta tal que contenga los puntos A y B y corte a la recta r . Razona la
respuesta.

Solución:

Los puntos A y B determinan un recta concreta, buscamos sus ecuaciones.

uuur B ( 2, 6, 5 )
Calculamos AB = ( 4,10, 8 ) . La recta está determinada por r , (el vector
v ( 2, 5, 4 )
r uuur
v ( 2, 5, 4 ) es paralelo al AB = ( 4,10, 8 ) ).

 x = 2 + 2t

Sus ecuaciones: r :  y = 6 + 5t .
z = 5 + 4t

Si sustituimos las ecuaciones de esta recta en las de la recta dada veremos si se


2 + 2t − 6 − 5t + 5 + 4t = 1
cortan o no,  , de la primera t = 0 .
2 ( 2 + 2t ) + 6 + 5t − 3 ( 5 + 4t ) = 2

Sustituyendo en la segunda ese valor 2 ( 2 ) + 6 + 0 − 3 ( 5 ) ≠ 2 , por tanto las rectas no


se cortan.

16-17
Proyecciones, distancias y ángulos Tema 16

x = t

R.14. Halla la distancia del punto P (1, 2, 3 ) a la recta r de ecuaciones r :  y = 6 − t ,
z = 2 + t

determinando el punto de la recta que dista menos de P.

Solución:

- Calculamos la ecuación del plano π que pasa por P y es perpendicular a r. Será


( x − 1) ( y − 2) + ( z − 3) = 0 → x − y + z − 2 = 0 .

- Calculamos el punto Q de corte de π con r resolviendo el sistema, dicho punto es el


de la recta que dista menos de r. Tenemos t − 6 + t + 2 + t − 2 = 0 → t = 2 → Q ( 2, 4, 4 ) , la

distancia pedida es la de P a Q d ( P,Q ) = 12 + 22 + 12 = 6 .

R.15. Dado el tetraedro de vértices ABCD con A (1, 2, 0 ) , B ( 2, 6, 0 ) , C ( 5, 3, 0 ) y D ( 3, 4, 3 ) .


a) Comprueba que los puntos medios de las aristas AB, BD, DC, y CA son
coplanarios y calcula la ecuación de ese plano.
b) Comprueba que el plano hallado es paralelo a las aristas CB y AD.
c) Calcula el volumen del tetraedro.

Solución:
3 
a) Calculamos los puntos medios pedidos, son: M ( A,B ) =  , 4, 0  ,
2 
 5 3   7 3   5 
M ( B,D ) =  , 5,  , M ( D,C ) =  4, ,  , M (C, A ) =  3, , 0  .
2 2  2 2  2 

Calculamos los vectores que van desde M ( A,B ) a cada uno de los otros tres puntos
r  3 r 5 1 3 r 3 3 
que son respectivamente v1 =  1,1,  , v 2 =  , − ,  y v 3 =  , − , 0  , si los cuatro puntos
 2   2 2 2  2 2 
son coplanarios el rango de estos vectores es 2, (si multiplicamos los tres vectores por 2 el
2 2 3
razonamiento no cambia); como 5 −1 3 = 0 los cuatro puntos son coplanarios. La ecuación
3 −3 0
3
x− y −4 z
2
del plano es 2 2 3 = 0.
3 −3 0

uuur uuur
b) Los vectores CB = ( −3, 3, 0 ) y AD = ( 2, 2, 3 ) están contenidos en el plano luego
dichas aristas son paralelas a éste.
uuur
c) Los vectores que forman tres aristas concurrentes del tetraedro son AB = (1, 4, 0 )
uuur uuur
AC = ( 4,1, 0 ) , y AD = ( 2, 2, 3 ) ,
 1 4 0
uuur uuur uuur
 AB, AC, AD  =  − 4 1 0  =
1 1 15
el volumen es v =   6 .
6  2 2 3 2
 

16-18
Proyecciones, distancias y ángulos Tema 16

R.16. Dados los puntos A ( 0, 3, −2 ) y B (1, 5, 0 ) Calcula la ecuación del plano perpendicular
al segmento AB y que pasa por su punto medio (plano mediador).

Solución:

Método I
uuur
El vector AB = (1, 2, 2 ) es un vector perpendicular al plano buscado y el punto
1 
M ( A,B ) =  , 4, −1 pertenece al plano.
 2 
 1
Su ecuación es  x −  + 2 ( y − 4 ) + 2 ( z + 1) = 0 → 2 x + 4 y + 4z − 13 = 0 .
 2

Método II

El plano pedido es el lugar geométrico de los puntos que equidistan de A y b y que por
2 2 2 2
tanto cumplen la condición x 2 + ( y − 3 ) + ( z + 2) = ( x − 1) + ( y − 5 ) + z 2 en la que
elevando al cuadrado y desarrollando se tiene

x 2 + y 2 + z 2 − 6 y + 4z + 9 + 4 = x 2 + y 2 + z 2 − 2 x − 10 y + 1 + 25

que, simplificando, lleva a la misma ecuación 2 x + 4 y + 4z − 13 = 0 .

16-19
Proyecciones, distancias y ángulos Tema 16

Ejercicios propuestos

Distancias
x = 0
P.1. Halla la distancia del punto A(1,2,3 ) a la recta r :  y la ecuación del plano que pasa
z = 0
por A y es perpendicular a r.

 x=s - t

P.2. Halla la distancia de A(1,2,3 ) al plano  y = 2 + 2s − 2t .
 z = − 1 − 3s + 2t

x −1 y z−2
P.3. Dado el punto A(1,0,1) , B(5,7,4 ) y los planos π : 2 x + 5 y − z + 7 = 0 π ′ : 1 0 2 =0.
−2 1 1
Calcula: a) Distancia entre A y B. b) Distancia de A a π . c) distancia entre π y π ′ .

x = t

P.4. Sea el punto A(1,1,3 ) y la recta r : y = 2 + t Halla: a) la ecuación del plano perpendicular a
z = 2t

la recta r que pasa por A. b) La intersección de este plano con la recta r. c) La distancia del
punto A a la recta r.

P.5. Comprueba que la recta x − 3 = y − 2 = 7 − z es paralela al plano x + 2y + 3z = 0 y halla la


distancia de la recta al plano.

x + z = 0 x = 0
P.6. Prueba que las rectas r :  , y s: ; se cruzan, y halla la distancia entre
y = 0 y = 4
ellas.

P.7. Halla la mínima distancia entre las rectas r1 y r2 en los siguientes casos:
 x = 2 + 3α
x −1 y − 2 
a) r1 : = = z − 3 ; r2 : y = 4 + α
2 −1 z = −1 − α

x = α
 x − 2y + z + 1 = 0 
b) r1 :  ; r2 : y = 1 − α
 2 x − 3 y + z + 4 = 0 z = 2 + 5α

x −2 z −1  x − 3 y − 11 = 0
P.8. Calcula la distancia entre las rectas r y s, siendo: r : =y = ; s: .
3 4 4 y − z + 4 = 0

 x = −1 + 2t
 x + 2 y −1 z − 3
P.9. Calcula la distancia entre las rectas r y s, siendo: r : y = t , s: = = ;
z = 3 + 2t −3 2 2

estudiando, previamente, la posición relativa entre ambas rectas.

P.10. Dados los puntos A(3,−2,0 ) y B (1,−2,−2) y la recta r : x = y = z . Calcula la distancia


desde el punto B al plano que contiene a r y al punto A.

16-20
Proyecciones, distancias y ángulos Tema 16

P.11. Para cada t real se considera el plano πt de ecuación:


(1 + 2t )x + (1 − t )y + (1 + 3t )z + 2t − 1 = 0 . Demuestra que todos los planos π t pasan por una
y +1 z − 2
recta r y halla la mínima distancia entre las rectas r y s, siendo s : x − 1 = = .
2 3

Ángulos

x = α − 2β

P.12. Calcula el ángulo diedro formado por los planos π : y = 3 + β y
z = −7 + 2α + β

π ′ : x + y + z + 13 = 0 .

x −1 y +1 z +1 − (x + 1) − (y + 2) z − 1
P.13. Halla el ángulo de las rectas r : = = y r′: = = .
3 4 5 3 4 5

 x = 2t

P.14. Halla el seno del ángulo α que forma la recta r : y = 1 + 3t con el plano
z = 2 + 4t

5 x + 4 y − 2z + 5 = 0 .

P.15. Dados los planos x + y = 0 , y + z = 0 , encuentra la ecuación de un plano que forme


ángulos iguales con los anteriores.

Proyecciones y simetrías

P.16. Halla el área de la proyección del triángulo de vértices (0,0,0 ), (2,1,3 ), (1,1,1) sobre el plano
x+y −z =0.

P.17. Halla el simétrico del punto A(2,2,1) respecto del plano x − 2y + z + 6 = 0 .

x − y = 0
P.18. Sean los planos π : x + y + z = 0 , π ′ : 3 x + 3 y + 2z = 0 y la recta r :  . Se pide:
x + y − z = 0
a) Proyecciones ortogonales de la recta sobre los planos. b) Posición relativa de dichas rectas.
c) Distancia entre ellas.

x−2 z−3
P.19. Dada la recta r : =y= y el punto P (3,−1,0 ) Se pide:
5 4
a) Calcula el punto H proyección ortogonal de P sobre r.
b) Calcula las coordenadas del punto P’ simétrico de P respecto a r.
c) Calcula la distancia de P a r

Otras determinaciones de rectas y planos

z −1 3−x z +1
P.20. Se consideran las rectas r : x − 1 = y − 2 = ; s: =3−y = . a) Comprueba
2 2 2
que se cortan y calcula las coordenadas del punto P de intersección. b) Determina la ecuación
de la recta que pasa por P y es perpendicular a r y s.

P.21. Halla las ecuaciones en forma continua de la recta que pasa por el punto (3,−1,2) , es
x − z = 0
paralela al plano π : 5 x − 9 y + 8z = 3 y es perpendicular a la recta r :  .
y = 0

16-21
Proyecciones, distancias y ángulos Tema 16

P.22. Halla las ecuaciones de las rectas que sean paralelas a la intersección de los planos:
π : x − 2y + 3z + 1 = 0 , π ′ : 3 x − 2y + z − 1 = 0 .

P.23. Escribe las ecuaciones de la perpendicular común a las rectas r :x =y =z,


s : x = y = 3z − 1.
 x = 1+ λ
y −1 z +1 
P.24. Dadas las rectas r : x − 2 = = y s : y = 2 − λ .
k −2  z = 2λ

a) Calcula el valor de k para que sean coplanarias.
b) Para ese valor de k Calcula la ecuación del plano que las contiene.
c) Calcula las ecuaciones de la perpendicular común a r y s.

 x = 1+ t  x=t
 
P.25. dadas las rectas r :  y = −t y s :  y = 1 + 3t . a) Demuestra que se cruzan. b) Calcula
z = 2 + 2t z = −1 − t
 
las ecuaciones de la perpendicular común p a r y s .

Producto vectorial y producto mixto. Áreas y volúmenes

P.26. A(1,3,2) y B (2,5,1) son dos vértices de un triángulo que tiene su tercer vértice situado en
x −2 y −4 z−3
un punto arbitrario (variable) de la recta: r : = = . Calcula el área de los
2 4 −2
diferentes triángulos formados por A, B y el tercer vértice en r. ¿El valor de dicha área depende
de dónde se sitúe el tercer vértice? .

P.27. Sean los puntos A(1,1,1) , B (3,0,2 ) , C (5,−2,2) y D(2,1, t ) . Halla el valor de t para que los
cuatro puntos determinen un plano. Halla el área de dicho cuadrilátero para el valor de t
calculado previamente.

P.28. Halla el volumen del tetraedro que forman los planos de ecuaciones y = 0, z = 0, x − y = 0
y 3 x + 2y + z − 15 = 0 .
x + y = 1
P.29. Calcula la ecuación del plano que pasa por la recta  , y determina con el primer
z = 0
octante un tetraedro de volumen 2.

P.30. Aplicando el producto vectorial, halla la dirección de la recta r intersección de los planos
π : 2 x + y − z + 6 = 0 , π ′ : 3 x − 2y + z − 1 = 0 .

P.31. Halla el área de la proyección del triángulo de vértices A(1,0,0 ) , B(2,1,0 ) y C (0,1,4 ) sobre
el plano coordenado XY.

P.32. Halla el área del triángulo de vértices: A(0,0,0 ) , B el punto de intersección de la recta
x +1 y +1
= = z + 2 con el plano XY y C (1,1,0 ) .
2 3

P.33. Determina t, para que los puntos A(1,1,1) , B(3,0,2) , C (5,−2,2 ) y D (2,1,t ) sean coplanarios.
Para el valor de t calculado anteriormente, obtener el área del polígono ABCD.

P.34. Razona si los puntos A(0,−1,2 ) , B (1,3,4 ) , C (0,1,7 ) y D(1−


, 1,2) son coplanarios o son los
vértices de un tetraedro. Si forman un tetraedro, halla su volumen.

16-22
Proyecciones, distancias y ángulos Tema 16

Soluciones

2 5 8
P.1. a) 10 , b) y − 2 = 0 . P.2. . P.3. a) d (A, B ) = 74 , b) d (A,π ) = , c)
5 30
7
d (π ,π ′) = (planos paralelos). P.4. a) x + y + 2z − 8 = 0 , b) (1,3,2) , c) 5 . P.5. a)
30
14 819
(1,1,−1) ⋅ (1,2,3) = 0 b) 2 14 . P.6. d = 4 . P.7. a) 2 , b) . P.8. . P.9. a) Se cruzan, b)
2 26
 x = 1 − 4α
8 6  x + y + z −1= 0  57
. P.10. . P.11. a) r :  o bien r :  y = −1 + α , b) . P.12.
153 38 2 x − y + 3z + 2 = 0 z = −1 + 3α 315

r r 14 3
v a (2,5,−1), v a′ (1,1,1) α = 50º 46 ′ 6 ′′ . P.13. 90º. P.14. . P.15. 3 x + y + 3z + 1 = 0 . P.16. .
9 15 3
 1 16 2  z z
P.17.  , ,−  . P.18. a) x = y = ,x = y = , b) Se cortan en (0,0,0 ) , c) d = 0 . P.19. a)
3 3 3 −2 −3
 22 4 47   19 13 94 
plano π perpendicular a r desde P 5 x + y + 4z − 14 = 0 , H  ,− ,  , b) P ′ − , , ,
 21 21 21   21 21 21 
199 x −1 y − 2 x −3 z−2
c) d (P, r ) = d (P, H ) = . P.20. a) (1,2,1) , b) = = z − 1 . P.21. = y +1= .
21 4 − 6 − 3 3
y − y0
P.22. Hay infinitas de la forma x − x 0 = = z − z0 tales que (x 0 , y 0 , z0 ) ∉ π ∩ π ′ . P. 23.
2
 1
 x = 2 − 2α
 1
y = + 2α . P.24. a) No pueden ser paralelas, se cortan para k = −1 , b) x + y − 3 = 0
 2
z = 1
 2
 5
x = 4 + α
 5 7 1  7
c) punto de corte  , ,  , perpendicular común y = + α
4 4 2  4
z = 1
 2
contiene a s
{ r r
}
P.25. a) rang Pr Ps (− 1,1,3 ),v r (1,−1,2),v s (1,3,−1) = 3 ; b) π : 
 paralelo a r
⇒ −5 x + 3 y + 4 z + 1 = 0

 1
 x = 25 − 5t
contiene a r  1 28 26   28
π′ :  ⇒ 5 x + 7 y + z − 7 = 0 Pc (s,π ′) =  , ,− ; p: y = + 3t .
perpendicular a π  25 25 25   25
z = − 26 + 4t
 25
30 5 3 75
P.26. , no. P.27. a) t = 2 , b) . P.28. . P.29. 12 x + 12y + z − 12 = 0 . P.30.
2 2 2
r 5 3 8
v (1,5,7 ) . P.31. S = 1 . P.32. S = 1 . P.33. a) t = 2 b) . P.34. a) tetraedro, b) .
2 3

16-23

Potrebbero piacerti anche